Chapter 9 Bio, Chapter 2, ECM, Bio Ch 18, Cell Biology Ultimate v1.1, Chapter 17 End of note questions, Chapter 11, Biology Lab Exam II, DNA Replication Questions, ch 17,18,20,22 practice questoions, Bio 1113, Ch. 6, Bio lab exam, Chapter 10 flashcar...

Pataasin ang iyong marka sa homework at exams ngayon gamit ang Quizwiz!

Although all protein structures are unique, there are common structural building blocks that are referred to as regular secondary structures. Some have alpha helices, some have beta sheets, and still others have a combination of both. What makes it possible for proteins to have these common structural elements? (a) specific amino acid sequences (b) side-chain interactions (c) the hydrophobic core interactions (d) hydrogen bonds along the protein backbone

(d) hydrogen bonds along the protein backbone

14-57The ATP synthase found in chloroplasts is structurally similar to theATP synthase in mitochondria. Given that ATP is being synthesized in the stroma, where will the F0portion of the ATP synthase be located? (a)thylakoid space (b)stroma (c)inner membrane (d)thylakoid membrane

(d) thylakoid membrane

Chloroplasts and Photosynthesis 14-54Photosynthesis is a process that takes place in chloroplasts and uses light energy to generate high-energy electrons, which are passed along an electron-transport chain. Where are the proteins of the electron-transport chain located in chloroplasts? (a)thylakoid space (b)stroma (c)inner membrane (d)thylakoid membrane

(d) thylakoid membrane

2.14. you have a concentrated stock solution of 10M NaOH and want to use it to produce a 150mL solution of 3 M NaOH. what volume of water and stock solutions will you measure out to make this new solution?

105mL water 45 mL NaOH stock

2.8. if the isotope 32S has 16 protons and 16 neutrons, how many protons, neutrons, and electrons will the isotope 35S have, respectively?

16,19,16

E

19) If glucose is available in the environment of E. coli, the cell responds with very low concentration of cAMP. When the cAMP increases in concentration, it binds to CAP. Which of the following would you expect would then be a measurable effect? A) decreased concentration of the lac enzymes B) increased concentration of the trp enzymes C) decreased binding of the RNA polymerase to sugar metabolism-related promoters D) decreased concentration of alternative sugars in the cell E) increased concentrations of sugars such as arabinose in the cell

An mRNA sequence is 5′AUG-GGC-ACU-CAU-ACU-UAA3′, where AUG is the start codon and UAA is the stop codon. How many distinct aminoacyl-tRNA synthetases are required to translate the mRNA sequence?

4

An mRNA sequence is 5′AUG-GGC-ACU-CAU-ACU-UAA3′, where AUG is the start codon and UAA is the stop codon. How many distinct aminoacyl-tRNA synthetases are required to translate the mRNA sequence? 2 3 4 5 6

4

2.9.b. what is the concentration, in grams per liter, of a .25M solution of glucose?

45g/L

C

58) A cell that remains entirely flexible in its developmental possibilities is said to be A) differentiated. B) determined. C) totipotent. D) genomically equivalent. E) epigenetic.

D

59) Differentiation of cells is not easily reversible because it involves A) changes in the nucleotide sequence of genes within the genome. B) changes in chromatin structure that make certain regions of the genome more accessible. C) chemical modifications of histones and DNA methylation. D) frameshift mutations and inversions. E) excision of some coding sequences.

If a DNA template strand has a sequence of 3′ TACAATGTAGCC 5′, then the RNA produced from it will be which sequence?

5′AUGUUACAUCGG3'

E

6) A lack of this nonprotein molecule would result in the inability of the cell to "turn off" genes: A) operon B) inducer C) promoter D) repressor E) corepressor

What happened to the six-carbon molecule found in the original glucose molecule?

6carbon--> pyruvate --> acytyl coA

2.9.c. how many molecules are there in 1 mole of glucose?

6x10^23 molecules

Kinesins were purified by adding the nonhydrolyzable analog AMP-PNP to cytoplasmic extracts containing microtubules, purifying the microtubules, and then releasing the kinesin proteins, which were still attached to the microtubules, by adding ATP. Would this trick have worked to purify myosin motors attached to actin filaments? Explain.

No, addition of a nonhydrolyzable form of ATP would not increase the affinity of myosin for actin. This is because when the myosin motor binds to ATP, the myosin head undergoes a conformational change that reduces its affinity for actin.

In which group of organisms would you expect to see polyribosomes? A) prokaryotes B) eukaryotes C) both D) neither

C) both

54.a. what would be the best model organism to use for the investigation of the process of: programmed cell death

C. elegans

22. some prokaryotes can live by utilizing entirely inorganic materials. which of the following inorganic molecules would you predict to be the predominant building block for fats, sugars, and proteins? a. O2 b. N2 c. CO2 d. H2

CO2

Explain how changes in leaf architecture help isolate rubisco in high CO2 areas but low O2 areas.

CO2 concentrations are maintained in the bundle sheeth so that photosynthesis will be favored over respiration

what would cause a proto-oncogene to become an oncogene?

DNA breakage point mutation viral infection

3.32. true or false: enzymes lower the free energy released by the reaction that they facilitate

False. Enzymes do not affect the initial energy of the reactants nor the final energy of the products after the reaction is complete, which are the values that determine the change in free energy of a reaction.

Who showed that DNA is the genetic material of a phage known as T2

Hershey and Chase

14-7The citric acid cycle generates NADH and FADH2, which are then used in the process of oxidative phosphorylation to make ATP. If the citric acid cycle (which does not use oxygen) and oxidative phosphorylation are separate processes, as they are, then why is it that the citric acid cycle stops almost immediately when O2 is removed?

The citric acid cycle stops almost immediately when oxygen is removed because several steps in the cycle require the oxidized forms of NAD+ and FAD. In the absence of oxygen, these electron carriers can be reduced by the reactions of the citric acid cycle but cannot be reoxidized by the electron-transport chain that participates in oxidative phosphorylation.

What is similar between Focal adhesions and Hemidesmosomes?

They both use anchor proteins

peptide bonds

connect amino acids together,covalent bonds

Which of the following is not associated with adhesive junctions? A) desmosomes B) hemidesmosomes C) plaques D) connexin E) cadherins

connexin

Pyruvate must move from the cytosol into the mitochondria, where it is oxidized to form CO2 and acetyl CoA by the pyruvate dehydrogenase complex. How many different enzymes and what total number of polypeptides, respectively, are required to perform this oxidation process in the mitochondrion? (a) 1; 60 (b) 3; 3 (c) 3; 26 (d) 3; 60

d

Which of the following processes do not take place in the mitochondria? (a) citric acid cycle (b) conversion of pyruvate to activated acetyl groups (c) oxidation of fatty acids to acetyl CoA Page 6 of 27 (d) glycogen breakdown

d

Transcription of gene begins at a site on DNA called_____ and ends at a site on DNA known as____. a. The start codon, the stop codon b. A promoter,the stop codon c. the start codon, the terminator d. a promoter, the terminator e.none of the above

d. a promoter, the terminator

62.b. match the following biological process with the model organism that is best suited or most specifically useful for its study: m. musculus (mouse)

development immunology

62.a. match the following biological process with the model organism that is best suited or most specifically useful for its study: a. thaliana (arabidopsis)

development photosynthesis

The small subnit of a ribosome is composed of a.protein b.an rRNA molecule c.many proteins d.many rRNA molecules e.Many proteins and one rRNA molecule.

e.Many proteins and one rRNA molecule.

How does amoeba eat?

engulfs food particle and brings it inwards by pinching off a membrane sac called a food vacuole in the cytoplasm

3.31. A chemical reaction is defined as spontaneous if there is a net loss of free energy during the reaction process. However, spontaneous reactions do not always occur rapidly. Favorable biological reactions require ______________ to selectively speed up reactions and meet the demands of the cell. (a) heat (b) ATP (c) ions (d) enzymes

enzymes

DNA methylation markers passed on to future generations

epigenetic inheritance

True/False: Allosteric interactions always result in enzyme inhibition.

false

14.a. true or false: the nucleus of an animal cell is round, small, and difficult to distinguish using light microscopy

false. the nucleus is one of the largest organelles and the easiest to discern within a typical cell

2.44. what monomer building block is necessary to assemble selectively permeable boundaries around and inside cells?

fatty acids

Which of the following monomer building blocks is necessary to assemble selectively permeable boundaries around and inside cells?

fatty acids

phloem

food distribution and transport

point of cytoplasmic contact between two cells

gap junction

the process by which information from a gene is used in the synthesis of a functional gene product

gene expression

4.b. good or bad to study in the model organism E. coli: DNA replication

good

Mutations in _____ genes caused the development of legs in the place of antennae.

homeotic

What type of interaction is directly responsible for the formation of the secondary structure?

hydrogen bonds between sections of the polypeptide backbone

palisade mesophyll

increase exposure of chloroplasts to light

56. drosophila melanogaster is a BLANK. this type of animal is the most abundant of all animal species, making it an appropriate choice as an experimental model.

insect

Glycolysis is an anaerobic process used to catabolize glucose. What does it mean for this process to be anaerobic? (a) no oxygen is required (b) no oxidation occurs (c) it takes place in the lysosome (d) glucose is broken down by the addition of electrons

a) Glycolysis takes place in the cytosol, and although oxidation of glucose is taking place, no molecular oxygen is used.

By convention, biologists write the sequence of amino acids in a polypeptide in which direction? carboxy- to amino-terminus polar residues to nonpolar residues charged residues to uncharged residues amino- to carboxy-terminus

amino- to carboxy-terminus

What enzyme catalyzes the attachment of amino acids to tRNA molecules? helicase topoisomerase aminoacyl-tRNA synthetase ribosome translation factor

aminoacyl-tRNA synthetase

A molecule with the chemical formula C6H12O6 is probably a _____

monosaccharide

2.45. diagram

phosphoryl

Ribosomes

play a key role in the synthesis of polypepties.

2.53.1.

polar head group

receptors for steroid hormones are located inside the cell instead of on the membrane surface like most other signal receptors. This is NOT a problem for steroids because ________.

steroid hormones are lipid soluble, so they can readily diffuse through the lipid bilayer of the cell membrane

19.c. some prokaryotes have cell walls surrounding the plasma membrane

true

3.19.b. true or false:The cycling of carbon through the biosphere first requires the incorporation of inorganic CO2 into organic molecules.

true

3.49.c. true or false: The cytosol is densely packed with molecules, creating what is more an aqueous gel than a solution.

true

35.b. true or false: the Golgi apparatus is made up of a series of membrane-enclosed compartments through which material destined for secretion must pass.

true

Identify the cytoskeletal structures (black lines) depicted in the epithelial cells shown in Figure Q17-1.

(A) microtubules (B) intermediate filaments (C) actin

Stroma

- Liquid surrounds membranes

Granum

- Stack of membranes

How many molecules of ATP are formed from 1 molecule of glucose?

2

prothallus (gametophytes)

2D sheet of cells

B

31) In eukaryotes, general transcription factors A) are required for the expression of specific protein-encoding genes. B) bind to other proteins or to a sequence element within the promoter called the TATA box. C) inhibit RNA polymerase binding to the promoter and begin transcribing. D) usually lead to a high level of transcription even without additional specific transcription factors. E) bind to sequences just after the start site of transcription.

Put the steps of the process of signal transduction in the order they occur:

31524

2.36. how many bonds can urea form if dissolved in water?

6

C

8) Which of the following is an example of post-transcriptional control of gene expression? A) the addition of methyl groups to cytosine bases of DNA B) the binding of transcription factors to a promoter C) the removal of introns and splicing together of exons D) gene amplification during a stage in development E) the folding of DNA to form heterochromatin

Which statement is NOT true about mutations?

A mutation is a result of asexual reproduction.

Consider this pathway: epinephrine → G-protein-coupled receptor → G-protein → adenylyl cyclase → cAMP. The second messenger in this pathway is ________.

A) cAMP

Chargaff's Rule

A=T and C=G

Which of the following statements in regards to cell adhesion is false?

All cells of multicellular organisms must adhere to survive

During the process of mitosis, chromatids separate during

Anaphase

what is fern medium + 2- thiouracil + excess uracil used for?

By proving that you can reverse the effect of 2- thiouracil by adding excess amounts of normal base

Which of the following would you NOT find in a plant cell?

C) collagen

Three base sequences that indicate a specific amino acid on mRNA are called

Codons

What process is DNA copied

DNA replication

Which of the following statements is true?

Disulfide bonds stabilize but do not change a protein's final conformation.

Which of the following is a modification made to eukaryotic mRNA before it is exported to the cytosol? A) the 5' end is capped B) a poly-A tail is added to the 3' end C) introns are removed D) Econ's are joined together E) all of the above

E) all of the above

The elasticity of the lungs, skin, and blood vessels is provided for by:

Elastin

Which of the following is a statement of the first law of thermodynamics?

Energy cannot be created or destroyed

What is the difference in size between the epithelial cheek cells and the Elodea cells?

Epithelial cells- 9000 um at 400x magnification Elodea cells- 2062.5 um at 400x magnification

A person carefully walking on a slippery sidewalk is most like which kind of bond?

Hydrogen

19) Where would you expect to find tight junctions?

In the epithelium of an animals stomach.

Desmosomes are made of ______________________to ensure long term use.

Intermediate filaments

Which of the following is most likely to be implicated in nephrotic syndrome and in neuromuscular junction/muscle innervation problems in children?

Lamimin

What is the source of energy that requires the electron in PS II

Light.

Which of the following is an example of a membranous organelle?

Lysosomes

Cytochalasins prevent addition of monomers to pre-existing ().

Microfilaments

The largest components of the cytoskeleton are

Microtubules

Synthesis of most ATP takes place in the

Mitochondria

The proper distribution of a cell's genetic material to two daughter cells is accomplished by the process of _______.

Mitosis

Which of the following results from Griffith's experiment is an example of transformation?

Mouse dies after being injected with a mixture of heat-killed S and living R cells.

more solid at room temperature because it is more saturated

Partially hydrogenated safflower oil compared to naturally occuring safflower oil is...

The cell membrane

Protects the cell and acts as a filter

The nucleus controls the synthesis of what molecule?

Proteins

Explain how lactic acid fermentation starts with glucose and yields lactate.

Pyruvate is reduced by NADH to form lactate, thus NADH --> NAD+

Membrane-bound proteins that allow recognition of a cell as "self" are called______ proteins.

Recognition

What does glycolysis literally mean?

Sugar splitting

What problem is 2-thiouracil causing?

The 2-thiouracil inhibits protein synthesis and limits the fern 1D growth

Examination of a sample of cells reveals large numbers of mitochondria compared to most other cells. What is the reason for this?

The cells have very high energy requirements

How do the leading and the lagging strands differ?

The leading strand is synthesized in the same direction as the movement of the replication fork, whereas the lagging strand is synthesized in the opposite direction.

Which of the following statements does not accurately describe the events involved in the propagation of an action potential?

The opening of transmitter-gated K+ channels helps to repolarize the membrane.

What does it mean to say that a signal is transduced?

The physical form of the signal changes from one form to another.

Porin proteins form large, barrel-like channels in the membrane. Which of the following is not true about these channels?

They are made primarily of α helices.

The process of synthesizing an mRNA chain through the use of DNA molecules is called

Transcription

Transfer RNA (tRNA)

Translates the language of mRNA into that of amino acids.

the hydrocarbon chain

What distinguishes a saturated fat from an unsaturated fat?

2.1. chemical reactions in living systems occur in an BLANK environment, within a narrow range of temperatures

aqueous

Fission

asexual reproduction

treadmilling

assembly and disassembly

intermediate filaments

can be composed of the protein vimentin.

2.37.

diagram

microvilli

increase cellular surface area

A unique feature of () is that they are tissue specific.

intermediate

54.c.what would be the best model organism to use for the investigation of the process of: immunology

mouse

2.5. what subatomic particles can vary between isotopes of the same element, without changing the observed chemical properties?

neutrons

2.43.a. inorganic or organice: glucose

o

2.53.2.

phosphate

N-terminus

refers to the presence of a nitrogen atom(N)

2.56.b. there are twenty possible BLANK that differ in structure and are generally referred to as "R."

side chains

2.25. what does not influence the length of a covalent bond?

the tendency of atoms to fill the outer electron shells

what describes proto-oncogenes?

they can code for proteins associated with cell growth

13. b. has the ability to resolve cellular components as small as 2nm

transmission electron

Amoeba

unicellular, eukaryotic organisms

Intermediate filaments

were first discovered in muscle cells.

Calc of percent change % increase in length

new length - old length ---------------------------- old length x 100

2.23.on the basis of the information in the chart and what you know about atomic surfaces, which elements form stable but reactive diatomic gases?

nitrogen, oxygen

Where does replication begin?

origin of replication

The "9+2" pattern of the axoneme is 9 () microtubules and 2 additional microtubules in center called ().

outer doublets; central pair

3.18. When elemental sodium is added to water, the sodium atoms ionize spontaneously. Uncharged Na becomes Na+. This means that the Na atoms have been _____________. (a) protonated. (b) oxidized. (c) hydrogenated. (d) reduced.

oxidized

Which of the following activities is not associated with cadherins? A) segregation of cells to specific tissues B) paracellular transport C) embryonic development D) establishment of synaptic connections E) loss of activity associated with cancer

paracellular transport

Which of the following is not a function of the cytoskeleton?

passive transport

giant protein complexes that bind to protein molecules and degrade them

proteasomes

Which subatomic particles contribute to the atomic number for any given element?

protons

Which of the following steps in cell crawling is accompanied by Arp2/3-dependent actin polymerization at the leading edge?

protrusion

channel protein

provide a doorway through the membrane

spongy mesophyll

provides surface for gas exchange

during development, individual cells of the same organism begin to produce different proteins because

specific genes are activated in the cells

Capping proteins such as CapZ () microtubules.

stabilize microtubules.

Which of the following statements about tRNA is FALSE?

tRNAs can be spliced by splicesomes.

Which of the following phenomena or processes does not contribute to the high fidelity of DNA replication?

telomerase

Cells use membranes to help maintain set ranges of ion concentrations inside and outside the cell. Which of the following ions is the most abundant outside a typical mammalian cell? (a) Na+ (b) K+ (c) Ca2+ (d) Cl-

(a)

RNA polymerase

the enzyme that synthesizes strands of RNA.

When an amino acid is specified by more than one codon, what is usually shared by the set of codons that specify this amino acid? their entire base sequence the third base the second base the first base the first and second bases

the first and second bases

Which of the following membrane lipids does not contain a fatty acid tail? (a) phosphatidylcholine (b) a glycolipid (c) phosphatidylserine (d) cholesterol

(d)

How many carbon atoms are fed into the citric acid cycle as a result of the oxidation of one molecule of pyruvate? A) 2 B) 4 C) 6 D) 8 E) 10

A

2.65.b. true or false: "nonpolar interactions" is simply another way of saying "van der waals attractions"

true

What is the %T in wheat DNA?

Approximately 28%

Compare the structure the cell wall of a cell from the tomato skin to a cell from the pulp

Both have chromoplasts, but the skin has a much more rigid cell wall while the pulp was much more fluid. Chromoplasts are found in plant cells. The cell walls are different because the functions of the two are different. The skin needs to protect the tomato, so it makes sense why it has a more rigid cell wall

3.57.b. Explain why temperature has this effect.

By increasing thermal motion, increasing the temperature increases the number of collisions of sufficient energy to overcome the activation energy. An increase in temperature will thus increase the reaction rate initially. However, enzymes are proteins and are held together by noncovalent interactions, so at very high temperatures the enzyme will begin to denature and the reaction rate will fall.

the tryptophan operon is a repressible organ that is

turned off whenever tryptophan is present

2.53.4.

unsaturated fatty acid

Carrier molecules are located within the

Cell membrane

In the body, ATP is made during the process of..?

Cellular respiration Cellular respiration takes place in both plants and animals

The most common form of Collagen is:

Collagen I

What is the difference between covalent bonds and ionic bonds?

Covalent bonds involve the sharing of electrons between atoms; ionic bonds involve the electrical attraction between charged atoms.

The folds of the internal membrane within mitochondria are called

Cristae

Who conducted the X-ray diffraction studies that were key to the discovery of the structure of DNA?

Franklin

15) Plasmodesmata in plant cells are most similar in function to which of the following structures in animal cells?

Gap Junctions

A(n) ______ is an organized unit of DNA sequences that enables a segment of DNA to be transcribed into RNA and ultimately results in the formation of a functional product.

Gene

Thee are associated with the "pinching in" of the cell membrane during cytokinesis.

Microfilaments

The citric acid cycle accounts for 2 additional reduced _______ molecules and 2 carbon dioxide molecules during the second cycle.

NAD+

Which of the following statements is true?

Nonpolar amino acids tend to be found in the interior of proteins.

A person is genotypically blood type A. However, this individual also does not produce the lipid to which N-acetylgalactosamine is added, so the lipid is therefore not expressed on the surface of erythrocytes. This situation is known as the Bombay phenotype. When tested, this person will appear to be blood type A) A. B) B. C) AB. D) O.

O

Fermentation allows for the production of ATP without using either ____ or any _____.

Oxygen, ETC.

What is the role of the ETC in forming the H+ gradient across the inner mitochondrial membrane?

The ETC helps create/maintain the H+ gradient by pumping H+s and the only way that they can diffuse across their gradient is through ATP synthase.

31.

a

Cilia and flagella are comprised of () connected to a basal body.

axoneme

Polarized cells are characterized by A) random orientation within a tissue. B) a polar flagellum. C) random orientation within an organ. D) distinct apical and basolateral sides.

distinct apical and basolateral sides.

Van der Waals interaction may result when ____

electrons are not symmetrically distributed in a molecule

ribosomes

in prokaryotes and eukaryotes

nocodazole

inhibits microtubule assembly

Heterotrophic

invest particles by a process called phagocytosis, amoeba

A triplet of nitrogenous bases on a tRNA molecule that interacts with the complementary codon on a mRNA strand is called a(n) ______.

Anticodon

14-18 Stage 1 of oxidative phosphorylation requires the movement of electrons along the electron-transport chain coupled to the pumping of protons into the intermembrane space. What is the final result of these electron transfers? (a)OH-is oxidized to O2 (b)pyruvate is oxidized to CO2 (c)O2is reduced to H2O (d)H-is converted to H

C O2 is reduced to H2O

10) All of the following proteins may be found in the extracellular matrix of animal cells EXCEPT ________.

C) actin

14-6Modern eukaryotes depend on mitochondria to generate most of the cell's ATP. How many molecules of ATP can a single molecule of glucose generate? (a)30 (b)2 (c)20 (d)36

Choice (a) is correct. Glycolysis of a single glucose molecule generates 2 ATP molecules. Oxidative phosphorylation in the mitochondria generates an additional 28 ATP molecules, making a total of 30 ATP molecules for each glucose molecule.

In phase two, the reduction stage, the reducing power of NADPH will donate electrons to the low energy acid 1,3-bisphophoglycerate to form the 3 carbon sugar ___

G3P

Oxygen stabilizes the electrons by combining with 2 hydrogen ions to form what compound?

H2O

Which of the following statements about RNA polymerase in bacteria is INCORRECT?

It catalyzes the addition of nucleotides to the 5′end of a growing RNA strand.

What does the light harvesting complex do?

It functions in gathering light to raise electrons to an exited state so that they can travel into the reaction center complex.

In photosystem II, the excited electron is eventually used by NADP+ reductase to join NADP+ and an H+ to form _____

NADPH

Cells make use of H+ electrochemical gradients in many ways. Which of the following proton transporters is used to regulate pH in animal cells?

Na+-H+ exchanger

Which of the following enzymes are responsible for dephosphorylating proteins?

None of these is correct.

The control center for gene expression is the

Nucleus

Transcription occurs in the

Nucleus

Which of the following indirectly determines the bonds that an atom can form?

Protons

In step 2 of the citric acid cycle, the enzyme aconitase generates isocitrate from citrate. Which of the following statements about this reaction is true?

The reaction sequence first generates one molecule of water and then consumes one molecule of water.

Which of the following statements about the mRNA start codon is INCORRECT? The start codon is only a few nucleotides from the ribosomal binding site. The start codon is usually GGA. The start codon is usually AUG. The start codon specifies the amino acid, methionine. The start codon defines the reading frame.

The start codon is usually GGA.

lagging strand

The strand that is synthesized in fragments using individual sections called Okazaki fragments

Codons for Pheylalanine (Phe)

UUU UUC

Microtubules and microfilaments differ with respect to each of the following except

a state of dynamic instability.

The structure depicted the accompanying figure shows the _____. α-helix secondary structure of a polypeptide 1-4 linkage of the β-glucose monomers of cellulose 1-4 linkage of the α-glucose monomers of starch double-helical structure of a DNA molecule β-pleated sheet secondary structure of a polypeptide

a-helix secondary structure of a polypeptide

3.27.d. A catalyst decreases the BLANK energy of reaction.

activation

Elongation of the microtubule occurs by the

addition of tubulin subunits to each end.

Cytoskeletal elements are found in the

all of the above

Microfilaments function in

all of the above

3.b. Despite the diversity, cells resemble each other to an astonishing degree in their chemistry. For example, the same 20 BLANK are used to make proteins.

amino acids

Side chains of amino acids ____

are all nonpolar

auricles

are located laterally from eyespot, they are chemosensors that make it easy to depict the head (ant) from tail (post)

Poly A tail

at the 3' end,most mature eukaryotic mRNAs have a string of adenine nucleotides. the poly A tail is not encoded in the gene sequence.A long poly A tail aids in the export of mRNAS from the nucleus.

37. which of the following choices best describes the role of the lysosome? a. transport of material to the golgi b. clean-up, recycling, and disposal of macromolecules d. the storage of excess macromolecules

b

49.c. good or bad to study in the model organism E. coli: how the actin cytoskeleton contributes to cell shape

bad

Synthesis of which new strand(s) would be affected by mutations in the enzyme DNA polymerase III?

both leading and lagging strands

primase

can start an RNA chain from scratch and adds RNA nucleotides one at a time using the parental DNA as a template

About 25 of the 92 natural elements are known to be essential to life. Which four of these 25 elements make up approximately 96% of living matter?

carbon, hydrogen, nitrogen, oxygen

Which two functional groups are always found in amino acids? carbonyl and amino groups ketone and methyl groups amino and sulfhydryl groups carboxyl and amino groups hydroxyl and carboxyl groups

carboxyl and amino groups

43. despite the differences between eukaryotic and prokaryotic cells, prokaryotes have proteins that are distantly related to eukaryotic actin filaments and microtubules. what is likely to be the most ancient function of the cytoskeleton?

cell division

62.e.match the following biological process with the model organism that is best suited or most specifically useful for its study: s. cerevisae

cell division

protein subunits associated with gap junctions

connexin

hollow cylinders associated with gap junctions

connexons

In what region does translation occur?

cytosol

24.d. If we were to strip away the plasma membrane from a eukaryotic cell and remove all of its membrane-enclosed organelles, we would be left with the BLANK, which contains many long, fine filaments of protein that are responsible for cell shape and structure and thereby form the cell's BLANK.

cytosol cytoskeleton

Glycolysis occurs in the _____

cytosol.

Because more than one codon can specify the same amino acid, the genetic code is said to be

degenerate.

button-like points of strong adhesion between two cells

desmosome

The activation of receptor tyrosine kinases is characterized by ________.

dimerization and phosphorylation

Helicases

enzymes that untwist the double helix at the replication forks

3.8.c. true or false: CO2 gas is fixed in a series of reactions that are light-dependent.

false, the fixation of carbon from co2 occurs independently of light

2.52.c. true or falseL the presence of double bonds in the hydrocarbon tail of a fatty acid does not greatly influence its structure

false. the presence of a double bond in the hydrocarbon chain of a fatty acid causes a kink in the chain, decreasing its flexibility and packing with neighboring hydrocarbon chains

The citric acid cycle is outlined in Figure Q13-44. Some of these reactions produce small molecules that are used in the electron-transport chain or as energy for other reactions. Select from the list below to fill in the empty boxes. Keep in mind that some choices may be used more than once and others not used at all. A. ATP B. ADP C. GTP D. GDP E. NAD+ F. NADH G. FADH H. FADH2

figure

A peptide bond _____. forms between the central carbon and the amino R-group of a single amino acid does not play a role in maintaining the tertiary structure of proteins forms between the functional R-groups of different amino acids forms the primary structure of proteins

forms the primary structure of proteins

2.49. which of the following are examples of isomers? a. glucose and galactose b. alanine and glycine c. adenine and guanine d. glycogen and cellulose

glucose and galactose

Coding strand

has the same sequence of bases as the resulting mRNA,except for thymine in the DNA is substituted for uracil in the RNA.It carries information that codes the polypeptide.

The organ where you would most likely find actin microfilaments continuous with adherens junctions is the A) liver. B) stomach. C) kidney. D) brain. E) heart.

heart

2.20. on the basis of the information in the chart and what you know about atomic surfaces, which elements are chemically inert?

helium, neon

An adhesive junction associated with the connection between an epithelial cell and the basal lamella is the A) hemidesmosome. B) desmosome. C) gap junction. D) plasmodesmata. E) focal contact.

hemidesmosome

2.43.d. inorganic or organice: water

i

cholesterol

important component of cell membranes; precursor from which other steroids are synthesized; common example: sex hormones

cytoskeleton

in prokaryotes and eukaryotes

plasma membrane

in prokaryotes and eukaryotes

36) A G-protein receptor with GTP bound to it ________.

is in its active state

Aminoacyl-tRNA synthetase is an enzyme whose function is to _____.

link a tRNA to its corresponding amino acid

euchromatin

loosely packed chromatin

According to the central dogma, what molecule should go in the blank? DNA → _____ → Proteins mRNA rRNA mtDNA tRNA

mRNA

The type of RNA that holds a copy of the genetic information is the

mRNA

2.29.

model

the basic mechanism for gene expression control in bacteria

operon model

36.d. what cell type has chloroplasts?

plant, bacterial

36.e. what cell type has cell walls?

plant, bacterial

collection of fibrous intracellular attachment proteins

plaque

25. The BLANK is made up of two concentric membranes and is continuous with the membrane of the endoplasmic reticulum

plasma membrane

When polymerization of a protein is complete, but a protein is still completely linear, what is the highest level of structure in the protein? primary secondary tertiary quaternary

primary

You disrupt all hydrogen bonds in a protein. What level of structure will be preserved?

primary structure

2.59.a. peptide: Pro-Val-Thr-Gly-Lys-Cys-Glu write out the sequence of amino acids.

proline valine threonine glycine lysine cysteine glutamic acid

Tyrosinase

responsible for a biochemical pathway that begins with tyrosine and leads to the formation of melanin

In general, a signal transmitted via phosphorylation of a series of proteins ________.

results in a conformational change to each protein

After fixing three carbon dioxides using the enzyme ____, the calvin cycle forms G3Ps with a total of __carbons. At this point the net gain of carbons is ____ or one net g3p molecule

rubisco, 18,3

Figure Q13-19 represents a cell lining the gut. Draw numbered, labeled lines to indicate exactly where inside a cell the following processes take place. Figure Q13-19 1. glycolysis 2. citric acid cycle 3. conversion of pyruvate to activated acetyl groups 4. oxidation of fatty acids to acetyl CoA 5. glycogen breakdown 6. release of fatty acids from triacylglycerols 7. oxidative phosphorylation

see figure

3.27.c. Enzymes act more BLANK than other catalysts.

selectively

2.16. a covalent bond between two atoms is formed as a result of the BLANK

sharing of electrons

Describe the function of the fine adjustment knobs

sharpens the focus of what is being observed under the microscope

Photosynthesis enables plants to capture the energy from sunlight. In this essential process, plants incorporate the carbon from CO2 into high-energy __________ molecules, which the plant cell mitochondria use to produce ATP.

sugar

elongation

the covalent bonding of amio acids to each other,one at a time,to produce a polypeptide.

3.52.The graph in Figure Q3-52 illustrates the relationship between reaction rates and substrate concentration for an enzyme-catalyzed reaction. What does the Km value indicate with respect to enzyme-substrate interactions? (a) the maximum rate of catalysis (b) the number of enzyme active sites (c) the enzyme-substrate binding affinity (d) the equilibrium rate of catalysis

the enzyme-substrate binding affinity

Codons

the groups of three nucleotide bases read

Which of the following is directly related to a single amino acid? the amino acetyl tRNA synthase the three-base sequence of mRNA the base sequence of the tRNA the complementarity of DNA and RNA

the three-base sequence of mRNA

Ions diffuse across membranes through specific ion channels down ____

their electrochemical gradients

Which protein influences the ability of RNA polymerase to transcribe genes?

transcription factors

secondary active transport (cotransport)

uses the concentration gradient created by an ATP-powered pump to power the transport of a molecule against its concentration gradient

then is followed by a light - yellow (visible as 2 bands)

xanthophyll

Initiation

Is a regonition step

They resist tensile forces and contain keratin to form the protective barrier of the skin.

Intermediate filaments

When yeast cells of the "a" mating type are exposed to a solution of "α" pheromones ________.

G-proteins will be activated in the cell-signaling pathway and the yeast cells will change shape

15-34Figure Q15-34shows the organization of a protein that normally resides in the plasma membrane. The boxes labeled 1 and 2 represent membrane-spanning sequences and the arrow represents a site of action of signal peptidase. Given this diagram, which of the following statements must be true? Figure Q15-34 (a)The N-terminus of this protein is cytoplasmic. (b)The C-terminus of this protein is cytoplasmic. (c)The mature version of this protein will span the membrane twice, with both the N-and C-terminusin the cytoplasm. (d)None of the above.

(b) The mature version of this protein will span the membrane once, with membrane- spanning segment 2 in the membrane and the C-terminus facing the cytoplasm.

15-39Molecules to be packaged into vesicles for transport are selected by ________. (a)clathrin. (b)adaptins. (c)dynamin. (d)SNAREs.

(b) adaptins.

Microtubules have an inner diameter of () and an outer diameter of ().

15 nm; 25 nm

D

4) This protein is produced by a regulatory gene: A) operon B) inducer C) promoter D) repressor E) corepressor

How many FADHs are formed from 1 molecule of glucose?

6

naturally occurring elements

92, 11 are fond in organisms- O,H, CO, N, P

Glycolipids are found on the surface of healthy cells, and contribute to the cell's defense against chemical damage and infectious agents. A. In which organelle are sugar groups added to membrane lipids? B. By what mechanism are glycolipids transported to the plasma membrane and presented to the extracellular environment? Draw a diagram to support your answer to part B.

A. The Golgi apparatus. B. Membranes that contain newly synthesized glycolipids bud from the Golgi apparatus to form vesicles. These vesicles then fuse with the plasma membrane. The glycolipids that were facing the lumen of the Golgi will now face the extracellular environment (Figure A11-26).

Indicate whether the following statements are true or false. If a statement is false, explain why it is false. A. Although cholesterol is a hydrophobic molecule, it has a hydrophilic head group like all other membrane lipids. B. Phosphatidylserine is the most abundant type of phospholipid found in cell membranes. C. Glycolipids lack the glycerol component found in phospholipids. D. The highly ordered structure of the lipid bilayer makes its generation and maintenance energetically unfavorable.

A. True. B. False. Phosphatidylcholine is the most abundant phospholipid found in cell membranes. C. True. D. False. The formation of a lipid bilayer is energetically favorable.

Start codon

AUG

impaired in cell-cell recognition

An animal cell lacking oligosaccharides on the external surface of its plasma membrane would likely be...

telomerase

An enzyme that catalyzes the lengthening of telomeres in eukaryotic germ cells.

Explain your three levels of exercise results in terms of aerobic respiration and anaerobic lactic acid fermentation.

As the rate of aerobic respiration increases the amount of CO2 increases. A drop in CO2 indicates an inadequate supply of O2 and the switch by some muscle cells to anaerobic respiration.

14-20 Which of the following components of the electron-transport chain does no tact as a proton pump? (a)NADH dehydrogenase (b)cytochrome c (c)cytochrome creductase (d)cytochrome coxidase

B Cytochrome C

When yeast cells of opposite mating types are in close proximity, cell changes occur inside those cells that result in the polymerization of actin filaments, allowing the yeast cells to mate. Those changes in cell behavior are initially due to signaling molecules (pheromones) released by the yeast cells. Pheromones are ________.

B) a type of hormone that binds cell-surface receptors causing cytoplasmic actin proteins to form filaments

In which group of organisms would you expect to see decoupled transcription and translation? A) prokaryotes B) eukaryotes C) both D) neither

B) eukaryotes

Lipid-soluble signaling molecules, such as testosterone, cross the membranes of all cells but affect only target cells because ________.

B) intracellular receptors are present only in target cells

Can signaling via a steroid hormone receptor lead to amplification of the original signal? If so, how?

Because the interactions of the signal molecule with its receptor and of the activated receptor with its gene are both one-to-one, there is no amplification in this part of the signaling pathway. The signal can, however, be amplified when the target genes are transcribed, because each activated gene produces multiple copies of mRNA, each of which is used to make multiple copies of the protein that the gene encodes.

Which of the following always exist, in part, inside of the cellular membrane?

CAMs

What is the difference between fermentation and cellular respiration?

Cellular respiration requires oxygen, fermentation can be undergone without oxygen

15-16A large protein that passes through the nuclear pore must have an appropriate _________. (a)sorting sequence, which typically contains the positively charged amino acids lysine and arginine.(b)sorting sequence, which typically contains the hydrophobic amino acids leucine and isoleucine.(c)sequence to interact with the nuclear fibrils. (d) Ran-interacting protein domain.

Choice (a) is correct. The nuclear import receptor interacts with the fibrils of the nuclear pore [choice (c)] and Ran [choice (d)].

Downhill route most electrons follow in cellular respiration: Glucose --> _____ --> ______ --> Oxygen

NADH, ETC.

15-2Which of the following statements about membrane-enclosed organelles is true? (a)In a typical cell, the area of the endoplasmic reticulum membrane far exceeds the area of plasma membrane. (b)The nucleus is the only organelle that is surrounded by a double membrane. (c)Other than the nucleus, most organelles are small and thus, in a typical cell, only about 10% of a cell's volume is occupied by membrane-enclosed organelles; the other 90% of the cell volume is the cytosol. (d)The nucleus is the only organelle that contains DNA.

Choice (a) is true; the area of the endoplasmic reticulum membrane is 20-30 times that of the plasma membrane in a typical cell. Chloroplasts and mitochondria are also surrounded by a double membrane [choice (b)]. The cytosol is about half the volume of a typical eukaryotic cell, with membrane-enclosed organelles making up the other half of the volume [choice (c)]. Chloroplasts and mitochondria also carry their own genome, whereas the nucleus carries the genome of the organism [choice (d)].

In cyclic electron flow no water is split, there is no production of ______, and there is no release of ________

NADPH; O2

Which of the following stages in the breakdown of the piece of toast you had for breakfast generates the most ATP? (a) the digestion of starch to glucose (b) glycolysis (c) the citric acid cycle (d) oxidative phosphorylation

Choice (d) is the correct answer. Oxidative phosphorylation produces about 28 ATP molecules. Choice (a) produces no ATP; choice (b) nets 2 ATP; choice (c) produces 1 GTP.

Which of the statements below about intermediate filaments is false? (a) They can stay intact in cells treated with concentrated salt solutions. (b) They can be found in the cytoplasm and the nucleus. (c) They can be anchored to the plasma membrane at a cell-cell junction. (d) Each filament is about 10 μm in diameter.

Choice (d) is untrue. Intermediate filaments are about 10 nm (not μm) in diameter. All the other statements are true.

Cells do not catabolize carbon dioxide because A) its double bonds are too stable to be broken. B) CO2 has fewer bonding electrons than other organic compounds. C) CO2 is already completely reduced. D) CO2 is already completely oxidized. E) the molecule has too few atoms.

D

In animal cells, the extracellular matrix plays important roles in all of the following processes except A) cell division. B) motility. C) DNA replication. D) differentiation. E) adhesion.

DNA replication

what is the direction of flow of genetic infomation?

DNA to RNA to protein

Out of the 3 stages, which has the most energy?

Electron Transport Chain. (ETC)

Most of the proteins located in the plasma membrane are produced in the

Endoplasmic reticulum

Coenzymes

Enzyme helpers, but they are not proteins

At puberty, an adolescent female body changes in both structure and function of several organ systems, primarily under the influence of changing concentrations of estrogens and other steroid hormones. How can one hormone, such as estrogen, mediate so many effects?

Estrogen binds to specific receptors inside many kinds of cells, each with different responses.

What are the colors of the visible spectrum?

ROY G BIV

head: hydrophilic tail: hydrophobic

What is the main difference between the tail and head of a phospholipid?

nonpolar molecules

What kind of molecules can easily pass through the membrane?

2.50.c. what type of molecule is figure q2-50

a fatty acid

Transcription factors

are proteins influenced by the ability of RNA polymerase to transcribe genes

The ________ serves as structural support, in tissue organization, and as a permeability barrier.

basal lamina

gelsolin

breaking and capping actin

3.28.Figure Q3-28 is an energy diagram for the reaction XY. Which equation below provides the correct calculation for the amount of free-energy change when X is converted to Y? (a) a + b - c (b) a - b (c) a - c (d) c - a *diagram

c-a

2.59.c.peptide: Pro-Val-Thr-Gly-Lys-Cys-Glu according to the conventional way of writing the sequence of a peptide or a protein, identify the c-terminal and n-terminal acids

c-terminal is glutamic acid n-terminal is proline

4 yellow greenish line

chlorophyll b

21.b. true or false: mitochondria are thought to have evolved from anaerobic bacteria

false. mitochondria use oxygen to generate energy and are thought to have evolved from aerobic bacteria

Ribosomal RNA(rRNA)

forms part of ribosomes,which provide the site where translation occurs.

2.53.5.

hydrophilic region

2.53.6.

hydrophobic region

Lipids that form membranes have what kind of structure?

polar heads and nonpolar tails; the polar heads interact with water

Fill in the spaces in the table below. For steps 1, 4, 5, and 8, name the correct substrates, enzymes, or products. For all the other steps, name the enzyme and draw the missing structure.

see figure

protonema

single chain of cells 1D

Where do light reactions occur?

thylakoid

35.a. true or false: membrane components in the cell are made in the endoplasmic reticulum.

true

60.b.true or false: the variation in genome size among protozoans is larger than that observed across all species of mammals, birds, and reptiles

true

filamin

two identical polypeptides joined end-to-end

2.69.b. choose the class of amino acids most important for the interaction: forming hydrogen bonds to aid solubility in water

uncharged polar

2.69.c. choose the class of amino acids most important for the interaction: binding to another water-soluble protein

uncharged polar, basic, and acidic

A group of membrane proteins can be extracted from membranes only by using detergents. All the proteins in this group have a similar amino acid sequence at their C-terminus: -KKKKKXXC (where K stands for lysine, X stands for any amino acid, and C stands for cysteine). This sequence is essential for their attachment to the membrane. What is the most likely way in which the C-terminal sequence attaches these proteins to the membrane? (a) The cysteine residue is covalently attached to a membrane lipid. (b) The peptide spans the membrane as an α helix. (c) The peptide spans the membrane as part of a β sheet. (d) The positively charged lysine residues interact with an acidic integral membrane protein.

(a)

Consider the apical location of a particular protein expressed in epithelial cells, illustrated in Figure Q11-50A. When a molecule that chelates calcium is added to the cell culture medium, you observe a redistribution of that protein around the entire cell, shown in Figure Q11-50B. Which is most likely to be true about the role of calcium in maintaining an apical distribution of protein A? (a) calcium is required to maintain the structural integrity of the junctional complex (b) calcium is required for the binding of the junctional proteins to the cell cortex (c) calcium is a structural component of protein A (d) calcium inhibits intracellular transport of protein A

(a)

Membrane lipids are capable of many different types of movement. Which of these does not occur spontaneously in biological membranes? (a) switching between lipid layers (b) lateral movement (c) rotation (d) flexing of hydrocarbon chains

(a)

Plasma membranes are extremely thin and fragile, requiring an extensive support network of fibrous proteins. This network is called the ____________. (a) cortex. (b) attachment complex. (c) cytoskeleton. (d) spectrin.

(a)

Which type of lipids are the most abundant in the plasma membrane? (a) phospholipids (b) glycolipids (c) sterols (d) triacylglycerides

(a)

Which of the following cells rely exclusively on glycolysis to supply them with ATP? (a) anaerobically growing yeast (b) aerobic bacteria (c) skeletal muscle cells (d) plant cells

(a) All the other cells can perform oxidative phosphorylation to generate additional ATP

The stage in a cell's life cycle in which the chromosomes are at opposite poles of the cell and the cellular cleavage is nearly complete is called

Telophase

The Alpha helices and Beta sheets are examples of protein __________________ structure. A protein such as hemoglobin, which is composed of more than one protein __________________, has __________________ structure. A protein's amino acid sequence is known as its __________________ structure. A protein __________________ is the modular unit from which many larger single-chain proteins are constructed. The three-dimensional conformation of a protein is its __________________ structure. allosteric ligand secondary domain primary subunit helix quaternary tertiary

The alpha helices and beta sheets are examples of protein *secondary* structure. A protein such as hemoglobin, composed of more than one protein *subunit*, has *quaternary* structure. A protein's amino acid sequence is known which is as its *primary* structure. A protein *domain* is the modular unit from which many larger single-chain proteins are constructed. The three- dimensional conformation of a protein is its *tertiary* structure.

Steps 7 and 10 of glycolysis result in substrate-level phosphorylation. Which of the following best describes this process?

The energy derived from substrate oxidation is coupled to the conversion of ADP to ATP.

15-63 If a lysosome breaks, what protects the rest of the cell from lysosomal enzymes?

The lysosomal enzymes are all acid hydrolases, which have optimal activity at the low pH (about 5.0) found in the interior of lysosomes. If a lysosome were to break, the acid hydrolases would find themselves at pH 7.2, the pH of the cytosol, and would therefore do little damage to cellular constituents.

Describe the relationship between the number of drops of NaOH required to neutralize the water and the amount of CO2 produced.

The more CO2, the more drops of NaOH that are required to return the solution to the original pH

What were the results of the pure water growth in the tomato plants?

The plant changed a small amount in weight and length, overall was malnourished with yellowish leaves and minimal roots

What is the function of the poly A tail?

The poly A tail increases mRNA stability in eukaryotes.

Use your observations to support or falsify your prediction. The geranium leaf

The prediction was supported. The geranium leaf exposed to light had the presence of starch, which was indicated by the dark-blue color when I2KI was added. The leaf that was kept in dark did not have starch present, which was indicated by the yellow color when I2KI was added.

Which of the following statements about the mRNA start codon is INCORRECT?

The start codon is usually GGA.

15-59 In a cell capable of regulated secretion, what are the three main classes of proteins that must be separated before they leave the transGolgi network?

The three main classes of protein that must be sorted before they leave the trans Golgi network in a cell capable of regulated secretion are (1) those destined for lysosomes, (2) those destined for secretory vesicles, and (3) those destined for immediate delivery to the cell surface.

Cytochalasin is a drug that caps actin filament plus ends, thus preventing actin polymerization. Phalloidin is a drug that binds to and stabilizes actin filaments, preventing actin depolymerization. Even though these drugs have opposite effects on actin polymerization, the addition of either of these drugs instantaneously freezes the cell movements that depends on actin filaments. Explain why drugs that have opposite effects on actin filaments can have a similar effect on cell movements.

These drugs both stop cell movements because actin polymerization and depolymerization are both required for this process. As cells move forward, the growth of actin filaments near the plasma membrane helps push out the membrane. As this occurs, continuous depolymerization of actin filaments occurs at the actin filaments away from the plasma membrane.

The process of forming a polypeptide from mRNA instructions is called

Translation

Water molecules and small ions enter a cell through

Transport proteins

Cells can adhere to each other or to the ECM.

True

DNA is used as a template to make mRNA, which contains the information to make a polypeptide.

True

The termination of translation occurs when a release factor recognizes the stop codon.

True

They form liposomes.

What happens when phospholipids are placed into water?

unsaturated fatty acids and short hydrocarbon chains

What increases membrane permeability of a substance?

fatty acids of different lengths and saturation

What influences membrane permeability?

cholesterol

What is a very common steroid?

transport proteins

What is used in facilitated diffusion to help diffuse polar molecules and ions passively across the membrane?

Think of squirrels entering and leaving a room as an analogy for a chemical reaction. When is the room at equilibrium?

When equal numbers of squirrels are entering and leaving the room

DNA

Which biological polymer has the least variety in structure?

Increased level of saturation of fatty acids

Which of the following would decrease membrane fluidity?

Rank the following cytoskeletal filaments from smallest to largest in diameter (1 = smallest in diameter, 4 = largest) ______ intermediate filaments ______ microtubules ______ actin filament ______ myofibril

__2___ intermediate filaments (10 nm diameter) __3___ microtubules (25 nm) __1___ actin filament (5-9 nm) __4___ myofibril (1-2 μm)

Given the generic signaling pathway in Figure Q16-12, write the number corresponding to the item on the line next to the descriptor below. Figure Q16-12 _________ receptor protein _________ effector proteins _________ intracellular signaling proteins _________ ligand

____2____ receptor protein ____4____ effector proteins ____3____ intracellular signaling proteins ____1___ ligand

3.64.The study of enzymes also includes an examination of how the activity is regulated. Molecules that can act as competitive inhibitors for a specific reaction are often similar in shape and size to the enzyme's substrate. Which variable(s) used to describe enzyme activity will remain the same in the presence and absence of a competitive inhibitor? (a) Vmax (b) V (c) Vmax and Km (d) Km

a

C-terminus

a carboxyl group is always found at the end of the polypeptide.

Protein folding can be studied using a solution of purified protein and a denaturant (urea), a solute that interferes with noncovalent interactions. Which of the following is observed after the denaturant is removed from the protein solution?

a) The polypeptide returns to its original conformation.

Proteins help cells with which task(s)? catalysis transport signaling all of the above

all of the above

The Golgi apparatus has a polarity, or sidedness, to its structure and function. Which of the following statements correctly describes this polarity?

all of the above

Which of the following cell junctions are associated with the proper alignment of intestinal epithelial cells? A) gap junctions B) adhesive junctions C) tight junctions D) desmosomes E) all of the above

all of the above

What is a coenzyme?

an organic nonprotein helper for catalytic reactions.

36.h. what cell type has Golgi apparatus?

animal, plant

36.a. what cell type has DNA?

animal, plant, bacterial

36.c. what cell type has plasma membrane?

animal, plant, bacterial

Dyneins are

associated with cilia and flagella.

49. a. good or bad to study in the model organism E. coli: formation of endoplasmic reticulum

bad

49.e. good or bad to study in the model organism E. coli: how mitochondria get distributed to cells during cell division

bad

Pyruvate is an important metabolic intermediate that can be converted into several other compounds, depending on which enzyme is catalyzing the reaction. Which of the following cannot be produced from pyruvate in a single enzyme-catalyzed reaction? (a) lactate (b) oxaloacetate (c) citrate (d) alanine

c

Which of the following is incorrectly identified in this illustration of a tRNA? a)5′ and 3′ ends b)hydrogen bond c)amino acid attachment site d)anticodon loop

c

7. Which statement is NOT true about mutations? a. A mutation is a change in the dna that can generate offspring less fit for survival than their parents b. A mutation can be a result of imperfect dna duplication. c. A mutation is a result of sexual reproduction d. A mutation is a change in the dna that can generate offspring that are less fit for survival as their parents are.

c. A mutation is a result of sexual reproduction

tumor suppressor genes

can encode proteins that promote DNA repair or cell-cell adhesion

45. Microtubules BLANK and are required to pull duplicated chromosomes to opposite poles of dividing cells

can rapidly organize

Of the following microtubule-interacting proteins, which might you potentially investigate as a mechanism for disrupting cell division in actively replicating tumor cells?

catastrophins

53. brewer's yeast, apart from being an irreplaceable asset in the brewery and in the bakery, is an experimental organism used to study eukaryotic cells. however, it does have some limitations. which of the processes below cannot be studied in yeast? a. dna replication b. cell motility c. exocytosis d. cell division

cell motility

Epithelial cells

cells on your skin that are constantly sloughing away

26. the nucleus, an organelle found in eukaryotic cell, confines the BLANK, keeping them separated from other components of the cell.

chromosomes

"Untested" B lymphocytes go through a process of testing before they will be released into the blood as memory or plasma cells. Only those with low affinity for "self" major histocompatibility complexes or peptides make it to the periphery. The rest undergo programmed cell death. This is called _____________.

clonal deletion

3.15.Oxidation is a favorable process in an aerobic environment, which is the reason cells are able to derive energy from the oxidation of macromolecules. Once carbon has been oxidized to _______________, its most stable form, it can only cycle back into the organic portion of the carbon cycle through __________________. (a) CO2, photosynthesis. (b) CH3, combustion. (c) CO2, respiration. (d) CO, reduction.

co2, photosynthesis

snRNPS

containsmall nuclear RNA and a set of proteins

nonmotor MAPs

control microtubule organization

ions pumps

crucial for maintaining electrochemical gradients and membrane potential for nerve impulses

Translation occurs in the _____. cytoplasm lysosome nucleus Golgi apparatus nucleoplasm

cytoplasm

In step 2 of the citric acid cycle, the enzyme aconitase generates isocitrate from citrate. Which of the following statements about this reaction is true? (a) There is a substantial free-energy difference between the reactants and products of this reaction. (b) The unbonded electrons from hydroxide ions provide energy for this reaction. (c) The aconitase enzyme functions as a mutase in this reaction. (d) The reaction sequence first generates one molecule of water and then consumes one molecule of water.

d

3.29.Enzymes facilitate reactions in living systems. Figure Q3-29 presents an energy diagram for the reaction XY. The solid line in the energy diagram represents changes in energy as the reactant is converted to product under standard conditions. The dashed line shows changes observed when the same reaction takes place in the presence of a dedicated enzyme. Which equation below indicates how the presence of an enzyme affects the activation energy of the reaction (catalyzed versus uncatalyzed)? (a) d - c versus b - c (b) d - a versus b - a (c) a + d versus a + b (d) d - c versus b - a *diagram

d-a versus b-a

During initiation of translation,the frist codon,______, enters the ____ and associates with the initiator tRNA. a.UAG,A site b.AUG,A site c.UAG, P site d.AUG, P site e.AUG,E Site

d.AUG, P site

Cystinuria

defect in carrier protein that transports cysteine across membrane of kidney cells

Type II diabetics have normal to elevated levels of insulin. What, then, might be causing their elevated blood glucose levels?

defective receptors

Because more than one codon can specify the same amino acid, the genetic code is said to be repetitive. redundant. reverse. degenerate. wobbly.

degenerate

2.58.d. five-carbon sugar found in DNA is called

deoxyribose

Which of the following is correctly paired with regard to similarity in function? A) adhesive junctions-tight junctions B) desmotubule-gap junction C) tight junctions-gap junctions D) plaques-plasmodesmata E) connexons-desmosomes

desmotubule-gap junction

62.f.match the following biological process with the model organism that is best suited or most specifically useful for its study: d. rerio (zebrafish)

development

62.g.match the following biological process with the model organism that is best suited or most specifically useful for its study: d. melongaster

development

38.

didinium engulfs prey by changing its shape, and for this it uses its cytoskeleton. bacteria have no cytoskeleton and cannot easily change their shape because they are generally surrounded by a tough cell wall.

Which of the following is associated with retrograde movement?

dynein

Chemiosmosis

energy stored in the form of H+ gradient across a membrane is used to drive cellular work.

3.8.b.H2 is the most stable and abundant form of hydrogen in the environment.

false, the most stable form of hydrogen is h2o

14.c. the cytosol is fairly empty, containing a limited number of organelles, which allows room for rapid movement via fusion.

false. the cytosol is actually brimming with individual proteins, protein fibers, membrane systems, transport vesicles, and small molecules. The rate of movement is limited by the space available.

48.c. true or false: the first eukaryotic cells on earth must have been aerobic; otherwise, they would not have been able to survive when the planet's atmosphere became oxygen-rich

false. the first eukaryotic cells likely contained a nucleus but no mitochondria. these ancestral eukaryotes subsequently adapted to survive in a world filled with oxygen by engulfing primitive aerobic prokaryotic cells

A collection of collagen fibrils is a collagen ________.

fiber

In order for a cell to crawl, it must produce a protrusion at the front of the cell (leading edge). One type is a pointed structure called ().

filopodium

Membrane synthesis in the cell requires the regulation of growth for both halves of the bilayer and the selective retention of certain types of lipids on one side or the other. Which group of enzymes accomplishes both of these tasks?

flippases

13. c.requires the use of two sets of filters. the first filter narrows the wavelength range that reaches the specimen and the second blocks out all wavelengths that pass back up to the eyepiece except for those emitted by the dye in the sample

fluorescence

axonemal microtubules

found in cilia and flagella

All of the following are moved by molecular motors except

glucose

starch???????

glucose production exceeds the immediate needs of a plant, the excess glucose is converted into starch and stored within the cells. starch can be a indirect measure of photosynthetic activity.

2.53.3.

glycerol

3 oxidation pathways

glycolysis, critic acid cycle, electron transport chain

2.58.f. linkage between the sugar and the base is called

glycosidic bond

49.d. good or bad to study in the model organism E. coli: how cells decode their genetic instructions to make proteins

good

2.43.g. inorganic or organice: calcium

i

2.43.j. inorganic or organice: iron

i

A hungry yeast cell lands in a vat of grape juice and begins to feast on the sugars there, producing carbon dioxide and ethanol in the process: C6H12O6 + 2ADP + 2Pi + H+ LaTeX: \Leftrightarrow ⇔ 2CO2 + 2CH3CH2OH + 2ATP + 2H2O Unfortunately, the grape juice is contaminated with proteases that attack some of the transport proteins in the yeast cell membrane, and the yeast cell dies. Which of the following could account for the yeast cell's demise?

inability to import sugar into the cell

when taken up by the cell, what binds to the repressor so that the repressor no longer binds to the operator?

inducer

2.46. diagram

ionic bonds

The microtubule-organizing center (MTOC)

is called a centrosome during interphase.

A. thaliana, or Arabidopsis, is a common weed. Biologists have selected it over hundreds of thousands of other flowering plant species to serve as an experimental model organism because __________________.

it can reproduce in 8-10 weeks.

In order for a cell to crawl, it must produce a protrusion at the front of the cell (leading edge). One type is a thin sheet called ().

lamellipodium

20. prokarytic cells are able to evolve very fast, which helps them to rapidly adapt to new food sources and develop resistance to antibiotics. what are the three main characteristics that support the rapid evolution of prokaryotic populations?

large population rapid growth can exchange DNA

12. what unit of length would you generally use to measure a typical plant or animal cell?

micrometers

Nucleic acids are polymers made up of which of the following monomers?

nucleotides

The endothelial cells found closest to the site of an infection express proteins called lectins. Each lectin binds to a particular ____________ that is presented on the surface of a target cell.

oligosaccharide

cancer-causing genes

oncogenes

autotroph

organism that makes its own food using sunlight or chemicals; plants

Diffusion

random movement of molecules due to their internal kinetic energy

2.53.7.

saturated fatty acid

Terminator

specifies the end of transcription.signals the end of transcription

Passive transport

substances can pass through them without any expenditure of energy by the cell

34. photosynthesis enables plants to capture the energy from sunlight. in this essential process, plants incorporate the carbon from CO2 into high-energy BLANK molecules, which the plant cell mitochondria use to produce ATP.

sugar

Which molecule carries an amino acid to the site of protein synthesis?

tRNA

What does the operon model attempt to explain?

the coordinated control of gene expression in bacteria

Template strand

the portion of the DNA strand that is used as a template for RNA synthesis

Translation

the process of synthesizing a specific polypeptide on a ribosome.

2.2.b. true or false:Carbon-based compounds make up the vast majority of molecules found in cells.

true

3.32.b. true or false: enzymes lower the activation energy for a specific reaction

true

3.49.b. true or false:Sequential pathways can help drive unfavorable reactions by siphoning off the products into the next energetically favorable reaction in the series.

true

How many hydrogen bonds are between A and T?

two

Which of the following is required for the secretion of neurotransmitters in response to an action potential?

voltage-gated Ca2+ channels

Net diffusion

when there is a difference in concentration between two regions of a system

regeneration

which is the process of replacing lost or damaging body part and can be observed in tadpoles, salamander, and cockroach

What did Frederick Griffith do?

worked with two strains of a bacterium, one pathogenic and one harmless

2.38.d. will the reaction in C occur more readily (be driven to the right) if the pH of the solution is high?

yes. if the pH is high, then the concentration of hydronium ions will be low. therefore the rightward reaction, which produces hydronium ions, will be favored.

The mathematical expression for the change in free energy is ΔG=ΔH-TΔS. Which of the following is (are) correct?

ΔG is the change in free energy

Even though proteins can form channels across biological membranes using either α helices or β sheets, channels made of α helices are more versatile. Explain the physical constraints on β-barrel structures and why these constraints do not apply to channels made of α helices.

β-Barrel structures are composed of individual β strands that form a β sheet that needs to be curved to make the structure of a pore in the membrane. The physical constraints are due to very specific positioning of each strand to maintain the necessary hydrogen- bonding network within the sheet. The relative positions of α helices can vary and still form strong interactions with other helices in the transmembrane region of a protein.

Which of the following statements about the function of the centrosome is false? (a) Microtubules emanating from the centrosome have alternating polarity such that some have their plus end attached to the centrosome while others have their minus end attached to the centrosome. (b) Centrosomes contain hundreds of copies of the γ-tubulin ring complex important for microtubule nucleation. (c) Centrosomes typically contain a pair of centrioles, which is made up of a cylindrical array of short microtubules. (d) Centrosomes are the major microtubule-organizing center in animal cells.

(a) Microtubules emanating from the centrosome are all arranged with their minus ends at the centrosomes and the plus ends extending into the cytoplasm.

Globular proteins fold up into compact, spherical structures that have uneven surfaces. They tend to form multisubunit complexes, which also have a rounded shape. Fibrous proteins, in contrast, span relatively large distances within the cell and in the extracellular space. Which of the proteins below is not classified as a fibrous protein? (a) elastase (b) collagen (c) keratin (d) elastin

(a) elastase

Eukaryotic cells are able to trigger the release of material from secretory vesicles to the extracellular space using a process called exocytosis. An example of materials commonly released this way is _____________.

(a) hormones.

Figure Q12-54 illustrates changes in membrane potential during the formation of an action potential. What membrane characteristic or measurement used to study action potentials is indicated by the arrow? (a) effect of a depolarizing stimulus (b) resting membrane potential (c) threshold potential (d) action potential

(b)

Formation of a lipid bilayer is energetically favorable. How does this arrangement result in higher entropy for the system, and thus make bilayer formation energetically favorable? (a) Polar head groups form a hydrogen-bonding network at the interface with water. (b) Water molecules form cagelike structures around hydrophobic molecules. (c) Hydrogen bonds form between neighboring polar head groups in the bilayer. (d) Fatty acid tails are highly saturated and flexible.

(b)

Membrane proteins, like membrane lipids, can move laterally by exchanging positions with other membrane components. Which type of membrane proteins is expected to be the least mobile, based on their function? (a) channels (b) anchors (c) receptors (d) enzymes

(b)

The lateral movement of transmembrane proteins can be restricted by several different mechanisms. Which mechanism best describes the process by which focal adhesions are formed to promote cell motility? (a) proteins are tethered to the cell cortex (b) proteins are tethered to the extracellular matrix (c) proteins are tethered to the proteins on the surface of another cell (d) protein movement is limited by the presence of a diffusion barrier

(b)

The Nernst equation can be used to calculate the membrane potential based on the ratio of the outer and inner ion concentration. In a resting cell, membrane potential is calculated taking only K+ ions into account. What is V when Co = 15 mM and Ci = 106 mM? (a) 438.1 mV (b) -52.7 mV (c) 52.7 mV (d) -5.3 mV

(b) Knowing that V = 62 × log(Co/Ci), substitute the outer and inner concentration values: V = 62 × log(15/106) V = 62 × -0.849 V = -52.7 mV

The activation of the serine/threonine protein kinase Akt requires phosphoinositide 3-kinase (PI 3-kinase) to _________. (a) activate the RTK. (b) create phosphorylated lipids that serve as docking sites that localize Akt to the plasma membrane. (c) directly phosphorylate Akt. (d) to create DAG.

(b) PI 3-kinase activity causes the localization of Akt to the plasma membrane, where it is phosphorylated by another protein kinase. The RTK is activated before PI 3-kinase is activated [choice (a)]. PI 3-kinase phosphorylates lipids, not proteins such as Akt [choice (c)]. DAG is created by phospholipase C and is not involved in this particular signaling pathway [choice (d)].

14-45Ubiquinone is one of two mobile electron carriers in the electron-transport chain. Where does the additional pair of electrons reside in the reduced ubiquinone molecule? (a)The electrons are added directly to the aromatic ring. (b)The electrons are added to each of two ketone oxygens on the aromatic ring. (c)The electrons are added to the hydrocarbon tail, which hides them inside the membrane bilayer. (d)Both electrons, and one proton, are added to a single ketone oxygen bound to the aromatic ring.

(b) The electrons are added to each of two ketone oxygens on the aromatic ring.

Which of the following is not a feature commonly observed in Beta sheets? (a) antiparallel regions (b) coiled-coil patterns (c) extended polypeptide backbone (d) parallel regions

(b) coiled-coil patterns

14-27The mitochondrial ATP synthase consists of several different protein subunits. Which subunit binds to ADP + Piand catalyzes the synthesis of ATP as a result of a conformational change? (a)transmembrane H+carrier (b)F1ATPase head (c)peripheral stalk (d)central stalk

(b)F1 ATPase head

Approximately, how many distinct synapses are established on the dendrites and cell body of a motor neuron in the spinal cord? (a) tens (b) hundreds (c) thousands (d) millions

(c)

Cells use membranes to help maintain set ranges of ion concentrations inside and outside the cell. Which of the following negatively charged ions is not primarily used to buffer positive charges inside the cell? (a) PO4 3- (b) OH- (c) Cl- (d) HCO 3-

(c)

New membrane phospholipids are synthesized by enzymes bound to the _____________ side of the _________________ membrane. (a) cytosolic, mitochondrial (b) luminal, Golgi (c) cytosolic, endoplasmic reticulum (d) extracellular, plasma

(c)

Unlike soluble, cytosolic proteins, membrane proteins are more difficult to purify. Which of the following substances is most commonly used to help purify a membrane protein? (a) high salt solution (b) sucrose (c) detergent (d) ethanol

(c)

14-69 Oxidative phosphorylation, as it occurs in modern eukaryotes, is a complex process that probably arose in simple stages in primitive bacteria. Which mechanism is proposed to have arisen first as this complex system evolved? (a)electron transfers coupled to a proton pump (b)the reaction of oxygen with an ancestor of cytochrome c oxidase (c)ATP-driven proton pumps (d)the generation of ATP from the energy of a proton gradient

(c) ATP-driven proton pumps

Glycolysis generates more stored energy than it expends. What is the net number of activated carrier molecules produced in this process (number and type of molecules produced minus the number of those molecules used as input)? (a) 6 ATP, 2 NADH (b) 4 ATP, 4 NADH (c) 2 ATP, 2 NADH (d) 4 ATP, 2 NADH

(c) Although 4 ATP and 2 NADH are produced, 2 ATP are converted to ADP in the early steps of glycolysis. In contrast, NADH is not converted to NAD+ in any of the reactions.

14-13 Which of the following statements describes the mitochondrial intermembrane space? (a)It is permeable to molecules with molecular mass as high as 5000 daltons. (b)It contains transporters for ATP molecules. (c)It contains proteins that are released during apoptosis. (d)It contains enzymes required for the oxidation of fatty acids.

(c) It contains proteins that are released during apoptosis.

15-54Vesicles from the ER enter the Golgi at the ______. (a)medial cisternae. (b)transGolgi network. (c)cisGolgi network. (d)transcisternae.

(c) cis Golgi network.

Negatively charged ions are required to balance the net positive charge from metal ions such as K+, Na+, and Ca2+. Which of the following negatively charged ions is the most abundant outside the cell and which ion does most often neutralize (written in parentheses)? (a) Cl-(Ca2+) (b) PO43-(K+) (c) PO43-(Ca2+) (d) Cl-(Na+)

(d)

Red blood cells have been very useful in the study of membranes and the protein components that provide structural support. Which of the following proteins is the principal fibrous protein in the cortex of the red blood cell? (a) tubulin (b) attachment proteins (c) actin (d) spectrin

(d)

15-49 Different glycoproteins can have a diverse array of oligosaccharides. Which of the statements below about this diversity is true?(a)Extensive modification of oligosaccharides occurs in the extracellular space.(b)Different oligosaccharides are covalently linked to proteins in the ER and the Golgi.(c)A diversity of oligosaccharyl transferases recognizes specific protein sequences, resulting in the linkage of a variety of oligosaccharides to proteins.(d)Oligosaccharide diversity comes from modifications that occur in the ER and the Golgi of the 14-sugar oligosaccharide added to the protein in the ER.

(d) Oligosaccharide diversity comes from modifications that occur in the ER and the Golgi of the 14-sugar oligosaccharide added to the protein in the ER.

15-11Proteins that are fully translated in the cytosol do not end up in _______. (a)the cytosol. (b)the mitochondria. (c)the interior of the nucleus. (d)transport vesicles.

(d) Proteins destined for transport vesicles will be translated on ribosomes associated with the endoplasmic reticulum.

Which of the following statements about the structure of microtubules is false? (a) Microtubules are built from protofilaments that come together to make a hollow structure. (b) The two ends of a protofilament are chemically distinct, with α-tubulin exposed at one end and β-tubulin exposed at the other end. (c) Within a microtubule, all protofilaments are arranged in the same orientation, giving the microtubule structural polarity. (d) α-Tubulin and β-tubulin are covalently bound to make the tubulin dimer that then assembles into protofilaments.

(d) α-Tubulin and β-tubulin bind with each other through noncovalent interactions.

We can estimate the relative mobility of a population of molecules along the surface of a living cell by fluorescently labeling the molecules of interest, bleaching the label in one small area, and then measuring the speed of signal recovery as molecules migrate back into the bleached area. What is this method called? What does the abbreviation stand for? (a) SDS (b) SPT (c) GFP (d) FRAP

(d), fluorescence recovery after photobleaching

Vesicular Transport 15-36 Which of the following choices reflects the appropriate order of locations through which a protein destined for the plasma membrane travels? (a)lysosome endosome plasma membrane (b)ER lysosome plasma membrane (c)Golgi lysosome plasma membrane (d)ER Golgi plasma membrane(d)ER Golgi plasma membrane ...

(d)ER Golgi plasma membrane ...

What are the three enzymes that are required for Argine synthesis?

+Omithine, +Citrulline, Arginine

The Nernst equation can be used to calculate the membrane potential based on the ratio of the outer and inner ion concentration. In a resting cell, membrane potential is calculated taking only K+ ions into account. What is V when Co = 15 mM and Ci = 106 mM?

-52.7 mV

B

10) Transcription of the structural genes in an inducible operon A) occurs continuously in the cell. B) starts when the pathway's substrate is present. C) starts when the pathway's product is present. D) stops when the pathway's product is present. E) does not result in the production of enzymes.

E

18) If she moves the repressor gene (lac I), along with its promoter, to a position at some several thousand base pairs away from its normal position, which will you expect to occur? A) The repressor will no longer be made. B) The repressor will no longer bind to the operator. C) The repressor will no longer bind to the inducer. D) The lac operon will be expressed continuously. E) The lac operon will function normally.

2.9a. if .5 mole of glucose weighs 90g, what is the molecular mass of glucose?

180 daltons. a mole of a substance has a mass equivalent to its molecular weight

When the polymer X-X-X... is broken down into monomers, it is "phosphorylyzed" rather than hydrolyzed, in the following repeated reaction: X-X-X... + P LaTeX: \leftrightarrow ↔ X-P + X-X... (reaction 1) Given the ΔG° values of the reactions listed in the following table, what is the expected ratio of X-phosphate (X-P) to free phosphate (P) at equilibrium for reaction 1?

1:1

How many different primary sequences can be generated by randomly assembling amino acids into peptides that are five residues long? 20^5=3,200,000 20 20×5=100. The number is infinite.

20^5=3,200,000

How many nucleotides are contained in a single codon?

3

Each NADH can form a maximum of ____ ATP molecules. Each FADH2, which donates electrons that activate only two proton pumps, makes ____ ATP molecules.

3,2

The genetic code consists of ____ codons that specify amino acids, and ____ codons that do not specify amino acids. 20/3 64/3 60/4 61/3 21/3

61/3

14-42A.Match each equation in column A with the corresponding standard redox potential in column B. Column A 1. H2O ↔½O2+ 2H++ 2 e- 2. reduced ubiquinone ↔oxidized ubiquinone + 2H++ 2 e- 3. NADH ↔NAD++ H++ 2 e-C. 4. reduced cytochrome c↔oxidized cytochrome c+ e- Column B. A.+30 mV B.+820 mV C.+230 mV D.-320 mV B.How do these standard redox potentials support our understanding of the stepwise electron transfers that occur in the electron-transport chain? C.Why would it not be advantageous for living systems to evolve a mechanism for the direct transfer of electrons from NADH to O2?

A. 1—B; 2—A; 3—D; 4—C B. Each successive member of the electron-transport chain is a better electron acceptor, which permits a unidirectional series of electron transfers until reaching O2, which is the best electron acceptor and the final destination of the electrons, forming water as oxygen is consumed. C. If NADH directly donated electrons to O2, a large amount of energy would be released as heat and lost, rather than used as a way for the cell to generate chemical energy in the form of ATP.

The graph in Figure Q17-30 shows the time course of the polymerization of pure tubulin in vitro. You can assume that the starting concentration of free tubulin is much higher than it is in cells. A. Explain the reason for the initial lag in the rate of microtubule formation. B. Why does the curve level out after point C?

A. Before they can polymerize to form microtubules, tubulin molecules have to form small aggregates that act as nucleation centers. This aggregation step is slow because the molecules have to come together in the right configuration. This is why there is a lag phase before microtubules start to be formed. B. After point C, an equilibrium point has been reached, where the rates of polymerization and depolymerization are exactly balanced.

3.43. a. Consider the reaction XY in a cell at 37°C. At equilibrium, the concentrations of X and Y are 50 μM and 5 μM, respectively. Use this information and the equations below to answer questions A-E. ΔG° = -0.616 ln Keq ΔG = ΔG° + 0.616 ln [Y]/[X] Recall that the natural log of a number z will have a negative value when z < 1, positive when z > 1, and 0 when z = 1. A. What is the value of Keq for this reaction?

A. Keq = [Y]/[X]

14-56Indicate whether the following statements are trueor false. If a statement is false, explain why it is false. A.Carbon fixation can be described as a process by which gaseous carbon-containing molecules are captured and incorporated into biological hydrocarbon molecules. B.The electron-transport proteins, utilized in stage 1 of photosynthesis, reside in the inner membrane of the chloroplast. C.Similar to oxidative phosphorylation, the electrons passed along the chloroplast electron-transport chain are ultimately passed on to a molecule of O2, to produce H2O. D.Stage 2 of photosynthesis involves a cycle of reactions that does not directly depend on energy derived from sunlight.

A. True. B. False. The electron-transport system in chloroplasts resides in the thylakoid membrane. C. False. The recipient of electrons in the chloroplast electron-transport chain is the NADP+ cofactor, which becomes reduced to NADPH. D. True.

In an attempt to define the protein domains of protein X, you treat it with a protease and use polyacrylamide gel electrophoresis to analyze the peptides produced. In the past, you have used chymotrypsin to perform this experiment, but the stock of this enzyme has been used up . You find a stock of elastase and decide to use it instead of waiting for a new stock of chymotrypsin to arrive. A. Give two reasons why elastase is a good substitute for chymotrypsin in this assay. B. Why might proteolysis of the same substrate by chymotrypsin or elastase yield different results?

A. You might assume that chymotrypsin and elastase would yield the same results because (1) they are both serine proteases and (2) they have a high degree of structural similarity. B. The slight structural differences of the substrates cause the enzymatic activity of the protease to differ. As a result, they have different substrate affinities and cleave the bond between a different set of amino acids.

What are require for the formation of the scar matrix?

All of the above

By which of the following molecular mechanisms do psychoactive drugs exert their effects?

All of these are correct.

Does the evidence you obtained from this exercise support or refute the diffusion gradient model? (i.e. What did you get? Did it agree with your predictions?) Explain

All or most of the planaria should have fully regenerated, but may not be fully grown.

The citric acid cycle is a critical sequence of reactions for energy production, which take place in the matrix of the mitochondria. The reaction cycle requires materials from the cytosol to be converted into acetyl CoA, which represents the starting point of a new cycle. Which of the following statements about acetyl CoA is true?

Amino acids can be converted into acetyl CoA.

Ribozyme

An RNA moecule that catalyzes a chemical reaction.

Explain how AMP stimulates cellular respiration while citrate and ATP inhibit it.

As ATP is used up, AMP accumulates which signals more ATP to be made. The more ATP there is, the slower glycolisis is.

17) For a tissue or an organ to function as a unit, ________.

B) there must be cell—cell communication among the cells within a tissue or organ

22) Intercalated discs are cell—cell junctions found between cardiac muscle cells. One feature of these intercalated discs is that they contain a large number of gap junctions, which means that ________.

B) water ions and small molecules can readily pass from one cardiac muscle cell to the next

Why is oxygen the final electron acceptor?

Because it is so electronegative

What are similarities and differences between the function of DNA polymerase and that of RNA polymerase?

Both DNA and RNA polymerase use DNA strands as a templaate and connect the nucleotides to each other in a 5' to 3' direction based on complementarity base pairing.One difference is that DNA polymerase needs a preexisting strand,such as an RNA primer to begin DNA replication.Whereas RNA polymerase can begin the syntheis of RNA on a bare template strand.Another key difference is that DNA polymerase connects deoxyribonucleotides,where as RNA polymerase connects ribonucleotides.

Protein kinase is an enzyme that ________.

C) activates or inactivates other proteins by adding a phosphate group to them

Intracellular calcium (Ca++) concentration is kept low by a ______________.

Ca++ ATPase pump

They are proteins that promote depolymerization of extant microtubules at the ends.

Catastrophins

The smallest living things are

Cells

3.37.Although the biochemical study of reaction rates and free energies is important for understanding each biological reaction individually, these studies do not provide an accurate picture of what is happening to reactants and products inside the cell. Why not?

Chemical reactions inside the cell do not reach a state of equilibrium because both reactants and products are typically used in more than one set of reactions, which means their concentrations are constantly fluctuating. As a result, the forward and reverse reaction rates are almost never identical.

All intermediate filaments are of similar diameter because ____________. (a) the central rod domains are similar in size and amino acid sequence. (b) the globular domains are similar in size and amino acid sequence. (c) covalent bonds among tetramers allow them to pack together in a similar fashion. (d) there is only a single type of intermediate filament in every organism.

Choice (a) is correct. Globular domains vary among intermediate filaments in size and have different types of amino acids [choice (b)]. The interactions among all intermediate filament subunits involve noncovalent bonding [choice (c)]. There are several classes of intermediate filaments and an organism can have more than one class (and sometimes, more than one member of each class) [choice (d)].

14-1 The link between bond-forming reactions and membrane transport processes in the mitochondria is called __________________. (a)chemiosmotic coupling. (b)proton pumping. (c)electron transfer. (d)ATP synthesis.

Choice (a)is correct. Choices (b), (c), and (d) are individual parts of the overall process of chemiosmotic coupling.

The last common ancestor to plants and animals was a unicellular eukaryote. Thus, it is thought that multicellularity and the attendant demands for cell communication arose independently in these two lineages. This evolutionary viewpoint accounts nicely for the vastly different mechanisms that plants and animals use for cell communication. Fungi use signaling mechanisms and components that are very similar to those used in animals. Which of the phylogenetic trees shown in Figure Q16-59 does this observation support?

Choice (b) is the correct answer (see Figure Q16-59). The similarities in signaling mechanisms between animals and fungi support the phylogenetic tree in which fungi branched from the animal lineage after plants and animals separated. This branching order is supported by a wide variety of other data, including genomic sequence comparisons.

The following happens when a G-protein-coupled receptor activates a G protein. (a) The β subunit exchanges its bound GDP for GTP. (b) The GDP bound to the α subunit is phosphorylated to form bound GTP. (c) The α subunit exchanges its bound GDP for GTP. (d) It activates the α subunit and inactivates the βγ complex.

Choice (c) is correct. Choice (d) is incorrect because the βγ complex can also activate downstream targets. The other statements are simply untrue.

Which reaction does the enzyme phosphoglucose isomerase catalyze? (a) glucose glucose 6-phosphate (b) fructose 6-phosphate fructose 1,6-bisphosphate (c) glucose 6-phosphate fructose 6-phosphate (d) glucose glucose 1-phosphate

Choice (c) is the correct answer. The isomerase part of the enzyme name indicates that it catalyzes an isomerization reaction, and the phosphoglucose part of the name indicates the type of substrate used. The enzymes that catalyzed reactions (a), (b), and (d) would be called kinases, because they transfer phosphate groups from one molecule to another.

Which of the following statements is false? (a) Nucleotides and amino acids can act as extracellular signal molecules. (b) Some signal molecules can bind directly to intracellular proteins that bind DNA and regulate gene transcription. (c) Some signal molecules are transmembrane proteins. (d) Dissolved gases such as nitric oxide (NO) can act as signal molecules, but because they cannot interact with proteins they must act by affecting membrane lipids.

Choice (d) is not true. NO can diffuse across the plasma membrane and directly activate intracellular proteins such as the enzyme guanylyl cyclase.

Which of the following statements regarding dynamic instability is false? (a) Each microtubule filament grows and shrinks independently of its neighbors. (b) The GTP cap helps protect a growing microtubule from depolymerization. (c) GTP hydrolysis by the tubulin dimer promotes microtubule shrinking. (d) The newly freed tubulin dimers from a shrinking microtubule can be immediately captured by growing microtubules and added to their plus end.

Choice (d) is untrue. A newly dissociated tubulin dimer will be bound to GDP; this GDP will need to be exchanged for GTP before it can be added to a newly growing microtubule.

What are cylindrical extensions of the plasma membrane?

Cilia

The most common eukaryotic ribosome carries out its function in the.

Cyrosol

14-21 Which component of the electron-transport chain is required to combine the pair of electrons with molecular oxygen? (a)cytochrome c (b)cytochrome b-c1complex (c)ubiquinone (d)cytochrome c oxidase

D cytochrome c oxidase

25) A cell with membrane-bound proteins that selectively bind a specific hormone is called that hormone's ________.

D) target cell

When electrons move closer to a more electronegative atom, what happens? A) Energy is released. B) Energy is consumed. C) The more electronegative atom is reduced. D) The more electronegative atom is oxidized. E) A and C are correct.

E

Where does glycolysis takes place? A) mitochondrial matrix B) mitochondrial outer membrane C) mitochondrial inner membrane D) mitochondrial intermembrane space E) cytosol

E

ATP synthase

Enzyme that synthesizes ATP. ATP is mainly produced in the mitochondria and is an important enzyme that provides energy for the cell to use through the synthesis of adenosine triphosphate (ATP).

The 2 electron carrier molecules that feed electrons into the ETC are _____ and _____.

FADH2 and NADH.

In eukaryotes, 40S and 60S combine to form a 100S ribosome.

False

3.56.c. true or false: Competitive inhibitors bind irreversibly to the enzyme active site, lowering Vmax.

False. Competitive inhibitors bind reversibly to an enzyme's active site.

2.2.c. true or false: The chemical reactions in living systems are loosely regulated, allowing for a wide range of products and more rapid evolution.

False. The chemical reactions in living systems are very tightly controlled, ensuring that events occur at the proper time and at the proper location inside the cell.

The process that occurs when a physical pressure pushes materials through a barrier is called

Filtration

The structure that allows locomotion in some cells is the _______.

Flagellum

built a model in which the backbones were antiparallel

Franklin

Who discovered the genetic role of DNA

Frederick Griffith in 1928

Match the class of cell-surface receptor with the best description of its function. Not all descriptors will be used. G-protein-coupled receptors ______ ion-channel-coupled receptors ______ enzyme-coupled receptors ______ A. alter the membrane potential directly by changing the permeability of the plasma membrane B. signal by opening and closing in a ligand-independent manner C. must be coupled with intracellular monomeric GTP-binding proteins D. all receptors of this class are polypeptides with seven transmembrane domains E. discovered for their role in responding to growth factors in animal cells

G-protein-coupled receptors ___D___ ion-channel-coupled receptors ___A___ enzyme-coupled receptors ___E___

As electrons fall between PSI and II the cytochrome complex uses the energy to pump ___ ions. This builds a proton gradient that is used in chemiosmosis to produce what?

H+; ATP

To break the last phosphate bond in ATP, what must be added?

H20 (water) The process of breaking the last phosphate bond is called Hydrolysis

3.12. Fill in the blanks, selecting from the choices below. Light + _________ + _________ _________ + heat + sugars CO, CO2, O2, H2, H2O, N2, NO

H2O, CO2, O2

What compound is the source of electrons for linear electron flow? This compound is also the source of ___ in the atmosphere.

H2O; O2.

Which of the following binds intermediate filaments to anchor proteins to integrins to the ECM fibrous proteins?

Hemidesmosomes

saturated structure is more linear unsaturated structure is bent

How are the structures of saturated and unsaturated fats different?

Write a prediction concerning the relationship between the presence of light and starch production (if/then).

If light ( independent) is present, then the plant will carry out photosynthesis and store glucose as starch. (dependent)

Which of the following is mismatched? A) Ig[chondroitin B) cadherinscalcium c) cadherinsembryonic development E) lectinscarbohydrates

Ig[chondroitin

The central dogma of gene expresssion at the molecular level

In bacteria, transcription and translation occur in the cytoplasm In eukaryotes,transcription and RNA processing occur in the nucleus,whereas translation takes place in the cytosol

Cilia are found

In cells that are required to move a fluid medium along their surface

hydrophobic and hydrophilic

In order for a protein to be an integral membrane protein, it would have to be (hydrophobic/hydrophilic/both).

What is the difference between oxidation and reduction?

In oxidation, you lose electrons, in reduction, you gain them.

3.30.b.The branched diagram in Figure Q3-30 represents several possible reaction pathways that substance X may follow. Each branch point represents two possible reactions, which would give rise to different products. Next to each of these points are values for the uncatalyzed activation energies and catalyzed activation energies (in kcal/mole), respectively. Order the final products in the relative amounts you expect them to be produced when enzymes for each reaction are present.

In the presence of the enzymes required to catalyze the two different reactions represented by each branch, we should expect to see a different distribution of products: G, H, E, F, A, B, D, C.

How doe temperature affect the enzyme reaction?

Increases reaction

As scientists were unraveling the mysteries associated with transcription and translation in eukaryotes, they discovered there was not a one-to-one correspondence between the nucleotide sequence of a gene and the base sequence of the mRNA it codes for. They proposed the genes-in-pieces hypothesis. How can the genes-in-pieces hypothesis be explained? Introns are noncoding segments of DNA that are present in the initial transcript, but are removed by splicing. Exons are noncoding segments of DNA that are present in the initial transcript, but are removed by splicing. Exons are noncoding segments of DNA that are not read or transcribed by RNA polymerase II. Introns are noncoding segments of DNA that are not read or transcribed by RNA polymerase II.

Introns are noncoding segments of DNA that are present in the initial transcript, but are removed by splicing

Which of the following techniques would not be useful in separating proteins on the basis of size?

Ion exchange chromatography

What does the primary electron accepter do?

It accepts electrons and functions in shuttling electrons from PS to ETC to produce ATP

What purpose does the phosphorylation of glucose to glucose 6-phosphate by the enzyme hexokinase serve as the first step in glycolysis?

It helps drive the uptake of glucose from outside the cell.

14-11 Which of the following statements describes the mitochondrial outer membrane? (a)It is permeable to molecules with molecular mass as high as 5000 daltons. (b)It contains transporters for ATP molecules. (c)It contains proteins that are released during apoptosis. (d)It contains enzymes required for the oxidation of fatty acids.

It is permeable to molecules with molecular mass as high as 5000 daltons.

If an anticodon in a tRNA molecule had the sequence 3'-ACC-5' which amino acid does it carry?

It would bind to a 5'-UGG-3' codon and it would carry tryptophan.

who introduced an elegant double-helical model for the structure of deoxyribonucleic acid, or DNA

James Watson and Francis Crick

The equilibrium constant (K) for the reaction Y LaTeX: \leftrightarrow ↔ X can be expressed with respect to the concentrations of the reactant and product molecules. Which of the expressions below shows the correct relationship between K, [Y], and [X]?

K = [X]/[Y]

3.44.Hydrolysis reactions are commonly used inside the cell to split high-energy covalent bonds. For each of the three reactions below, use the ΔG° for each reaction to determine the equilibrium constants (K). Assume that each reaction occurs independently of the other two. ΔG° (kcal/mole) Reaction 1: acetyl-P actetate + P -10.3 Reaction 2: ATP ADP + P -7.3 Reaction 3: glucose 6-P glucose + P -3.3

K can be easily calculated from the standard free-energy values by solving the standard free-energy equation (ΔG° = -1.42 log K) for K (K =10ΔG°/-1.42). Reaction 1: K = 107.25 Reaction 2: K = 105.14 Reaction 3: K = 102.32

Although membrane proteins contribute roughly 50% of the total mass of the membrane, there are about 50 times more lipid molecules than there are protein molecules in cellular membranes. Explain this apparent discrepancy.

Membrane proteins are much larger molecules than the membrane lipids. Thus, fewer are required to represent the same total mass contributed by the lipid components of the membrane. By this estimation, the molecular weight of the average membrane protein is 50 times that of the average membrane lipid.

The following codons and the amino acids they encode is as follows: AUG = Start or Met; UUU, UUC = Phe; UUA, UUG = Leu; UCU, UCC = Ser; CCU, CCC = Pro; ACU, ACC = Thr; UGA = Stop. The 5′ACU-UUC-ACU-AUG-UUU-UUA-UCC-UCC-ACU-CCU-UGA3′ mRNA transcript results in which of the following polypeptide sequences?

Met-Phe-Leu-Ser-Ser-Thr-Pro

Glyceraldehyde 3-phosphate dehydrogenase operates by stripping a hydride ion from its substrate. Which molecule is the recipient of the proton and two electrons during this transfer?

NAD+

Although ATP and NADH are both important activated carrier molecules, ATP hydrolysis provides the direct molecular energy for most biochemical reactions. Why do the mitochondria also need to generate high levels of NADH?

NADH is an activated carrier molecule used as a cofactor for many enzymes that catalyze redox reactions. NADH also donates electrons to the electron-transport chain, which is essential for the production of ATP.

Note that the light reactions store chemical energy in _____ and _______, which shuttle the energy to the carbohydrate-producing __ cycle

NADPH and ATP; Calvin

Eukaryotic cells have membranous organelles which share many similarities with the cell plasma membrane. Which of the following is not a characteristic shared by the plasma membrane and organellar membranes?

NOTflexible, two-dimensional field

Based on the periodic table shown here, which elements most likely form an ionic bond?

Na, Cl, Li, F

Neurons communicate with each other through specialized sites called __________________. Many neurotransmitter receptors are ligand-gated ion channels that open transiently in the __________________ cell membrane in response to neurotransmitters released by the __________________ cell. Ligand- gated ion channels in nerve cell membranes convert __________________ signals into __________________ ones. Neurotransmitter release is stimulated by the opening of voltage-gated __________________ in the nerve-terminal membrane. acetylcholine receptor GABA receptor postsynaptic Ca2+ channels K+ channels presynaptic chemical Na+ channels synapses electrical

Neurons communicate with each other through specialized sites called synapses. Many neurotransmitter receptors are ligand-gated ion channels that open transiently in the postsynaptic cell membrane in response to neurotransmitters released by the presynaptic cell. Ligand-gated ion channels in nerve cell membranes convert chemical signals into electrical ones. Neurotransmitter release is stimulated by the opening of voltage-gated Ca2+ channels in the nerve-terminal membrane.

The cell's RNA is produced by the

Nucleolus

Record behavioral observations of planaria before cuts.

Observations recorded. Should see movement away from light and toward food.

The movement of water across a membrane from an area of higher water concentration to an area of lower water concentration is known as

Osmosis

2.59.b.peptide: Pro-Val-Thr-Gly-Lys-Cys-Glu write the single-letter code for amino acids

PVTGKCE

Compare the exact amounts of ATP and CO2 produced from each glucose molecule in aerobic respiration and anaerobic lactic acid fermentation in muscle cells.

Per glucose molecule. ATP CO2 Aerobic ~36 6 Anaerobic 2 0

14-61 Use the terms provided below to fill in the blanks. Not all words or phrases will be used; each word or phrase may be used more than once.Photons from sunlight that are in the ______________ wavelength range are preferentially absorbed by chlorophyll molecules to raise the energy levels of electrons in the __________ ring. The __________ reflected are lower in energy, which is indicated in the ________, green wavelengths detected by the human eye. benzene longer porphyrin blue orange red electrons photons shorter heme

Photons from sunlight that are in the red wavelength range are preferentially absorbed by chlorophyll molecules to raise the energy levels of electrons in the porphyrin ring. The photons reflected are lower in energy, which is indicated in the longer, green wavelengths detected by the human eye.

What is the significance of photosynthesis?

Photosynthesis fixes carbon from CO2 to produce organic compounds. These organic molecules along with other inorganic raw materials obtained from the environment are the ultimate source of organic compounds for all non-photosynthetic organisms.

The process by which vesicles containing fluid are formed on the surface of a cell for transport into the cell is called

Pinocytosis

Explain what is meant by a C3 plant

Plant that uses calvin cycle for carbon fixation to form G3P

Glycolysis and the citric acid cycle comprise two different sets of oxidation reactions. The reaction sequence for glycolysis is linear, whereas the reaction sequence for the citirc acid cycle forms a circle. How does this difference in the arrangement of reactions influence the rate of these processes when an excess amount of a single intermediate is added?

Primarily, what is seen is that the citric acid cycle occurs more rapidly after the addition of any one of the intermediates. This means that if one intermediate is added, levels of all of them increase. In glycolysis, the intermediates downstream of the intermediate being added will be affected.

What 3 organic macromolecules are often utilized to make ATP by cellular respiration?

Proteins, lipids, carbohydrates

When compounds lose energy, they ___ energy; when compounds gain electrons, they ____ energy.

Release (lose); absorb (gain)

Explain what happens in the light reactions stage of photosynthesis?

Solar energy is converted to chemical energy. Light is absorbed by chlorophyll and by doing so there is a transfer of electrons and H+ ions to NADP+. Thus NADP+ is reduced to NADPH. ATP is also produced and when the energy from light is used to help catalyze the reaction between ADP and Pi, it is called photophosphorylation.

D

Suppose an experimenter becomes proficient with a technique that allows her to move DNA sequences within a prokaryotic genome. 16) If she moves the promoter for the lac operon to the region between the beta galactosidase gene and the permease gene, which of the following would be likely? A) Three structural genes will no longer be expressed. B) RNA polymerase will no longer transcribe permease. C) The operon will no longer be inducible. D) Beta galactosidase will be produced. E) The cell will continue to metabolize but more slowly.

D

48) Which of the following describes the function of an enzyme known as Dicer? A) It degrades single-stranded DNA. B) It degrades single-stranded mRNA. C) It degrades mRNA with no poly(A) tail. D) It trims small double-stranded RNAs into molecules that can block translation. E) It chops up single-stranded DNAs from infecting viruses.

In addition to ATP, what are the end products of glycolysis? A) CO2 and H2O B) CO2 and pyruvate C) NADH and pyruvate D) CO2 and NADH E) H2O, FADH2, and citrate

C

15-28Which of the following statements about a protein in the lumen of the ER is false? (a)A protein in the lumen of the ER is synthesized by ribosomes on the ER membrane. (b)Some of the proteins in the lumen of the ER can end up in the extracellular space. (c)Some of the proteins in the lumen of the ER can end up in the lumen of an organelle in the endomembrane system. (d)Some of the proteins in the lumen of the ER can end up in the plasma membrane.

Choice (d) is correct. ER signal sequences are typically at the N-terminus [choice (a)]. More than one ribosome can bind to an mRNA molecule [choice (b)]. Hydrophobic stop- transfer sequences are found on membrane-inserted proteins and not on soluble proteins [choice (c)].

We know the detailed molecular structure and mechanism of action of the transmembrane protein bacteriorhodopsin. This protein uses sunlight as the source of energy to pump ______ out of the cell.

H+

when hydrogens are added to unsaturated fats

How are trans-fats made?

Codons for Leu

UUA UUG CUU CUC CUA CUG

3.53.The graph in Figure Q3-53 illustrates the change in the rate of an enzyme-catalyzed reaction as the concentration of substrate is increased. Which of the values listed below is used to calculate the enzyme turnover number? (a) ½Vmax (b) Km (c) Vmax (d) Vmax - Km

Vmax

Who built models of a double helix to conform to the X-rays and chemistry of DNA?

Watson and Crick

glycerol and fatty acid

What are fats constructed from?

Actin can adopt a variety of shapes. Match the name of the actin form with the type of actin structure depicted as black lines within the cells in Q17-43. _____ lamellipodia _____ contractile bundles _____ contractile ring _____ microvilli

__C__ lamellipodia __B__ contractile bundles __D__ contractile ring __A__ microvilli

In Meselson and Stahl's experiment proving semiconservative DNA replication, they started with bacteria grown in a heavy isotope of nitrogen and then switched them to a light isotope. They then observed the DNA density after one and two rounds of replication. What was the result after one round of replication?

all hybrid DNA

the thin yellow line at the top

carotene

62.c.match the following biological process with the model organism that is best suited or most specifically useful for its study: s. pombe

cell division

3.42.In the case of a simple conversion reaction such as XY, which value of ΔG° is associated with a larger concentration of X than Y at equilibrium? (Hint: How is ΔG° related to K?) (a) ΔG° = -5 (b) ΔG° = -1 (c) ΔG° = 0 (d) ΔG° = 1

d

Golgi apparatus

only in eukaryotes

D

40) Transcription factors in eukaryotes usually have DNA binding domains as well as other domains also specific for binding. In general, which of the following would you expect many of them to be able to bind? A) repressors B) ATP C) protein-based hormones D) other transcription factors E) tRNA

What numbers must be placed as coefficients in the blanks for the chemical reaction below in order to ensure that matter is conserved? Fe3O4 + __C -> __Fe + __CO

4; 3; 4

What is the formula for photosynthesis?

6CO2 + 6H2O + energy --> C6H12O6 + 6O2

How does pyruvate enter the mitochondrion? A) active transport B) diffusion C) facilitated diffusion D) through a channel E) through a pore

A

One characteristic of life and living systems is that they are able to adapt. In general, cells interact with other cells and their environment through the action of their ________.

A) plasma membrane and extracellular matrix

When a mutation caused by a nucleotide-pair insertion occurs in a protein-coding region, which of the following types of mutations cannot occur? A) silent B) frameshift C) missense D) nonsense E) none of the above

A) silent

Signals from the extracellular matrix to the cytoskeleton may be transmitted by ________.

C) integrins

Osmotic pressure is determined by the _______ of solute molecules in a solution.

Concentration

If a cell wanted long term adhesion in the small intestine, it would need.....?

Desmosomes

The process by which molecules such as oxygen are moved into cells along their concentration gradient directly through the cell membrane is called

Diffusion

Which of the following techniques is used to monitor (single) ion channel activity?

Patch clamp

Elongation

RNA polymerase synthesizes the RNA transcript for this to occur thesigma factor is released and RNA polymerase slides along the DNA in a way that maintains an open complex as it goes the DNA strand is used as a template for RNA synthesis .

One function of the nucleolus is to produce ______.

Ribosomes

Explain the correlation between an absorption spectra, and an action spectrum.

The action spectrum shows that the chlorophylls in the absorption spectrum are most important because they correlate closest to the action spectrum.

What is the function of the anticodon?

The function of the anticodon in tRNA is to recognize a codon in an mRNA.

What were the results of the excess nitrogen in the tomato plants?

The plant changed a large amount in weight and length, overall was unhealthy and had leaves that were wilting and brown roots

3.74.a. Coenzyme A can be converted to acetyl CoA, which is an important activated carrier molecule that has a central role in metabolism and can be used to add two carbons in each successive cycle of fatty acid synthesis. Identify the location and type of high-energy bond in the acetyl CoA molecule shown in Figure Q3-74.

The thioester bond is the high-energy bond, boxed in Figure A3-74.

Why do the epithelial cheek cells lack an elaborate internal structure?

They are found on the outside of animals and are constantly sloughing off. Since they aren't very temporary and don't perform complex functions, there is no need for elaborate internal structures

What do the cell walls of plants and the extracellular matrix of animal cells have in common?

They have functional connections with the cytoskeleton inside the cell.

Molecular chaperones can work by creating an "isolation chamber." What is the purpose of this chamber?

This chamber serves to protect unfolded proteins from interacting with other proteins in the cytosol, until protein folding is completed.

On which of the following processes would a defective ribosome have the most detrimental effect?

Translation

Hemidesmosomes adheres with anchor proteins to the basal lamina.

True

More than one codon can specify the same amino acid.

True

51. biologist cannot possibly study all living species. instead they try to understand cell behavior by studying a select subset of them. which of the following characteristics are useful in an organism chosen for use as a model in laboratory studies? a. amenability to genetic manipulation b. ability to grow under controlled conditions c. rapid rate of reproduction d. all of the above

all of the above

A specific gene is known to code for three different but related proteins. This could be due to which of the following?

alternative RNA splicing

36.f. what cell type has lysosomes?

animal, plant

54.b.what would be the best model organism to use for the investigation of the process of: chloroplast function

arabidopsis

3.35.Which of the following is true for a reaction at equilibrium? (a) ΔG = ΔG° (b) ΔG° + RT ln [X]/[Y] = 0 (c) RT ln [X]/[Y] = 0 (d) ΔG + ΔG° = RT ln [X]/[Y]

b

The final metabolite produced by glycolysis is ___________. (a) acetyl CoA. (b) pyruvate. (c) 3-phosphoglycerate. (d) glyceraldehyde 3-phosphate.

b

During eukaryotic RNA processing,the nontranslated sequences that are removed are called a.exons b.introns c.promoters. d.codons e.ribozymes

b.introns

Which of the following lists of structures is in the correct order from inside to outside of the cell?

basal body, transition zone, axoneme

Steroid hormones bind to receptors inside the cell and alter their conformation. The hormone—receptor complex is then transported into the nucleus, where it can directly affect gene expression. To get from the location where the receptor binds the hormone to its site of action, the hormone—receptor complex must ________.

be transported through the nuclear pore complex

Imagine a bacterial replication fork. Synthesis of which new strand(s) would be affected by mutations in the enzyme DNA polymerase III?

both leading and lagging

3.47.The equilibrium constant for complex formation between molecules A and B will depend on their relative concentrations, as well as the rates at which the molecules associate and dissociate. The association rate will be larger than the dissociation rate when complex formation is favorable. The energy that drives this process is referred to as ___________. (a) dissociation energy. (b) association energy. (c) binding energy. (d) releasing energy.

c

3.66. Energy cannot be created or destroyed, but it can be converted into other types of energy. Cells use potential kinetic energy to generate stored chemical energy in the form of activated carrier molecules, which are often employed to join two molecules together in _____________ reactions. (a) oxidation (b) hydrolysis (c) condensation (d) reduction

c

Electron Transport Chain. (ETC)

chain of proteins that move electrons from protein to protein to protein

Where does photosynthesis take place?

chloroplasts

3.33.ΔG measures the change of free energy in a system as it converts reactant (Y) into product (X). When [Y] =[X], ΔG is equal to _____________. (a) ΔG° + RT (b) RT (c) ln [X]/[Y] (d) ΔG°

d

The region of the tRNA that is complementary to a codon in mRNA is a. the acceptor stem b.the codon c.the peptidyl site d.the anticodon e.the adaptor loop.

d.the anticodon

The advantage to the cell of the gradual oxidation of glucose during cellular respiration compared with its combustion to CO2 and H2O in a single step is that ________________.

energy can be extracted in usable amounts.

3.26.Catalysts are molecules that lower the activation energy for a given reaction. Cells produce their own catalysts called _____________. (a) proteins. (b) enzymes. (c) cofactors. (d) complexes.

enzymes

2.19.b. true or false: there is no limit to the number of electrons that can occupy the fourth electron shell

false, the fourth electron shell has the capacity to hold 18 electrons

Proton-motive force

force formed by the pumping of hydrogen ions across a biological membrane during chemiosmosis

The processes of transcription and translation are collectively known as

gene expression.

sorts and packages macromolecules

golgi apparatus

where does transcription occur in eukaryotes?

in the nucleus

What unit of length would you generally use to measure a typical plant or animal cell?

micrometers

Where does respiration occur?

mitochondria

2.7. carbon 14 is an unstable isotope of carbon that decays very slowly. compared to the common, stable carbon 12 isotope, carbon 14 has two additional...

neutrons

Arp2/3

nucleating actin branches

2.58.h. sugar linked to a base and a phosphate

nucleotide

2.58.h. sugar linked to a base and a phosphate is called

nucleotide

17. by definition, prokaryotic cells do not possess a...

nucleus

2.43.f. inorganic or organice: adenosine

o

mitochondria

only in eukaryotes

stoma

passage of gases and water vapor into and out of the leaf

F-actin filaments ARE

polymerized G-actin monomers wound around each other in a helix.

Instead of studying one or two proteins or protein complexes present in the cell at any given time, we can now look at a snapshot of all proteins being expressed in cells being grown in specific conditions. This large-scale, systematic approach to the study of proteins is called _______________.

proteomics.

2.3. which subatomic particles contribute to the atomic number for any given element?

protons

3.27.a. By definition, catalysis allows a reaction to occur more BLANK.

rapidly

photosynethis

reactions that light to produce to glucose

The stimulation of auditory nerves depends on the opening and closing of channels in the auditory hair cells. Which type of gating mechanism do these cells use?

stress-gated

If they had measured after a third round of replication, what would they have observed?

three times as much light as hybrid DNA

The type of cellular junction prominent in the stomach and kidney is the A) plasmodesmata. B) gap junction. C) adhesive junction. D) tight junction. E) laminin junction.

tight junction

Paracellular transport is associated with which of the following? A) adherens junctions B) integrins C) desmosomes D) tight junctions E) hemidesmosomes

tight junctions

create a permeability barrier

tight junctions

xylem

transport of water and minerals

One of the most important roles of proteoglycans is to trap ________.

water

Figure Q12-56 illustrates changes in membrane potential during the formation of an action potential. What membrane characteristic or measurement used to study action potentials is indicated by the arrow? (a) effect of a depolarizing stimulus (b) resting membrane potential (c) threshold potential (d) action potential

(a)

Pumps are transporters that are able to harness energy provided by other components in the cells to drive the movement of solutes across membranes, against their concentration gradient. This type of transport is called _____________. (a) active transport. (b) free diffusion. (c) facilitated diffusion. (d) passive transport.

(a)

The endothelial cells found closest to the site of an infection express proteins called lectins. Each lectin binds to a particular ____________ that is presented on the surface of a target cell. (a) oligosaccharide (b) aminophospholipid (c) polysaccharide (d) sphingolipid

(a)

The lateral movement of transmembrane proteins can be restricted by several different mechanisms. Which mechanism best describes the process by which a budding yeast cell designates the site of new bud formation during cell division? (a) proteins are tethered to the cell cortex (b) proteins are tethered to the extracellular matrix (c) proteins are tethered to the proteins on the surface of another cell (d) protein movement is limited by the presence of a diffusion barrier

(a)

The stimulation of a motor neuron ultimately results in the release of a neurotransmitter at the synapse between the neuron and a muscle cell. What type of neurotransmitter is used at these neuromuscular junctions? (a) acetylcholine (b) glutamate (c) GABA (d) glycine

(a)

15-21 Which of the following statements about peroxisomes is false? (a)Most peroxisomal proteins are synthesized in the ER. (b)Peroxisomes synthesize phospholipids for the myelin sheath. (c)Peroxisomes produce hydrogen peroxide. (d)Vesicles that bud from the ER can mature into peroxisomes.

(a) Although peroxisomes can get some membrane-embedded proteins from the ER, most peroxisomal proteins are imported from the cytosol.

15-61Which of the following statements about phagocytic cells in animals is false? (a)Phagocytic cells are important in the gut to take up large particles of food. (b)Phagocytic cells scavenge dead and damaged cells and cell debris. (c)Phagocytic cells can engulf invading microorganisms and deliver them to their lysosomes for destruction. (d)Phagocytic cells extend pseudopods that surround the material to be ingested.

(a) Although some unicellular eukaryotes ingest food particles by phagocytosis, phagocytosis is not involved in digestion in the animal gut.

Which of the following is not a feature commonly observed in alpha helices? (a) left-handedness (b) one helical turn every 3.6 amino acids (c) cylindrical shape (d) amino acid side chains that point outward

(a) left-handedness

In the photosynthetic archaean Halobacterium halobium, a membrane transport protein called bacteriorhodopsin captures energy from sunlight and uses it to pump protons out of the cell. The resulting proton gradient serves as an energy store that can later be tapped to generate ATP. Which statement best describes how bacteriorhodopsin operates? (a) The absorption of sunlight triggers a contraction of the β barrel that acts as the protein's central channel, squeezing a proton out of the cell. (b) The absorption of sunlight triggers a shift in the conformation of the protein's seven, membrane spanning α helices, allowing a proton to leave the cell. (c) The absorption of sunlight triggers a restructuring of bacteriorhodopsin's otherwise unstructured core to form the channel through which a proton can exit the cell. (d) The absorption of sunlight triggers the activation of an enzyme that generates ATP.

(b)

The plasma membrane serves many functions, many of which depend on the presence of specialized membrane proteins. Which of the following roles of the plasma membrane could still occur if the bilayer were lacking these proteins? (a) intercellular communication (b) selective permeability (c) cellular movement (d) import/export of molecules

(b)

We know the detailed molecular structure and mechanism of action of the transmembrane protein bacteriorhodopsin. This protein uses sunlight as the source of energy to pump ______ out of the cell. (a) ATP (b) H+ (c) K+ (d) Na+

(b)

Which of the following conditions is likely to decrease the likelihood of skeletal muscle contraction? (a) partial depolarization of the T-tubule membrane, such that the resting potential is closer to zero (b) addition of a drug that blocks Ca2+ binding to troponin (c) an increase in the amount of ATP in the cell (d) a mutation in tropomyosin that decreases its affinity for the actin filament

(b) Ca2+ binding to troponin leads to a conformational change that causes a movement in tropomyosin so that myosin can bind to actin to initiate contraction. Thus, if troponin cannot bind Ca2+, the likelihood of contraction decreases. Partial depolarization of the T-tubule membrane will make it easier to depolarize the membrane, increasing the likelihood of muscle contraction [choice (a)]. ATP is required for myosin movement, so increasing the amount of ATP in the cell will not decrease contraction [choice (c)]. Tropomyosin normally binds to actin and blocks myosin binding, so a mutation in tropomyosin that decreases its affinity for actin should not decrease the likelihood of muscle contraction [choice (d)].

If Na+ channels are opened in a cell that was previously at rest, how will the resting membrane potential be affected? (a) The membrane potential is not affected by Na+ (b) It becomes more negative. (c) It becomes more positive. (d) It is permanently reset.

(c) As Na+ ions move into the cell, the net charge becomes more positive (less negative) and the membrane potential changes to reflect the Co/Ci for both Na+ and K+ ions.

14-58Stage 2 of photosynthesis, sometimes referred to as the dark reactions, involves the reduction of CO2 to produce organic compounds such as sucrose. What cofactor is the electron donor for carbon fixation? (a)H2O (b)NADH (c)FADH2 (d)NADPH

(d) NADPH

Step 3 in glycolysis requires the activity of phosphofructokinase to convert fructose 6-phosphate into fructose 1,6-bisphosphate. Which of the following molecules is an allosteric inhibitor of this enzyme? (a) Pi (b) AMP (c) ADP (d) ATP

(d) When ATP levels are high, the cell does not need to break down more glucose to generate ATP. Thus, with ATP acting as an allosteric inhibitor of a key glycolytic step, there is a rapid on-off switch for this pathway.

14-17Electron transport is coupled to ATP synthesis in mitochondria, in chloroplasts, and in the thermophilic bacterium Methanococcus. Which of the following is likely to affect the coupling of electron transport to ATP synthesis in allof these systems? (a)a potent inhibitor of cytochrome c oxidase (b)the removal of oxygen (c)the absence of light (d)an ADP analog that inhibits ATP synthase

(d) an ADP analog that inhibits ATP synthase

Chloroplasts are found only in eukaryotic cells that carry out photosynthesis: plants and algae. Plants and algae appear green as a result of the presence of chlorophyll. Where is chlorophyll located in the chloroplast?

(d) in the third, innermost membrane D

2.28.b. if .5 kilocalories of energy are required to break 6x10^23 bonds of a particular type, what is the strength of this bond?

.5 kcal/mole

If the isotope 32S has 16 protons and 16 neutrons, how many protons, neutrons, and electrons will the isotope 35S have, respectively?

16, 19, 16

An atom of oxygen has an atomic number of 8. How many electrons are in the first, second, and third electron shells, respectively?

2, 6, 0

Arrange the following molecules with respect to their relative levels of oxidation from the least oxidized (first) to the most oxidized (last). 1: CH2O (formaldehyde) 2: CH4 (methane) 3: CHOOH (formic acid) 4: CH3OH (methanol) 5: CO2 (carbon dioxide)

2,4,1,3,5

How many nucleotides are contained in a single codon? 1 3 4 6 9

3

2.50.b. how many hydrogens does figure q2-50 have?

31 hydrogen atoms

C

91) One of the human leukemias, called CML (chronic myelogenous leukemia) is associated with a chromosomal translocation between chromosomes 9 and 22 in somatic cells of bone marrow. Which of the following allows CML to provide further evidence of this multi-step nature of cancer? A) CML usually occurs in more elderly persons (late age of onset). B) The resulting chromosome 22 is abnormally short; it is then known as the Philadelphia chromosome. C) The translocation requires breaks in both chromosomes 9 and 22, followed by fusion between the reciprocal pieces. D) CML involves a proto-oncogene known as abl. E) CML can usually be treated by chemotherapy.

11) Which component is a protein fiber of the extracellular matrix?

A

The ATP made during glycolysis is generated by A) substrate-level phosphorylation B) electron transport C) photophosphorylation. D) chemiosmosis. E) oxidation of NADH to NAD+.

A

During a normal translation cycle, where would you expect to find a tRNA attached to a single amino acid?

A site

Reaction rate formula

A540/ reaction time

Each codon shown below specifies an amino acid. For which one is it possible that a change in a single base could create a stop codon? CAC ACA AAA ACC

AAA

Which of these is an example of carrier-mediated transport?

Active transport/facilitated diffusion

ATP stands for

Adenosine Triphosphate

Which of the following statements is true regarding cell-cell adhesion? A) Adhesion is coordinated with many cellular processes except cell survival. B) Adhesion molecules include plakoglobulin, which cements two cells together. C) Adherens junctions are exclusive to plant cells.D) Adhesive junctions are dynamic in response to cellular signals.

Adhesive junctions are dynamic in response to cellular signals.

What changes did you notice in the Elodea cells between the different environments?

After the NaCl was added, the plasma membrane and cytoplasm become separated from the cell wall. This is because the cells were in a hypertonic solution

________ enables a single gene to encode two or more polypeptides that are different in their amino acid sequence.

Alternative splicing

________ enables a single gene to encode two... What process enables a single gene to encode two or more polypeptides that are different in their amino acid sequence. Reverse transcription Self-splicing Capping Alternative splicing Regulatory splicing

Alternative splicing

Several different classes of enzymes are needed for the catabolism of carbohydrates. Which of the following descriptions best matches the function of a dehydrogenase?

An enzyme that catalyzes the oxidation of a molecule by removing a hydride ion.

The molecule that functions as the reducing agent (electron donor) in a redox or oxidation-reduction reaction A) gains electrons and gains energy. B) loses electrons and loses energy. C) gains electrons and loses energy. D) loses electrons and gains energy. E) neither gains nor loses electrons, but gains or loses energy

B

What is the term for metabolic pathways that release stored energy by breaking down complex molecules? A) anabolic pathways B) catabolic pathways C) fermentation pathways D) thermodynamic pathways E) bioenergetic pathways

B

14-19Osmosis describes the movement of water across a biological membrane and down its concentration gradient. In chemiosmosis, useful energy is harnessed by the cell from the movement of _______________ across the inner mitochondrial membrane into the matrix _________________ a concentration gradient. (a)ATP, against (b)protons, down (c)electrons, down (d)ADP, against

B Protons, down

The template strand of a given gene includes the sequence 3'-GCCACGTATCA-5'. What is the sequence of the nontemplate strand? A) 3'-CGGTGCATAGT-5' B) 5'-CGGTGCATAGT-3' C) 5'-CGGUGCAUAGU-3' D) 3'-CGGUGCAUAGU-5'

B) 5'-CGGTGCATAGT-3'

After transcription begins, several steps must be completed before the fully processed mRNA is ready to be used as a template for protein synthesis on the ribosomes. Which three statements correctly describe the processing that takes place before a mature mRNA exits the nucleus? A. A translation stop codon is added at the 3' end of the pre-mRNA. B. A poly-A tail (50-250 adenine nucleotides) is added to the 3' end of the pre-mRNA. C. Coding sequences called exons are spliced out by ribosomes. D. Noncoding sequences called introns are spliced out by molecular complexes called spliceosomes. E. A cap consisting of a modified guanine nucleotide is added to the 5' end of the pre-mRNA.

B, D, E

65. The antibiotic streptomycin inhibits protein synthesis in bacteria. If this antibiotic is added to a culture of animal cells, protein synthesis in the cytosol continues normally. However, over time, the population of mitochondria in the cell becomes depleted. Specifically, it is observed that the protein-synthesis machinery inside the mitochondria is inhibited.B. What do you expect to observe if, in a new experiment, animal cells are treated with diphtheria toxin, a compound that is known to block cytosolic protein synthesis but does not have any impact on bacterial growth?

B. We would expect that although cytosolic protein synthesis would stop, mitochondrial protein synthesis should still occur normally (at least for a little while). This result would lend further support to the idea that mitochondria are derived from a noneukaryotic organism. If this were not the case, these compounds would be expected to affect protein synthesis at both locations.

When a glucose molecule loses a hydrogen atom as the result of an oxidation-reduction reaction, the molecule becomes A) dehydrogenated. B) hydrogenated. C) oxidized. D) reduced. E) an oxidizing agent.

C

Where do the catabolic products of fatty acid breakdown enter into the citric acid cycle? A) pyruvate B) malate or fumarate C) acetyl CoA D) -ketoglutarate E) succinyl CoA

C

Which of the following types of mutations can result in a frameshift? A) nucleotide-pair substitution in a coding region B) nucleotide-pair substitution in a noncoding region C) 1 nucleotide-pair insertion in a coding region D) 1 nucleotide-pair insertion in a noncoding region E) 3 nucleotide-pair insertion in a coding region

C) 1 nucleotide-pair insertion in a coding region

The nontemplate strand of a portion of a gene reads 5'-TTCACTGGTTCA. What is the sequence of the resulting transcript for this portion? A) 5'-AAGUGACCAAGU B) 5'-UGAACCAGUGAA C) 5'-UUCACYGGUUCA D)5'-ACUUGGUCACTT

C) 5'-UUCACYGGUUCA

7) Which of the following statements is FALSE?

C) A plant cell continues growing after the secondary cell wall forms but stops growing with the formation of the tertiary cell wall.

During a normal translation cycle, where would you expect to find a tRNA attached to a single amino acid? A) E site B) P site C) A site D) alternating between tRNAs attaches to the P site and the A site

C) A site

26) Different body cells can respond differently to the same peptide hormones because

C) a target cell's response is determined by the components of its signal transduction pathways

What kind of molecules can be transcription factors? A) DNA and RNA B) RNA and proteins C) proteins D) lipids E) lipids and carbohydrates

C) proteins

3.43.c. Consider the reaction XY in a cell at 37°C. At equilibrium, the concentrations of X and Y are 50 μM and 5 μM, respectively. Use this information and the equations below to answer questions A-E. ΔG° = -0.616 ln Keq ΔG = ΔG° + 0.616 ln [Y]/[X] Recall that the natural log of a number z will have a negative value when z < 1, positive when z > 1, and 0 when z = 1. What is the value of the standard free energy? Refer to Table 3-1 in the textbook or use a calculator.

C. ΔG° = -0.616 ln Keq = -0.616 ln 0.1 = (-0.616) (-2.3) = 1.4 kcal/mole.

What is the chemical formula for cellular respiration?

C6H12O6 + 6O2 yields 6CO2 + 6H2O + Energy (ATP + Heat); glucose + oxygen yields carbon dioxide + water + energy in the form of ATP and Heat

The starting product of glycolysis is the six-carbon sugar _______, and the ending product is two ______ carbon compounds termed _____.

C6H12O6, organic, pyruvate

2.38.c. completer the following reaction: CH3COOH + H2O <-> BLANK

CH3COO- + H30+

________ are the adhesive glycoproteins associated with most animal cells that function in calcium-mediated cell-cell recognition and adhesion.

Cadherins

Explain what is meant by C4 plant?

Calvin cycle is preceded by reactions that incorporate CO2 into 4 carbon component whose end product supplies CO2 for the Calvin cycle.

The carbons in the citric acid cycle are lost as the molecule _________.

Carbon dioxide

________proteins interact with substances by binding to them and changing shape to regulate the passage of materials through the cell membrane.

Carrier

Explain the role of PEP carboxylase in C4 plants.

Catalyzes the first step in the C4 pathway when CO2 is added, forming oxaloacetate.

Cellular reproduction is known as_____.

Cell division

What is a concept of the cell theory?

Cells are the basic structural unit of life

Foreign substances like nicotine, morphine, and menthol exert their initial effects by _____. (a) killing cells immediately, exerting their physiological effects by causing cell death. (b) diffusing through cell plasma membranes and binding to transcription factors to change gene expression. (c) interacting with cell-surface receptors, causing the receptors to transduce signal inappropriately in the absence of the normal stimulus. (d) removing cell-surface receptors from the plasma membrane.

Choice (c) is correct. Although some foreign substances will remove cell-surface receptors from the plasma membrane [choice (d)], this is a long-term response and not part of the initial response.

Keratins, neurofilaments, and vimentins are all categories of intermediate filaments. Which of the following properties is not true of these types of intermediate filaments? (a) They strengthen cells against mechanical stress. (b) Dimers associate by noncovalent bonding to form a tetramer. (c) They are found in the cytoplasm. (d) Phosphorylation causes disassembly during every mitotic cycle.

Choice (d) is untrue. Keratins, neurofilaments, and vimentins are cytoplasmic intermediate filaments [choice (c)], which tend to be very stable once formed. The nuclear intermediate filaments are disassembled and reformed during mitosis; this process is regulated by phosphorylation.

unsaturated fatty acids are liquid at room temperature so they freeze at much lower temperatures compared to saturated fatty acids

Compared to tropical fish, arctic fish oils have more unsaturated fatty acids or saturated fatty acids?

Structurally, the plasma membrane

Contains some imbedded proteins/is composed of a bilayer of lipids/contains some carbohydrate molecules/contains some small holes, or pores

Starting with citrate, which of the following combinations of products would result from three turns of the citric acid cycle? A) 1 ATP, 2 CO2, 3 NADH, and 1 FADH2 B) 2 ATP, 2 CO2, 1 NADH, and 3 FADH2 C) 3 ATP, 3 CO2, 3 NADH, and 3 FADH2 D) 3 ATP, 6 CO2, 9 NADH, and 3 FADH2 E) 38 ATP, 6 CO2, 3 NADH, and 12 FADH2

D

In a long, random sequence of A,C, G, or U, how often should a stop codon appear? A) almost never B) 1/3 C) 1/64 D) 3/64

D) 3/64

24) Pemphigus vulgaris is an autoimmune disorder in humans in which antibodies are produced against the cadherins of desmosomes. The blistering of the skin and mucous membranes characteristic of this disorder is probably a result of ________.

D) a loss in cell—cell adhesion

Hormones are chemical substances produced in one organ that are released into the bloodstream and affect the function of a target organ. For the target organ to respond to a particular hormone, it must ________.

D) have receptors that recognize and bind the hormone molecule

3.c. Similarly, the genetic information of all cells is stored in their BLANK.

DNA

Genotype is to _____ as phenotype is to _____. heredity; DNA base sequence gene regulation; translation transcription; amino acid sequence DNA base sequence; physical traits that are products of the proteins produced

DNA base sequence; physical traits that are products of the proteins produced

What are the steps (in the correct order) that link a change in the base sequence of a gene to a change in the phenotype of an organism like a mouse or a human? DNA—>Protein—>RNA RNA—>DNA—>Protein DNA—>RNA—>Protein Protein—>RNA—>DNA

DNA-RNA-Protein

Which of the following is the strongest evidence that protein structure and function are correlated? Proteins function best at certain temperatures. Denatured (unfolded) proteins do not function normally. Enzymes tend to be globular in shape. Proteins have four distinct levels of structure and many functions.

Denatured (unfolded) proteins do not function normally

During cellular respiration, acetyl CoA accumulates in which location? A) cytosol B) mitochondrial outer membrane C) mitochondrial inner membrane D) mitochondrial intermembrane space E) mitochondrial matrix

E

For each molecule of glucose that is metabolized by glycolysis and the citric acid cycle, what is the total number of NADH + FADH2 molecules produced? A) 4 B) 5 C) 6 D) 10 E) 12

E

4. How does cellular specialization serve multicellular organisms and how might a high degree of specialization be detrimental?

Each type of cells in a multicellular organism, relies on the other cells types for survival, which is the specialization component of cells that creates the division of labor between cells. This specialization of cells is essential for the organism as a whole, because if one type of specialized cells were to be removed, then that lack of capabilities would discontinue the ability to generate offspring and would not live very long.

During active transport, a cell must expend ________ to accomplish the movement of a substance.

Energy

Each member of the ETC is lower in free ____ than the preceding member of the chain, but higher in _______. The molecule at zero free energy, which is _____, is lowest of all the molecules in free energy and highest in electronegativity.

Energy, electronegativity, O2

3.60.Chemical reactions are reversible; they can proceed in both the forward and reverse directions. If the ΔG° for the reaction YX is energetically favorable, how can you explain the fact that not all of the Y molecules will be converted to X molecules?

Even when the forward reaction is highly favorable, it is important to keep in mind that molecules exist as part of a population, and each member of a given population has a varying level of energy per molecule. Statistically speaking, there will always be some molecules that have sufficient energy to reach the energy of activation for the back reaction XY, even though the proportion of molecules with this energy will be much lower than that for the forward reaction YX. As more and more X molecules are converted to Y molecules, eventually the Y molecules in the mixture outnumber the X molecules to such a large extent that the fluxes in the backward and forward directions become equal; it is here that the reaction reaches its equilibrium point.

Which of the following terms does not directly pertain to translation? A) anticodon B) tRNA C) aminoacyl-tRNA synthetase D) polypeptide E) rRNA F) DNA

F) DNA

The movement of substances from an area of high concentration to an area of low concentration by interacting with a carrier molecule is an example of

Facilitated diffusion

19.a. true or false: the terms "prokaryote" and "bacterium" are synonyms

False. archaea are significantly different from bacteria

The carbohydrate produced directly from the Calvin cycle is not glucose, but the three-carbon compound ______. Each turn of the Calvin cycle fixes one molecule of CO2; therefore it will take ____ turns of the calvin cycle to net one G3P

G3P,3

Pumps require energy for transport, but channels and carriers do not use energy.

How does a pump differ from a channel or a carrier?

Write a prediction for the cuts you made concerning the regeneration of the planarians if the diffusion gradient model is correct (if/then).

If the diffusion gradient model is correct, then the planaria will regenerate the missing parts of their body. The result will be a fully formed, normal planarian.

Which of the following classes of molecules serve primarily to link proteins embedded in the plasma membrane with structural proteins in the ECM?

Integrins

________ indirectly link the extracellular matrix and the cytoskeleton of animal cells

Integrins

Which of the following statements about RNA processing in eukaryotes is INCORRECT?

Introns are simply excised out of pre-mRNA to produce the mature mRNA.

Which of the following is true regarding the basal lamina?

Laminin in the basal lamina is a frequent ligand of integrins

Explain the relationship between wavelength and energy.

Longer wavelength means less energy, shorter wavelength means higher energy.

Do you expect the cell to produce more ATP from one glucose molecule or from one fatty acid molecule? Explain your answer.

More ATP is produced from fat catabolism than from glucose catabolism. Most ATP is generated in the mitochondria, and the amount depends on the production of the NADH and FADH2 cofactors in the Krebs cycle. The Krebs cycle relies on the input of acetyl CoA. Each glucose molecule can be converted into two acetyl CoA molecules. A molecule of fat will have three fatty acid chains, with an average length of 12-16 carbons. Even if we assume very short fatty acid chains of six carbons each (the length of a glucose molecule), this would mean the production of three acetyl CoA molecules per chain, and nine total for the triacylglycerol.

Carbon-14 dating works for fossils up to about 75,000 years old. Most dinosaurs went extinct 65.5 million years ago. Can 14C be used to date dinosaur bones?

No; the 14C present in the dinosaur when it died would decay too much to be measured after 65.5 million years

Using O18 as the basis of your discussion, explain how we know that oxygen released in photosynthesis comes from water?

O18 is a heavy isotope that was used as a tracer, and only when the H20 was the source of O18 would the O2 released be labeled with the isotope.

Which of the following steps or processes in aerobic respiration include the production of carbon dioxide? (a) breakdown of glycogen (b) glycolysis (c) conversion of pyruvate to acetyl CoA (d) oxidative phosphorylation

Only choice (c) is correct. One carbon is oxidized and released as carbon dioxide when pyruvate is converted to acetyl CoA.

A ________ bond holds the amino acids of a growing polypeptide chain together during the elongation stage of translation.

Peptide

Self-splicing

RNA itself can catalyze the removal of its own intron.portions of te RNA act like an enzyme to cleave the covalent bonds at the intron-exon boundaries and connect the exons together.

The classes of membrane proteins that recognize extracellular structures are called

Receptor proteins

What is a function of the cell membrane?

Sensitivity to changes in the extracellular fluid

Voltage-gated channels contain charged protein domains, which are sensitive to changes in membrane potential. By responding to a threshold in the membrane potential, these voltage sensors trigger the opening of the channels. Which of the following best describes the behavior of a population of channels exposed to such a threshold?

Some channels remain closed and some open completely.

Fill in Table Q12-23. In the "Type of transport" column, designate whether the transporter works by uniport, symport, or antiport mechanisms.

Table A12-23

_________ causes microtubules to form tight bundles in nerve cells.

Tau

Which agar block would have the most efficient surface area-to-volume ratio?

The 3cm agar block would be more efficient because it has a surface area to volume ration of 2:1

Which describes the hydrolysis of ATP.why is ATP needed to charge a tRNA?

The attachment of an amino acid to a tRNA is an endergonic reaction.ATP provides the energy to catalyze this reaction.

3.43.e. Consider the reaction XY in a cell at 37°C. At equilibrium, the concentrations of X and Y are 50 μM and 5 μM, respectively. Use this information and the equations below to answer questions A-E. ΔG° = -0.616 ln Keq ΔG = ΔG° + 0.616 ln [Y]/[X] Recall that the natural log of a number z will have a negative value when z < 1, positive when z > 1, and 0 when z = 1. Imagine starting conditions in which the reaction XY is unfavorable, yet the cell needs to produce more Y. Describe two ways in which this may be accomplished.

The cell may directly couple the unfavorable reaction to a second, energetically favorable reaction whose negative ΔG has a value larger than the positive ΔG of the XY reaction; the coupled reaction will have a ΔG equal to the sum of the component reactions. Alternatively, more X will be converted to Y if the concentration of Y drops; this may happen if Y is converted to Z in a second reaction or if Y is exported from the cell or compartment where the XY reaction occurs.

Explain what you observed when Elodea was placed in a tube containing dissolved CO2 and phenol red.

The color changed from yellow to pink because the plant used CO2 dissolved in the water which lowered the pH.

Scientists learned that cell death is a normal and even important part of life by studying the development of the nematode worm C. elegans. What was the most important feature of C. elegans for the study of programmed cell death?

The developmental pathway of each cell in the adult worm was known.

Nitrogenous bases are paired in specific combinations. Which of the following does not provide evidence to support this conclusion?

The pairs of nitrogenous bases are held together by hydrogen bonds.

What were the results of the nitrogen in the tomato plants?

The plant changed a medium amount in the weight and length, overall was healthy with green leaves and long roots

a significant component of hydrocarbons

The three types of lipids found in cells (fats, steroids, and phospholipids) are insoluble in water because they all possess _____.

Which of the following is *not* a function provided by the ECM?

To provide plasma membrane support.

Adheren junctions and occluding junctions are not long term.

True

Most cells must adhere to survive but a few cells are free.

True

End codons/stop condons/termination codons

UAA UAG UGA

phopholipids

What are lipids and proteins considered to be?

integral: span the membrane with segments both inside and outside of the cell peripheral: loosely bound to surface of membrane (often interact with exposed surface of integral proteins)

What are the 2 major types of membrane proteins?

omega-3

What is an essential fatty acid?

cis is bent trans is straight

What is the difference between a cis transfiguration and a trans configuration?

passive transport (no energy)

What kind of transport is diffusion and facilitated diffusion considered?

ions and polar molecules

What kinds of molecules cannot easily pass through the membrane?

presence of aquaporins

What would increase the rate of osmosis?

they are all hydrophobic

Why re lipids typically all grouped together?

Does the distribution of bases in sea urchin DNA and salmon DNA follow Chargaff's rules?

Yes, because the %A approximately equals the %T and the %G approximately equals the %C in both species.

Radioactive uranium-235 has a half-life of 704 million years. If it was incorporated into dinosaur bones, could it be used to date the dinosaur fossils?

Yes; after 65.5 million years, only about one-tenth of the 235U would have decayed, leaving plenty to measure in the fossils

3.20.Arrange the following molecules in order with respect to their relative levels of oxidation (assign 5 to the most oxidized and 1 to the most reduced). _______ CH2O (formaldehyde) _______ CH4 (methane) _______ CHOOH (formic acid) _______ CH3OH (methanol) _______ CO2 (carbon dioxide)

__3__ CH2O (formaldehyde) __1__ CH4 (methane) __4__ CHOOH (formic acid) __2__ CH3OH (methanol) __5__ CO2 (carbon dioxide)

Foods are broken down into simple molecular subunits for distribution and use throughout the body. Which type of simple subunits, listed below, is used preferentially as an energy source? (a) simple sugars (b) proteins (c) free fatty acids (d) glycerol

a

18. although there are many distinct prokaryotic species, most have a small range of shapes, sizes, and growth rates. Which of the following characteristics are NOT observed in prokaryotes? a. a highly structured cytoplasm b. endoplasmic reticulum c. the ability to divide rapidly d. a cell wall

a highly structured cytoplasm endoplasmic reticulum

During RNA processing a(n) _____ is added to the 3' end of the RNA. 3' untranslated region a long string of adenine nucleotides 5' untranslated region coding segment modified guanine nucleotide

a long string of adenine nucleotides

3.17. Oxidation is the process by which oxygen atoms are added to a target molecule. Generally, the atom that is oxidized will experience which of the following with respect to the electrons in its outer shell? (a) a net gain (b) a net loss (c) no change (d) an equal sharing

a net loss

Oxidation is the process by which oxygen atoms are added to a target molecule. Generally, the atom that is oxidized will experience which of the following with respect to the electrons in its outer shell?

a net loss

Protein structures have several different levels of organization. The primary structure of a protein is its amino acid sequence. The secondary and tertiary structures are more complicated. Consider the definitions below and select the one that best fits the term "protein domain."

a protein segment that folds independently

Coding sequence

a region that specified the linear amino acid sequence of a polypeptide.

Which of the following food items is like a compound?

a sandwich containing ham, lettuce, and tomato

The stimulation of a motor neuron ultimately results in the release of a neurotransmitter at the synapse between the neuron and a muscle cell. What type of neurotransmitter is used at these neuromuscular junctions?

acetylcholine

Pumps are transporters that are able to harness energy provided by other components in the cells to drive the movement of solutes across membranes, against their concentration gradient. This type of transport is called _____________.

active transport.

Match the target of the G protein with the appropriate signaling outcome. adenylyl cyclase ________ A. cleavage of inositol phospholipids ion channels _________ B. increase in cAMP levels phospholipase C _________ C. changes in membrane potential

adenylyl cyclase ___B_____ ion channels ___C_____ phospholipase C ___A_____

Plasmodesmata are cell—cell junctions that are found between ________.

adjacent plant cells

Describe the function of the diaphragm

adjusts the contrast and light on the slide being observed

air spaces

allows air to come in contact with cells

6. Proteins are important architectural and catalytic components within the cell, helping to determine its chemistry, its shape, and its ability to respond to changes in the environment. Remarkably, all of the different proteins in a cell are made from the same 20 BLANK. By linking them in different sequences, the cell can make protein molecules with different conformations and surface chemistries and therefore different functions.

amino acids

What enzyme catalyzes the attachment of amino acids to tRNA molecules?

aminoacyl-tRNA synthetase

Which of the following channels would not be expected to generate a change in voltage by movement of its substrate across the membrane where it is found?

an aquaporin

Axonemal microtubules

are organized around a central shaft or axoneme.

Lamellipodia

are thin sheets of cytoplasm use for cell movement.

2.6. atomic number and weight of carbon

atomic number=6 atomic weight=12

Several different classes of enzymes are needed for the catabolism of carbohydrates. Which of the following descriptions best matches the function of a mutase? (a) An enzyme that catalyzes the rearrangement of bonds within a single molecule. (b) An enzyme that catalyzes a change in the position of a specific chemical group within a single molecule. (c) An enzyme that catalyzes the oxidation of a molecule by removing a hydride ion. (d) An enzyme that catalyzes the addition of phosphate groups to other molecules.

b

Step 6 of the citric acid cycle is catalyzed by succinate dehydrogenase. Keeping in mind that dehydrogenases catalyze redox reactions, which are the products of the reaction in which succinate is oxidized? (a) fumarate, NADH (b) fumarate, FADH2 (c) fumarate, FADH2 (d) succinyl CoA, NADH

b

Steps 7 and 10 of glycolysis result in substrate-level phosphorylation. Which of the following best describes this process? (a) ATP is being hydrolyzed to phosphorylate the substrate. (b) The energy derived from substrate oxidation is coupled to the conversion of ADP to ATP. (c) Two successive phosphates are transferred, first to AMP, then to ADP, finally forming ATP. (d) The substrate is hydrolyzed using ATP as an energy source.

b

The concentration of H+ ions inside the mitochondrial matrix is lower than it is in the cytosol or the mitochondrial intermembrane space. What would be the immediate effect of a membrane-permeable compound that carries and releases protons into the mitochondrial matrix? (a) inhibition of the electron-transport chain (b) inhibition of ATP synthesis (c) increased import of ADP into the matrix (d) inhibition of the citric acid cycle

b

The oxygen-dependent reactions required for cellular respiration were originally thought to occur in a linear pathway. By using a competitive inhibitor for one enzyme in the pathway, investigators discovered that these reactions occur in a cycle. Which product in the reaction pathway builds up when the inhibitor is added? (a) citrate (b) succinate (c) fumarate (d) malate

b

Which of the following is the best example of gene expression? a)A frog adapts to variation in its environmental temperature b)Mouse fur color results from pigment formed by gene-encoded enzymes. c)DNA is replicated during the S phase of the cell cycle. d)The percent of A versus a alleles in a population is altered by natural selection. e)Mutation alters the sequence of a region of DNA.

b

Which of the following polymers of glucose is used as a vehicle to store energy reserves in animal cells? (a) glucagon (b) glycogen (c) starch (d) glycerol

b

11. which statement is NOT true about the events/conclusions from studies during the mid- 1800s surrounding the discovery of the cells? a. cells came to be known as the smallest universal building block of living organisms b. scientists came to the conclusion that new cels can form spontaneously from the remnants of ruptured cells. c. light microscopy was essential in demonstrating the commonalities between plan and animal tissues. d. new cells arise from the growth and division of previously existing cells.

b. scientists came to the conclusion that new cells can form spontaneously from the remnants of ruptured cells.

The synthesis of a polypeptide occurs during which stage of translation? a.initiation b.elgongation c.termination d.splicing

b.elgongation

What types of viruses were used in research

bacteriophages (or phages), are widely used in molecular genetics research

Melanin

black pigment

3.36. The equilibrium constant (K) for the reaction YX can be expressed with respect to the concentrations of the reactant and product molecules. Which of the expressions below shows the correct relationship between K, [Y], and [X]? (a) K = [Y]/[X] (b) K = [Y] * [X] (c) K = [X]/[Y] (d) K = [X] - [Y]

c

Glyceraldehyde 3-phosphate dehydrogenase operates by stripping a hydride ion from its substrate. Which molecule is the recipient of the proton and two electrons during this transfer? (a) oxygen (b) acetyl CoA (c) NAD+ (d) FADH

c

The____ is the site where the translation process takes place. a.mitochondria b.nucleus c.ribosome d.lysosome e.ribozyme

c.ribosome

inducible operons usually function as part of ___________ pathways

catabolic

axoneme

central shaft

The minus (-) ends of microtubules are often anchored at the (), such that the dynamics associated with microtubules is at the plus (+) ends.

centrosome

Velocity

change of absorbance (A540)/ change in time

carrier proteins

change shape to shuttle molecules across the membrane

3.4.The energy used by the cell to generate specific biological molecules and highly ordered structures is stored in the form of _____________.

chemical bonds

In which of the following organs would you expect to find adhesive junctions in humans? A) heart muscle B) skin epithelium C) uterus D) both choices A and B E) choices A, B, and C

choices A, B, and C

The three most common glycosaminoglycans (GAGs) are ________, ________, and ________.

chondroitin sulfate, keratan sulfate, hyaluronate

Which of the following is not a cellular activity associated with microtubules?

cilial structure

3.13. During respiration, energy is retrieved from the high-energy bonds found in certain organic molecules. Which of the following, in addition to energy, are the ultimate products of respiration? (a) CO2, H2O (b) CH3, H2O (c) CH2OH, O2 (d) CO2, O2

co2, h2o

blastema

collection of undifferenced cells called neoblasts- have the ability to become any type of cell the body needs

Functional protein

compsed of two or more different polypeptides

unsaturated fat

contains one or more double bonds; fewer hydrogens because of the double bonded carbons; bad fats

Which of the following is not associated with the basal lamina? A) serves as a structural support B) allows for movement of some small molecules C) can influence cell migrations in certain areas of the body D) contains collagen type IV, proteoglycans, laminins, and entactin E) contains the most abundant supply of fibronectins.

contains the most abundant supply of fibronectins

2.57. dna and rna are different types of nucleic acid polymer. which of the following is true of dna but not true of rna? a. contains uracil b. contains thymine c. is single stranded d. it has 5' to 3' directionality

contains thymine

Topoisomerase

corrects "overwinding" ahead of replication forks by breaking, swiveling, and rejoining DNA strands

The cell () is a dense network of microfilaments just beneath the plasma membrane.

cortex

transcription and translation both occur in the same location, where dies this occur and what type of cell is this ?

cytoplasm, bacteria

How do amoeba move?

cytoplasmic extensions called pseudopodia which anchor to a solid surface and pull the organism forwards

Integrins are integral membrane proteins. They are often attached to ________.

cytoskeletal proteins and proteins in the extracellular matrix

The ________ is a complex network of interconnected filaments and tubules that extends throughout the cytosol, from the nucleus to the inner surface of the plasma membrane.

cytoskeleton

3.55.The study of enzyme kinetics is usually performed with purified components and requires the characterization of several aspects of the reaction, including the rate of association with the substrate, the rate of catalysis, and _____________. (a) the enzyme's structure. (b) the optimal pH of the reaction. (c) the subcellular localization of the enzyme. (d) the regulation of the enzyme activity.

d

Osmosis

diffusion of water molecules through a semipermeable membrane

54.d.what would be the best model organism to use for the investigation of the process of: development of a multicellular tissue

drosophila

The () model was proposed by Mitchison and Kirschner to explain how polymerization and depolymerization of microtubules occur simultaneously.

dynamic instability

motor MAPs

dynein

Which of the following is a modification made to eukaryotic mRNA before it is exported to the cytosol? a)The 5′ end is capped. b)A poly-A tail is added to the 3′ end. c)Introns are removed. d)Exons are joined together. e)all of the above

e

Which of the following is mismatched with regard to similar excellular structures in plant and animal cells? A) elastins-extensins B) extracellular matrix-cell wall C) collagens-cellulose D) proteoglycans-hemicelluloses E) fibronectins-pectins

elastins-extensins

3.11.In the first stage of photosynthesis, light energy is converted into what other form of energy? (a) electrical (b) chemical (c) potential (d) kinetic

electrical kinetic

What was the result after two rounds of replication?

equal amounts of light and hybrid DNA

8. Changes in dna sequence from one generation to the next may result in offspring that are altered in fitness compared with their parents. the process of change and selection over the course of many generations is the basis of BLANK.

evolution

Changes in DNA sequence from one generation to the next may result in offspring that are altered in fitness compared with their parents. The process of change and selection over the course of many generations is the basis of __________.

evolution.

Forms of the Ras protein found in tumors usually cause which of the following?

excessive cell division

60.c. true or false: the vast majority of our genome encodes functional RNA molecules or proteins and most of the intervening DNA is nonfunctional

false, it is a relatively small proportion of our DNA that encodes RNA and protein molecules. the majority of nonencoding sequences is probably involved in critical regulatory processes

2.65.a. true or false: "nonpolar interactions" is simply another way of saying "van der Waals attractions"

false, van der waals attractions describe the general attractive forces between all atoms. the contact distance between any two nonbonded atoms is the sum of van der waals radii. nonpolar interactions are based on the exclusion of hydrophobic molecules from a hydrophilic environment

32.b. the number of mitochondria inside a cell remains constant over the life of the cell

false. mitochondria have their own division cycle and their numbers change based on the rate of division

In animal cells, adhesive glycoproteins such as ________ and ________ attach cells to the matrix.

fibronectins; laminins

type of adherens junction connecting cells with the extracellular matrix

focal contact

A peptide bond ____

forms the primary structure of proteins

where are epithelial cells found

found on the outside of animals and line interior cavities and ducts

Ring-shaped complexes in the centrosome contain ().

g-tubulin

Which protein would be a good target for drug therapy if one needed to interfere with breaking down the microfilament network?

gelsolin

An organized unit of DNA sequences that enables a segment of DNA to be transcribed into RNA and ultimately results in the formation of a functional product is called a _______. chromosome trait allele gene expression

gene

Match the basic protein functions in the left column with a specific example of that type of protein in the column on the right. ___ gene regulatory A. insulin ___ motor B. carboxylase ___ storage C. rhodopsin ___ enzyme D. hemoglobin ___ transport E. ferritin ___ structural F. myosin ___ special purpose G. green fluorescent protein ___ receptor H. tubulin ___ signal I. homeodomain proteins

gene regulatory __*I*_ motor __*F*_ storage __*E*_ enzyme __*B*_ transport __*D*_ structural __*H*_ special purpose __*G*_ receptor __*C*_ signal __*A*_

Which of the following gated ion channels are involved in inhibitory synaptic signaling?

glycine-gated Cl- channels

3.5.At first glance, it may seem that living systems are able to defy the second law of thermodynamics. However, on closer examination it becomes clear that although cells create organization from raw materials in the environment, they also contribute to disorder in the environment by releasing _____________.

heat

At first glance, it may seem that living systems are able to defy the second law of thermodynamics. However, on closer examination, it becomes clear that although cells create organization from raw materials in the environment, they also contribute to disorder in the environment by releasing _____________.

heat.

Arrange the following terms in order from smallest to largest size: nucleosome, looped domain, metaphase chromosome, histone, 30-nm fiber.

histone, nucleosome, 30-nm fiber, looped domain, metaphase chromosome

Of the proteoglycans associated with the extracellular matrix in vertebrates, which of the following is associated with the lubrication of joints? A) chondroitin sulfate B) glucuronate C) hyaluronate D) keratan sulfate E) mucoproteins

hyaluronate

2.43.i. inorganic or organice: oxygen

i

Where does most new membrane synthesis take place in a eukaryotic cell?

in the endoplasmic reticulum

33. chloroplasts are found only in eukaryotic cells that carry out photosynthesis: plants and algae. plants and algae appear green as a result of the presence of chlorophyll. where is chlorophyll located in the chloroplast?

in the third, innermost membrane

Proteins bind selectively to small-molecule targets called ligands. The selection of one ligand out of a mixture of possible ligands depends on the number of weak, noncovalent interactions in the protein's ligand-binding site. Where is the binding site typically located in the protein structure?

inside a cavity on the protein surface

Which of the following is characteristic of a steroid hormone action?

internal receptor binding

RNA splicing

introns are removed and the remaining exons are connected to eachother.

Intervening sequences that are transcribed, but not translated into protein are called

introns.

Which one of the following is true? A codon _____. can code for up to four different amino acids consists of four nucleotides is the basic unit of the genetic code extends from one end of a tRNA molecule

is the basic until of the genetic code

55. A. thaliana or Arabidopsis, is a common weed. biologists have selected it over hundreds or thousands of other flowering plant species to serve as an experimental model organism because BLANK

it can reproduce in 8-10 weeks

2.28.a. in which scientific unit is the strength of a chemical bond usually expressed?

kilocalories per mole

Consider the replication bubble diagrammed at the right. Which letters represent leading strands?

l and o

Imagine a bacterial replication fork. Synthesis of which new strand(s) would be affected by mutations in the enzyme primase?

lagging strand

Synthesis of which new strand(s) would be affected by mutations in the enzyme DNA ligase?

lagging strand

hypotonic

less solutes in environment; water moves into the cell, cell swells

2.26. double covalent bonds are both shorter and stronger than single covalent bonds, but they also limit the geometry of the molecule because they....

limit the rotation of the bonded atoms

Double covalent bonds are both shorter and stronger than single covalent bonds, but they also limit the geometry of the molecule because they ____________________.

limit the rotation of the bonded atoms.

polypeptide

linear sequence of amino acid; it denotes structure.

Which of the following contains hydrolytic enzymes?

lysosome

digests worn-out organelles

lysosome

Consider the replication bubble diagrammed at the right. Which letters represent places where one could find Okazaki fragments?

m and n

2.22.on the basis of the information in the chart and what you know about atomic surfaces, which elements will form ions with a net charge of +2 in solution?

magnesium, calcium

3.a. Cells can be very diverse: superficially, they come in various sizes, ranging from bacterial cells such as lactobacillus, which is a few BLANK in length, to larger cells such as frog's egg, which has a diameter of about one BLANK.

micrometers; millimeter

What are prions? mobile segments of DNA tiny circular molecules of RNA that can infect plants misfolded versions of normal protein that can cause disease viruses that invade bacteria viral DNA that attaches itself to the host genome and causes disease

misfolded versions of normal protein that can cause disease

During RNA processing a(n) _____ is added to the 5' end of the RNA. 3' untranslated region a long string of adenine nucleotides 5' untranslated region coding segment modified guanine nucleotide

modified guanine nucleotide

hypertonic

more solutes in environment; water moves out of the cell, cell shrivels

diffusion

movement of any molecule down its concentration gradient (from high to low); spontaneous

active transport

movement of molecules against their concentration gradient; requires energy

diffusion

movement of molecules from an area of high concentration to one of low concentration.

chemiosmosis

movement of protons through Electron Transport Chain for ATP Synthesis

Describe the function of the course

moves the stage up and down- affecting how close the slide is to the objective

Carbon 14 is an unstable isotope of carbon that decays very slowly. Compared to the common, stable carbon 12 isotope, carbon 14 has two additional ______________.

neutrons.

2.18.c. equal sharing of electrons yields a BLANK covalent bond.

nonpolar

2.69.d. choose the class of amino acids most important for the interaction: localizing an "integral membrane" protein that spans a lipid bilayer

nonpolar

2.43.e. inorganic or organice: cholesterol

o

a cluster of related genes can be under coordinated control by a single "on-off switch", called the ___________

operator

negative phototaxis

organism should move away from light to more shaded regions

2.32. what describes the calculation of pH?

pH=-log10[H+]

Transcribed region

part of this region contains the infomation that specifies an amino acid sequence.

2.56.d. when a protein is made, amino acids are linked together through BLANK, which are formed by condensation reactions between the carboxyl end of the last amino acid and the BLANK end of the next amino acid to be added to the growing chain

peptide bonds amino

41. BLANK are fairly small organelles that provide a safe place within the cell to carry out certain biochemical reactions that generate harmful, highly reactive oxygen species. these chemicals are both generated and broken down in the same location

perioxisomes

breaks down hydrogen peroxide

peroxisomes

2.58.g. linkage between phosphate groups is called

phosphoanhydride bond

2.58.a. the linkage between two nucleotides is called

phosphodiester bond

2.58.b. the linkage between the 5' sugar hydroxyl and a phosphate group is called

phosphoester bond

Which of the following types of molecules are the major structural component of the cell membrane?

phospholipids and proteins

2.56.a. proteins are BLANK built from amino acids, which each have an amino group and a BLANK group attached to the central BLANK.

polypeptides carboxyl a-carbon

________ is directly produced from the transcription of a eukaryotic gene.

pre-mRNA

Watson and Crick's semiconservative model of replication

predicts that when a double helix replicates, each daughter molecule will have one old strand (derived or "conserved" from the parent molecule) and one newly made strand

capping protein

prevents loss of subunits

Sporophyte

produce spores via meiosis

function of the tomato fruit

protect and support development of the seeds

Epithelial cells function

protect from water loss, mechanical injury and foreign invaders

5' caps and 3' poly(A) tails of eukaryotic mRNAs ______. act as sites for the start and stop of translation, respectively protect mRNA from degradation and enhance translation mark the beginning and end of introns, respectively protect mRNA from degradation and enhance transcription

protect mRNA from degradation and enhance translation

According to the original central dogma, what macromolecule or phenomenon is directly responsible for an organism's phenotype? mutation DNA transcription protein RNA

protein

The phosphorylation of a protein is typically associated with a change in activity, the assembly of a protein complex, or the triggering of a downstream signaling cascade. The addition of ubiquitin, a small polypeptide, is another type of covalent modification that can affect the protein function. Ubiquitylation often results in ______________.

protein degradation.

Initiation factors

proteins that facilitate the interactions between components.

aquaporins

proteins that form water channels in the membrane

5. The flow of genetic information is controlled by a series of biochemical reactions that result in the production of proteins, each with its own specific order of amino acids. Choose the correct series of biochemical reactions from the options presented here:

replication, transcription, translation

can be inhibited when a specific molecule binds to a regulatory site

repressible operon

A gap junction is a channel that connects adjacent cells. Which one of the following cannot pass through a gap junction?

ribosomes

Which of the following molecules is not an ionophore that functions as an ion channel or transporter and was originally used as an antibiotic?

ricin

After death, lack of ATP stops dissociation of cross-bridges, the muscle is locked stiff state called

rigor mortis

After death, lack of ATP stops dissociation of cross-bridges, the muscle is locked stiff state called ().

rigor mortis

Which level of protein structure do the α-helix and the β-pleated sheet represent? primary secondary tertiary quaternary primary, secondary, tertiary, and quaternary

secondary

Exons

sequences contained in the mature mRNA.they are expressed in regions whereas introns are intervening regions that are not expressed because they are removed from the mRNA.

Regulatory sequence

site for the binding of regulatory proteins.the role of regulatory proteins is to influence the rate of transcription.

Genetic Code

specifies the relationship between the sequence of nucleotides in the mRNA and the sequences of amino acids in a polypeptide.

Specialized hair cells that contain special actin-rich sensory structures called () are located within the organ of Corti in the cochlea of the ear

stereocilia

28. mitochondria perform cellular respiration, a process that uses oxygen, generates carbon dioxide, and produces chemical energy for the cell. Which answer below indicates a correct pairing of material "burned" and the form of energy produced during cellular respiration? a. fat, ADP b. sugar, fat c. sugar, ATP d. fat, protein

sugar, ATP

2.10. which of the following elements is least abundant in living organisms? a. sulfur b. carbon c. oxygen d. nitrogen

sulfur

Amino acids are carried to the ribosomes to be incorporated into polypeptide chains by ________.

tRNA

what enzyme compensates for replication-associated shortening of linear chromosomes?

telomerase

What component of amino acid structure varies among different amino acids? the long carbon-hydrogen tails of the molecule the presence of a central C atom the glycerol molecule that forms the backbone of the amino acid the components of the R-group

the components of the R-group

Critical concentration is

the concentration of tubulin dimers at which assembly is balanced with disassembly.

osmosis

the diffusion of water

diffusion gradient model

there is a chemical gradient that allows undifferentiated cells to sense their position in the body according to the concentration of chemical substances known as morphogens they will regenerate tissues in a posterior to anterior direction

How many hydrogen bonds between g and c

three

58. zebrafrish (danio rerio) are especially useful in the study of early development because their embryos are BLANK

transparents

3.21.Oxidation and reduction states are relatively easy to determine for metal ions, because there is a measurable net charge. In the case of carbon compounds, oxidation and reduction depend on the nature of polar covalent bonds. Which of the following is the best way to describe these types of bond? (a) hydrogen bonds in a nonpolar solution (b) covalent bonds in an aqueous solution (c) unequal sharing of electrons across a covalent bond (d) equal sharing of electrons across a covalent bond

unequal sharing of electrons across a covalent bond

3.d. Although BLANK contain the same types of molecules as cells, their inability to reproduce themselves by their own efforts means that they are not considered living matter.

viruses

For the reaction Y LaTeX: \leftrightarrow ↔ X at standard conditions with [Y] = 1 M and [X] = 1 M, ΔG is initially a large negative number. As the reaction proceeds, [Y] decreases and [X] increases until the system reaches equilibrium. How do the values of ΔG and ΔG° change as the reaction equilibrates?

ΔG becomes less negative and ΔG° stays the same.

Anaerobically growing yeast further metabolizes the pyruvate produced by glycolysis to CO2 and ethanol as part of a series of fermentation reactions. A. What other important reaction occurs during this fermentation step? Page 7 of 27 B. Why is this reaction (that is, the answer to part A) essential for the anaerobically growing cell?

A. NADH NAD+. B. Under anaerobic conditions, it is the only means of regenerating the NAD+ required for glycolysis, the main energy-generating pathway of an anaerobically growing yeast cell.

18) In plant cells, the middle lamella ________.

Allows adjacent cells to adhere one another

Which statement is most accurate concerning the function(s) of the 5' cap and the 3' poly(A) tail of eukaryotic mRNAs? Both structures are important for initiating translation and extending the lifespan of the mRNA. Neither structure has any known function. The poly(A) tail stabilizes the mRNA, but plays no direct role in its translation to protein. The 5' cap is important for initiating translation but plays no role in stabilizing mRNA.

Both structures are important for initiating translation and extending the lifespan of the mRNA

In the reaction cycle involved in the oxidation of pyruvate, what are the advantages of having three enzyme activities contained in a single large complex instead of having three smaller and physically independent enzymes?

By co-localizing three enzyme activities in a large, layered complex, the substrates are already bound and properly positioned for rapid enzyme catalysis, and the free energy released by one reaction can be readily harnessed for the next.

During glycolysis, when glucose is catabolized to pyruvate, most of the energy of glucose is A) transferred to ADP, forming ATP. B) transferred directly to ATP. C) retained in the pyruvate. D) stored in the NADH produced. E) used to phosphorylate fructose to form fructose-6-phosphate.

C

How many molecules of carbon dioxide (CO2) would be produced by five turns of the citric acid cycle? A) 2 B) 5 C) 10 D) 12 E) 60

C

What are the products of cellular respiration?

Carbon dioxide Water Energy. 38 ATP (adenosine triphosphate) are produced through aerobic respiration. 2 ATP are produced through anaerobic respiration.

They are proteins that promote depolymerization of extant microtubules at the ends

Catastrophins

Which of the following statements about organellar movement in the cell is false? (a) Organelles undergo saltatory movement in the cell. (b) Only the microtubule cytoskeleton is involved in organellar movement. (c) Motor proteins involved in organellar movement use ATP hydrolysis for energy. (d) Organelles are attached to the tail domain of motor proteins.

Choice (b) is untrue; both the actin cytoskeleton and the microtubule cytoskeleton are involved in organellar movement.

The watery medium inside a cell is known as

Cytosol

Which protein is NOT a fibrous protein of the extracellular matrix?

Elastin

If Na+ channels are opened in a cell that was previously at rest, how will the resting membrane potential be affected?

It becomes more positive.

Why is Triton X-100 a better detergent than SDS to help you extract and study the function of membrane proteins?

It is a non-denaturing detergent.

The conversion of glyceraldehyde 3-phosphate to 1,3 bisphosphoglycerate in step 6 of glycolysis generates a "high energy" phosphoanhydride bond. Which of the following best describes what happens to that bond in step 7?

It is hydrolyzed to drive the formation of ATP.

What does it mean when we say the genetic code is redundant? A single codon can specify the addition of more than one amino acid. More than one codon can specify the addition of the same amino acid. The genetic code is universal (the same for all organisms). The genetic code is different for different domains of organisms.

More than one codon can specify the addition of the same amino acid

Pyruvate can be converted into many other molecules by various biosynthetic and metabolic pathways, which makes it a central hub in the regulation of cellular metabolism. Which of the following molecules is not made from pyruvate?

NADH

Which of the following is NOT a component of the extra cellular matrix?

None of the above

Mitochondria contain their own genome, are able to duplicate, and actually divide on a different time line from the rest of the cell. Nevertheless, mitochondria cannot function for long when isolated from the cell because they are __________________.

None of these is correct.

What type of bond is very prevalent in lipids and gives lipids their hydrophobic properties?

Nonpolar covalent

What is the function of the ETC in cellular respiration?

To break the fall of electrons to oxygen into several energy releasing steps.

Where do the Calvin cycle take place?

Stroma

How does the simple primary and secondary structure of DNA hold the information needed to code for the many features of multicellular organisms? The amino acids that make up the DNA molecule contain the information needed to make cellular proteins. The width of the double helix changes at each gene due to differences in hydrogen bonds. The hydrogen bonding among backbone constituents carries coded information. The base sequence of DNA carries the information needed to code for proteins.

The base sequence of DNA carries the information needed to code for proteins

Recall that the other question you were studying was "Is RNA synthesis required for transition from 1-D to 2-D growth?" Using your data to support your answer, is it YES or NO?

Yes, RNA synthesis is required for transition from 1-D to 2-D growth. The gametophytes in the normal fern medium had 2-D growth. The gametophytes in the experimental treatments (2-thiouracil 5mg/L and 2- thiouracil 10mg/L) did NOT have 2D growth only 1-D growth. The gametophytes in the 2-thiouracil + uracil had 2-D growth.

steroids

characterized by a carbon skeleton with 4 fused rings; distinguished by the different chemical groups attached to the rings

Replication fork

a Y-shaped region where new DNA strands are elongating

2.58.e. sugar unit linked to a base is called

nucleoside

RNA processing

the RNA transcript, termed pre-mRNA is modified in ways that make it functionally active mRNA.

respriation

the process by which living cells release the chemical energy that is stored in glucose

intermediate filaments associated with plaques

tonofilaments

Introns

transcribed but not translated into proteins.

The process that produces mRNA from DNA is called

transcription

Explain the difference in energy usage between the catabolic reactions of cellular respiration, and the anabolic pathways of biosynthesis.

Catabolic: energy is generation, anabolic: energy is consumed

A

81) Which of the following is characteristic of the product of the p53 gene? A) It is an activator for other genes. B) It speeds up the cell cycle. C) It causes cell death via apoptosis. D) It allows cells to pass on mutations due to DNA damage. E) It slows down the rate of DNA replication by interfering with the binding of DNA polymerase.

C

A researcher has arrived at a method to prevent gene expression from Drosophila embryonic genes. The following questions assume that he is using this method. 51) The researcher in question measures the amount of new polypeptide production in embryos from 2—8 hours following fertilization and the results show a steady and significant rise in polypeptide concentration over that time. The researcher concludes that A) his measurement skills must be faulty. B) the results are due to building new cell membranes to compartmentalize dividing nuclei. C) the resulting new polypeptides are due to translation of maternal mRNAs. D) the new polypeptides were inactive and not measurable until fertilization. E) polypeptides were attached to egg membranes until this time.

How does ATP synthase use the flow of hydrogen ions to produce ATP?

ATP synthase acts as a rotor which allows for the production of ATP from ADP +Pi by activating catylic sites in the knob.

The release of intracellular materials by a vesicle at the cell surface is called

Exocytosis

Who were the electrophysiologists whose pioneering work on Loligo made enormous contributions to our understanding of neuron excitability?

Hodgkin & Huxley

65. The antibiotic streptomycin inhibits protein synthesis in bacteria. If this antibiotic is added to a culture of animal cells, protein synthesis in the cytosol continues normally. However, over time, the population of mitochondria in the cell becomes depleted. Specifically, it is observed that the protein-synthesis machinery inside the mitochondria is inhibited. a.Explain this observation based on what you know about the origins of the modern eukaryote.

If the mitochondria originated from an ancient aerobic bacterium that was engulfed by an ancient eukaryote, as postulated, it is possible that an antibiotic that inhibits protein synthesis in bacteria could also block that process in mitochondria.

Which of the following amino acids is of a type that is different from the others?

NOT All of these amino acids are of a similar type.

The ______ of a membrane is the property that determines it's effectiveness as a barrier

Permeability

CAM photosynthesis

Photosynthesis that takes place in desert plants

________ is to transcription as ________ are to translation?

RNA polymerase; ribosomes

What happens when a plant undergoes photorespiration?

Rubisco binds to O2 thus no carbohydrate is generated.

Which of the following CAMs is involved with leukocyte rolling?

Selectin

Not all intercellular signals require transduction. Which one of the following signals would be processed without transduction?

a lipid-soluble signal

Which of the following best represents the centreal dogma of gene expression? a. During transcription,DNA codes for polypeptides. b. During transcription,DNA codes for mRNA,which codes for polypeptides during translation. c.During translation,DNA codes for mRNA,which codes for polypeptides during transcription. d.none of the above.

b. During transcription,DNA codes for mRNA,which codes for polypeptides during translation.

Several different classes of enzymes are needed for the catabolism of carbohydrates. Which of the following descriptions best matches the function of a kinase? (a) An enzyme that catalyzes the rearrangement of bonds within a single molecule. (b) An enzyme that catalyzes a change in the position of a specific chemical group within a single molecule. (c) An enzyme that catalyzes the oxidation of a molecule by removing a hydride ion. (d) An enzyme that catalyzes the addition of phosphate groups to other molecules.

d

In signal transduction, phosphatases ________.

inactivate protein kinases and turn off the signal transduction

42. the cytoskeleton provides support, structure, motility, and organization, and it forms tracks to direct organelle and vesicle transport. which of the cytoskeletal elements listed below is the thickest? a. actin filaments b. microtubules c. intermediate filaments

microtubules

Which of the following is the correct order for the synthesis of the plant cell wall, from the outermost layer to the innermost layer? A) middle lamella; primary cell wall; secondary cell wall; plasma membrane B) primary cell wall; middle lamella; secondary cell wall; plasma membrane C) plasma membrane; secondary cell wall; primary cell wall; middle lamella D) middle lamella; secondary cell wall; primary cell wall; middle lamella E) primary cell wall; secondary cell wall; plasma membrane; middle lamella

middle lamella; primary cell wall; secondary cell wall; plasma membrane

Which of the following expressions accurately describes the calculation of pH?

pH = -log10[H+]

2.39. aromatic carbon compounds such as benzene are planar and very stable. double bond character extends around the entire ring, which is why it is often drawn as a hexagon with a circle inside. this characteristic is caused by election...

resonance

In performing research, you treat frog zygotes with antibodies that bind E-cadherin. As a result, you expect A) the cells to become exclusively muscle tissue. B) the cells to be loosely associated and abort development. C) the cells to be loosely associated and develop into a novel strain of frog. D) the cells to divide uncontrollably. E) no change with regard to development.

the cells to be loosely associated and abort development.

During transcription, in which direction is (1) RNA synthesized and in which direction is (2) the DNA template "read"? (1) 5'—>3' (2) 3'—>5' (1) 3'—>5' (2) 5'—>3' (1) 5'—>3' (2) 5'—>3' (1) 3'—>5' (2) 3'—>5'

(1) 5' --> 3' (2) 3'-->5'

Membrane synthesis in the cell requires the regulation of growth for both halves of the bilayer and the selective retention of certain types of lipids on one side or the other. Which group of enzymes accomplishes both of these tasks? (a) flippases (b) phospholipases (c) convertases (d) glycosylases

(a)

Most animal fats form a solid at room temperature, while plant fats remain liquid at room temperature. Which of the following is a feature of lipids in plant membranes that best explains this difference? (a) unsaturated hydrocarbons (b) longer hydrocarbon tails (c) higher levels of sterols (d) larger head groups

(a)

Porin proteins form large, barrel-like channels in the membrane. Which of the following is not true about these channels? (a) They are made primarily of α helices. (b) They are made primarily of β sheets. (c) They cannot form narrow channels. (d) They have alternating hydrophobic and hydrophilic amino acids.

(a)

Transporters, in contrast to channels, work by ________________. (a) specific binding to solutes. (b) a gating mechanism. (c) filtering solutes by charge. (d) filtering solutes by size.

(a)

Water molecules readily form hydrogen bonds with other polar molecules, and when they encounter nonpolar molecules they must form hydrogen-bonding networks with neighboring water molecules. Which of the following molecules will cause a "cage" of water to form? (a) 2-methylpropane (b) acetone (c) methanol (d) urea

(a)

Which of the following phenomena will be observed if a cell's membrane is pierced? (a) the membrane reseals (b) the membrane collapses (c) a tear is formed (d) the membrane expands

(a)

Which of the following phospholipid precursors is the most hydrophobic? (a) triacylglycerol (b) diacylglycerol (c) phosphate (d) glycerol

(a)

Activated protein kinase C (PKC) can lead to the modification of the membrane lipids in the vicinity of the active PKC. Figure Q16-38 shows how G proteins can indirectly activate PKC. You have discovered the enzyme activated by PKC that mediates the lipid modification. You call the enzyme Rafty and demonstrate that activated PKC directly phosphorylates Rafty, activating it to modify the plasma membrane lipids in the vicinity of the cell where PKC is active; these lipid modifications can be detected by dyes that bind to the modified lipids. Cells lacking Rafty do not have these modifications, even when PKC is active. Which of the following conditions would lead to signal-independent modification of the membrane lipids by Rafty? Figure Q16-38 (a) the expression of a constitutively active phospholipase C (b) a mutation in the GPCR that binds the signal more tightly (c) a Ca2+ channel in the endoplasmic reticulum with an increased affinity for IP3 (d) a mutation in the gene that encodes Rafty such that the enzyme can no longer be phosphorylated by PKC

(a) A constitutively active phospholipase C will lead to the constitutive production of IP3 and diacylglycerol, leading to activation of PKC in a signal-independent manner; thus, Rafty activation and the lipid modification will be signal-independent. Choices (b) and (c) will increase activity of the signal transduction pathway in a signal-dependent manner. Choice (d) will prevent PKC from activating Rafty and will thus prevent the lipid modifications.

Which of the situations below will enhance microtubule shrinkage? (a) addition of a drug that inhibits GTP exchange on free tubulin dimers (b) addition of a drug that inhibits hydrolysis of the GTP carried by tubulin dimers (c) addition of a drug that increases the affinity of tubulin molecules carrying GDP for other tubulin molecules (d) addition of a drug that blocks the ability of a tubulin dimer to bind to γ-tubulin

(a) A drug that inhibits GTP exchange on free tubulin dimers will effectively decrease the available pool of GTP-bound tubulin dimers available for addition to microtubule ends, thus tipping the balance in favor of microtubule disassembly. A drug that inhibits hydrolysis of GTP carried by tubulin dimers [choice (b)] or that increases the affinity of GDP-bound tubulin dimers for each other [choice (c)] will stabilize growing microtubules. Blocking the ability of a tubulin dimer to bind to γ-tubulin will decrease the rate of new microtubule formation but should not enhance microtubule shrinkage [choice (d)].

14-5Which of the following statements describes the phosphorylation event that occurs during the process known as oxidative phosphorylation? (a)A phosphate group is added to ADP. (b)ATP is hydrolyzed in order to add phosphate groups to protein substrates. (c)A phosphate group is added to molecular oxygen. (d)Inorganic phosphate is transported into the mitochondrial matrix, increasing the local phosphate concentration.

(a) A phosphate group is added to ADP.

Compared to the normal situation, in which actin monomers carry ATP, what do you predict would happen if actin monomers that bind a nonhydrolyzable form of ATP were incorporated into actin filaments? (a) Actin filaments would grow longer. (b) Actin filaments would grow shorter because depolymerization would be enhanced. (c) Actin filaments would grow shorter because new monomers could not be added to the filaments. (d) No change, as addition of monomers binding nonhydrolyzable ATP would not affect actin filament length.

(a) Addition of monomers carrying a nonhydrolyzable form of ATP would stabilize the interactions between the monomers of a filament, stabilizing the filament and inhibiting depolymerization, resulting in longer actin filaments.

Which of the following channels would not be expected to generate a change in voltage by movement of its substrate across the membrane where it is found? (a) an aquaporin (b) a sodium channel (c) a calcium channel (d) a proton channel

(a) Aquaporin channels are found in the plasma membrane of some cells, where they facilitate diffusion of water across the membrane. Because water is an uncharged molecule, its movement would not be expected to alter the voltage across the membrane.

Consider the consequences of a mutation in the DNA template sequence 5'ATG3' to 5'TTG3': What is (a) the resulting change in the mRNA codon and (b) the effect on the corresponding amino acid? (a) AUG to UUG; (b) Met to Leu (a) UAC to AAC; (b) Tyr to Asn (a) CAU to CAA; (b) No change (a) CAU to CAA; (b) His to Gln

(a) CAU to CAA; (b) His to Gln

15-38Which of the following statements about vesicle budding from the Golgi is false? (a)Clathrin molecules are important for binding to and selecting cargoes for transport. (b)Adaptins interact with clathrin. (c)Once vesicle budding occurs, clathrin molecules are released from the vesicle. (d)Clathrin molecules act at the cytosolic surface of the Golgi membrane.

(a) Cargo binds to cargo receptors. Adaptin molecules capture cargo receptors, which bind to the appropriate cargo molecules for incorporation into the vesicle.

When Ras is activated, cells will divide. A dominant-negative form of Ras clings too tightly to GDP. You introduce a dominant-negative form of Ras into cells that also have a normal version of Ras. Which of the following statements is true? (a) The cells you create will divide less frequently than normal cells in response to the extracellular signals that typically activate Ras. (b) The cells you create will run out of the GTP necessary to activate Ras. (c) The cells you create will divide more frequently compared to normal cells in response to the extracellular signals that typically activate Ras. (d) The normal Ras in the cells you create will not be able to bind GDP because the dominant-negative Ras binds to GDP too tightly.

(a) Dominant-negative Ras is considered "dominant" because it prevents normal Ras from doing its job. This mutant protein, when overexpressed in a cell, binds to—and essentially monopolizes—other signaling partners in the pathway, but it cannot activate the target proteins that lie downstream. In this way, dominant-negative Ras mutants block Ras signaling and inhibit cell proliferation. The creation of GTP in the cell is independent of Ras and its associated signaling pathways [choice (b)]. An activated Ras mutant will cause cells to divide more frequently. In this situation, the Ras pathway does not work and cells should not divide more frequently [choice (c)]. The binding of GDP to dominant-negative Ras should not affect the ability of normal Ras to bind GDP, as the concentration of GTP in the cell is quite high, and the normal Ras is inactive and bound to the GDP it created from GTP the last time it was active.

We can test the relative permeability of a phospholipid bilayer by using a synthetic membrane that does not contain any protein components. Some uncharged, polar molecules are found to diffuse freely across these membranes, to varying degrees. Which of the following has the lowest rate of diffusion across an artificial membrane? Why? (a) glucose (b) water (c) glycerol (d) ethanol

(a) Glucose will have a negligible degree of diffusion across the synthetic bilayer. Not only is it polar, it is also larger than other molecules that are able to diffuse across the membrane.

You are interested in cell-size regulation and discover that signaling through a GPCR called ERC1 is important in controlling cell size in embryonic rat cells. The G protein downstream of ERC1 activates adenylyl cyclase, which ultimately leads to the activation of PKA. You discover that cells that lack ERC1 are 15% smaller than normal cells, while cells that express a mutant, constitutively activated version of PKA are 15% larger than normal cells. Given these results, which of the following treatments to embryonic rat cells should lead to smaller cells? (a) addition of a drug that causes cyclic AMP phosphodiesterase to be hyperactive (b) addition of a drug that prevents GTP hydrolysis by Gα (c) addition of a drug that activates adenylyl cyclase (d) addition of a drug that mimics the ligand of ERC1

(a) Hyperactivating cyclic AMP phosphodiesterase will degrade the cAMP, terminating the signal more quickly than usual. All other answers will lead to larger cells.

In the final stage of the oxidation of food molecules, a gradient of protons is formed across the inner mitochondrial membrane, which is normally impermeable to protons. If cells were exposed to an agent that causes the membrane to become freely permeable to protons, which of the following effects would you expect to observe? (a) The ratio of ATP to ADP in the cytoplasm would fall. (b) NADH would build up. (c) Carbon dioxide production would cease. (d) The consumption of oxygen would fall.

(a) If the inner mitochondrial membrane became permeable to protons, the electron-transport chain would continue to oxidize NADH to NAD+, transport electrons, and pump protons, so the consumption of oxygen would not fall. However, without the energy stored in a proton gradient, there is no way of driving the synthesis of ATP.

Voltage-gated channels contain charged protein domains, which are sensitive to changes in membrane potential. By responding to a threshold in the membrane potential, these voltage sensors trigger the opening of the channels. Which of the following best describes the behavior of a population of channels exposed to such a threshold? (a) Some channels remain closed and some open completely. (b) All channels open completely. (c) All channels open partly, to the same degree. (d) All channels open partly, each to a different degree.

(a) Individual channels are either completely open or completely closed. However, in a given population, there will be a mixture of open and closed channels.

15-12Proteins that are fully translated in the cytosol and lack a sorting signal will end up in ____. (a)the cytosol. (b)the mitochondria. (c)the interior of the nucleus. (d)the nuclear membrane.

(a) Proteins produced in the cytosol that lack sorting signals remain in the cytosol. Proteins produced in the cytosol and destined for the mitochondria [choice (b)] or the interior of the nucleus [choice (c)] will have a sorting signal to direct the protein to its proper location. Proteins destined for the nuclear membrane [choice (d)] are not translated in the cytosol.

The graph in Figure Q17-31 shows the time course of the polymerization of pure tubulin in vitro. Assume that the starting concentration of free tubulin is higher than it is in cells. Three parts of the curve are labeled above it as A, B, and C. You conduct a similar in vitro tubulin-polymerization experiment, only you include purified centrosomes in your preparation. When you plot your data, which part of your graph should be most dissimilar to the curve shown in Figure Q17-31? (a) A (b) B (c) C (d) None. The shape of my graph should be identical to the graph produced when tubulin is polymerized in the absence of purified centrosomes.

(a) Purified centrosomes should enhance the nucleation of microtubules, and thus decrease the lag time (seen in part A of the graph) for microtubule polymerization that occurs when microtubules are polymerized from only pure tubulin.

Which of the following mechanisms is not directly involved in inactivating an activated RTK? (a) dephosphorylation by serine/threonine phosphatases (b) dephosphorylation by protein tyrosine phosphatases (c) removal of the RTK from the plasma membrane by endocytosis (d) digestion of the RTK in lysosomes

(a) RTKs are phosphorylated on tyrosines by their dimerization partner, which is also a tyrosine kinase, and thus the reversal of these phosphorylations involves protein tyrosine phosphatases, and not protein serine/threonine phosphatases. Endocytosis of the receptor and its ultimate digestion in the lysosome are other methods that the cell uses to down- regulate active receptors.

15-31Figure Q15-31 shows the organization of a protein that resides on the ER membrane. The N-and C-termini of the protein are labeled. Boxes 1, 2, and 3 represent membrane-spanning sequences. Non-membrane-spanning regions of the protein are labeled "X,""Y,"and "Z." Once this protein is fully translocated, where will region Y be? (a) in the cytoplasm (b) in the ER lumen (c) inserted into the ER membrane (d) degraded by signal peptidase

(a) The final topology of the protein on the ER membrane is diagrammed in Figure A15- 31.

Which of the following items is not important for flagellar movement? (a) sarcoplasmic reticulum (b) ATP (c) dynein (d) microtubules

(a) The sarcoplasmic reticulum is important for muscle contraction. All other items are important for flagellar movement.

4-31 The relationship of free-energy change (ΔG) to the concentrations of reactants and products is important because it predicts the direction of spontaneous chemical reactions. In the hydrolysis of ATP to ADP and inorganic phosphate (Pi), the standard free-energy change (ΔG°) is -7.3 kcal/mole. The free-energy change depends on concentrations according to the following equation: ΔG= ΔG°+ 1.42 log10([ADP] [Pi]/[ATP]) In a resting muscle, the concentrations of ATP, ADP, and Piare approximately 0.005 M, 0.001 M, and 0.010 M, respectively. What is the ΔGfor ATP hydrolysis in resting muscle? (a)-11.1 kcal/mole (b)-8.72 kcal/mole (c)6.01 kcal/mole (d)-5.88 kcal/mole

(a) The ΔG for ATP hydrolysis is -11.1 kcal/mole. This result is calculated by substituting values into the equation given: ΔG = -7.3 kcal/mole + 1.42 log10 ([0.001 M] [0.010 M]/[0.005 M]) = -7.3 kcal/mole + 1.42 log10 (0.002) = -11.1 kcal/mole.

14-46Electron-transfer reactions occur rapidly. Which of the following statements best describes how the diffusion of ubiquinone is controlled in order to ensure its proximity to the other enzyme complexes? (a)Ubiquinone is anchored directly in the inner mitochondrial membrane via its hydrocarbon tail, and can only diffuse laterally. (b)Ubiquinone is present at high concentrations, minimizing the impact of diffusion on the electron-transport chain. (c)Ubiquinone becomes covalently attached to the other enzyme complexes. (d)The intermembrane space in the mitochondrionis relatively small, and therefore the random diffusion of these molecules is not a problem.

(a) Ubiquinone is anchored directly in the inner mitochondrial membrane via its hydrocarbon tail, and can only diffuse laterally.

The process of generating monoclonal antibodies is labor-intensive and expensive. An alternative is to use polyclonal antibodies. A subpopulation of purified polyclonal antibodies that recognize a particular antigen can be isolated by chromatography. Which type of chromatography is used for this purpose? (a) affinity (b) ion-exchange (c) gel-filtration (d) any of the above

(a) affinity

You have isolated two mutants of a normally pear-shaped microorganism that have lost their distinctive shape and are now round. One of the mutants has a defect in a protein you call A and the other has a defect in a protein you call B. First, you grind up each type of mutant cell and normal cells separately and separate the plasma membranes from the cytoplasm, forming the first cell extract. Then you set aside a portion of each fraction for later testing. Next, you wash the remaining portion of the membrane fractions with a low concentration of urea (which will unfold proteins and disrupt their ability to interact with other proteins) and centrifuge the mixture. The membranes and their constituent proteins form a pellet, and the proteins liberated from the membranes by the urea wash remain in the supernatant. When you check each of the fractions for the presence of A or B, you obtain the results given below. Which of the following statements are consistent with your results (more than one answer may apply)? (a) Protein A is an integral membrane protein that interacts with B, a peripheral membrane protein that is part of the cell cortex. (b) Protein B is an integral membrane protein that interacts with A, a peripheral membrane protein that is part of the cell cortex. (c) Proteins A and B are both integral membrane proteins. (d) The mutation in A affects its ability to interact with B.

(a) and (d) The results from the extracts of normal cells show that protein A is an integral membrane protein that remains in the membrane through all the treatments, whereas protein B is a peripheral membrane protein that can be removed from the membrane by urea. In the cell extracts from the mutants with a defect in A, the A protein still remains in the membrane, but the B protein does not. This is consistent with the mutation in A affecting its interaction with B. The same results are obtained when the B protein is mutant, which is consistent with the idea that A and B interact. The loss of an interaction between an integral membrane protein and a protein in the cortex would be more likely to result in a change in cell shape than the loss of an interaction between an integral membrane protein and a protein on the exterior of the cell.

Instead of studying one or two proteins or protein complexes present in the cell at any given time, we can now look at a snapshot of all proteins being expressed in cells being grown in specific conditions. This large-scale, systematic approach to the study of proteins is called _______________. (a) proteomics (b) structural biology (c) systems biology (d) genomics

(a) proteomics

For some proteins, small molecules are integral to their structure and function. Enzymes can synthesize some of these small molecules, whereas others, called vitamins, must be ingested in the food we eat. Which of the following molecules is not classified as a vitamin but does require the ingestion of a vitamin for its production? (a) retinal (b) biotin (c) zinc (d) heme

(a) retinal

The three-dimensional coordinates of atoms within a folded protein are determined experimentally. After researchers obtain a protein's structural details, they can use different techniques to highlight particular aspects of the structure. What visual model best displays a protein's secondary structures (alpha helices and beta sheets)? (a) ribbon (b) space-filling (c) backbone (d) wire

(a) ribbon

14-55In stage 1 of photosynthesis, a proton gradient is generated and ATP is synthesized. Where do protons become concentrated in the chloroplast? (a)thylakoid space (b)stroma (c)inner membrane (d)thylakoid membrane

(a) thylakoid space

You have two purified samples of protein Y: the wild-type (nonmutated) protein and a mutant version with a single amino acid substitution. When washed through the same gelfiltration column, mutant protein Y runs through the column more slowly than the normal protein. Which of the following changes in the mutant protein is most likely to explain this result? (a) the loss of a binding site on the mutant protein surface through which protein Y normally forms dimers (b) a change that results in the mutant protein's acquiring an overall positive instead of a negative charge (c) a change that results in the mutant protein's being larger than the wild-type protein (d) a change that results in the mutant protein's having a slightly different shape from the wild-type protein

(a). Dimers formed by a normal protein will run through the gel-filtration column faster than a mutant protein Y monomer. Choice (b) is unlikely, because gel-filtration columns separate proteins on the basis of size, not charge or affinity for small molecules. Choice (c) is unlikely, because if the mutant protein were larger than normal it would be less able to enter the porous beads and would run through the column faster than the normal protein. Choice (d) is unlikely, because a small change in shape without a change in size would be unlikely to have a major effect on the behavior of a protein in a gel-filtration column.

A bacterium is suddenly expelled from a warm human intestine into the cold world outside. Which of the following adjustments might the bacterium make to maintain the same level of membrane fluidity? (a) Produce lipids with hydrocarbon tails that are longer and have fewer double bonds. (b) Produce lipids with hydrocarbon tails that are shorter and have more double bonds. (c) Decrease the amount of cholesterol in the membrane. (d) Decrease the amount of glycolipids in the membrane.

(b)

Although the extracellular environment has a high sodium ion concentration and the intracellular environment has a high potassium ion concentration, both must be neutralized by negatively charged molecules. In the extracellular case, what is the principal anion? (a) HCO3- (b) Cl- (c) PO4cv3- (d) OH-

(b)

Cells use membranes to help maintain set ranges of ion concentrations inside and outside the cell. Which of the following ions is the most abundant inside a typical mammalian cell? (a) Na+ (b) K+ (c) Ca2+ (d) Cl-

(b)

Consider the apical location of a particular protein expressed in epithelial cells, illustrated in Figure Q11-49A. Which type of defect described below is the most likely to cause the redistribution of that protein around the entire cell, shown in Figure Q11-49B? (a) a nonfunctional protein glycosylase (b) the deletion of a junctional protein (c) the truncation of a protein found in the extracellular matrix (d) a nonfunctional flippase

(b)

There are two properties of phospholipids that affect how tightly they pack together: the length of the hydrocarbon chain and the number of double bonds. The degree of packing, in turn, influences the relative mobility of these molecules in the membrane. Which of the following would yield the most highly mobile phospholipid (listed as number of carbons and number of double bonds, respectively)? (a) 24 carbons with 1 double bond (b) 15 carbons with 2 double bonds (c) 20 carbons with 2 double bonds (d) 16 carbons with no double bonds

(b)

When a signal needs to be sent to most cells throughout a multicellular organism, the signal most suited for this is a ___________. (a) neurotransmitter. (b) hormone. (c) dissolved gas. (d) scaffold.

(b)

Where does most new membrane synthesis take place in a eukaryotic cell? (a) in the Golgi apparatus (b) in the endoplasmic reticulum (c) in the plasma membrane (d) in the mitochondria (e) on ribosomes

(b)

Which of the following best describes the behavior of a gated channel? (a) It stays open continuously when stimulated. (b) It opens more frequently in response to a given stimulus. (c) It opens more widely as the stimulus becomes stronger. (d) It remains closed if unstimulated.

(b)

Which of the following statements best reflects the nature of synaptic plasticity? (a) New synapses are created due to the postnatal generation of neurons. (b) Synaptic response changes in magnitude depending on frequency of stimulation. (c) There is a change in the type of neurotransmitter used at the synapse. (d) Neuronal connections are pruned during normal development.

(b)

Figure Q16-18 shows the pathway through which nitric oxide (NO) triggers smooth muscle relaxation in a blood-vessel wall. Which of the following situations would lead to relaxation of the smooth muscle cells in the absence of acetylcholine? Figure Q16-18 (a) a smooth muscle cell that has a defect in guanylyl cyclase such that it cannot bind NO (b) a muscle cell that has a defect in guanylyl cyclase such that it constitutively converts GTP to cyclic GMP (c) a muscle cell that has cyclic GMP phosphodiesterase constitutively active (d) a drug that blocks an enzyme involved in the metabolic pathway from arginine to NO

(b) A constitutively active guanylyl cyclase will produce cyclic GMP (cGMP) even in the absence of a signal and thus will lead to relaxation of smooth muscle cells in the absence of acetylcholine. Choice (a) would lead to a block in the production of cGMP such that even if NO were to reach the smooth muscle cells, relaxation would not occur. Choice (c) would not lead to muscle-cell relaxation independently of acetylcholine, because cyclic GMP phosphodiesterase is involved in the degradation of cGMP. Choice (d) will lead to a block in the production of NO.

Active transport requires the input of energy into a system so as to move solutes against their electrochemical and concentration gradients. Which of the following is not one of the common ways to perform active transport? (a) Na+-coupled (b) K+-coupled (c) ATP-driven (d) light-driven

(b) Because K+ is a positively charged ion and the outside of the plasma membrane is positively charged, K+ has a very small electrochemical gradient across the membrane even though its concentration gradient is large. Because there is little net movement across the membrane for K+, it would not make a good source of energy to drive the transport of other molecules against their respective gradients.

Intermediate filaments are made from elongated fibrous proteins that are assembled into a ropelike structure. Figure Q17-10 shows the structure of an intermediate filament subunit. You are interested in how intermediate filaments are formed, and you create an intermediate filament subunit whose α-helical region is twice as long as that of a normal intermediate filament by duplicating the normal α-helical region while keeping a globular head at the N-terminus and a globular tail at the C-terminus; you call this subunit IFαd. If you were to assemble intermediate filaments using IFαd as the subunit, which of the following predictions describes the most likely outcome? Figure Q17-10 (a) Filaments assembled using IFαd will interact with different cytoskeletal components. (b) Filaments assembled using IFαd will form dimers that are twice as long as dimers assembled from normal intermediate filaments. (c) Sixteen tetramers assembled from IFαd will be needed for a ropelike structure to form. (d) Dimers of IFαd will form by interactions with the N-terminal globular head and the C-terminal globular tail.

(b) Because the α-helical region is twice as long, you would predict that a coiled-coil dimer made up of two IFαd subunits would be about twice as long as a dimer assembled from a normal intermediate filament subunit. Because the globular head and tail regions usually interact with other cellular components, doubling the size of the α-helical region without changes in the globular regions is unlikely to cause changes in protein interactions [choice (a)]. Eight tetramers are usually needed to form a ropelike filament, and it is unlikely that 16 will be needed with IFαd, because it is the length of the coiled- coil region that is most affected by a doubling in size of the α-helical region [choice (c)]. Interactions in the α-helical region are important for dimerization, and thus choice [d] is untrue.

The growth factor RGF stimulates proliferation of cultured rat cells. The receptor that binds RGF is a receptor tyrosine kinase called RGFR. Which of the following types of alteration would be most likely to prevent receptor dimerization? (a) a mutation that increases the affinity of RGFR for RGF (b) a mutation that prevents RGFR from binding to RGF (c) changing the tyrosines that are normally phosphorylated on RGFR dimerization to alanines (d) changing the tyrosines that are normally phosphorylated on RGFR dimerization to glutamic acid

(b) Binding of a ligand to RTKs leads to their dimerization, and thus a mutation that prevents RGFR from binding to RGF will prevent dimerization. A mutation that increases the affinity of RGFR for RGF will increase dimerization in the presence of ligand [choice (a)]. RTKs become phosphorylated on dimerization. However, changing the relevant tyrosines to alanine will block receptor activation but should not cause or prevent dimerization [choice (c)]. Because glutamic acid is negatively charged, it can mimic the addition of a phosphate to an amino acid; thus, changing the relevant tyrosines to glutamic acid may mimic receptor activation, but it should not cause or prevent receptor dimerization [choice (d)].

Which of the following occur without coupling transport of the solute to the movement of a second solute? (a) import of glucose into gut epithelial cells (b) export of Ca2+ from the cytosol (c) export of H+ from animal cells for pH regulation (d) the export of Na+ from cells to maintain resting membrane potential

(b) Ca2+ is exported using ATP-powered pumps. There are no other solutes that are being moved by these pumps.

In the absence of oxygen, yeast cells can switch to a completely anaerobic metabolism called fermentation. Which of the following is a final product of fermentation in yeast?

(b) Ethanol and CO2 are the products of fermentation in yeast when grown anaerobically. Option (a) is lactate, (c) is acetaldehyde, and (d) is pyruvate.

You discover a protein, MtA, and find that it binds to the plus ends of microtubules in cells. The hypothesis that best explains this localization is________________. (a) MtA is involved in stabilizing microtubules. (b) MtA binds to GTP-bound tubulin on microtubules. (c) MtA is important for the interaction of microtubules with the centrosome. (d) MtA will not bind to purified microtubules in a test tube.

(b) GTP-bound tubulin molecules are found at the growing end of a microtubule, which is its plus end. A function of MtA in stabilizing microtubules cannot be inferred simply from its localization, as protein factors that stabilize and destabilize microtubules can bind to plus ends [choice (a)]. The localization of MtA at the plus end of the microtubule means that MtA is at the end of the microtubule furthest from the centrosome [choice (c)]. Whether MtA will bind to purified microtubules in a test tube cannot be inferred from the discovery of its localization at the plus end of microtubules in cells [choice (d)].

Ion channels are classified as membrane transport proteins. Channels discriminate by size and charge. In addition to Na+, which one of the following ions would you expect to be able to freely diffuse through a Na+ channel? Explain your answer. (a) Mg2+ (b) H+ (c) K+ (d) Cl-

(b) If an ion channel is open, it will allow any ion that is under a certain size and that has the correct charge to pass through. H+ is the only ion listed that is both smaller and has the same charge of +1.

You have generated antibodies that recognize the extracellular domain of the Ca2+-pump. Adding these antibodies to animal cells blocks the active transport of Ca2+ from the cytosol into the extracellular environment. What do you expect to observe with respect to intracellular Ca2+? (a) Ca2+-pumps in vesicle membranes keep cytosolic calcium levels low. (b) Ca2+-pumps in the endoplasmic reticulum membrane keep cytosolic calcium levels low. (c) Ca2+-pumps in the Golgi apparatus keep cytosolic calcium levels low. (d) Ca2+ concentrations in the cytosol increase at a steady rate.

(b) In addition to the Ca2+-pumps in the plasma membrane, Ca2+-pumps are also found in the membrane of the endoplasmic reticulum (ER). Those in the ER membrane will continue to remove calcium ions from the cytosol, keeping calcium levels low.

Which of the following statements is false? (a) Cytochalasins prevent actin polymerization. (b) Actin filaments are usually excluded from the cell cortex. (c) Integrins are transmembrane proteins that can bind to the extracellular matrix. (d) ARPs can promote the formation of branched actin filaments.

(b) Much of the actin in the cell is concentrated in the cell cortex, the region of the cell just beneath the plasma membrane.

Male cockroaches with mutations that strongly decrease the function of an RTK called RTKX are oblivious to the charms of their female comrades. This particular RTK binds to a small molecule secreted by sexually mature females. Most males carrying loss-of-function mutations in the gene for Ras protein are also unable to respond to females. You have just read a paper in which the authors describe how they have screened cockroaches that are mutant in RTKX for additional mutations that partly restore the ability of males to respond to females. These mutations decrease the function of a protein that the authors call Z. Which of the following types of protein could Z be? Explain your answer. (a) a protein that activates the Ras protein by causing Ras to exchange GDP for GTP (b) a protein that stimulates hydrolysis of GTP by the Ras protein (c) an adaptor protein that mediates the binding of the RTKX to the Ras protein (d) a transcriptional regulator required for the expression of the Ras gene

(b) Mutations that increase the activity of Ras should mimic the effect of stimulating RTKX in a receptor-independent fashion. Because the intracellular concentration of GTP is higher than that of GDP, some proportion of the Ras molecules is expected to be GTP- bound and active; ridding the cells of a protein that stimulates GTP hydrolysis will increase this pool of active Ras. Mutants that cannot stimulate Ras to exchange GDP for GTP will have the same phenotype as mutants lacking Ras [choice (a)], as will mutants lacking a transcriptional regulator required for expression of the Ras gene [choice (d)]. Defects in an adaptor protein that mediates the binding of receptor X to Ras will have no further effect on a mutant already lacking the receptor [choice (c)].

14-15 NADH contains a high-energy bond that, when cleaved, donates a pair of electrons to the electron-transport chain. What are the immediate products of this bond cleavage? (a)NAD++ OH- (b)NAD++ H- c)NAD-+ H+ (d)NAD + H

(b) NAD+ + H-

15-64 You are working in a biotech company that has discovered a small-molecule drug called H5434. H5434 binds to LDL receptors when they are bound to cholesterol. H5434 binding does not alter the conformation of the LDL receptor's intracellular domain. Interestingly, in vitro experiments demonstrate that addition of H5434 increases the affinity of LDL for cholesterol and prevents cholesterol from dissociating from the LDL receptor even in acidic conditions. Which of the following is a reasonable prediction of what may happen when you add H5434 to cells? (a)Cytosolic cholesterol levels will remain unchanged relative to normal cells. (b)Cytosolic cholesterol levels will decrease relative to normal cells. (c)The LDL receptor will remain on the plasma membrane. (d)The uncoating of vesicles will not occur.

(b) Normally, cholesterol dissociates from the LDL receptor in the acidic environment of the endosomes and is released into the cytosol. If the drug prevents cholesterol from dissociating from the LDL receptor in acidic conditions, cholesterol may not become released into the cytosol, and thus cytosolic cholesterol levels are likely to decrease relative to those in normal cells. There is no reason to believe that the LDL receptor will remain on the plasma membrane [choice (c)], because the cytosolic region of the receptor is not directly altered by the drug. Vesicle uncoating is also unlikely to be altered [choice (d)], because this occurs after vesicles are pinched off from the membrane.

15-20Which of the following statements about transport into mitochondria and chloroplasts is false? (a)The signal sequence on proteins destined for these organelles is recognized by a receptor protein in the outer membrane of these organelles. (b)After a protein moves through the protein translocator in the outer membrane of these organelles, the protein diffuses in the lumen until it encounters a protein translocator in the inner membrane. (c)Proteins that are transported into these organelles are unfolded as they are being transported. (d)Signal peptidase will remove the signal sequence once the protein has been imported into these organelles.

(b) Once a protein is bound to the import receptor, the protein—in a complex that includes the protein translocator—will diffuse along the outer membrane until it reaches a specialized site where the inner and outer membranes contact each other, and will then be translocated simultaneously across the inner and outer membranes.

15-51 Which of the following statements about the protein quality control system in the ER is false? (a)Chaperone proteins help misfolded proteins fold properly. (b)Proteins that are misfolded are degraded in the ER lumen. (c)Protein complexes are checked for proper assembly before they can exit the ER. (d)A chaperone protein will bind to a misfolded protein to retain it in the ER.

(b) Proteins that are misfolded are exported from the ER into the cytosol, where they are degraded.

You are interested in further understanding the signal transduction pathway that controls the production of Pig1, a protein important for regulating cell size. Activation of the TRK receptor leads to activation of the GTP-binding protein, Ras, which then activates a protein kinase that phosphorylates the SZE transcription factor. SZE only interacts with the nuclear transport receptor when it is phosphorylated. SZE is a gene activator for the Pig1 gene. This pathway is diagrammed in Figure Q16-50. Normal cells grown under standard conditions (without ligand) are 14 μm in diameter while normal cells exposed to TRK ligand are 10.5 μm in diameter. Given this situation, which of the following conditions do you predict will more likely lead to smaller cells? (a) addition of TRK ligand and a drug that stimulates the GTPase activity of Ras (b) addition of TRK ligand and a drug that inhibits the activity of the phosphatase that acts on SZE (c) addition of TRK ligand and a drug that stimulates the degradation of Pig1 (d) addition of TRK ligand and a drug that inhibits Pig1 binding to DNA

(b) The activation of the TRK receptor and its downstream signaling pathway leads to smaller cells. Thus, by inhibiting the dephosphorylation of SZE, SZE will likely activate the expression of Pig1 longer than it would in normal cells, leading to smaller cells. All other scenarios interfere with TRK receptor signaling, and should lead to cells that are not as small.

When using the Nernst equation to calculate membrane potential, we are making several assumptions about conditions in the cell. Which of the following is not a good assumption? (a) The temperature is 37°C. (b) The plasma membrane is primarily permeable to Na+ (c) At rest, the interior of the cell is more negatively charged than the exterior. (d) K+ is the principal positive ion in the cell.

(b) The cell has K+ leak channels. At rest, the cell is mostly permeable to K+ because of the presence of these channels.

Figure Q17-57shows an electron micrograph of a skeletal muscle fiber, where various points along a fiber and various regions have been labeled. Figure Q17-57 Which of the following statements is true about muscle contraction? (a) Point A will move closer to point B. (b) Point B will move closer to point C. (c) Region D will become smaller. (d) Region E will shrink in size.

(b) The dark region in the center of the micrograph corresponds to the thick filaments of a myofibril and is composed of many myosin molecules. The light regions (labeled A, B, and C) correspond to actin filaments, which are attached to the Z discs (also see Figure A17-57). During muscle contraction, the myosin filaments will travel along the actin filaments, bringing points B and C closer together. Points A and B will not move relative to each other, as contraction occurs within each sarcomere [choice (a)]. The absolute size of region D will not change during contraction [choice (c)], and the width of the myofibril will not shrink [choice (d)]. Figure A17-57

Ca2+-pumps in the plasma membrane and endoplasmic reticulum are important for _____________. (a) maintaining osmotic balance. (b) preventing Ca2+ from altering the activity of molecules in the cytosol. (c) providing enzymes in the endoplasmic reticulum with Ca2+ ions that are necessary for their catalytic activity. (d) maintaining a negative membrane potential.

(b) The major purpose of the Ca2+-pumps is to keep the cytosolic concentration of Ca2+ low. When Ca2+ does move into the cytosol, it alters the activity of many proteins; hence Ca2+ is a powerful signaling molecule. It is not involved in the catalytic activity of endoplasmic reticulum enzymes [choice (c)]. Because the levels of Ca2+ are very low relative to the levels of K+ and Na+, the Ca2+ gradient does not have a significant effect on the osmotic balance of the cell [choice (a)] or the membrane potential [choice (d)].

The simultaneous oxidation and phosphorylation of glyceraldehyde 3-phosphate forms a highly reactive covalent thioester bond between a cysteine side chain (reactive group -SH) on the enzyme (glyceraldehyde 3-phosphate dehydrogenase) and the oxidized intermediate (see arrow in Figure Q13-31A). If the enzyme had a serine (reactive group -OH) instead of a cysteine at this position, which could form only a much-lower-energy bond to the oxidized substrate (see arrow in Figure Q13-31B), how might this new enzyme act? (a) It would oxidize the substrate and phosphorylate it without releasing it. (b) It would oxidize the substrate but not release it. (c) It would phosphorylate the substrate on the 2 position instead of the 1 position. (d) It would behave just like the normal enzyme.

(b) The phosphorylation and release of the product from the normal enzyme is possible because a phosphate molecule can attack the high-energy thioester bond formed between the oxidized substrate and enzyme. If the bond between the oxidized substrate and enzyme is of much lower energy, the enzyme will not be able to transfer the oxidized substrate to a phosphate group, and substrate and enzyme will remain covalently bound. Choices (a), (c), and (d) could not happen, because none of the bonds in the substrate molecule is reactive enough to be broken by a phosphate group.

14-62If you shine light on chloroplasts and measure the rate of photosynthesis as a function of light intensity, you get a curve that reaches a plateau at a fixed rate of photosynthesis, x, as shown in Figure Q14-62.Figure Q14-62Which of the following conditions will increase the value of x? (a)increasing the number of chlorophyll molecules in the antenna complexes (b)increasing the number of reaction centers (c)adding a powerful oxidizing agent (d)decreasing the wavelength of light used

(b) The rate of photosynthesis will increase with increasing light intensity until photons hit all of the reaction centers directly. At saturating levels of light, the number of reaction centers that are still capable of being excited limits the rate of photosynthesis, which can be increased only by increasing the number of reaction centers or by increasing the rate at which the reaction centers are restored to their low-energy state. Increasing the number of chlorophyll molecules in the antenna complexes, the energy per photon of light, or the rate at which chlorophyll molecules are able to transfer energy electrons to one another will have no effect on either of these parameters. Adding a powerful oxidizing agent might, if anything, interfere with the reduction of the reaction center back to its resting state.

Consider the mechanism by which actin and tubulin polymerize. Which of the items below does not describe something similar about the polymerization mechanisms of actin and microtubules? (a) Although both filaments can grow from both ends, the growth rate is faster at the plus ends. (b) Depolymerization initiates at the plus ends of filaments. (c) Nucleotide hydrolysis promotes depolymerization of filaments. (d) Free subunits (actin and tubulin) carry nucleoside triphosphates.

(b) The shrinkage of microtubules that occurs involves a switch from growth to shrinkage only at the plus end of microtubules. However, actin loses subunits from its minus end during actin treadmilling.

To study how proteins fold, scientists must be able to purify the protein of interest, use solvents to denature the folded protein, and observe the process of refolding at successive time points. What is the effect of the solvents used in the denaturation process? (a) The solvents break all covalent interactions. (b) The solvents break all noncovalent interactions. (c) The solvents break some of the noncovalent ineractions, resulting in a misfolded protein. (d) The solvents create a new protein conformation.

(b) The solvents break all noncovalent interactions.

Studies conducted with a lysozyme mutant that contains an Asp -> Asn change position 52 and a Glu->Gln change at position 35 exhibited almost a complete loss in enzymatic activity. What is the most likely explanation for the decrease in enzyme activity in the mutant? (a) increased affinity for substrate (b) absence of negative charges in the active site (c) change in the active site scaffold (d) larger amino acids in the active site decrease the affinity for substrate

(b) absence of negative charges in the active site

Proteins bind selectively to small molecule targets called ligands. The selection of one ligand out of a mixture of possible ligands depends on the number of weak, noncovalent interactions in the protein's ligand-binding site. Where is the binding site typically located in the protein structure? (a) on the surface of the protein (b) inside a cavity on the protein surface (c) buried in the interior of the protein (d) forms on the surface of the protein in the presence of ligand

(b) inside a cavity on the protein surface

14-48Cytochrome c oxidase is an enzyme complex that uses metal ions to help coordinate the transfer of four electrons to O2. Which metal atoms are found in the active site of this complex? (a)two iron atoms (b)one iron atom and one copper atom (c)one iron atom and one zinc atom (d)one zinc atom and one copper atom

(b) one iron atom and one copper atom

The phosphorylation of a protein is typically associated with a change in activity, the assembly of a protein complex, or the triggering of a downstream signaling cascade. The addition of ubiquitin, a small polypeptide, is another type of covalent modification that can affect the protein function. Ubiquitylation often results in ______________. (a) membrane association (b) protein degradation (c) protein secretion (d) nuclear translocation

(b) protein degradation

14-60The photosystems in chloroplasts contain hundreds of chlorophyll molecules, most of which are part of _______________. (a)plastoquinone. (b)the antenna complex. (c)the reaction center. (d)the ferredoxin complex.

(b) the antenna complex.

It is possible to follow the movement of a single molecule or a small group of molecules. This requires the use of antibodies linked to small particles of gold, which appear as dark spots when tracked through video microscopy. What is this method called? What does the abbreviation stand for? (a) SDS (b) SPT (c) GFP (d) FRAP

(b), single-particle tracking

14-12 Which of the following statements describes the mitochondrial inner membrane? (a)It is permeable to molecules with molecular mass as high as 5000 daltons. (b)It contains transporters for ATP molecules. (c)It contains proteins that are released during apoptosis. (d)It contains enzymes required for the oxidation of fatty acids.

(b)It contains transporters for ATP molecules.

14-59In the electron-transport chain in chloroplasts, ________-energy electrons are taken from __________. (a)high; H2O. (b)low; H2O. (c)high; NADPH. (d)low; NADPH.

(b)low; H2O.

Both glycoproteins and proteoglycans contribute to the carbohydrate layer on the surface of the cell. Which of the following is not true of glycoproteins? (a) They can be secreted into the extracellular environment. (b) They have only one transmembrane domain. (c) They have long carbohydrate chains. (d) They are recognized by lectins.

(c)

Figure Q12-53 illustrates changes in membrane potential during the formation of an action potential. What membrane characteristic or measurement used to study action potentials is indicated by the arrow? (a) effect of a depolarizing stimulus (b) resting membrane potential (c) threshold potential (d) action potential

(c)

Some cells have aquaporins—channels that facilitate the flow of water molecules through the plasma membrane. For these cells, what regulates the rate and direction of water diffusion across the membrane? (a) aquaporin conformation (b) resting membrane potential (c) solute concentrations on either side of the membrane (d) availability of ATP

(c)

The lateral movement of transmembrane proteins can be restricted by several different mechanisms. Which mechanism best describes the process by which neutrophils are recruited by endothelial cells? (a) proteins are tethered to the cell cortex (b) proteins are tethered to the extracellular matrix (c) proteins are tethered to the proteins on the surface of another cell (d) protein movement is limited by the presence of a diffusion barrier

(c)

The lateral movement of transmembrane proteins can be restricted by several different mechanisms. Which mechanism best describes the process by which an antigen-presenting cell triggers an adaptive immune response? (a) proteins are tethered to the cell cortex (b) proteins are tethered to the extracellular matrix (c) proteins are tethered to the proteins on the surface of another cell (d) protein movement is limited by the presence of a diffusion barrier

(c)

The local mediator nitric oxide stimulates the intracellular enzyme guanylyl cyclase by ________________. (a) activating a G protein. (b) activating a receptor tyrosine kinase. (c) diffusing into cells and stimulating the cyclase directly. (d) activating an intracellular protein kinase.

(c)

Three phospholipids X, Y, and Z are distributed in the plasma membrane as indicated in Figure Q11-14. For which of these phospholipids does a flippase probably exist? Figure Q11-14 (a) X only (b) Z only (c) X and Y (d) Y and Z

(c)

When the net charge on either side of the plasma membrane is zero, what else is true? (a) There is an equal number of K+ ions on each side of the plasma membrane. (b) The K+ leak channels are open. (c) The electrochemical potential across the membrane is zero. (d) The resting membrane potential is between -20 mV and -200 mV.

(c)

Which of the following gated ion channels are involved in inhibitory synaptic signaling? (a) voltage-gated Na+ channels (b) voltage-gated Ca2+ channels (c) glycine-gated Cl- channels (d) glutamate-gated cation channels

(c)

Which of the following statements about resting membrane potential is not true? (a) The resting membrane potential for most animal cells is 0 mV, because the positive and negative ions are in balance. (b) The resting membrane potential for most animal cells is positive, because Na+ ions are so plentiful inside cells. (c) The resting membrane potential for most animal cells is negative, because the inside of the cell is more negatively charged than the outside of the cell. (d) At the resting membrane potential, no ions enter or exit the cell.

(c)

Fully folded proteins typically have polar side chains on their surfaces, where electrostatic attractions and hydrogen bonds can form between the polar group on the amino acid and the polar molecules in the solvent. In contrast, some proteins have a polar side chain in their hydrophobic interior. Which of following would not occur to help accommodate an internal, polar side chain? (a) A hydrogen bond forms between two polar side chains. (b) A hydrogen bond forms between a polar side chain and protein backbone. (c) A hydrogen bond forms between a polar side chain and an aromatic side chain. (d) Hydrogen bonds form between polar side chains and a buried water molecule.

(c) A hydrogen bond forms between a polar side chain and an aromatic side chain.

You are examining a cell line in which activation of the Rho family member Rac promotes lamellipodia formation. Which of the following statements is most likely to be true? (a) Cells carrying a Rac mutation that makes Rac act as if it is always bound to GTP will polymerize more unbranched actin filaments than normal cells. (b) Cells carrying a Rac mutation that makes Rac unable to exchange GDP for GTP will polymerize more unbranched actin filaments than normal cells. (c) Cells carrying a Rac mutation that makes Rac act as if it is always bound to GTP will polymerize more branched actin filaments than normal cells. (d) Cells carrying a Rac mutation that makes Rac unable to exchange GDP for GTP will polymerize more branched actin filaments than normal cells.

(c) Activation of Rac promotes lamellipodia formation by enhancing actin nucleation using the ARP complex, which promotes the formation of branched actin filaments. Because lamellipodia formation involves branched actin filaments, a mutation that creates a constitutively active form of Rac (a GTP-bound form of Rac) will promote the formation of a greater number of branched actin filaments. Rac that is mutated and unable to exchange GDP for GTP will not be active.

Which of the following statements is correct? Kinesins and dyneins ___________________. (a) have tails that bind to the filaments. (b) move along both microtubules and actin filaments. (c) often move in opposite directions to each other. (d) derive their energy from GTP hydrolysis.

(c) All other answers are false. The motor heads bind to the filaments [choice (a)]. Both motors move along microtubules [choice (b)] and use ATP hydrolysis for energy [choice (d)].

14-33The relationship of free-energy change (ΔG) to the concentrations of reactants and products is important because it predicts the direction of spontaneous chemical reactions. Consider, for example, the hydrolysis of ATP to ADP and inorganic phosphate (Pi). The standard free-energy change (ΔG°) for this reaction is -7.3 kcal/mole. The free-energy change depends on concentrations according to the following equation: ΔG= ΔG°+ 1.42 log10([ADP] [Pi]/[ATP]) In a resting muscle, the concentrations of ATP, ADP, and Piare approximately 0.005 M, 0.001 M, and 0.010 M, respectively. At [Pi] = 0.010 M, what will be the ratio of [ATP] to [ADP] at equilibrium? (a)1.38 ×106 (b)1 (c)7.2 ×10-8 (d)5.14

(c) At equilibrium, the ΔG is equal to zero by definition. The ratio of [ATP] to [ADP] at equilibrium is less than 1:107. This result is calculated by setting ΔG = 0, so that 1.42 log10 ([ADP] [Pi]/[ATP]) = -ΔG° = 7.3 kcal/mole. Solving for [ADP]/[ATP], the equation becomes log10 ([ADP] [0.010]/[ATP]) = 7.3/1.42 = 5.14; then [ADP]/[ATP] = (105.14)/(0.010) = 13.8 × 106. Thus, the reciprocal [ATP]/[ADP] is 7.2 × 10-8.

Figure Q17-40A shows how the movement of dynein causes the flagellum to bend. If instead of the normal situation, the polarity of the adjacent doublet of microtubules were to be reversed (see Figure Q17-40B), what do you predict would happen? (a) No bending would occur. (b) Bending would occur exactly as diagrammed in Figure Q17-40A. (c) Bending would occur, except that the right microtubule doublet would move down relative to the left one. (d) The two microtubule doublets would slide away from each other.

(c) Because the polarity of the microtubule bundle is reversed, the dynein motors should walk in the opposite direction from the normal situation diagrammed in Figure Q17-40A. Microtubule sliding would occur if the linking proteins were absent [choice (d)], which is not true here.

Figure Q17-52 shows the leading edge of a lamellipodium. Which of the following statements is false? Figure Q17-52 (a) Nucleation of new filaments near the leading edge pushes the plasma membrane forward. (b) ARP proteins nucleate the branched actin filaments in the lamellipodium. (c) Capping proteins bind to the minus end of actin filaments. (d) There is more ATP-bound actin at the leading edge than in the actin filaments away from the leading edge.

(c) Capping protein binds to the plus end of actin filaments, preventing further assembly or disassembly from the growing end.

Which of the following statements is true? (a) Disulfide bonds are formed by the cross-linking of methionine residues. (b) Disulfide bonds are formed mainly in proteins that are retained within the cytosol. (c) Disulfide bonds stabilize but do not change a protein's final conformation. (d) Agents such as mercaptoethanol can break disulfide bonds through oxidation.

(c) Disulfide bonds stabilize but do not change a protein's final conformation.

15-41 Your friend has just joined a lab that studies vesicle budding from the Golgi and has been given a cell line that does not form mature vesicles. He wants to start designing some experiments but wasn't listening carefully when he was told about the molecular defect of this cell line. He's too embarrassed to ask and comes to you for help. He does recall that this cell line forms coated pits but vesicle budding and the removal of coat proteins don't happen. Which of the following proteins might be lacking in this cell line? (a)clathrin (b)Rab (c)dynamin (d)adaptin

(c) Given that coated pits can form but no vesicle budding is seen, dynamin is the most likely answer. Since coated pits are formed, clathrin and adaptin are unlikely to be the answer, because they are involved in the initial shaping of the vesicle into the pit [choices (a) and (d)]. Rab proteins are involved in the recognition of the transport vesicle with its target membrane and not with vesicle budding [choice (b)].

Figure Q16-63 shows how normal signaling works with a Ras protein acting downstream of an RTK. You examine a cell line with a constitutively active Ras protein that is always signaling. Which of the following conditions will turn off signaling in this cell line? Figure Q16-63 (a) addition of a drug that prevents protein X from activating Ras (b) addition of a drug that increases the affinity of protein Y and Ras (c) addition of a drug that blocks protein Y from interacting with its target (d) addition of a drug that increases the activity of protein Y

(c) If protein Y cannot interact with its target, signaling will not occur. Increasing the activity of protein Y [choice (d)] or increasing the affinity of protein Y and Ras [choice (b)] would not turn off signaling. Preventing protein X from activating Ras [choice (a)] would have no effect in a cell line with a constitutively active Ras protein, because Ras is already active.

14-63If you add a compound to illuminated chloroplasts that inhibits the NADP+reductase, NADPH generation ceases, as expected. However, ferredoxin does not accumulate in the reduced form because it is able to donate its electrons not only to NADP+(via NADP+reductase) but also back to the cytochrome b6-fcomplex. Thus, in the presence of the compound, a "cyclic" form of photosynthesis occurs in which electrons flow in a circle from ferredoxin, to the cytochromeb6-fcomplex, to plastocyanin, to photosystem I, to ferredoxin. What will happen if you now also inhibit photosystem II? (a)Less ATP will be generated per photon absorbed.(b)ATP synthesis will cease. (c)Plastoquinone will accumulate in the oxidized form.(d)Plastocyanin will accumulate in the oxidized form.

(c) If you now inhibit photosystem II, you will deprive plastoquinone, which can still donate its electrons to the cytochrome b6-f complex, of an electron source. Hence, plastoquinone will accumulate in its oxidized form. In contrast, all of the other components downstream of plastoquinone will be able to cycle between their oxidized and reduced states. ATP synthesis will continue, because electrons are still being fed through the cytochrome b6-f complex, and the same amount of ATP will be generated.

The intermediates of the citric acid cycle are constantly being depleted because they are used to produce many of the amino acids needed to make proteins. The enzyme pyruvate carboxylase converts pyruvate to oxaloacetate to replenish these intermediates. Bacteria, but not animal cells, have additional enzymes that can carry out the reaction acetyl CoA + isocitrate oxaloacetate + succinate. Which of the following compounds will not support the growth of animal cells when used as the major source of carbon in food, but will support the growth of nonphotosynthetic bacteria? (a) pyruvate (b) glucose (c) fatty acids (d) fructose

(c) In oxidative metabolism, fatty acids can only be converted to acetyl CoA, which is completely oxidized to carbon dioxide through the citric acid cycle. In addition, bacteria can use some of this acetyl CoA as a source of carbon atoms to replenish the components of the citric acid cycle, whereas animals cannot.

A protein kinase can act as an integrating device in signaling if it ___________________. (a) phosphorylates more than one substrate. (b) catalyzes its own phosphorylation. (c) is activated by two or more proteins in different signaling pathways. (d) initiates a phosphorylation cascade involving two or more protein kinases.

(c) Integrating devices are able to relay signals from more than one signaling pathway. Being activated by two or more proteins in different signaling pathways allows a kinase (or any other signaling molecule) to be affected by more than one upstream signal. Choices (a), (b), and (d) affect the output signal that a kinase is able to produce, not its ability to integrate upstream signals from more than one signaling pathway.

15-6Which of the following organelles isnot part of the endomembrane system? (a)Golgi apparatus (b)the nucleus (c)mitochondria (d)lysosomes

(c) Mitochondria are not part of the endomembrane system, which is thought to have arisen initially through invagination of the plasma membrane. Instead, mitochondria (and chloroplasts) are thought to have evolved from a bacterium that was engulfed by a primitive eukaryotic cell.

14-10Which of the following statements about mitochondrial division is true? (a)Mitochondria divide in synchrony with the cell. (b)The rate of mitochondrial division is the same in all cell types. (c)Mitochondrial division is mechanistically similar to prokaryotic cell division. (d)Mitochondria cannot divide and produce energy for the cell at the same time.

(c) Mitochondrial division is mechanistically similar to prokaryotic cell division.

Which of the following statements is true? (a) Peptide bonds are the only covalent bonds that can link together two amino acids in proteins. (b) The polypeptide backbone is free to rotate about each peptide bond. (c) Nonpolar amino acids tend to be found in the interior of proteins. (d) The sequence of the atoms in the polypeptide backbone varies between different proteins.

(c) Nonpolar amino acids tend to be found in the interior of proteins.

14-23In oxidative phosphorylation, ATP production is coupled to the events in the electron-transport chain. What is accomplished in the final electron-transfer event in the electron-transport chain? (a)OH-is oxidized to O2 (b)pyruvate is oxidized to CO2 (c)O2is reduced to H2O (d)NAD+is reduced to NADH

(c) O2 is reduced to H2O

14-25Some bacteria can live both aerobically and anaerobically. How does the ATP synthase in the plasma membrane of the bacterium help such bacteria to keep functioning in the absence of oxygen?

(c) O2 is reduced to H2O

15-48 Cells have oligosaccharides displayed on their cell surface that are important for cell-cell recognition. Your friend discovered a transmembrane glycoprotein, GP1, on a pathogenic yeast cell that is recognized by human immune cells. He decides to purify large amounts of GP1 by expressing it in bacteria. To his purified protein he then adds a branched 14-sugar oligosaccharide to the asparagine of the only Asn-X-Ser sequence found on GP1 (Figure Q15-48). Unfortunately, immune cells do not seem to recognize this synthesized glycoprotein. Which of the following statements is a likely explanation for this problem?Figure Q15-48(a)The oligosaccharide should have been added to the serine instead of the asparagine.(b)The oligosaccharide should have been added one sugar at a time.(c)The oligosaccharide needs to be further modified before it is mature.(d)The oligosaccharide needs a disulfide bond.

(c) Oligosaccharides are usually further modified by enzymes in the ER and the Golgi before the glycoprotein is inserted into the plasma membrane. The other choices are untrue, and thus are not good explanations. Oligosaccharides are added to the Asn and not the Ser [choice (a)] and are added as a branched 14-sugar oligosaccharide [choice (b)]. Disulfide bonding occurs between cysteines of proteins and not in sugars [choice (d)].

The microtubules in a cell form a structural framework that can have all the following functions except which one? (a) holding internal organelles such as the Golgi apparatus in particular positions in the cell (b) creating long, thin cytoplasmic extensions that protrude from one side of the cell (c) strengthening the plasma membrane (d) moving materials from one place to another inside a cell

(c) One function of actin filaments, but not microtubules, is to provide a meshwork beneath the plasma membrane that helps to form and strengthen this membrane. Microtubules have all of the other functions that are listed.

__________ are fairly small organelles that provide a safe place within the cell to carry out certain biochemical reactions that generate harmful, highly reactive oxygen species. These chemicals are both generated and broken down in the same location.

(c) Peroxisomes

15-23Most proteins destined to enter the endoplasmic reticulum_________. (a)are transported across the membrane after their synthesis is complete. (b)are synthesized on free ribosomes in the cytosol.(c)begin to cross the membrane while still being synthesized. (d)remain within the endoplasmic reticulum.

(c) Proteins destined to enter the endoplasmic reticulum have an N-terminal signal sequence that leads to the docking of the ribosome synthesizing the protein onto the ER and the entry of the protein across the ER membrane as the polypeptide chain is being synthesized.

The growth factor Superchick stimulates the proliferation of cultured chicken cells. The receptor that binds Superchick is a receptor tyrosine kinase (RTK), and many chicken tumor cell lines have mutations in the gene that encodes this receptor. Which of the following types of mutation would be expected to promote uncontrolled cell proliferation? (a) a mutation that prevents dimerization of the receptor (b) a mutation that destroys the kinase activity of the receptor (c) a mutation that inactivates the protein tyrosine phosphatase that normally removes the phosphates from tyrosines on the activated receptor (d) a mutation that prevents the binding of the normal extracellular signal to the receptor

(c) RTKs are usually activated by signal-induced dimerization, which allows the receptors to phosphorylate themselves and activate intracellular signaling proteins that are stimulated by the phosphorylated receptor. After it is activated, the receptor is dephosphorylated, and thereby inactivated, by a protein tyrosine phosphatase. Therefore, a mutation in the gene that encodes the protein tyrosine phosphatase will inappropriately increase the activity of the receptor and promote uncontrolled cell proliferation. Mutations that prevent dimerization of the receptor (including mutations that prevent ligand binding) or autophosphorylation (which requires the kinase activity of the receptor) will inactivate the receptor.

15-43Which of the following statements about vesicular membrane fusion is false? (a)Membrane fusion does not always immediately follow vesicle docking. (b)The hydrophilic surfaces of membranes have water molecules associated with them that must be displaced before vesicle fusion can occur. (c)The GTP hydrolysis of the Rab proteins provides the energy for membrane fusion. (d)The interactions of the v-SNAREs and the t-SNAREs pull the vesicle membrane and the target organelle membrane together so that their lipids can intermix.

(c) Rab proteins are important for docking, but are not involved in the catalysis of membrane fusion.

3.41.When the polymer X-X-X... is broken down into monomers, it is "phosphorylyzed" rather than hydrolyzed, in the following repeated reaction: X-X-X... + P X-P + X-X... (reaction 1) Given the ΔG° values of the reactions listed in the following table, what is the expected ratio of X-phosphate (X-P) to free phosphate (P) at equilibrium for reaction 1? (a) 1:106 (b) 1:104 (c) 1:1 (d) 104:1

(c) Reaction 1 can be written as the sum of the three reactions given, because the ATP used in step 2 is restored in step 3. X-X-X... + H2O X + X-X... ΔG° = -4.5 kcal/mole X + ATP X-P + ADP ΔG° = -2.8 kcal/mole ADP + P ATP + H2O ΔG° = +7.3 kcal/mole Because ΔG° values are additive, ΔG°total = 0, and if ΔG° = 0, Keq = 1, meaning that [products]/[reactants] = 1, and the ratio of X-P to P is 1:1.

15-25In which cellular location would you expect to find ribosomes translating mRNAs that encode ribosomal proteins? (a)the nucleus (b)on the rough ER (c)in the cytosol (d)in the lumen of the ER

(c) Ribosomes are cytoplasmic proteins and thus their protein components are translated in the cytosol.

Which of the following structures shorten during muscle contraction? (a) myosin filaments (b) flagella (c) sarcomeres (d) actin filaments

(c) Sarcomeres contain actin filaments and myosin filaments that slide past each other during muscle contraction, leading to shortening of the sarcomere; the actin filaments and myosin filaments do not change in length. Flagella are microtubule-based structures that are not present on muscle cells.

15-13 Signal sequences that direct proteins to the correct compartment are _________. (a)added to proteins through post-translational modification. (b)added to a protein by a protein translocator.(c)encoded in the amino acid sequence and sufficient for targeting a protein to its correct destination. (d)always removed once a protein is at the correct destination.

(c) Signal sequences are found within the amino acid sequence of proteins. They are sometimes removed when the protein is at the correct destination [choice (d)], but not all are removed. For example, nuclear import signals are not removed once a protein is inside the nucleus. A protein translocator resides in the membrane and helps transport soluble proteins across the membrane [choice (b)], but does not add signal sequences to proteins.

15-57 Figure Q15-57 shows the orientation of the Krt1 protein on the membrane of a Golgi-derived vesicle that will fuse with the plasma membrane. Figure Q15-57Given this diagram, which of the following statements is true? (a) When this vesicle fuses with the plasma membrane, the entire Krt1 protein will be secreted into the extracellular space. (b) When this vesicle fuses with the plasma membrane, the C-terminus of Krt1 will be inserted into the plasma membrane. (c) When this vesicle fuses with the plasma membrane, the N-terminus of Krt1 will be in the extracellular space. (d) When this vesicle fuses with the plasma membrane, the N-terminus of Krt1 will be cytoplasmic.

(c) The orientation of Krt1 as the vesicle fuses with the plasma membrane is shown in Figure A15-57. The darker-colored lines in the membrane represent the membranes contributed by the vesicle during fusion.

14-53Which of the following statements about cytochrome c is true? (a)Cytochrome c shuttles electrons between the NADH dehydrogenase complex and cytochrome c reductase complex. (b)When cytochrome c becomes reduced, two cysteines (sulfur-containing amino acids) become covalently bound to a heme group. (c)The pair of electrons accepted by cytochrome care added to the porphyrin ring of the bound heme group. (d)Cytochrome cis the last protein in the electron-transport chain, passing its electrons directly to molecular oxygen, a process that reduces O2to H2O.

(c) The pair of electrons accepted by cytochrome c are added to the porphyrin ring of the bound heme group.

The Na+-K+ ATPase is also known as the Na+-K+ pump. It is responsible for maintaining the high extracellular sodium ion concentration and the high intracellular potassium ion concentration. What happens immediately after the pump hydrolyzes ATP? (a) Na+ is bound (b) ADP is bound (c) the pump is phosphorylated (d) the pump changes conformation

(c) The phosphorylation of the pump causes the conformational change, and it occurs after the binding of Na+

Which of the following statements about the cytoskeleton is false? (a) The cytoskeleton is made up of three types of protein filaments. (b) The cytoskeleton controls the location of organelles in eukaryotic cells. (c) Covalent bonds between protein monomers hold together cytoskeletal filaments. (d) The cytoskeleton of a cell can change in response to the environment.

(c) The protein monomers of the cytoskeleton are held together by noncovalent interactions between the protein monomers. All other statements are true.

15-52Which of the following statements about the unfolded protein response (UPR) is false? (a)Activation of the UPR results in the production of more ER membrane. (b)Activation of the UPR results in the production of more chaperone proteins. (c)Activation of the UPR occurs when receptors in the cytoplasm sense misfolded proteins. (d)Activation of the UPR results in the cytoplasmic activation of gene regulatory proteins.

(c) The receptors for the unfolded proteins are on the ER membrane, and they sense the misfolded proteins using their luminal domains.

Which of the following statements is true? (a) Phospholipids will spontaneously form liposomes in nonpolar solvents. (b) In eukaryotes, all membrane-enclosed organelles are surrounded by one lipid bilayer. (c) Membrane lipids diffuse within the plane of the membrane. (d) Membrane lipids frequently flip-flop between one monolayer and the other.

(c) The remaining answers are false. Phospholipids form bilayers only in polar solvents. Nuclei and mitochondria are enclosed by two membranes. Membrane lipids rarely flip- flop between one monolayer and the other.

Which of the following statements about the carbohydrate coating of the cell surface is false? (a) It is not usually found on the cytosolic side of the membrane. (b) It can play a role in cell-cell adhesion. (c) The arrangement of the oligosaccharide side chains is highly ordered, much like the peptide bonds of a polypeptide chain. (d) Specific oligosaccharides can be involved in cell-cell recognition.

(c) The sugars in an oligosaccharide side chain attached to the cell surface can be joined together in many different ways and in varied sequences.

14-64The enzyme ribulose bisphosphate carboxylase (Rubisco) normally adds carbon dioxide to ribulose 1,5-bisphosphate. However, it will also catalyze a competing reaction in which O2is added to ribulose 1,5-bisphosphate to form 3-phosphoglycerate and phosphoglycolate. Assume that phosphoglycolate is a compound that cannot be used in any further reactions. If O2and CO2have the same affinity for Rubisco, which of the following is the lowest ratio of CO2to O2at which a net synthesis of sugar can occur? (a)1:3 (b)1:2 (c)3:1 (d)2:1

(c) Three molecules of O2 are required to form three molecules of 3-phosphoglycerate and three molecules of phosphoglycolate. To break even (that is, simply to keep the Calvin cycle going with no net sugar produced), you need to have enough 3-phosphoglycerate to synthesize ribulose 1,5-bisphosphate again. Therefore, for every three molecules of O2 that react with ribulose 1,5-bisphosphate, you need to generate two additional molecules of 3-phosphoglycerate. For every three molecules of CO2 that go into the Calvin cycle, one molecule of 3-phosphoglycerate is formed. So if you have at least six molecules of CO2 per three molecules of O2 going through the Calvin cycle, you will break even. Only if you have a ratio of CO2 to O2 higher than 6:3 (2:1) can you have a net synthesis of carbohydrate.

15-18 Your friend works in a biotechnology company and has discovered a drug that blocks the ability of Ran to exchange GDP for GTP. What is the most likely effect of this drug on nuclear transport? (a)Nuclear transport receptors would be unable to bind cargo. (b)Nuclear transport receptors would be unable to enter the nucleus. (c)Nuclear transport receptors would be unable to release their cargo in the nucleus. (d)Nuclear transport receptors would interact irreversibly with the nuclear pore fibrils.

(c) When Ran-GTP binds to the nuclear transport receptor, cargo is released. If Ran could not exchange its GDP for GTP, this would not happen. Ran-GTP is not needed for cargo binding, for nuclear entry, or for interactions with the nuclear pore fibrils during nuclear import.

Which of the following mechanisms best describes the manner in which lysozyme lowers the energy required for its substrate to reach its transition state conformation? (a) by binding two molecules and orienting them in a way that favors a reaction between them (b) by altering the shape of the substrate to mimic the conformation of the transition state (c) by speeding up the rate at which water molecules collide with the substrate (d) by binding irreversibly to the substrate so that it cannot dissociate

(c) by speeding up the rate at which water molecules collide with the substrate

Which of the following methods used to study proteins is limited to proteins with a molecular weight of 50 kD or less? (a) x-ray crystallography (b) fingerprinting (c) nuclear magnetic resonance (d) mass spectroscopy

(c) nuclear magnetic resonance

14-52Which of the following is not an electron carrier that participates in the electron-transport chain? (a)cytochrome (b)quinone (c)rhodopsin (d)copper ion

(c) rhodopsin

Membranes undergo spontaneous rearrangement if torn. Which of the following would happen if a cell membrane had a large tear? Figure 11-17

(c) vesicles form

A hungry yeast cell lands in a vat of grape juice and begins to feast on the sugars there, producing carbon dioxide and ethanol in the process: C6H12O6 + 2ADP + 2Pi + H+ ! 2CO2 + 2CH3CH2OH + 2ATP + 2H2O Unfortunately, the grape juice is contaminated with proteases that attack some of the transport proteins in the yeast cell membrane, and the yeast cell dies. Which of the following could account for the yeast cell's demise? (a) toxic buildup of carbon dioxide inside the cell (b) toxic buildup of ethanol inside the cell (c) diffusion of ATP out of the cell (d) inability to import sugar into the cell

(d)

Cholesterol serves several essential functions in mammalian cells. Which of the following is not influenced by cholesterol? (a) membrane permeability (b) membrane fluidity (c) membrane rigidity (d) membrane thickness

(d)

Diversity among the oligosaccharide chains found in the carbohydrate coating of the cell surface can be achieved in which of the following ways? (a) varying the types of sugar monomers used (b) varying the types of linkages between sugars (c) varying the number of branches in the chain (d) all of the above

(d)

During nervous-system development in Drosophila, the membrane-bound protein Delta acts as an inhibitory signal to prevent neighboring cells from developing into neuronal cells. Delta is involved in ______________ signaling. (a) endocrine (b) paracrine (c) neuronal (d) contact-dependent

(d)

Figure Q12-55 illustrates changes in membrane potential during the formation of an action potential. What membrane characteristic or measurement used to study action potentials is indicated by the arrow? (a) effect of a depolarizing stimulus (b) resting membrane potential (c) threshold potential (d) action potential

(d)

In a method called patch-clamping, a glass capillary can be converted into a microelectrode that measures the electrical currents across biological membranes. Which of the following is not true about the patch-clamp method? (a) The glass capillary adheres to a "patch" of membrane through the application of suction. (b) The aperture in the glass capillary used to make a microelectrode is about 1 μm in diameter. (c) If the experimental conditions are held constant, fluctuations in electrical currents across the patch of membrane are still observed. (d) Single-channel patch-clamp recordings have demonstrated that gated membrane channels will only open and close in response to specific stimuli.

(d)

K+ leak channels are found in the plasma membrane. These channels open and close in an unregulated, random fashion. What do they accomplish in a resting cell? (a) They set the K+ concentration gradient to zero. (b) They set the membrane potential to zero. (c) They disrupt the resting membrane potential. (d) They keep the electrochemical gradient for K+ at zero.

(d)

Membrane curvature is influenced by the differential lipid composition of the two membrane monolayers. Which factor do you think has the largest impact on the curvature of biological membranes? (a) amount of cholesterol (b) charge of the lipid head group (c) length of the hydrocarbon tails (d) size of the lipid head group

(d)

Some lipases are able to cleave the covalent bonds between the glycerol backbone and the attached fatty acid. What final products do you expect to accumulate through the action of the enzyme monoacylglycerol lipase? (a) phosphoglycerol and free fatty acid (b) sterol and glycerol (c) free phosphate and glycerol (d) glycerol and free fatty acid

(d)

The lateral movement of transmembrane proteins can be restricted by several different mechanisms. Which mechanism best describes the process by which nutrients are taken up at the apical surface of the epithelial cells that line the gut and released from their basal and lateral surfaces? (a) proteins are tethered to the cell cortex (b) proteins are tethered to the extracellular matrix (c) proteins are tethered to the proteins on the surface of another cell (d) protein movement is limited by the presence of a diffusion barrier

(d)

The length of time a G protein will signal is determined by _______. (a) the activity of phosphatases that turn off G proteins by dephosphorylating Gα. (b) the activity of phosphatases that turn GTP into GDP. (c) the degradation of the G protein after Gα separates from Gβγ. (d) the GTPase activity of Gα.

(d)

Which of the following statements about GABA receptors is not true? (a) They are located on postsynaptic membranes. (b) They are ligand-gated channels. (c) They inhibit synaptic signaling. (d) They promote neuronal uptake of Na+

(d)

Motor proteins use the energy in ATP to transport organelles, rearrange elements of the cytoskeleton during cell migration, and move chromosomes during cell division. Which of the following mechanisms is sufficient to ensure the unidirectional movement of a motor protein along its substrate? (a) A conformational change is coupled to the release of a phosphate (Pi). (b) The substrate on which the motor moves has a conformational polarity. (c) A conformational change is coupled to the binding of ADP. (d) A conformational change is linked to ATP hydrolysis.

(d) A conformational change is linked to ATP hydrolysis.

Adrenaline stimulates glycogen breakdown in skeletal muscle cells by ultimately activating glycogen phosphorylase, the enzyme that breaks down glycogen, as depicted in Figure Q16-36. Figure Q16-36 Which of the following statements is false? (a) A constitutively active mutant form of PKA in skeletal muscle cells would lead to a decrease in the amount of unphosphorylated phosphorylase kinase. (b) A constitutively active mutant form of PKA in skeletal muscle cells would not increase the affinity of adrenaline for the adrenergic receptor. (c) A constitutively active mutant form of PKA in skeletal muscle cells would lead to an excess in the amount of glucose available. (d) A constitutively active mutant form of PKA in skeletal muscle cells would lead to an excess in the amount of glycogen available.

(d) A constitutively active mutant form of PKA in skeletal muscle cells would lead to a decrease in the amount of glycogen available, because active PKA stimulates enzymes that are responsible for the breakdown of glycogen so that glucose can be produced. All the other statements are true.

For both actin and microtubule polymerization, nucleotide hydrolysis is important for ______. (a) stabilizing the filaments once they are formed. (b) increasing the rate at which subunits are added to the filaments. (c) promoting nucleation of filaments. (d) decreasing the binding strength between subunits on filaments.

(d) ATP hydrolysis in actin polymerization decreases the binding strength between monomers in the actin filaments; GTP hydrolysis during tubulin polymerization decreases the binding strength between the tubulin subunits in the microtubule.

14-4Which of the following is not part of the process known as oxidative phosphorylation? (a) Molecular oxygen serves as a final electron acceptor. (b)FADH2and NADH become oxidized as they transfer a pair of electrons to the electron-transport chain. (c)The electron carriers in the electron-transport chain toggle between reduced and oxidized states as electrons are passed along. (d)ATP molecules are produced in the cytosol as glucose is converted into pyruvate.

(d) ATP molecules are produced in the cytosol as glucose is converted into pyruvate.

15-40Which of the following protein families are not involved in directing transport vesicles to the target membrane? (a)SNAREs (b)Rabs (c)tethering proteins (d)adaptins

(d) Adaptins are involved in vesicle budding and are removed during the uncoating process, and thus should not be present when the vesicle reaches its target.

Which of the following statements about G-protein-coupled receptors (GPCRs) is false? (a) GPCRs are the largest family of cell-surface receptors in humans. (b) GPCRs are used in endocrine, paracrine, and neuronal signaling. (c) GPCRs are found in yeast, mice, and humans. (d) The different classes of GPCR ligands (proteins, amino acid derivatives, or fatty acids) bind to receptors with different numbers of transmembrane domains.

(d) Although it is true that many types of ligands can bind to and activate GPCRs, all GPCRs have a similar structure with seven transmembrane domains.

15-47 Which of the following statements about disulfide bond formation is false? (a)Disulfide bonds do not form under reducing environments. (b)Disulfide bonding occurs by the oxidation of pairs of cysteine side chains on the protein. (c)Disulfide bonding stabilizes the structure of proteins. (d)Disulfide bonds form spontaneously within the ER because the lumen of the ER is oxidizing.

(d) An enzyme in the ER lumen catalyzes disulfide bond formation.

How We Know: How Chemiosmotic Coupling Drives ATP Synthesis 14-36Experimental evidence supporting the chemiosmotic hypothesis was gathered by using artificial vesicles containing a protein that can pump protons in one direction across the vesicle membrane to create a proton gradient. Which protein was used to generate the gradient in a highly controlled manner? (a)cytochrome c oxidase (b)NADH dehydrogenase (c)cytochrome c (d)bacteriorhodopsin

(d) Bacteriorhodopsin is a transmembrane protein that pumps protons across the membrane when exposed to light. The other proteins pump protons but are part of the electron-transport chain, so they would not be good options to show that it is only the proton gradient that is required in this system rather than any specific intermediate in the electron-transport chain.

14-70 Below is a list of breakthroughs in energy metabolism in living systems. Which is the correct order in which they are thought to have evolved? A.H2O-splitting enzyme activity B.light-dependent transfer of electrons from H2S to NADPH C.the consumption of fermentable organic acids D.oxygen-dependent ATP synthesis (a)A, C, D, B (b)C, A, B, D (c)B, C, A, D (d)C, B, A, D

(d) C, B, A, D

Which of the following statements about actin is false? (a) ATP hydrolysis decreases actin filament stability. (b) Actin at the cell cortex helps govern the shape of the plasma membrane. (c) Actin filaments are nucleated at the side of existing actin filaments in lamellipodia. (d) The dynamic instability of actin filaments is important for cell movement.

(d) Dynamic instability is a phenomenon associated with microtubules and not actin. Actin disassembly and assembly are both important for cell movement. However, this differs from dynamic instability in that the growth of actin filaments occurs at the leading edge; this growth occurs in a directed fashion because of actin-binding proteins that promote the formation of new filaments at the leading edge. Actin-binding proteins that destabilize actin filaments promote actin disassembly away from the leading edge. The actin assembly and disassembly in moving cells differs from the stochastic growth and disassembly of the microtubules.

Below is a list of molecules with different chemical characteristics. Knowing that all molecules will eventually diffuse across a phospholipid bilayer, select the option below that most accurately predicts the relative rates of diffusion of these molecules (fastest to slowest). alanine estrogen propanol sodium (a) alanine > propanol > sodium > estrogen (b) sodium > propanol > alanine > estrogen (c) estrogen > propanol > sodium > alanine (d) estrogen > propanol > alanine > sodium

(d) Estrogen is a steroid hormone and will diffuse the fastest across the membrane. Propanol is a small, uncharged molecule with a polar group. Alanine is an amino acid, and although it has a small, nonpolar side group, amino acids are charged molecules. Sodium is an ion and will move the slowest across the bilayer.

The Aeroschmidt weed contains an ATP-driven ion pump in its vacuolar membrane that pumps potentially toxic heavy metal ions such as Zn2+ and Pb2+ into the vacuole. The pump protein exists in a phosphorylated and an unphosphorylated form and works in a similar way to the Na+-K+ pump of animal cells. To study its action, you incorporate the unphosphorylated form of the protein into phospholipid vesicles containing K+ in their interiors. (You ensure that all of the protein molecules are in the same orientation in the lipid bilayer.) When you add Zn2+ and ATP to the solution outside such vesicles, you find that Zn2+ is pumped into the vesicle lumen. You then expose vesicles containing the pump protein to the solutes shown in Table Q12-25A. Table Q12-25A You then determine the amount of phosphorylated and unphosphorylated ATP- driven ion pump protein in each sample. Your results are summarized in Table Q12-25B, where a minus sign indicates an absence of a type of protein and a plus sign indicates its presence. Table Q12-25B What would you expect to happen if you treat vesicles as in lane F, but before determining the phosphorylation state of the protein, you wash away the outside buffer and replace it with a buffer containing only Zn2+? (a) Nothing will happen. (No Zn2+ will move into the vesicle; no K+ will move out of the vesicle; the phosphorylation state of the protein will not change.) (b) No Zn2+ will move into the vesicle; no K+ will move out of the vesicle; the protein will become unphosphorylated. (c) A small amount of Zn2+ will move into the vesicle; no K+ will move out of the vesicle; the phosphorylation state of the protein will not change. (d) A small amount of Zn2+ will move into the vesicle; no K+ will move out of the vesicle; the protein will become unphosphorylated.

(d) If the pump is mechanistically similar to the Na+-K+ pump, then the transport of ions is driven by ATP hydrolysis and the pump is transiently phosphorylated; phosphorylation is stimulated by one ion and dephosphorylation is stimulated by the other ion. Because all of the protein is in the phosphorylated form in the absence of Zn2+ (lane F), Zn2+ is probably required for dephosphorylation. K+, then, probably binds to the dephosphorylated form and stimulates the ATPase/autophosphorylation. So, if Zn2+ is added to the phosphorylated pump, Zn2+ will stimulate dephosphorylation, trigger a conformational change, and be injected into the vesicle. K+ will stimulate the kinase activity of the pump, but because there is no ATP to be hydrolyzed in the interior of the vesicle, no phosphorylation and hence no movement of K+ will occur.

Both excitatory and inhibitory neurons form junctions with muscles. By what mechanism do inhibitory neurotransmitters prevent the postsynaptic cell from firing an action potential? (a) by closing Na+ channels (b) by preventing the secretion of excitatory neurotransmitters (c) by opening K+ channels (d) by opening Cl- channels

(d) Inhibitory neurons release inhibitory neurotransmitters such as GABA and glycine. They bind to and open ligand-gated Cl- channels. If Na+ channels are open, Cl- ions will rush into the cell as well, neutralizing the positive charges carried by Na+

14-65 Which of the following statements is not trueabout the possible fates of glyceraldehyde 3-phosphate? (a)It can be exported from the chloroplast to the cytosol for conversion into sucrose. (b)It can be used to make starch, which is stored inside the stroma of the chloroplast. (c)It can be used as a precursor for fatty acid synthesis and stored as fat droplets in the stroma. (d)It can be transported into the thylakoid space for use as a secondary electron acceptor downstream of the electron-transport chain.

(d) It can be transported into the thylakoid space for use as a secondary electron acceptor downstream of the electron-transport chain.

14-14Which of the following statements describes the mitochondrial matrix? (a)It is permeable to molecules with molecular mass as high as 5000 daltons. (b)It contains transporters for ATP molecules. (c)It contains proteins that are released during apoptosis. (d)It contains enzymes required for the oxidation of fatty acids.

(d) It contains enzymes required for the oxidation of fatty acids.

Your friend works in a biotech company that has just discovered a drug that seems to promote lamellipodia formation in cells. Which of the following molecules is unlikely to be involved in the pathway that this drug affects? (a) Rac (b) ARP (c) actin (d) myosin

(d) Myosins are not directly involved in lamellipodia formation. Lamellipodium formation involves branched actin structures that use ARP for their formation, and thus ARP and actin are likely to be involved [choices (b) and (c)]. The Rho family member Rac triggers lamellipodia formation when activated, and thus may be involved [choice (a)].

15-10Where are proteins in the chloroplast synthesized? (a)in the cytosol (b)in the chloroplast (c)on the endoplasmic reticulum (d)in both the cytosol and the chloroplast

(d) Proteins in the chloroplast are synthesized in the cytosol and in the chloroplast. The chloroplast proteins that are encoded by the nuclear DNA are synthesized in the cytosol, and the sorting signals on the protein direct them to the chloroplast. The chloroplast proteins encoded by the chloroplast DNA are synthesized on ribosomes inside the chloroplast.

Pyruvate can be converted into many other molecules by various biosynthetic and metabolic pathways, which makes it a central hub in the regulation of cellular metabolism. Which of the following molecules is not made from pyruvate? (a) oxaloacetate (b) ethanol (c) lactate (d) NADH

(d) Pyruvate cannot be converted into NADH, but it can be converted into the other metabolites in one or two steps.

The stimulation of auditory nerves depends on the opening and closing of channels in the auditory hair cells. Which type of gating mechanism do these cells use? (a) voltage-gated (b) extracellular ligand-gated (c) intracellular ligand-gated (d) stress-gated

(d) Sound waves cause vibrations of the tectorial membrane. These vibrations cause the bundles of stereocilia to tilt. This tilting physically pulls the filament that links a cilium to the ion channel in neighboring cilia, which then pulls the gate on that ion channel open.

Which of the following statements is true? (a) Amoebae have transporter proteins that actively pump water molecules from the cytoplasm to the cell exterior. (b) Bacteria and animal cells rely on the Na+-K+ pump in the plasma membrane to prevent lysis resulting from osmotic imbalances. (c) The Na+-K+ pump allows animal cells to thrive under conditions of very low ionic strength. (d) The Na+-K+ pump helps to keep both Na+ and Cl- ions out of the cell.

(d) The Na+-K+ pump keeps Na+ out directly by pumping it out and keeps Cl- out indirectly by helping to maintain the negative membrane potential. Cells do not have pumps for moving water molecules across the membrane [choice (a)], since the lipid bilayer is permeable to water. Bacteria do not have Na+-K+ pumps in their plasma membranes [choice (b)]. The Na+-K+ pump cannot directly remove water molecules from the cell; it helps maintain osmotic balance by pumping out the Na+ that leaks in, which would not help if the cell were in a solution of very low ionic strength [choice (c)].

The citric acid cycle is a series of oxidation reactions that removes carbon atoms from substrates in the form of CO2. Once a molecule of acetyl CoA enters the citric acid cycle, how many complete cycles are required for both of the carbon atoms in its acetyl groupto be oxidized to CO2? (a) 1 (b) 2 (c) 3 (d) 4

(d) The carbons from acetyl CoA are carried through the entire first reaction cycle. In the second cycle, one of these carbons is oxidized to CO2. Two more cycles later, the second carbon molecule from that same acetyl CoA is oxidized to CO2.

Cells make use of H+ electrochemical gradients in many ways. Which of the following proton transporters is used to regulate pH in animal cells? (a) light-driven pump (b) H+ ATPase (c) H+ symporter (d) Na+-H+ exchanger

(d) The high extracellular concentration of Na+ is employed by a transporter that pumps protons out of animal cells as Na+ is brought into the cell. The other transporters are found only in bacterial cells.

The hydrolysis of GTP to GDP carried out by tubulin molecules ________________. (a) provides the energy needed for tubulin to polymerize. (b) occurs because the pool of free GDP has run out. (c) tips the balance in favor of microtubule assembly. (d) allows the behavior of microtubules called dynamic instability.

(d) The hydrolysis of GTP to GDP occurs after a GTP-bound tubulin molecule is incorporated into a microtubule, and it makes the microtubule more susceptible to disassembly. It is the resulting switch in microtubule stability that gives rise to the phenomenon known as dynamic instability.

Which of the following statements does not accurately describe the events involved in the propagation of an action potential? (a) An initial influx of Na+ through a small cluster of channels causes local depolarization of the membrane. (b) Local depolarization causes nearby Na+ channels to open. (c) Channels in depolarized regions of the membrane are inactivated until the resting membrane potential is reestablished. (d) The opening of transmitter-gated K+ channels helps to repolarize the membrane.

(d) The outflow of K+ through voltage-gated K+ channels helps bring the membrane back to its resting potential.

You are interested in understanding the regulation of nuclear lamina assembly. To create an in vitro system for studying this process you start with partly purified nuclear lamina subunits to which you will add back purified cellular components to drive nuclear lamina assembly. Before you start doing experiments, your instructor suggests that you consider what type of conditions would be most amenable to the assembly of the nuclear lamina from its individual subunits in vitro. Which of the following conditions do you predict would be most likely to enhance the assembly of the nuclear lamina? (a) addition of phosphatase inhibitors (b) addition of ATP (c) addition of a concentrated salt solution that is 10 times the concentration normally found in the nucleoplasm (d) addition of protein kinase inhibitors

(d) The phosphorylation of nuclear lamins by protein kinases induces conformational changes that weaken the binding between nuclear lamin tetramers; thus, inhibiting protein kinases may enhance assembly of the nuclear lamina. Adding phosphatase inhibitors [choice (a)] or ATP [choice (b)] will enhance the activity of any co-purifying protein kinases and enhance disassembly. Because noncovalent protein-protein interactions hold the nuclear lamina together, the addition of a very concentrated salt solution will inhibit proper nuclear lamina assembly [choice (c)].

Cell movement involves the coordination of many events in the cell. Which of the following phenomena is not required for cell motility? (a) Myosin-mediated contraction at the rear of the moving cell. (b) Integrin association with the extracellular environment. (c) Nucleation of new actin filaments. (d) Release of Ca2+ from the sarcoplasmic reticulum.

(d) The release of Ca2+ from the sarcoplasmic reticulum is important for muscle contraction, not cell motility.

15-24 After isolating the rough endoplasmic reticulum from the rest of the cytoplasm, you purify the RNAs attached to it. Which of the following proteins do you expect the RNA from the rough endoplasmic reticulum to encode? (a)soluble secreted proteins (b)ER membrane proteins (c)plasma membrane proteins (d)all of the above

(d) The rough ER consists of ER membranes and polyribosomes that are in the process of translating and translocating proteins into the ER membrane and lumen. Thus, all proteins that end up in the lysosome, Golgi apparatus, or plasma membrane, or are secreted, will be encoded by the RNAs associated with the rough ER.

14-32 The relationship of free-energy change (ΔG) to the concentrations of reactants and products is important because it predicts the direction of spontaneous chemical reactions. In the hydrolysis of ATP to ADP and inorganic phosphate (Pi), the standard free-energy change (ΔG°) is -7.3 kcal/mole. The free-energy change depends on concentrations according to the following equation: ΔG= ΔG°+ 1.42 log10([ADP] [Pi]/[ATP]) In a resting muscle, the concentrations of ATP, ADP, and Piare approximately 0.005 M, 0.001 M, and 0.010 M, respectively. What is the ΔGfor ATP synthesis in resting muscle? (a)-6.01 kcal/mole (b)5.88 kcal/mole (c)8.72 kcal/mole (d)11.1kcal/mole

(d) The ΔG for ATP hydrolysis is -11.1 kcal/mole. This result is calculated by substituting values into the equation given: ΔG = -7.3 kcal/mole + 1.42 log10 ([0.001 M] [0.010 M]/[0.005 M]) = -7.3 kcal/mole + 1.42 log10 (0.002) = -11.1 kcal/mole. The ΔG for synthesis is +11.1 kcal/mole because the forward and reverse reactions always have the same numerical value for ΔG but with the sign reversed.

The correct folding of proteins is necessary to maintain healthy cells and tissues. Unfolded proteins are responsible for such neurodegenerative disorders as Alzheimer's, Huntington's, and Creutzfeld-Jacob disease (the specific faulty protein is different for each disease). What is the ultimate fate of these disease-causing, unfolded proteins? (a) They are degraded. (b) They bind a different target protein. (c) They form structured filaments. (d) They form protein aggregates.

(d) They form protein aggregates.

Which of the following is required for the secretion of neurotransmitters in response to an action potential? (a) neurotransmitter receptors (b) Na+-K+ pumps (c) voltage-gated K+ channels (d) voltage-gated Ca2+ channels

(d) Voltage-gated Ca2+ channels open in response to the depolarization caused as the action potential moves toward the nerve terminal. The influx of calcium from outside the cell causes the synaptic vesicles to fuse with the plasma membrane and release large amounts of neurotransmitter into the synaptic cleft.

The Ras protein is a GTPase that functions in many growth-factor signaling pathways. In its active form, with GTP bound, it transmits a downstream signal that leads to cell proliferation; in its inactive form, with GDP bound, the signal is not transmitted. Mutations in the gene for Ras are found in many cancers. Of the choices below, which alteration of Ras activity is most likely to contribute to the uncontrolled growth of cancer cells? (a) a change that prevents Ras from being made (b) a change that increases the affinity of Ras for GDP (c) a change that decreases the affinity of Ras for GTP (d) a change that decreases the rate of hydrolysis of GTP by Ras

(d) a change that decreases the rate of hydrolysis of GTP by Ras

Protein structures have several different levels of organization. The primary structure of a protein is its amino acid sequence. The secondary and tertiary structures are more complicated. Consider the definitions below and select the one that best fits the term "protein domain." (a) a small cluster of Alpha helices and Beta sheets (b) the tertiary structure of a substrate-binding pocket (c) a complex of more than one polypeptide chain (d) a protein segment that folds independently

(d) a protein segment that folds independently

Lysozyme is an enzyme that specifically recognizes bacterial polysaccharides, which renders it an effective antibacterial agent. Into what classification of enzymes does lysozyme fall? (a) isomerase (b) protease (c) nuclease (d) hydrolase

(d) hydrolase

Energy required by the cell is generated in the form of ATP. ATP is hydrolyzed to power many of the cellular processes, increasing the pool of ADP. ADP molecules then bind to glycolytic enzymes, which will lead to the production of more ATP. The best way to describe how oxidation energy is converted to ATP energy during glycolysis is by ___________. (a) feedback inhibition (b) allosteric conformation (c) allosteric activation (d) substrate-level phosphorylation

(d) substrate-level phosphorylation

Secretory Pathways 15-45 N-linked oligosaccharides on secreted glycoproteins are attached to________. (a)nitrogenatoms in the polypeptide backbone. (b)the serine or threonine in the sequence Asn-X-Ser/Thr. (c)the N-terminus of the protein. (d)the asparagine in the sequence Asn-X-Ser/Thr.

(d) the asparagine in the sequence Asn-X-Ser/Thr.

The flow of genetic information is controlled by a series of biochemical reactions that result in the production of proteins, each with its own specific order of amino acids. Choose the correct series of biochemical reactions from the options presented here.

-None of these is correct. transcription, translation, replication translation, transcription, replication translation, replication, transcription replication, translation, transcription

16. what is the smallest distance two points can be separated and still resolved using light microscopy?

0.2 um

How many FADH2 have been formed during the citric acid cycle?

1 per pyruvate

How many ATPs are formed from the citric acid cycle?

1 per pyruvate.

What concentration of hydronium ions does a solution of pH 8 contain?

1 x 10-8 M

D

1) If a particular operon encodes enzymes for making an essential amino acid and is regulated like the trp operon, then the A) amino acid inactivates the repressor. B) enzymes produced are called inducible enzymes. C) repressor is active in the absence of the amino acid. D) amino acid acts as a corepressor. E) amino acid turns on transcription of the operon.

What are the three necessary steps in going from glycolysis to the citric acid cycle?

1) Pyruvate's carboxyl group is removed and given off as CO2. 2) Enxyme transfers extracted electrons to NAD+, storing them as NADH. 3) CoA is attached to acetate by an unstable bond forming acetyle CoA

A

1) What does the operon model attempt to explain? A) the coordinated control of gene expression in bacteria B) bacterial resistance to antibiotics C) how genes move between homologous regions of DNA D) the mechanism of viral attachment to a host cell E) horizontal transmission of plant viruses

What do nucleotides consist of?

1. Nitrogenous base 2. Pentose sugar 3. Phosphate group

15-46 Name two types of protein modification that can occur in the ER but not in the cytosol.

1. Proteins in the ER can undergo disulfide bond formation. (This does not occur in the cytosol because of its reducing environment.) 2. Proteins in the ER can undergo glycosylation. (Glycosylating enzymes are not found in the cytosol.) (Signal-sequence cleavage is also an acceptable answer, although not really what this question is referring to.)

15-62 Name three possible fates for an endocytosed molecule that has reached the endosome

1. recycled to the original membrane 2. destroyed in the lysosome 3. transcytosed across the cell to a different membrane

What information regarding an enzyme-catalyzed reaction is obtained in a plot of the inverse of the initial velocities against the inverse of the corresponding substrate concentrations?

1/Vmax and 1/Km

B

10) Proto-oncogenes can change into oncogenes that cause cancer. Which of the following best explains the presence of these potential time bombs in eukaryotic cells? A) Proto-oncogenes first arose from viral infections. B) Proto-oncogenes normally help regulate cell division. C) Proto-oncogenes are genetic "junk." D) Proto-oncogenes are mutant versions of normal genes. E) Cells produce proto-oncogenes as they age.

47. On average, eukaryotic cells are BLANK times longer and have BLANK times more volume than prokaryotic cells

10, 1000

You have a concentrated stock solution of 10 M NaOH and want to use it to produce a 200 mL solution of 2.5 M NaOH. What volume of water and stock solutions will you measure out to make this new solution?

105 mL of water, 45 mL of NaOH stock

2.38.b. what concentration of hydronium ions does a solution of pH 8 contain?

10^-8 M

D

11) How does active CAP induce expression of the genes of the lactose operon? A) It terminates production of repressor molecules. B) It degrades the substrate allolactose. C) It stimulates splicing of the encoded genes. D) It stimulates the binding of RNA polymerase to the promoter. E) It binds steroid hormones and controls translation.

C

12) For a repressible operon to be transcribed, which of the following must occur? A) A corepressor must be present. B) RNA polymerase and the active repressor must be present. C) RNA polymerase must bind to the promoter, and the repressor must be inactive. D) RNA polymerase cannot be present, and the repressor must be inactive. E) RNA polymerase must not occupy the promoter, and the repressor must be inactive.

B

13) Allolactose induces the synthesis of the enzyme lactase. An E. coli cell is presented for the first time with the sugar lactose (containing allolactose) as a potential food source. Which of the following occurs when the lactose enters the cell? A) The repressor protein attaches to the regulator. B) Allolactose binds to the repressor protein. C) Allolactose binds to the regulator gene. D) The repressor protein and allolactose bind to RNA polymerase. E) RNA polymerase attaches to the regulator.

C

14) Altering patterns of gene expression in prokaryotes would most likely serve the organism's survival in which of the following ways? A) organizing gene expression so that genes are expressed in a given order B) allowing each gene to be expressed an equal number of times C) allowing the organism to adjust to changes in environmental conditions D) allowing young organisms to respond differently from more mature organisms E) allowing environmental changes to alter the prokaryote's genome

The calvin cycle must be turned three times. Each turn will require a starting molecule of ribulose biphosphate, a five-carbon compound. This means we start with ___ carbons distributed in 3 RUBPs.

15

There are two properties of phospholipids that affect how tightly they pack together: the length of the hydrocarbon chain and the number of double bonds. The degree of packing, in turn, influences the relative mobility of these molecules in the membrane. Which of the following would yield the most highly mobile phospholipid (listed as number of carbons and number of double bonds, respectively)?

15 carbons with 2 double bonds

B

15) In response to chemical signals, prokaryotes can do which of the following? A) turn off translation of their mRNA B) alter the level of production of various enzymes C) increase the number and responsiveness of their ribosomes D) inactivate their mRNA molecules E) alter the sequence of amino acids in certain proteins

D

17) If she moves the operator to the far end of the operon (past the transacetylase gene), which of the following would likely occur when the cell is exposed to lactose? A) The inducer will no longer bind to the repressor. B) The repressor will no longer bind to the operator. C) The operon will never be transcribed. D) The structural genes will be transcribed continuously. E) The repressor protein will no longer be produced.

15-30Match the components involved with ER transport with the appropriate cellular location. Locations can be used more than once, or not at all. Components 1. signal-recognition particle _____ 2. protein translocator _____ 3. mRNA _____ 4. SRP receptor _____ 5. active site of signal peptidase ____ Location A. cytosol B. ER lumen C. ER membrane

1—A(cytosol); 2—C(ER membrane); 3—A(cytosol); 4—C(ER membrane0; 5—B(ER Lumen)

Match the numbered lines in the diagram with the following structures: A. nerve terminal B. cell body C. axon D. dendrite

1—B; 2—D; 3—C; 4—A

How many molecules of FADH2s are formed from 1 molecule of glucose?

2

How many protein subunit binding sites are needed to form helices?

2

Glycolysis generates more stored energy than it expends. What is the net number of activated carrier molecules produced in this process (number and type of molecules produced minus the number of those molecules used as input)?

2 ATP, 2 NADH

How many total carbons are lost as pyruvate is oxidized during the citric acid cycle?

2 per pyruvate.

A

2) Muscle cells differ from nerve cells mainly because they A) express different genes. B) contain different genes. C) use different genetic codes. D) have unique ribosomes. E) have different chromosomes.

E

2) The role of a metabolite that controls a repressible operon is to A) bind to the promoter region and decrease the affinity of RNA polymerase for the promoter. B) bind to the operator region and block the attachment of RNA polymerase to the promoter. C) increase the production of inactive repressor proteins. D) bind to the repressor protein and inactivate it. E) bind to the repressor protein and activate it.

It is known that an RNA molecule that contains 2-thiouracil is defective. Based on your knowledge of RNA's used in the cell, what problem do you think the 2-thiouracil is causing? (hint: consider gene expression).

2-thiouracil disrupts translation and ultimately protein synthesis. It is a base analog that inhibits mRNA synthesis by competing with uracil.

Avogadro's number, 6 x 1023, tells us how many atoms or molecules are in a mole. Sulfur has a molecular weight of 32. How many moles and atoms are there in 90 grams of sulfur?

2.81 / 1.69 x 1024

2.50.a. how many carbon atoms does the molecule represented in figure q2-50

20 carbon atoms

D

20) Muscle cells and nerve cells in one species of animal owe their differences in structure to A) having different genes. B) having different chromosomes. C) using different genetic codes. D) having different genes expressed. E) having unique ribosomes.

2.13. the first task you are assigned in your summer laboratory job is to prepare a concentrated NaOH stock solution. the molecular weight of NaOH is 40. how many grams of solid NaOH will you need to weigh out to obtain a 500mL solution that has a concentation of 10 M?

200g

A

21) Which of the following mechanisms is (are) used to coordinately control the expression of multiple, related genes in eukaryotic cells? A) organization of the genes into clusters, with local chromatin structures influencing the expression of all the genes at once B) each of the genes sharing a common control element, allowing several activators to turn on their transcription, regardless of their location in the genome C) organizing the genes into large operons, allowing them to be transcribed as a single unit D) a single repressor able to turn off several related genes E) environmental signals that enter the cell and bind directly to their promoters

C

22) If you were to observe the activity of methylated DNA, you would expect it to A) be replicating nearly continuously. B) be unwinding in preparation for protein synthesis. C) have turned off or slowed down the process of transcription. D) be very actively transcribed and translated. E) induce protein synthesis by not allowing repressors to bind to it.

D

23) Genomic imprinting, DNA methylation, and histone acetylation are all examples of A) genetic mutation. B) chromosomal rearrangements. C) karyotypes. D) epigenetic phenomena. E) translocation.

E

24) Approximately what proportion of the DNA in the human genome codes for proteins or functional RNA? A) 83% B) 46% C) 32% D) 13% E) 1.5%

D

25) Two potential devices that eukaryotic cells use to regulate transcription are A) DNA methylation and histone amplification. B) DNA amplification and histone methylation. C) DNA acetylation and methylation. D) DNA methylation and histone acetylation. E) histone amplification and DNA acetylation.

The first task you are assigned in your summer laboratory job is to prepare a concentrated NaOH stock solution. The molecular weight of NaOH is 40. How many grams of solid NaOH will you need to weigh out to obtain a 500 mL solution that has a concentration of 12.5 M?

250 g

A

26) In both eukaryotes and prokaryotes, gene expression is primarily regulated at the level of A) transcription. B) translation. C) mRNA stability. D) mRNA splicing. E) protein stability.

E

27) In eukaryotes, transcription is generally associated with A) euchromatin only. B) heterochromatin only. C) very tightly packed DNA only. D) highly methylated DNA only. E) both euchromatin and histone acetylation.

Suppose a double-stranded DNA molecule was shown to have 22% guanine bases. What would be the expected percentage of adenine bases in that molecule?

28%

A

28) A geneticist introduces a transgene into yeast cells and isolates five independent cell lines in which the transgene has integrated into the yeast genome. In four of the lines, the transgene is expressed strongly, but in the fifth there is no expression at all. Which is a likely explanation for the lack of transgene expression in the fifth cell line? A) A transgene integrated into a heterochromatic region of the genome. B) A transgene integrated into a euchromatic region of the genome. C) The transgene was mutated during the process of integration into the host cell genome. D) The host cell lacks the enzymes necessary to express the transgene. E) A transgene integrated into a region of the genome characterized by high histone acetylation.

C

29) During DNA replication, A) all methylation of the DNA is lost at the first round of replication. B) DNA polymerase is blocked by methyl groups, and methylated regions of the genome are therefore left uncopied. C) methylation of the DNA is maintained because methylation enzymes act at DNA sites where one strand is already methylated and thus correctly methylates daughter strands after replication. D) methylation of the DNA is maintained because DNA polymerase directly incorporates methylated nucleotides into the new strand opposite any methylated nucleotides in the template. E) methylated DNA is copied in the cytoplasm, and unmethylated DNA in the nucleus.

2.51. most types of molecules in the cell have asymmetric (chiral) carbons. consequently there is the potential to have two different molecules that look much the same but are mirror images of each other and therefore not equivalent. these special types of isomers are called stereoisomers. which of the four carbons circles is the asymmetric carbon that determines whether the amino acid is a d or l stereoisomer?

3

How many NADHs are formed for each turn in the citric acid cycle?

3 per pyruvate

A particular triplet of bases in the template strand of DNA is 5' AGT 3'. The corresponding codon for the mRNA transcribed is _____. 3' UGA 5' 5' TCA 3' 3' ACU 5' 3' UCA 5' either UCA or TCA, depending on wobble in the first base

3' UCA 5'

E

3) The tryptophan operon is a repressible operon that is A) permanently turned on. B) turned on only when tryptophan is present in the growth medium. C) turned off only when glucose is present in the growth medium. D) turned on only when glucose is present in the growth medium. E) turned off whenever tryptophan is added to the growth medium.

D

3) What would occur if the repressor of an inducible operon were mutated so it could not bind the operator? A) irreversible binding of the repressor to the promoter B) reduced transcription of the operon's genes C) buildup of a substrate for the pathway controlled by the operon D) continuous transcription of the operon's genes E) overproduction of catabolite activator protein (CAP)

2.12. avogadro's number, calculated from the atomic weight of hydrogen, tells us how many atoms or molecules are in a mole.The resulting base for all calculations of moles and molarity (how many molecules are present when you weigh out a substance or measure from a stock solution) is the following: 1 g of hydrogen atoms = 6 × 1023 hydrogen atoms = 1 mole of hydrogen Sulfur has a molecular weight of 32. How many moles and atoms are there in 120 grams of sulfur? (a) 3.75 and 6 × 1023 (b) 32 and 6 × 1023 (c) 1.75 and 1.05 ×1024 (d) 3.75 and 2.25 × 1024

3.75 and 2.25x10^24

A

30) Eukaryotic cells can control gene expression by which of the following mechanisms? A) histone acetylation of nucleosomes B) DNA acetylation C) RNA induced modification of chromatin structure D) repression of operons E) induction of operators in the promoter

C

32) This binds to a site in the DNA far from the promoter to stimulate transcription: A) enhancer B) promoter C) activator D) repressor E) terminator

D

33) This can inhibit transcription by blocking the binding of positively acting transcription factors to the DNA: A) enhancer B) promoter C) activator D) repressor E) terminator

E

34) This is the site in the DNA located near the end of the final exon, encoding an RNA sequence that determines the 3' end of the transcript: A) enhancer B) promoter C) activator D) repressor E) terminator

D

35) Steroid hormones produce their effects in cells by A) activating key enzymes in metabolic pathways. B) activating translation of certain mRNAs. C) promoting the degradation of specific mRNAs. D) binding to intracellular receptors and promoting transcription of specific genes. E) promoting the formation of looped domains in certain regions of DNA.

B

36) A researcher found a method she could use to manipulate and quantify phosphorylation and methylation in embryonic cells in culture. In one set of experiments using this procedure in Drosophila, she was readily successful in increasing phosphorylation of amino acids adjacent to methylated amino acids in histone tails. Which of the following results would she most likely see? A) increased chromatin condensation B) decreased chromatin concentration C) abnormalities of mouse embryos D) decreased binding of transcription factors E) inactivation of the selected genes

A

37) A researcher found a method she could use to manipulate and quantify phosphorylation and methylation in embryonic cells in culture. In one set of experiments she succeeded in decreasing methylation of histone tails. Which of the following results would she most likely see? A) increased chromatin condensation B) decreased chromatin concentration C) abnormalities of mouse embryos D) decreased binding of transcription factors E) inactivation of the selected genes

E

38) A researcher found a method she could use to manipulate and quantify phosphorylation and methylation in embryonic cells in culture. One of her colleagues suggested she try increased methylation of C nucleotides in a mammalian system. Which of the following results would she most likely see? A) increased chromatin condensation B) decreased chromatin concentration C) abnormalities of mouse embryos D) decreased binding of transcription factors E) inactivation of the selected genes

C

39) A researcher found a method she could use to manipulate and quantify phosphorylation and methylation in embryonic cells in culture. She tried decreasing the amount of methylation enzymes in the embryonic stem cells and then allowed the cells to further differentiate. Which of the following results would she most likely see? A) increased chromatin condensation B) decreased chromatin concentration C) abnormalities of mouse embryos D) decreased binding of transcription factors E) inactivation of the selected genes

If an mRNA sequence is 5′AUG-GGC-ACU-CAU3′, what would the anticodon sequence be?

3′UAC-CCG-UGA-GUA5′

A

4) The functioning of enhancers is an example of A) transcriptional control of gene expression. B) a post-transcriptional mechanism for editing mRNA. C) the stimulation of translation by initiation factors. D) post-translational control that activates certain proteins. E) a eukaryotic equivalent of prokaryotic promoter functioning.

C

41) Gene expression might be altered at the level of post-transcriptional processing in eukaryotes rather than prokaryotes because of which of the following? A) Eukaryotic mRNAs get 5' caps and 3' tails. B) Prokaryotic genes are expressed as mRNA, which is more stable in the cell. C) Eukaryotic exons may be spliced in alternative patterns. D) Prokaryotes use ribosomes of different structure and size. E) Eukaryotic coded polypeptides often require cleaving of signal sequences before localization.

B

42) Which of the following experimental procedures is most likely to hasten mRNA degradation in a eukaryotic cell? A) enzymatic shortening of the poly(A) tail B) removal of the 5' cap C) methylation of C nucleotides D) memethylation of histones E) removal of one or more exons

A

43) Which of the following is most likely to have a small protein called ubiquitin attached to it? A) a cyclin that usually acts in G1, now that the cell is in G2 B) a cell surface protein that requires transport from the ER C) an mRNA that is leaving the nucleus to be translated D) a regulatory protein that requires sugar residues to be attached E) an mRNA produced by an egg cell that will be retained until after fertilization

A

44) The phenomenon in which RNA molecules in a cell are destroyed if they have a sequence complementary to an introduced double-stranded RNA is called A) RNA interference. B) RNA obstruction. C) RNA blocking. D) RNA targeting. E) RNA disposal.

D

45) At the beginning of this century there was a general announcement regarding the sequencing of the human genome and the genomes of many other multicellular eukaryotes. There was surprise expressed by many that the number of protein-coding sequences is much smaller than they had expected. Which of the following accounts for most of the rest? A) "junk" DNA that serves no possible purpose B) rRNA and tRNA coding sequences C) DNA that is translated directly without being transcribed D) non-protein coding DNA that is transcribed into several kinds of small RNAs with biological function E) non-protein coding DNA that is transcribed into several kinds of small RNAs without biological function

A

46) Which of the following best describes siRNA? A) a short double-stranded RNA, one of whose strands can complement and inactivate a sequence of mRNA B) a single-stranded RNA that can, where it has internal complementary base pairs, fold into cloverleaf patterns C) a double-stranded RNA that is formed by cleavage of hairpin loops in a larger precursor D) a portion of rRNA that allows it to bind to several ribosomal proteins in forming large or small subunits E) a molecule, known as Dicer, that can degrade other mRNA sequences

C

47) One of the hopes for use of recent knowledge gained about non-coding RNAs lies with the possibilities for their use in medicine. Of the following scenarios for future research, which would you expect to gain most from RNAs? A) exploring a way to turn on the expression of pseudogenes B) targeting siRNAs to disable the expression of an allele associated with autosomal recessive disease C) targeting siRNAs to disable the expression of an allele associated with autosomal dominant disease D) creating knock-out organisms that can be useful for pharmaceutical drug design E) looking for a way to prevent viral DNA from causing infection in humans

C

49) In a series of experiments, the enzyme Dicer has been inactivated in cells from various vertebrates, and the centromere is abnormally formed from chromatin. Which of the following is most likely to occur? A) The usual mRNAs transcribed from centromeric DNA will be missing from the cells. B) Tetrads will no longer be able to form during meiosis I. C) Centromeres will be euchromatic rather than heterochromatic and the cells will soon die in culture. D) The cells will no longer be able to resist bacterial contamination. E) The DNA of the centromeres will no longer be able to replicate.

Use this model of a eukaryotic transcript to answer the following question. E = exon and I = intron 5' UTR E1 I1 E2 I2 E3 I3 E4 UTR 3' Which components of the previous molecule will also be found in mRNA in the cytosol? 5' E1 E2 E3 E4 3' 5' UTR I1 I2 I3 UTR 3' 5' UTR E1 E2 E3 E4 UTR 3' 5' E1 I1 E2 I2 E3 I3 E4 3' 5' I1 I2 I3 3'

5' UTR E1 E2 E3 E4 UTR 3'

C

5) A mutation in this section of DNA could influence the binding of RNA polymerase to the DNA: A) operon B) inducer C) promoter D) repressor E) corepressor

C

5) Absence of bicoid mRNA from a Drosophila egg leads to the absence of anterior larval body parts and mirror-image duplication of posterior parts. This is evidence that the product of the bicoid gene A) is transcribed in the early embryo. B) normally leads to formation of tail structures. C) normally leads to formation of head structures. D) is a protein present in all head structures. E) leads to programmed cell death.

E

50) Since Watson and Crick described DNA in 1953, which of the following might best explain why the function of small RNAs is still being explained? A) As RNAs have evolved since that time, they have taken on new functions. B) Watson and Crick described DNA but did not predict any function for RNA. C) The functions of small RNAs could not be approached until the entire human genome was sequenced. D) Ethical considerations prevented scientists from exploring this material until recently. E) Changes in technology as well as our ability to determine how much of the DNA is expressed have now made this possible.

D

52) The researcher continues to study the reactions of the embryo to these new proteins and you hypothesize that he is most likely to see which of the following (while embryonic genes are still not being expressed)? A) The cells begin to differentiate. B) The proteins are evenly distributed throughout the embryo. C) Larval features begin to make their appearance. D) Spatial axes (anterior posterior, etc.) begin to be determined. E) The embryo begins to lose cells due to apoptosis from no further gene expression.

B

54) You are given an experimental problem involving control of a gene's expression in the embryo of a particular species. One of your first questions is whether the gene's expression is controlled at the level of transcription or translation. Which of the following might best give you an answer? A) You explore whether there has been alternative splicing by examining amino acid sequences of very similar proteins. B) You measure the quantity of the appropriate pre-mRNA in various cell types and find they are all the same. C) You assess the position and sequence of the promoter and enhancer for this gene. D) An analysis of amino acid production by the cell shows you that there is an increase at this stage of embryonic life. E) You use an antibiotic known to prevent translation.

A

55) In humans, the embryonic and fetal forms of hemoglobin have a higher affinity for oxygen than that of adults. This is due to A) nonidentical genes that produce different versions of globins during development. B) identical genes that generate many copies of the ribosomes needed for fetal globin production. C) pseudogenes, which interfere with gene expression in adults. D) the attachment of methyl groups to cytosine following birth, which changes the type of hemoglobin produced. E) histone proteins changing shape during embryonic development.

A

56) The process of cellular differentiation is a direct result of A) differential gene expression. B) morphogenesis. C) cell division. D) apoptosis. E) differences in cellular genomes.

A

57) The fact that plants can be cloned from somatic cells demonstrates that A) differentiated cells retain all the genes of the zygote. B) genes are lost during differentiation. C) the differentiated state is normally very unstable. D) differentiated cells contain masked mRNA. E) differentiation does not occur in plants.

If a DNA template strand has a sequence of 3′ TACAATGTAGCC 5′, then the RNA produced from it will be which sequence? 3′TACAATGTAGCC5′ 5′ATGTTACATCGG3′ 5′AUGUUACAUCGG3' 3′AUGUUACAUCGG5′ 3′ATGTTACATCGG5′

5′AUGUUACAUCGG3'

In step 3 of the citric acid cycle, the oxidation of isocitrate and the production of CO2 are coupled to the reduction of NAD+, generating NADH and an α-ketoglutarate molecule. In the isocitrate molecule shown below, which carbon is lost as CO2 and which is converted to a carbonyl carbon?

6 and 4

E

6) Which of the following statements about the DNA in one of your brain cells is true? A) Most of the DNA codes for protein. B) The majority of genes are likely to be transcribed. C) Each gene lies immediately adjacent to an enhancer. D) Many genes are grouped into operon-like clusters. E) It is the same as the DNA in one of your heart cells.

A

60) In animals, embryonic stem cells differ from adult stem cells in that A) embryonic stem cells are totipotent, and adult stem cells are pluripotent. B) embryonic stem cells are pluripotent, and adult stem cells are totipotent. C) embryonic stem cells have more genes than adult stem cells. D) embryonic stem cells have fewer genes than adult stem cells. E) embryonic stem cells are localized to specific sites within the embryo, whereas adult stem cells are spread throughout the body.

B

61) Which of the following statements is true about stem cells? A) Stem cells can continually reproduce and are not subject to mitotic control. B) Stem cells can differentiate into specialized cells. C) Stem cells are found only in bone marrow. D) Stem cells are found only in the adult human brain. E) Stem cell DNA lacks introns.

B

62) What is considered to be the first evidence of differentiation in the cells of an embryo? A) cell division B) the occurrence of mRNAs for the production of tissue-specific proteins C) determination D) changes in the size and shape of the cell E) changes resulting from induction

C

63) In most cases, differentiation is controlled at which level? A) replication of the DNA B) nucleosome formation C) transcription D) translation E) post-translational activation of the proteins

A

64) Which of the following serve as sources of developmental information? A) cytoplasmic determinants such as mRNAs and proteins produced before fertilization B) signal molecules produced by the maturing zygote C) ubiquitous enzymes such as DNA polymerase and DNA ligase D) paternally deposited proteins E) specific operons within the zygote genome

B

65) The MyoD protein A) can promote muscle development in all cell types. B) is a transcription factor that binds to and activates the transcription of muscle-related genes. C) was used by researchers to convert differentiated muscle cells into liver cells. D) magnifies the effects of other muscle proteins. E) is a target for other proteins that bind to it.

A

66) The gene for which protein would most likely be expressed as a result of MyoD activity? 17 A) myosin B) crystallin C) albumin D) hemoglobin E) DNA polymerase

A

67) The general process that leads to the differentiation of cells is called A) determination. B) specialization. C) identification. D) differentialization. E) cellularization.

E

68) Your brother has just purchased a new plastic model airplane. He places all the parts on the table in approximately the positions in which they will be located when the model is complete. His actions are analogous to which process in development? A) morphogenesis B) determination C) induction D) differentiation E) pattern formation

A

69) Which of the following is established prior to fertilization in Drosophila eggs? A) the anterior-posterior and dorsal-ventral axes B) the position of the future segments C) the position of the future wings, legs, and antennae D) A and B only E) A, B, and C

A

7) Cell differentiation always involves the A) production of tissue-specific proteins, such as muscle actin. B) movement of cells. C) transcription of the myoD gene. D) selective loss of certain genes from the genome. E) cell's sensitivity to environmental cues such as light or heat.

B

7) When this is taken up by the cell, it binds to the repressor so that the repressor no longer binds to the operator: A) operon B) inducer C) promoter D) repressor E) corepressor

A

70) The product of the bicoid gene in Drosophila provides essential information about A) the anterior-posterior axis. B) the dorsal-ventral axis. 18 C) the left-right axis. D) segmentation. E) lethal genes.

Photosystem I is refered to by the wavelength at which its reaction center best absorbs light, or P____; photosystem II is also known by this characteristic, or P_____

700;680

B

71) If a Drosophila female has a homozygous mutation for a maternal effect gene, A) she will not develop past the early embryonic stage. B) all of her offspring will show the mutant phenotype, regardless of their genotype. C) only her male offspring will show the mutant phenotype. D) her offspring will show the mutant phenotype only if they are also homozygous for the mutation. E) only her female offspring will show the mutant phenotype.

A

72) Mutations in these genes lead to transformations in the identity of entire body parts: A) homeotic genes B) segmentation genes C) egg-polarity genes D) morphogens E) inducers

C

73) These genes are expressed by the mother, and their products are deposited into the developing egg: A) homeotic genes B) segmentation genes C) egg-polarity genes D) morphogens E) inducers

B

74) These genes map out the basic subdivisions along the anterior-posterior axis of the Drosophila embryo: A) homeotic genes B) segmentation genes C) egg-polarity genes D) morphogens E) inducers

D

75) These genes form gradients and help establish the axes and other features of an embryo: A) homeotic genes B) segmentation genes C) egg-polarity genes D) morphogens E) inducers

B

76) Gap genes and pair-rule genes fall into this category: A) B) segmentation genes C) egg-polarity genes D) morphogens E) inducers

B

77) The product of the bicoid gene in Drosophila could be considered a(n) A) tissue-specific protein. B) cytoplasmic determinant. C) maternal effect. D) inductive signal. E) fertilization product.

D

78) The bicoid gene product is normally localized to the anterior end of the embryo. If large amounts of the product were injected into the posterior end as well, which of the following would occur? A) The embryo would grow to an unusually large size. B) The embryo would grow extra wings and legs. C) The embryo would probably show no anterior development and die. D) Anterior structures would form in both sides of the embryo. E) The embryo would develop normally.

A

79) What do gap genes, pair-rule genes, segment polarity genes, and homeotic genes all have in common? A) Their products act as transcription factors. B) They have no counterparts in animals other than Drosophila. C) Their products are all synthesized prior to fertilization. D) They act independently of other positional information. E) They apparently can be activated and inactivated at any time of the fly's life.

A

8) A mutation that inactivates the regulatory gene of a repressible operon in an E. coli cell would result in A) continuous transcription of the structural gene controlled by that regulator. B) complete inhibition of transcription of the structural gene controlled by that regulator. C) irreversible binding of the repressor to the operator. D) inactivation of RNA polymerase by alteration of its active site. E) continuous translation of the mRNA because of alteration of its structure.

3.51.The small molecule cyclic AMP (cAMP) takes about 0.2 second to diffuse 10 μm, on average, in a cell. Suppose that cAMP is produced near the plasma membrane on one end of the cell; how long will it take for this cAMP to diffuse through the cytosol and reach the opposite end of a very large cell, on average? Assume that the cell is 200 μm in diameter. (a) 4 seconds (b) 16 seconds (c) 80 seconds (d) 200 seconds

80 seconds

A

80) Which of the following statements describes proto-oncogenes? A) They can code for proteins associated with cell growth. B) They are introduced to a cell initially by retroviruses. C) They are produced by somatic mutations induced by carcinogenic substances. D) Their normal function is to suppress tumor growth E) They are underexpressed in cancer cells

C

82) Tumor suppressor genes A) are frequently overexpressed in cancerous cells. B) are cancer-causing genes introduced into cells by viruses. C) can encode proteins that promote DNA repair or cell-cell adhesion. D) often encode proteins that stimulate the cell cycle. E) all of the above

D

83) The incidence of cancer increases dramatically in older humans because A) the Ras protein is more likely to be hyperactive after age sixty. B) proteasomes become more active with age. C) as we age, normal cell division inhibitors cease to function. D) the longer we live, the more mutations we accumulate. E) tumor-suppressor genes are no longer able to repair damaged DNA.

A

84) The cancer-causing forms of the Ras protein are involved in which of the following processes? A) relaying a signal from a growth factor receptor B) DNA replication C) DNA repair D) cell-cell adhesion E) cell division

E

85) Forms of the ras protein found in tumors usually cause which of the following? A) DNA replication to stop B) DNA replication to be hyperactive C) cell-to-cell adhesion to be nonfunctional D) cell division to cease E) growth factor signaling to be hyperactive

A

86) A genetic test to detect predisposition to cancer would likely examine the APC gene for involvement in which type(s) of cancer? A) colorectal only B) lung and breast C) small intestinal and esophageal D) lung only E) lung and prostate

E

87) Which of the following can contribute to the development of cancer? A) random spontaneous mutations B) mutations caused by X-rays C) transposition D) chromosome translocations E) all of the above

D

88) One hereditary disease in humans, called xeroderma pigmentosum (XP), makes homozygous individuals exceptionally susceptible to UV-induced mutation damage in the cells of exposed tissue, especially skin. Without extraordinary avoidance of sunlight exposure, patients soon succumb to numerous skin cancers. Which of the following best describes this phenomenon? A) inherited cancer taking a few years to be expressed B) embryonic or fetal cancer C) inherited predisposition to mutation D) inherited inability to repair UV-induced mutation E) susceptibility to chemical carcinogens

D

9) The lactose operon is likely to be transcribed when A) there is more glucose in the cell than lactose. B) the cyclic AMP levels are low. C) there is glucose but no lactose in the cell. D) the cyclic AMP and lactose levels are both high within the cell. E) the cAMP level is high and the lactose level is low.

B

9) Within a cell, the amount of protein made using a given mRNA molecule depends partly on A) the degree of DNA methylation. B) the rate at which the mRNA is degraded. C) the presence of certain transcription factors. D) the number of introns present in the mRNA. E) the types of ribosomes present in the cytoplasm.

The net production of 1 G3P requires ___ molecules of ATP and ___ moleculs of NADPH

9,6

B

90) In colorectal cancer, several genes must be mutated in order to make a cell a cancer cell, supporting Knudsen's hypothesis. Which of the following kinds of genes would you expect to be mutated? A) genes coding for enzymes that act in the colon B) genes involved in control of the cell cycle C) genes that are especially susceptible to mutation D) the same genes that Knudsen identified as associated with retinoblastoma E) the genes of the bacteria that are abundant in the colon

14-2 Describe how a standard flashlight battery can convert energy into useful work and explain how this is similar to the energy conversions in the mitochondria.

A battery contains chemicals that generate negatively charged ions at one pole, and it is able to cause the continuous transfer of electrons along a metal wire if that pole is connected to the other end of the battery. The energy released by the electron-transfer process driven by the battery can be harnessed to do useful work, as when it is used to run an electric motor. Likewise, the energy released by the electron transfers that occur between the protein complexes in the electron-transport chain does useful work when it drives the movement of protons to one side of the membrane, since the resulting proton gradient is then used to generate chemical energy in the form of ATP

For each of the following sentences, fill in the blanks with the best word or phrase selected from the list below. Not all words or phrases will be used; each word or phrase should be used only once. A carbon atom in a CO2 molecule in the atmosphere eventually becomes a part of one of the enzymes that catalyzes glycolysis in one of your cells. The CO2 first enters a cell in a corn leaf, where photosynthesis fixes the carbon to make it part of a sugar molecule; this travels from the leaf to an ear of corn, where it is stored as part of a polysaccharide __________________ molecule in the corn seed. You then eat a corn chip made from the corn seed. You digest the corn seed, and the free __________________ travels in your bloodstream, eventually being taken up by a liver cell and stored as __________________. When required, this storage molecule breaks down into glucose 1-phosphate, which enters the glycolytic pathway. Glycolysis produces __________________, which is converted into acetyl CoA, which enters the __________________. Several intermediates in this Page 16 of 27 process can provide the carbon skeleton for the production of __________________, which are then incorporated into the enzymes that catalyze steps in glycolysis. amino acids insulin carbon fixation lactate citric acid cycle nucleotides fatty acid oxidative phosphorylation fermentation pyruvate galactose starch glucose triacylglycerol glycogen

A carbon atom in a CO2 molecule in the atmosphere eventually becomes a part of one of the enzymes that catalyzes glycolysis in one of your cells. The CO2 first enters a cell in a corn leaf, where photosynthesis fixes the carbon to make it part of a sugar molecule; this travels from the leaf to an ear of corn, where it is stored as part of a polysaccharide starch molecule in the corn seed. You then eat a corn chip made from the corn seed. You digest the corn seed, and the free glucose travels in your bloodstream, eventually being taken up by a liver cell and stored as glycogen. When required, this storage molecule breaks down into glucose 1-phosphate, which enters the glycolytic pathway. Glycolysis produces pyruvate, which is converted into acetyl CoA, which enters the citric acid cycle. Several intermediates in this process can provide the carbon skeleton for the production of amino acids, which are then incorporated into the enzymes that catalyze steps in glycolysis.

Which of the following observations may have resulted in the hypothesis that a codon is made up of three bases? A codon of three bases in length, from four different bases, would code for a maximum of twelve different amino acids. A codon of two bases in length, from four different bases, would code for a maximum of thirty-two different amino acids. A codon of three bases in length, from four different bases, would code for a maximum of sixty-four different amino acids. A codon of four bases in length, from four different bases, would code for a maximum of twenty-four different amino acids.

A codon of three bases in length, from four different bases, would code for a maximum of sixty-four different amino acids

When ions are moved across the cell membrane

A concentration gradient may be established/an electrical gradient may be established/an osmotic gradient may be established

A

A few decades ago, Knudsen and colleagues proposed a theory that, for a normal cell to become a cancer cell, a minimum of two genetic changes had to occur in that cell. Knudsen was studying retinoblastoma, a childhood cancer of the eye. 89) If there are two children born from the same parents, and child one inherits a predisposition to retinoblastoma (one of the mutations) and child two does not, but both children develop the retinoblastoma, which of the following would you expect? A) an earlier age of onset in child one B) a history of exposure to mutagens in child one but not in child two C) a more severe cancer in child one D) increased levels of apoptosis in both children E) decreased levels of DNA repair in child one

15-7 You discover a fungus that contains a strange star-shaped organelle not found in any other eukaryotic cell you have seen. On further investigation, you find the following. 1.The organelle possesses a small genome in its interior. 2.The organelle is surrounded by two membranes. 3.Vesicles do not pinch off from the organelle membrane. 4.The interior of the organelle contains proteins similar to those of many bacteria. 5.The interior of the organelle contains ribosomes. How might this organelle have arisen?

A genome, a double membrane, ribosomes, and proteins similar to those found in bacteria are evidence for an organelle having evolved from an engulfed bacterium.

A molecule moves down its concentration gradient by __________________ transport, but requires __________________ transport to move up its concentration gradient. Transporter proteins and ion channels function in membrane transport by providing a __________________ pathway through the membrane for specific polar solutes or inorganic ions. __________________ are highly selective in the solutes they transport, binding the solute at a specific site and changing conformation so as to transport the solute across the membrane. On the other hand, __________________ discriminate between solutes mainly on the basis of size and electrical charge. active hydrophilic noncovalent amino acid hydrophobic passive amphipathic ion channels transporter proteins

A molecule moves down its concentration gradient by passive transport, but requires active transport to move up its concentration gradient. Transporter proteins and ion channels function in membrane transport by providing a hydrophilic pathway through the membrane for specific polar solutes or inorganic ions. Transporter proteins are highly selective in the solutes they transport, binding the solute at a specific site and changing conformation so as to transport the solute across the membrane. On the other hand, ion channels discriminate between solutes mainly on the basis of size and electrical charge.

A newly synthesized protein generally folds up into a __________________ conformation. All the information required to determine a protein's conformation is contained in its amino acid __________________. On being heated, a protein molecule will become __________________ as a result of breakage of __________________ bonds. On removal of urea, an unfolded protein can become __________________. The final folded conformation adopted by a protein is that of __________________ energy. composition irreversible reversible covalent lowest sequence denatured noncovalent stable highest renatured unstable

A newly synthesized protein generally folds up into a *stable* conformation. All the information required to determine a protein's conformation is contained in its amino acid *sequence*. On being heated, a protein molecule will become *denatured* as a result of breakage of *noncovalent* bonds. On removal of urea, an unfolded protein can become *renatured*. The final folded conformation adopted by a protein is that of *lowest* energy.

15-3Name the membrane-enclosed compartments in a eukaryotic cell where each of the functions listed below takes place. A.photosynthesis B.transcription C.oxidative phosphorylation D.modification of secreted proteins E.steroid hormone synthesis F.degradation of worn-out organelles G.new membrane synthesis H.breakdown of lipids and toxic molecules

A photosynthesis = chloroplast B. transcription = nucleus C. oxidative phosphorylation = mitochondrion D. modification of secreted proteins = Golgi apparatus and rough endoplasmic reticulum (ER) E. steroid hormone synthesis = smooth ER F. degradation of worn-out organelles = lysosome G. new membrane synthesis = ER H. breakdown of lipids and toxic molecules = peroxisome

What promotes the binding between the mRNA and the small ribosomal subunit?

A region near the 5' end of the mRNA is complementary to a region of rRNa is the small subunit.These complementary regions hydrogen-bond with each other to promote the binding of the mRNA to the small ribosomal subunit.

Scientists have found that extracellular matrix components may induce specific gene expression in embryonic tissues such as the liver and testes. For this to happen there must be direct communication between the extracellular matrix and the developing cells. Which kind of transmembrane protein would most likely be involved in this kind of induction?

A) integrins

50) Which of the following statements about quorum sensing is FALSE? Quorum sensing ________.

A) is cell—cell communication in eukaryotes

Aminoacyl-tRNA synthetase is an enzyme whose function is to_______? A) link a tRNA to its corresponding amino acid B) remove introns from pre-RNA transcript C) cause the RNA polymerase to detach from the DNA D) join together RNA nucleotides complementary to the DNA template strand

A) link a tRNA to its corresponding amino acid

The flow of ions through a gated channel can be studied using a method called "patch-clamp recording." A. How is a detached patch-clamp experiment set up, and what exactly does it mean to "clamp" an ion channel? B. How is it possible to collect the recordings shown in Figures Q12-37A and Q12-37B from a single ion channel?

A. A detached patch-clamp experiment requires the removal of a portion of the cell membrane by sealing the microelectrode to the membrane surface. After lifting the patch of membrane stuck to the microelectrode, it is placed into a solution of controlled medium. The voltage applied to the membrane patch can be fixed (clamped) while other parameters are studied. B. By manipulating ion concentrations in the two chambers or simply reversing the direction of the current in the system, the ion flow through the channel can be reversed, resulting in the recording of negative values for current when the channel opens.

The rod photoreceptors in the eye are extremely sensitive to light. The cells sense light through a signal transduction cascade involving light activation of a GPCR that activates a G protein that activates cyclic GMP phosphodiesterase. How would you expect the addition of the following drugs to affect the light-sensing ability of the rod cells? Explain your answers. A. a drug that inhibits cyclic GMP phosphodiesterase B. a drug that is a nonhydrolyzable analog of GTP

A. A drug that inhibits cyclic GMP phosphodiesterase would decrease any light response in the rod cell. Normally, cyclic GMP is continuously being produced in the eye. The perception of light by a rod cell normally leads to the activation of cyclic GMP phosphodiesterase, which then hydrolyzes cyclic GMP molecules. This causes Na+ channels to close, which changes the membrane potential and alters the signal sent to the brain. If cyclic GMP phosphodiesterase were blocked, levels of cyclic GMP would remain high and there would be no cellular response to light. B. A drug that is a nonhydrolyzable analog of GTP would lead to a prolonged response to light. This is because a nonhydrolyzable analog of GTP would prevent the G protein from turning itself off by hydrolyzing its bound GTP to GDP. Continued activation of the G protein would keep cyclic GMP phosphodiesterase levels higher than normal, leading to a prolonged period of lowered levels of cyclic GMP. This in turn would cause Na+ channels to be closed for longer than normal, leading to a prolonged change in the membrane potential and an extended light response.

14-35Human infants have a much larger portion of brown adipose tissue than adult humans. It was found that the mitochondria in brown adipocytes (brown fat cells) have a novel protein in the inner mitochondrial membrane. This protein, called the uncoupling protein (UCP), was found to transport protons from the intermembrane space into the matrix. A.What is the impact of UCP on oxidative phosphorylation in the mitochondria of brown fat? B.Propose an explanation for the higher proportion of brown fat cells in infants compared to adults.

A. A protein that transports protons into the mitochondrial matrix would diminish the proton gradient. Without the proton gradient, ATP will not be generated. However, the electron-transport chain can still work, as long as oxygen is present. The UCP, therefore, is a biological uncoupler of the oxidative phosphorylation process. The electron-transport chain will run in a futile cycle that does not convert the energy from redox reactions into chemical energy (ATP), but instead releases this energy as heat. B. The thermogenesis resulting from the action of UCP is important for helping infants maintain a constant body temperature. As our body mass increases with age, our body temperatures are probably less susceptible to fluctuations, and therefore adults do not require the same amount of brown fat as infants.

Receipt of extracellular signals can change cell behavior quickly (for example, in seconds or less) or much more slowly (for example, in hours). A. What kind of molecular changes could cause quick changes in cell behavior? B. What kind of molecular changes could cause slow changes in cell behavior? C. Explain why the response you named in A results in a quick change, whereas the response you named in B results in a slow change.

A. Any answer that involves the modification of existing cell components is correct. Protein phosphorylation, protein dephosphorylation, protein ubiquitylation, lipid phosphorylation, and lipid cleavage are all examples of correct answers. B. Responses that involve alterations in gene expression occur slowly. C. Modification of existing cell components can happen quickly, whereas responses that depend on changes in gene expression take much longer, because the genes will need to be transcribed, the mRNAs will need to be translated, and the proteins need to accumulate to high-enough levels to instigate change.

A calmodulin-regulated kinase (CaM-kinase) is involved in spatial learning and memory. This kinase is able to phosphorylate itself such that its kinase activity is now independent of the intracellular concentration of Ca2+. Thus, the kinase stays active after Ca2+ levels have dropped. Mice completely lacking this CaM-kinase have severe spatial learning defects but are otherwise normal. A. Each of the following mutations also leads to similar learning defects. For each case explain why. (1) a mutation that prevents the kinase from binding ATP (2) a mutation that deletes the calmodulin-binding part of the kinase (3) a mutation that destroys the site of autophosphorylation B. What would be the effect on the activity of CaM-kinase if there were a mutation that reduced its interaction with the protein phosphatase responsible for inactivating the kinase?

A. Because a complete lack of the CaM-kinase causes a learning defect, we can assume that mutations leading to inactivation of the kinase would also have a similar effect. (1) Protein kinases have a binding site for ATP, which is the source of the phosphate used for phosphorylating their target proteins; if the kinase cannot bind ATP, it will be inactive. (2) Because binding to calmodulin in the presence of Ca2+ activates CaM-kinases, deletion of the calmodulin-binding portion would inactivate the kinase. (3) A mutation that destroys the site of autophosphorylation will also impair the normal function of the kinase, because the kinase will become inactive as soon as Ca2+ levels decrease. B. The kinase would stay active for longer after a transient increase in intracellular Ca2+ concentration.

You are testing the rate of glucose transport into vesicles using the Na+-glucose pump. A. In experiment 1, you employ liposomes that have the pump in the same orientation as that found in the plasma membrane in epithelial cells. These liposomes contain glucose but no Na+ ions. You then transfer the liposomes to a series of tubes with solutions containing the same glucose concentration as that inside the vesicle and 0, 1, 2, 3, or 10 mM Na+. You measure the initial rates of glucose transport and plot your results (Figure Q12-33). Why do the initial rates of glucose transport into the liposome reach a plateau as the concentration of Na+ increases? B. In experiment 2, there is one new variable: you have included leaky Na+ channels in the liposomal membrane. Figure Q12-33 shows your results. Explain the reason for obtaining such different results in experiment 2 from those in experiment 1.

A. Because transporters require recognition and binding of the transport substrate before moving it across the membrane, the transporters can reach a maximum capacity (saturation) at which we observe the maximum rate at which transport can occur, regardless of any further increase in the concentration of transport substrate. In this case, it seems that 10 mM Na+ is close to saturating the experimental system being used. B. The leaky Na+ channels allow rapid equilibration of Na+ ions across the liposomal membranes, destroying the gradient required for glucose transport into the liposome. Thus, there is no net movement of glucose into the liposome at any of the Na+ ion concentrations tested.

Your friend is studying a segment of a newly discovered virus that carries an enhancer of gene expression that confers responsiveness to glucocorticoid (a hormone) on genes that are linked to it. He constructs two versions of a reporter gene: one has only a minimal promoter linked to it (which contains sites for RNA polymerase binding); the other reporter gene has both this minimal promoter plus the viral enhancer attached to it. The reporter gene allows him to measure the amount of transcription that occurs from each construct. Your friend puts each of these constructs into two different cell lines and examines the expression of the reporter gene in each cell line, as shown in Figure Q16-25. He is puzzled by these findings and asks for your help in interpreting them. Figure Q16-25 A. From these data, can you tell whether both cell lines contain glucocorticoid receptors? Why? B. What might account for the difference in the transcription of the reporter gene in cell lines 1 and 2 after introduction of the construct containing the viral enhancer in the presence of glucocorticoid?

A. Both cell lines seem to contain the glucocorticoid receptor, because both cell lines demonstrate increases in reporter gene expression in response to the addition of glucocorticoid. With the introduction of the construct containing the glucocorticoid-responsive viral enhancer, cell line 1 showed a 1000-fold increase and cell line 2 showed an 80-fold increase. B. There are four reasonable explanations for this difference in glucocorticoid responses in the two cell lines (any one would be correct). 1. Cell line 1 contains a protein that does not exist in cell line 2. This protein cooperates with the glucocorticoid receptor to activate the transcription of the reporter gene, leading to higher levels of the reporter gene in cell line 1 than in cell line 2. 2. Cell line 2 contains a slightly defective glucocorticoid receptor that does not bind DNA as strongly as the normal glucocorticoid receptor in cell line 1, thus leading to lower levels of reporter gene activity in cell line 2 than in cell line 1. 3. Cell line 1 contains a mutant glucocorticoid receptor that binds DNA much more strongly than the normal glucocorticoid receptor found in cell line 2. Therefore, higher levels of reporter gene activity are seen in cell line 1 than in cell line 2. 4. Cell line 2 contains a protein that does not exist in cell line 1. This protein acts as an antagonist to the glucocorticoid receptor to decrease the level of transcription from the reporter gene, leading to lower levels of reporter gene expression in cell line 2 than in cell line 1.

Acetylcholine acts at a GPCR on heart muscle to make the heart beat more slowly. It does so by ultimately opening K+ channels in the plasma membrane (as diagrammed in Figure Q16-32), which decreases the cell's excitability by making it harder to depolarize the plasma membrane. Indicate whether each of the following conditions would increase or decrease the effect of acetylcholine. A. addition of a drug that stimulates the GTPase activity of the Gα subunit B. mutations in the K+ channel that keep it closed all the time C. modification of the Gα subunit by cholera toxin D. a mutation that decreases the affinity of the βγ complex of the G protein for the K+ channel E. a mutation in the acetylcholine receptor that prevents its localization on the cell surface F. adding acetylcholinesterase to the external environment of the cell

A. Decrease. An increase in the GTPase activity of the Gα subunit will decrease the length of time that the G protein is active. B. Decrease. If the K+ channel remains closed, acetylcholine will not slow the heart. C. Increase. Cholera toxin inhibits the GTPase activity of the Gα subunit, keeping the subunit in an active state for a longer time. D. Decrease. The activated βγ complex binds to and activates the K+ channel; decreasing their affinity for each other will decrease the time that the K+ channel is open, effectively decreasing the effect of acetylcholine. E. Decrease. If there is no receptor on the cell surface, cells will be unable to respond to acetylcholine. F. Decrease. Acetylcholinesterase degrades acetylcholine and thus will decrease the effect of acetylcholine.

15-35Figure Q15-35 shows the orientation of a multipass transmembrane protein after it has completed its entry into the ER membrane (part A) and after it gets delivered to the plasma membrane (part B). This protein has an N-terminal signal sequence (depicted as the dark gray membrane-spanning box), which signal peptidase cleaves off in the endoplasmic reticulum. The other membrane-spanning domains in the protein are represented as open boxes. Given that any hydrophobic membrane-spanning domain can act as either a start-transfer region or a stop-transfer region, draw the final consequences of the actions described below on the orientation of the protein in the plasma membrane. Indicate on your drawing the extracellular space, the cytosolic face, and the plasma membrane, as well as the N-and C-terminus of the protein. Figure Q15-35A.deleting the first signal sequenceB.changing the hydrophobic amino acids in the first, cleaved sequence to charged amino acidsC.changing the hydrophobic residues in every other transmembrane sequence to charged residues, starting with the first, cleaved signal sequence

A. Deleting the first signal sequence completely would convert the next membrane- spanning domain into an internal start-transfer signal and would invert the orientation of the protein (see Figure A15-35A). B. Changing the hydrophobic amino acids to charged amino acids destroys the ability of the sequence both to act as a signal sequence and to become a membrane-spanning sequence. Therefore, the adjacent membrane-spanning domain will now become an internal start-transfer sequence and the protein will be inverted, as seen above in part A. The mutated signal sequence would not get cleaved off, because it would remain on the cytoplasmic side of the membrane and signal peptidase is found only inside the ER (see Figure A15-35B). C. Mutating every other membrane-spanning region so that they are now charged (and thus cannot span the membrane) would decrease the number of transmembrane regions and increase the size of the loops between membrane- spanning regions (see Figure A15-35C).

Indicate whether the following statements are true or false. If a statement is false, explain why it is false. A. When a mouse cell is fused with a human cell, the movement of the respective membrane proteins is restricted to their original locations at the time of fusion. B. Epithelial cell membranes are asymmetric, and proteins from the apical side of the cell membrane cannot diffuse into the basal side of the membrane. C. The longest carbohydrates found on the surfaces of cells are linked to lipid molecules. D. The only role of the carbohydrate layer on the cell surface is to absorb water, which creates a slimy surface and prevents cells from sticking to each other.

A. False. After about 1 hour, the mouse and human proteins present on the surface of the fused cell are found evenly dispersed throughout the plasma membrane. B. True. C. False. The very long, branched polysaccharides that are attached to integral membrane proteins are much longer than the oligosaccharides covalently attached to membrane lipids. D. False. Although the absorption of water is an important role of the carbohydrates on the surface of the plasma membrane, a second critical role is that of cell-cell recognition, which is important in immune responses, wound healing, and other processes that rely on cell-type-specific interactions.

14-29Indicate whether the following statements are true or false. If a statement is false, explain why it is false. A.The driving force that pulls protons into the matrix is called the proton-motive force, which is a combination of the large force due to the pH gradient and the smaller force that results from the voltage gradient across the inner mitochondrial membrane. B.Under anaerobic conditions, the ATP synthase can hydrolyze ATP instead of synthesizing it. C.ATP is moved out of the matrix, across the inner mitochondrial membrane, in a co-transporter that also brings ADP into the matrix. D.Brown fat cells make less ATP because they have an inefficient ATP synthase.

A. False. Although it is true that both the pH gradient and the voltage gradient are components of the proton-motive force, it is the voltage gradient (also referred to as the membrane potential) that is the greater of the two. B. True. C. True. D. False. The inner mitochondrial membranes in brown fat cells contain a transport protein that allows protons to move down their gradient without passing through the ATP synthase. As a result, less ATP is made and most of the energy from the proton gradient is released as heat.

Indicate whether the following statements are true or false. If a statement is false, explain why it is false. A. The proteins of the electron-transport chain remove a pair of high-energy electrons from the cofactors NADH and FADH2, after which the electrons move across the inner mitochondrial membrane to maintain the voltage gradient. B. Gluconeogenesis is a linear reaction pathway that the cell employs to generate glucose from pyruvate and is exactly the reverse of the reactions in the glycolytic pathway. C. With respect to the amount of energy stored in molecules of the body, 6 g of glycogen is the equivalent of 1 g of fat. D. Glycogen phosphorylase cleaves glucose monomers from the glycogen polymer, phosphorylating them at the same time so that they can be fed unchanged into the glycolytic pathway.

A. False. Although the proteins of the electron-transport chain collect electrons from the NADH and FADH2 cofactors, these high-energy electrons go through a series of transfers along the electron-transport chain. The energy released with each transfer moves protons across the inner mitochondrial membrane. It is this proton gradient that provides the energy to synthesize ATP. B. False. Gluconeogenesis can begin with pyruvate as a building block to make glucose, but there are three reactions in glycolysis that are irreversible because of a large free-energy barrier. Alternative enzymes and reaction pathways are used to bypass this problem, and they require the input of energy in the form of ATP and GTP. C. True. D. False. When glycogen phosphorylase cleaves a glucose monomer from glycogen, the product is glucose 1-phosphate. Before it can be used in glycolysis, it needs to be isomerized to glucose 6-phosphate.

Indicate whether the following statements are true or false. If a statement is false, explain why it is false. A. During glycolysis, glucose molecules are broken down to yield CO2 and H2O. B. The cleavage of fructose 1,6-bisphosphate yields two molecules of glyceraldehyde 3-phosphate. C. Anaerobic respiration is not the same as fermentation, as only the former requires an electron-transport chain. D. When subjected to anaerobic conditions, glycolysis in mammalian cells continues and causes a buildup of pyruvate in the cytosol. E. The pyruvate dehydrogenase complex catalyzes three different, but linked, enzymatic reactions. Page 10 of 27 F. Amino acids can be transported into the mitochondria and converted into acetyl CoA.

A. False. At the end of a series of the 10 different reactions involved in glycolysis, the final products are two molecules of pyruvate. Pyruvate will later be broken down into CO2 and H2O in the citric acid cycle. B. False. When fructose 1,6-bisphosphate is cleaved, the products are dihydroxyacetone phosphate and glyceraldehyde 3-phosphate. Only after the Page 23 of 27 subsequent isomerization of dihydroxyacetone phosphate is the second molecule of glyceraldehyde 3-phosphate produced. C. True. D. False. Under anaerobic conditions, mammalian cells convert pyruvate to lactate in a fermentation process. The lactate is subsequently excreted from the cell. E. True. F. True.

Indicate whether the following statements are true or false. If a statement is false, explain why it is false. A. Feedback inhibition is defined as a mechanism of down-regulating enzyme activity by the accumulation of a product earlier in the pathway. B. If an enzyme's allosteric binding site is occupied, the enzyme may adopt an alternative conformation that is not optimal for catalysis. C. Protein phosphorylation is another way to alter the conformation of an enzyme and serves exclusively as a mechanism to increase enzyme activity. D. GTP binding proteins typically have GTPase activity, and the hydrolysis of GTP transforms them to the "off" conformation.

A. False. Feedback inhibition occurs when an enzyme acting early in a metabolic pathway is inhibited by the accumulation of a product late in the pathway. The inhibitory product binds to a site on the enzyme that lowers its catalytic activity. B. True. C. False. Although phosphorylation of a protein can change its conformation, this modification may be either as a positive or a negative regulator of enzyme activity, depending on the protein in question. D. True.

Indicate whether the statements below are true or false. If a statement is false, explain why it is false. A. Gap junctions are large pores that connect the cytosol to the extracellular space. B. Aquaporin channels are found in the plasma membrane, and allow the rapid passage of water molecules and small ions in and out of cells. C. The ion selectivity of a channel depends solely on the charge of the amino acids lining the pore inside the channel. D. Most ion channels are gated, which allows them to open and close in response to a specific stimulus, rather than allowing the constant, unregulated flow of ions.

A. False. Gap junctions are used to connect the cytosol of adjacent cells, allowing the sharing of ions and small metabolites. Because gap junctions are large channels, if they were open while facing the extracellular environment, the ability of the plasma membrane to serve as a permeability barrier would be greatly reduced. B. False. Charged molecules (even protons, which are very small) are not able to pass through aquaporins. C. False. Selectivity depends on three parameters: the diameter, shape, and charge of the ion trying to pass through the pore of the channel. D. True.

Indicate whether the following statements are true or false. If a statement is false, explain why it is false. A. Lipid-linked proteins are classified as peripheral membrane proteins because the polypeptide chain does not pass through the bilayer. B. A protein can be embedded on the cytosolic side of the membrane bilayer by employing a hydrophobic α helix. C. A protein that relies on protein-protein interactions to stabilize its membrane association is classified as a peripheral membrane protein because it can be dissociated without the use of detergents. D. Membrane proteins that pump ions in and out of the cell are classified as enzymes.

A. False. Lipid-linked proteins are classified as integral membrane proteins because although they are not transmembrane proteins, they are covalently bound to membrane lipids and cannot be dissociated without disrupting the membrane's integrity. B. False. An embedded protein employs an amphipathic helix. The hydrophobic side interacts with the fatty acid tails of the membrane lipids, and the hydrophilic portion interacts with the aqueous components of the cytosol. C. True. D. False. Membrane proteins that pump ions in either direction across the membrane are in the functional class of transporters.

14-66 Indicate whether the following statements are trueor false. If a statement is false, explain why it is false. A.The dark reactions of photosynthesis occur only in the absence of light. B.Much of the glyceraldehyde 3-phosphate made in the chloroplast ends up producing the molecules needed by the mitochondria to produce ATP. C.Ribulose 1,5-bisphosphate is similar to oxaloacetate in the Krebs cycle in that they are both regenerated atthe end of their respective cycles. D.Each round of the Calvin cycle uses five molecules of CO2to produce one molecule of glyceraldehyde 3-phosphate and one of pyruvate.

A. False. The dark reactions are those involved in carbon fixation and are named as such because they do not require light. B. True. C. True. D. False. Three molecules of CO2 are required for each round of the Calvin cycle, and the product is one molecule of glyceraldehyde 3-phosphate and the recycling of the ribulose 1,5-bisphosphate molecule.

Indicate whether the following statements are true or false. If a statement is false, explain why it is false. A. The amino acids in the interior of a protein do not interact with the ligand and do not play a role in selective binding. B. Antibodies are Y shaped and are composed of six different polypeptide chains. C. ATPases generate ATP for the cell. D. Hexokinase recognizes and phosphorylates only one of the glucose stereoisomers.

A. False. The interior amino acids form a structural scaffold that maintains the specific orientation for those that directly interact with the ligand. Changes to these interior amino acids can change the protein shape and render it nonfunctional. B. False. Although antibodies are Y shaped, they are composed of four, not six, polypeptide chains. There are two heavy chains and two light chains. C. False. ATPases hydrolyze ATP; they do not produce it. These enzymes enable the cell to harness the chemical energy stored in the high-energy phosphate bonds. D. True.

Indicate whether the following statements are true or false. If a statement is false, explain why it is false. A. Generally, the total number of nonpolar amino acids has a greater effect on protein structure than the exact order of amino acids in a polypeptide chain. B. The "polypeptide backbone" refers to all atoms in a polypeptide chain, except for those that form the peptide bonds. C. The chemical properties of amino acid side chains include charged, uncharged polar, and nonpolar. D. The relative distribution of polar and nonpolar amino acids in a folded protein is determined largely by hydrophobic interactions, which favor the clustering of nonpolar side chains in the interior.

A. False. The order in which amino acids are linked is unique for each protein and is the most important factor in determining overall protein structure. B. False. Peptide bonds are planar amide bonds that are central to the polypeptide backbone formation. The atoms in the amino acid side chains are not considered to be part of the backbone. C. True. D. True.

Indicate whether the following statements are true or false. If a statement is false, explain why it is false. A. In order to study the activity of isolated transmembrane proteins, the membrane lipids must be completely stripped away. B. FRAP is a method used to study the movement of individual proteins. C. SDS is a mild detergent that is useful for the reconstitution of membrane components. D. The speed of fluorescent signal recovery during a FRAP assay is a measure of lateral mobility for the molecule of interest.

A. False. The region of the protein that normally crosses the membrane must be stabilized by the presence of phospholipids for the purified protein to be active. For this reason, purified membrane proteins are often reconstituted into artificial lipid bilayers. B. False. The FRAP method involves photobleaching of a small region of the membrane, which contains hundreds of target molecules, and follows the displacement of these molecules with neighboring molecules that have not been bleached. C. False. SDS is a strong, ionic detergent that will break up membrane bilayers and also denature proteins. D. True.

Indicate whether the statements below are true or false. If a statement is false, explain why it is false. A. Facilitated diffusion can be described as the favorable movement of one solute down its concentration gradient being coupled with the unfavorable movement of a second solute up its concentration gradient. B. Transporters undergo transitions between different conformations, depending on whether the substrate-binding pocket is empty or occupied. C. The electrochemical gradient for K+ across the plasma membrane is small. Therefore, any movement of K+ from the inside to the outside of the cell is driven solely by its concentration gradient. D. The net negative charge on the cytosolic side of the membrane enhances the rate of glucose import into the cell by a uniporter.

A. False. This describes coupled transport, which is one type of active transport. Facilitated diffusion can also be called passive transport, in which a solute always moves down its concentration gradient. B. True. C True. D. False. Glucose is an uncharged molecule, and its import is not directly affected by the voltage difference across the membrane if glucose is being transported alone. If the example given were the Na/glucose symporter, we would have to consider the charge difference across the membrane.`

14-44Indicate whether the following statements are true or false. If a statement is false, explain why it is false. A.Ubiquinone is associated with the inner mitochondrial membrane as a protein-bound electron carrier molecule. B.Ubiquinone can transfer only one electron in each cycle. C.The iron-sulfur centers in NADH dehydrogenase are relatively poor electron acceptors. D. Cytochrome c oxidase binds O2using an iron-heme group, where four electrons are shuttled one at a time.

A. False. Ubiquinone is an aromatic compound that uses its long hydrocarbon tail to associate with the inner mitochondrial membrane. B. False. Ubiquinone can transfer one or two electrons. In the case in which only one electron is transferred, the molecule contains an unpaired electron, which is highly reactive. C. True. D. True.

A. How does the generation of ATP by oxidative phosphorylation differ from ATP generation by substrate-level phosphorylation? B. What catabolic process uses substrate-level phosphorylation, and how many ATP molecules are generated in this way in the reaction pathway? C. Where does oxidative phosphorylation take place, and what other processes are required for this to occur?

A. In oxidative phosphorylation, molecular oxygen is required to produce ATP, by means of a multistage, chemiosmotic process (see Chapter 14), while in substrate-level phosphorylation, the energy released by the enzyme-catalyzed oxidation of a substrate generates ATP directly. B. Glycolysis uses substrate-level phosphorylation and generates two ATP molecules for each glucose molecule oxidized to pyruvate molecules (four total ATPs, but two of these are hydrolyzed in the first few steps of the pathway). C. Oxidative phosphorylation takes place in mitochondria as part of a multistage oxidation process. First, the citric acid cycle generates NADH and FADH2, which donate their high-energy electrons to the electron-transport chain. These electrons ultimately reduce molecular oxygen to water, and the energy of their oxidation is used to pump protons across a membrane. The proton gradient thereby produced is in turn harnessed by the enzyme ATP synthase to drive the production of ATP from ADP and inorganic phosphate.

Studies on the squid giant axon were instrumental to our current understanding of how action potentials are generated. You decide to do some experiments on the squid giant axon yourself. A. You remove the cytoplasm in an axon and replace it with an artificial cytoplasm that contains twice the normal concentration of K+ by adding KOAc, where OAc- is an anion to which the membrane is impermeable. In this way, you double the internal concentration of K+ while maintaining the bulk electrical balance of the cytoplasmic solution. Will this make the resting potential of the membrane more or less negative? B. You add NaCl to the extracellular fluid and effectively double the amount of extracellular Na+ ions. How does this affect the action potential? C. You replace half of the NaCl in the extracellular fluid with choline chloride. (Choline is a monovalent cation much larger than Na+. Note that the presence of choline will not impede the flow of Na+ through its channels.) How will this affect the action potential?

A. Increasing the concentration of K+ in the cytoplasm of the squid axon will make the membrane potential more negative. Doubling the amount of K+ increases the driving force for K+ to move out of the cell, leaving the inside of the cell more negative and thus decreasing the membrane potential. (Remember, from the Nernst equation, the driving force for an ion across a membrane is proportional to the ratio of the concentration of the ion on the outside to the concentration of the ion on the inside.) B. Doubling the amount of Na+ in the extracellular fluid will increase the height of the peak of the action potential. Again, this is because now the driving force for Na+ to enter the cell is greater than it was before. Thus, when Na+ channels open, the flux of Na+ ions is now greater. (Remember that flux is the number of ions entering per second.) C. The action potential in this case will reach a height that is less than that normally achieved. (Choline is added in this case to maintain bulk electrical neutrality. Because Na+ channels are not permeable to choline, choline does not contribute to the electrochemical gradient.) You have now halved the concentration of Na+ and thus decreased the driving force for Na+ to enter the cell.

In the budding yeast, activation of the GTP-binding protein Cdc42 occurs on binding of an external signal (pheromone) to a G-protein-coupled receptor. Activation of Cdc42 promotes actin polymerization. Predict what would happen to actin polymerization, in comparison with pheromone-treated cells, in the following cases. A. You add pheromone to an inhibitor of G-protein-coupled receptors. B. You add pheromone to a nonhydrolyzable analog of GTP.

A. Less actin polymerization. Cdc42 will not be able to be activated by the G- protein-coupled receptor. B. More actin polymerization. Cdc42 will be more active, because it will bind the nonhydrolyzable form of GTP and will not be able to be turned off.

Make predictions for what stage of development you would expect to see in all four treatments based on the statements below

A. New mRNA synthesis is NOT required for transition from 1-D to 2-D growth? If new mRNA synthesis is NOT required for transition from 1-D to 2-D growth then the experimental treatments will exhibit 2-D growth B. New mRNA synthesis IS required for transition from 1-D to 2-D growth? If de novo mRNA synthesis IS required for transition from 1-D to 2-D growth then the experimental treatments will exhibit 1-D growth only.

Indicate by writing "yes" or "no" whether amplification of a signal could occur at the particular steps described below. Explain your answers. A. An extracellular signaling molecule binds and activates a GPCR. B. The activated GPCRs cause Gα to separate from Gβ and Gγ. C. Adenylyl cyclase produces cyclic AMP. D. cAMP activates protein kinase A. E. Protein kinase A phosphorylates target proteins.

A. No. Each signaling molecule activates only one receptor molecule. B. Yes. Each activated GPCR activates many G-protein molecules. C. Yes. Each activated adenylyl cyclase molecule can generate many molecules of cAMP. D. No. In unstimulated cells, protein kinase A is held inactive in a protein complex. Binding of cAMP to the complex induces a conformational change, releasing the active protein kinase A. Therefore, one cAMP cannot activate more than one molecule of protein kinase A. E. Yes. Each activated protein kinase A molecule can phosphorylate many molecules of each type of target protein.

Two protein kinases, PK1 and PK2, work sequentially in an intracellular signaling pathway. You create cells that contain inactivating mutations in the genes that encode either PK1 or PK2 and find that these cells no longer respond to a particular extracellular signal. You also create cells containing a version of PK1 that is permanently active and find that the cells behave as though they are receiving the signal even when the signal is not present. When you introduce the permanently active version of PK1 into cells that have an inactivating mutation in PK2, you find that these cells also behave as though they are receiving the signal even when no signal is present. A. From these results, does PK1 activate PK2, or does PK2 activate PK1? Explain your answer. B. You now create a permanently active version of PK2 and find that cells containing this version behave as though they are receiving the signal even when the signal is not present. What do you predict will happen if you introduce the permanently active version of PK2 into cells that have an inactivating mutation in PK1?

A. Normally, PK2 activates PK1. We are told that PK1 and PK2 normally work sequentially in an intracellular signaling pathway. If PK1 is permanently activated, a response is seen independently of whether or not PK2 is present. If PK1 activated PK2, no response should be seen if PK1 were activated in the absence of PK2. B. You would predict that no response to the signal would be observed. This is because PK2 normally needs to activate PK1 for the cells to respond to the signal. When PK2 is permanently activated in the absence of PK1, PK1 is not there to relay the signal.

The field of neurobiology is seeing rapid advances in our understanding of neural circuitry in the brain. Part of this work involves the physical mapping of all synapses (a project dubbed the connectome); another critical advance is in the area of optogenetics, which allows scientists to dissect neural circuits that determine specific behaviors in a range of organisms from fruit flies to monkeys. A. Describe the method of optogenetics. B. Optogenetics has great potential for deepening our understanding of behavior, learning, memory, and cognitive development. However, it is not as likely to be used directly for treatment of problems such as depression or anxiety. Why not? Consider the limitations of the method when answering this question.

A. Optogenetics is a method in which genetic engineering techniques are used to introduce light-gated channels into a selected set of target neurons. Light of a specific wavelength is then used to open the channel, which allows the investigator to directly control the activity of these neurons in the living organism. When the channels used are light-gated Na+ channels, stimulation will allow Na+ to enter the neurons, triggering an action potential. Light-gated Cl- channels can similarly be used to inhibit neural activity. B. Optogenetics currently uses viral vectors to introduce the genes encoding the light-gated channels into the neurons of interest. This approach can be potentially problematic, as insertion of this engineered DNA at off-target sites in the genome could cause mutations that would alter or destroy the activity of critical genes. In addition, the stimulation of neurons by optogenetics requires a light source. In the case of a human patient, this would mean inserting a fiber optic light source into the region of the brain that contains the foreign channels. This would represent a highly experimental treatment, hence it is currently only used in animal models to explore circuitry we do not yet fully understand.

14-9In which of the four compartments of a mitochondrion are each of the following located? A.porin B.the mitochondrial genome C.citric acid cycle enzymes D.proteins of the electron-transport chain E.ATP synthase F.membrane transport protein for pyruvate

A. Porin is in the outer membrane. B. The mitochondrial genome is in the matrix. C. The citric acid cycle enzymes are in the matrix. D. The proteins of the electron-transport chain are in the inner membrane. E. ATP synthase is in the inner membrane. F. The transport protein for pyruvate is in the inner membrane.

Data for the mobility of three different proteins (X, Y, and Z) using fluorescence recovery after photobleaching (FRAP) are shown in Figure Q11-60A. Separately, single-particle tracking (SPT) data were collected for these samples, as shown in Figure Q11-60B A. Assign an SPT profile (A, B, or C) to each of these proteins on the basis of the respective FRAP profiles. B. It is important to remember that in each of these experiments, the results reflect a real, physical difference in the way in which these proteins are situated in the plasma membrane. Provide a plausible explanation for the differences observed in proteins X, Y, and Z.

A. Protein X should have an SPT profile as shown in (B); FRAP data for protein Y correlates with SPT data shown in (A); and Z seems to have intermediate mobility, as shown in the SPT profile (C). B. Protein X is highly mobile, so it is probably a small protein that is not anchored to anything and is not part of a larger, multiprotein complex. Protein Y is relatively immobile, indicating that it is probably anchored to the cell cortex. Protein Z is mobile, but less so than protein X. If protein Z bound to other proteins as part of a complex, the higher molecular weight would retard its observed movement in the membrane.

When activated by the extracellular signal protein platelet-derived growth factor (PDGF), the PDGF receptor phosphorylates itself on multiple tyrosines (as indicated in Figure Q16-55A by the circled Ps; the numbers next to these Ps indicate the amino acid number of the tyrosine). These phosphorylated tyrosines serve as docking sites for proteins that interact with the activated PDGF receptor. These proteins are indicated in the figure, and include the proteins A, B, C, and D. One of the cell's responses to PDGF is an increase in DNA synthesis, which can be measured by the incorporation of radioactive thymidine into the DNA. To determine which protein or proteins—A, B, C, or D—are responsible for the activation of DNA synthesis, you construct mutant versions of the PDGF receptor that retain one or more tyrosine phosphorylation sites. You express these mutant versions in cells that do not make their own PDGF receptor. In these cells, the various mutant versions of the PDGF receptor are expressed normally, and, in response to PDGF binding, become phosphorylated on whichever tyrosines remain. You measure the level of DNA synthesis in cells that express the various mutant receptors and obtain the data shown in Figure Q16-55B. Figure Q16-55 A. From these data, which, if any, of proteins A, B, C, and D are involved in the stimulation of DNA synthesis by PDGF? Explain your answer. B. Which, if any, of these proteins inhibit DNA synthesis? Explain your answer. C. Which, if any, of these proteins seem to have no detectable role in DNA synthesis? Explain your answer.

A. Proteins A and D stimulate DNA synthesis. PDGF receptors that can bind to only A or D (see experiments 2 and 5) can stimulate DNA synthesis to about 50% of normal amounts (which is represented by experiment 1). Proteins A and D are both needed and are used in an additive fashion; this is evident from experiment 6: when a PDGF receptor can bind both A and D, the DNA synthesis level is close to that obtained with the normal receptor. B. Protein B is an inhibitor of DNA synthesis. Consequently, receptors with binding sites for B and D (see experiment 7) stimulate a lower DNA synthesis rate than do receptors that bind only D (experiment 5). C. Protein C has no detectable role in DNA synthesis. Receptors that can bind only C (experiment 4) activate DNA synthesis about as much as receptors that do not bind any of the four proteins (experiment 9; the negative control). Furthermore, the binding of protein C does not affect the response mediated by protein D when the receptor can bind both C and D (experiment 8).

Sodium dodecyl sulfate (SDS) and Triton X-100 are both detergents that can be used to lyse cells. A. If the goal is to study the activity of membrane proteins after cell lysis, explain why SDS would not be a good choice. B. How does Triton X-100 work in cell lysis, and why is it a better choice of detergent to help you extract proteins?

A. SDS is a strong ionic detergent. When cells are exposed to SDS, membrane proteins are not only extracted from the membrane, they are completely unfolded. After denaturation, they cannot be studied as functional molecules. B. Triton X-100 has a smaller nonpolar portion and a polar but uncharged end, allowing it to mimic more closely the type of solvation effect of the membrane lipids. Triton X-100 forms a shell around the hydrophobic portion of the protein without disrupting the existing structure. This makes it possible to then place the protein into a new, synthetic membrane bilayer for study.

Figure Q17-41 shows two isolated outer-doublet microtubules from a eukaryotic flagellum with their associated dynein molecules. A. Sketch what will happen to this structure if it is supplied with ATP. B. Sketch what will happen to this structure if the linking proteins are removed and it is supplied with ATP. C. In a complete flagellum, what would happen if all the dynein molecules were active at the same time?

A. See Figure A17-41A. B. See Figure A17-41B. [Note to instructor: this question should be marked as correct only if the microtubule is shown bending in the correct direction (in A) and the correct microtubule is shown pushed forward (in B)]. C. The flagellum will not bend because there is no significant relative motion of one microtubule doublet to another: each is trying to push its neighbor forward at the same time. For the flagellum to bend, sets of dynein molecules on one side of the flagellum must be selectively activated.

You are interested in cell-size regulation and discover that signaling through an enzyme-coupled receptor is important for the growth (enlargement) of mouse liver cells. Activation of the receptor activates adenylyl cyclase, which ultimately leads to the activation of PKA, which then phosphorylates a transcription factor called TFS on threonine 42. This phosphorylation is necessary for the binding of TFS to its specific sites on DNA, where it then activates the transcription of Sze2, a gene that encodes a protein important for liver cell growth. You find that liver cells lacking the receptor are 15% smaller than normal cells, whereas cells that express a constitutively activated version of PKA are 15% larger than normal liver cells. Given these results, predict whether you would expect the cell's size to be bigger or smaller than normal cells if cells were treated in the following fashion. A. You change threonine 42 on TFS to an alanine residue. B. You create a version of the receptor that is constitutively active. C. You add a drug that inhibits adenylyl cyclase. D. You add a drug that increases the activity of cyclic AMP phosphodiesterase. E. You mutate the cAMP-binding sites in the regulatory subunits of PKA, so that the complex binds cAMP more tightly.

A. Smaller. This mutation will make a TFS that cannot be phosphorylated by PKA. B. Bigger. C. Smaller. D. Smaller. cAMP phosphodiesterase is involved in converting cAMP to AMP and will down-regulate this signaling pathway. E. Bigger. Higher affinity of the PKA complex for cAMP will increase its activity, and thus cells will be bigger.

Although the outer mitochondrial membrane is permeable to all small molecules, the inner mitochondrial membrane is essentially impermeable in the absence of specific transport proteins. Consider this information and what you have learned about the citric acid cycle to address the following questions. A. The ATP generated by oxidative respiration is used throughout the cell. The majority of ATP production occurs in the mitochondrial matrix. How do you think ATP is made accessible to enzymes in the cytosol and other organelles? B. If the inner mitochondrial membrane were rendered as permeable as the outer membrane, how would that affect oxidative phosphorylation? Which specific processes would stop and which remain? C. Present two types of benefits derived from separating the reactions of glycolysis in the cytosol from those that occur during the citric acid cycle in the mitochondrion.

A. The ATP must be transported across the inner mitochondrial membrane, after which it freely diffuses into the cytosol through the permeable outer membrane. Embedded in the inner membrane are dedicated ADP/ATP antiporters that serve the dual purpose of exporting ATP and bringing in new ADP, which can then be converted into ATP during oxidative phosphorylation. B. During oxidative phosphorylation, the NADH and FADH2 generated by the citric acid cycle donate their electrons to an electron-transport chain in the inner mitochondrial membrane. As the electrons move along this chain, the energy released is used to drive protons across the inner mitochondrial membrane. This movement of protons produces a proton gradient across the membrane, which Page 25 of 27 then serves as a source of energy for the generation of ATP. If the inner mitochondrial membrane were made "leaky," the proton gradient would dissipate. Thus although acetyl CoA would continue to be oxidized by the citric acid cycle, and electrons donated to the electron-transport chain, these processes could no longer promote the production of ATP. C. Compartmentalization provides a basic mechanism for the regulation of independent sets of reactions, including the citric acid cycle, fatty acid oxidation, glycolysis, and gluconeogenesis. In some cases, metabolic reactions are physically compartmentalized to separate anabolic from catabolic reactions. For example, in the mitochondrial matrix, oxaloacetate is used in the citric acid cycle to help oxidize the acetyl carbons of acetyl CoA. However, oxaloacetate in the cytosol tends to be consumed by biosynthetic enzymes that use the molecule as a precursor for the production of amino acids such as aspartate. By keeping these reactions separate, the cell can control whether a molecule is used in an anabolic or catabolic reaction. A second advantage of compartmentalization is the co-localization and concentration of enzymes with their substrates, which can enhance reaction rates.

For each of the following sentences, fill in the blank with the appropriate type of gating for the ion channel described. You can use the same type of gating mechanism more than once. A. The acetylcholine receptor in skeletal muscle cells is a(n) _________ ion channel. B. _________ ion channels are found in the hair cells of the mammalian cochlea. C. _________ ion channels in the mimosa plant propagate the leaf-closing response. D. _________ ion channels respond to changes in membrane potential. E. Many receptors for neurotransmitters are _________ ion channels.

A. The acetylcholine receptor in skeletal muscle cells is a ligand-gated ion channel. B. Stress-gated ion channels are found in the hair cells of the mammalian cochlea. C. Voltage-gated ion channels in the mimosa plant propagate the leaf-closing response. D. Voltage-gated ion channels respond to changes in membrane potential. E. Many receptors for neurotransmitters are ligand-gated ion channels.

You are curious about the dynamic instability of microtubules and decide to join a lab that works on microtubule polymerization. The people in the lab help you grow some microtubules in culture using conditions that allow you to watch individual microtubules under a microscope. You can see the microtubules growing and shrinking, as you expect. The professor who runs the lab gets in a new piece of equipment, a very fine laser beam that can be used to sever microtubules. She is very excited and wants to sever growing microtubules at their middle, using the laser beam. A. Do you predict that the newly exposed microtubule plus ends will grow or shrink? Explain your answer. B. What do you expect would happen to the newly exposed plus ends if you were to grow the microtubules in the presence of an analog of GTP that cannot be hydrolyzed, and you then severed the microtubules in the middle with a laser beam?

A. The newly exposed microtubule plus ends will most probably shrink if you sever the microtubules in the middle. This is because a microtubule grows by adding GTP-carrying subunits to the plus end. The GTP is hydrolyzed over time, leaving only a cap of GTP-carrying subunits at the plus end, with the remainder of the tubulin protofilament containing GDP-carrying subunits. Therefore, if you sever a growing microtubule in its middle, you will most probably create a plus end that contains GDP-carrying subunits. The GDP-carrying subunits are less tightly bound than the GTP-carrying subunits and will peel away from each other, causing depolymerization of the microtubule and shrinkage. B. If you were to polymerize the microtubules in the presence of a nonhydrolyzable analog of GTP and you then severed the microtubules with a laser, the newly exposed plus end would contain a GTP cap and so would probably continue to grow.

Molecular Mechanisms of Electron Transport and Proton Pumping 14-39Mitochondrial structure and the reaction products generated inside the matrix are critical for generating stores of energy. Answer the following questions based on what you know about mitochondrial structure and processes. A.The gradients used to generate ATP are maintained across the inner mitochondrial membrane. Why don't we observe a similar gradient generation across the outer mitochondrial membrane? B.The proton-motive force created by the electrochemical proton gradient is the source of free energy utilized in ATP formation. Describe the two components contributing to the total proton-motive force.

A. The outer mitochondrial membrane contains large, channel-forming proteins called porins, which makes this membrane permeable to small molecules. Protons, other ions, nucleotides, and many other small molecules flow freely across this membrane, making it impossible to establish a gradient of any of these molecules on either side of this membrane. B. One component of the proton-motive force is the concentration gradient of protons (or pH gradient) across the membrane. This pH gradient makes it energetically favorable for protons to flow back into the matrix. A second component of the proton-motive force is a charge differential across the membrane, referred to as the membrane potential. Because the matrix side of the membrane has a net negative charge, and protons have a positive charge, this component of the proton-motive force also drives the movement of protons back into the matrix.

15-26 What would happen in each of the following cases? Assume in each case that the protein involved is a soluble protein, not a membrane protein. A.You add a signal sequence (for the ER) to the N-terminal end of a normally cytosolic protein. B.You change the hydrophobic amino acids in an ER signal sequence into charged amino acids. C.You change the hydrophobic amino acids in an ER signal sequence into other hydrophobic amino acids. D.You move the N-terminal ER signal sequence to the C-terminal end of the protein.

A. The protein will now be transported into the ER lumen. B. The altered signal sequence will not be recognized and the protein will remain in the cytosol. C. The protein will still be delivered into the ER. It is the distribution of hydrophobic amino acids that is important, not the actual sequence. D. The protein will not enter the ER. Because the C-terminus of the protein is the last part to be made, the ribosomes synthesizing this protein will not be recognized by the signal-recognition particle (SRP) and hence not carried to the ER.

15-33Using genetic engineering techniques, you have created a set of proteins that contain two (and only two) conflicting signal sequences that specify different compartments. Predict which signal would win out for the following combinations. Explain your answers. A.Signals for import into the nucleus and import into the ER. B.Signals for export from the nucleus and import into the mitochondria. C.Signals for import into mitochondria and retention in the ER.

A. The protein would enter the ER. The signal for a protein to enter the ER is recognized as the protein is being synthesized and the protein will end up either in the ER or on the ER membrane. Cytosolic nuclear transport proteins recognize proteins destined for the nucleus once those proteins are fully synthesized and fully folded. B. The protein would enter the mitochondria. For a nuclear export signal to work, the protein would have to end up in the nucleus first and thus would need a nuclear import signal for the nuclear export signal to be used. C. The protein would enter the mitochondria. To be retained in the ER, the protein needs to enter the ER. Because there is no signal for ER import, the ER retention signal would not function.

14-38The respiratory chain is relatively inaccessible in the experimental manipulation of intact mitochondria. After disrupting mitochondria with ultrasound, however, it is possible to isolate functional submitochondrial particles, which consist of broken cristae that have resealed inside-out into small,closed vesicles. In these vesicles,the components that originally faced the matrix are now exposed to the surrounding medium. A.How might such an arrangement aid in the study of electron transport and ATP synthesis? B.Consider an anaerobic preparation of such submitochondrial particles. If a small amount of oxygen is added, do you predict that the preparation will consume oxygen in respiration reactions? Will the medium outside the particles become more acidic or morebasic? What, if anything, will change if the flow of protons through ATP synthase is blocked by an inhibitor? Explain your answer.

A. This arrangement of components within the vesicles allows the experimental manipulation of the medium surrounding the vesicles, which permits the consequences of different conditions in the mitochondrial matrix to be examined. The medium can be altered by changing pH, adding electron carriers and oxygen, and providing ADP and Pi, for example. The oxidation of electron carriers, the consumption of oxygen, and the production of ATP can be measured in the medium. By changing the composition of the medium, it should be possible, for example, to identify the electron carriers that can donate electrons from the matrix to the transport chain (the side of the membrane that normally faces the matrix is now on the outside), to assess the redox potentials of various components of the transport chain, and to determine the dependence of ATP synthesis on the pH gradient across the membrane and on the ATP/ADP ratio. B. Respiration reactions will rapidly consume at least some of the added oxygen. During the anaerobic conditions, the electron carriers in the electron-transport chain were reduced; on the addition of oxygen, electrons will be transferred to oxygen, thereby reducing the oxygen and oxidizing the carriers. Concomitantly with the electron flow, protons will be pumped from the medium into the vesicles, thereby making the medium slightly more basic and the inside of the vesicles acidic. Inhibition of the ATP synthase will not have an immediate effect on oxygen consumption or proton pumping. However, the proton concentration inside the vesicles will quickly become too high to continue the activity of the electron-transport-coupled proton pumping, and thus electron transport and oxygen consumption will cease.

Indicate whether the statements below are true or false. If a statement is false, explain why it is false. A. Neurotransmitters are small molecules released into the synaptic cleft after the fusion of synaptic vesicles with the presynaptic membrane. B. Action potentials are usually mediated by voltage-gated Ca2+ channels. C. Voltage-gated Na+ channels become automatically inactivated shortly after opening, which ensures that the action potential cannot move backward along the axon. D. Voltage-gated K+ channels also open immediately in response to local depolarization, reducing the magnitude of the action potential.

A. True. B. False. Action potentials are usually mediated by voltage-gated Na+ channels. C. True. D. False. Voltage-gated K+ channels respond more slowly than the voltage-gated Na+ channels. Because voltage-gated K+ channels do not open until the action potential reaches its peak, they do not affect its magnitude. Instead, they help to restore the local membrane potential quickly while the voltage-gated Na+ channels are in the inactivated conformation.

Indicate whether the following statements are true or false. If a statement is false, explain why it is false. A. CO2 and H2O are generated during the oxidation of food molecules. B. Activated carrier molecules store heat energy for the cell to use later. C. Catabolism is a general term that refers to the processes by which large molecules are synthesized from smaller molecules. D. The oxidation of sugar is an energetically favorable process.

A. True. B. False. Activated carriers have high-energy bonds that can drive other reactions when broken. Heat may be released during these reactions and may increase the reaction rates, but is not a form of energy that is stored in biological systems. C. False. Catabolism comprises the metabolic reactions that are involved in breaking large molecules into smaller molecules. Anabolism encompasses the reverse types of reactions: synthesizing larger molecules from smaller molecules. D. True.

14-8 Indicate whether the following statements are true or false. If a statement is false, explain why it is false. A.The number and location of mitochondria within a cell can change, depending on both the cell type and the amount of energy required. B.The inner mitochondrial membrane contains porins, which allow pyruvate to enter for use in the citric acid cycle. C.The inner mitochondrial membrane is actually a series of discrete,flattened,membrane-enclosed compartments called cristae, similar to what is seen in the Golgi apparatus. D.The intermembrane space of the mitochondria is chemically equivalent to the cytosol with respect to pH and the small molecules present.

A. True. B. False. The outer mitochondrial membrane contains porins, allowing the passage of all molecules with a mass of less than 5000 daltons. Although pyruvate must pass through the inner membrane, it does so in a highly regulated manner via specific transporter channels. C. False. Although the cristae do look like individual compartments on the basis of the images of the inner structure of the mitochondria, the inner membrane is a single, albeit highly convoluted, membrane. D. True.

Indicate whether the following statements are true or false. If a statement is false, explain why it is false. A. CO2 and O2 are water-soluble molecules that diffuse freely across cell membranes. B. The differences in permeability between artificial lipid bilayers and cell membranes arise from variations in phospholipid content. C. Transporters are similar to channels, except that they are larger, allowing folded proteins as well as smaller organic molecules to pass through them. D. Cells expend energy in the form of ATP hydrolysis so as to maintain ion concentrations that differ from those found outside the cell.

A. True. B. False. The primary difference between cell membranes and artificial membranes is that cell membranes have proteins responsible for creating a selective permeability, which varies with the location and function of the membrane. C. False. Transporters work by changing conformation after specific binding of the solute to be transported. Channels exclude molecules on the basis of size and charge, but do not depend on specific recognition of the molecules moving through. D. True.

Indicate whether the statements below are true or false. If a statement is false, explain why it is false. A. The extracellular concentration of Ca2+ is approximately 104-fold higher than the concentration of Ca2+ in the cytosol. B. The low cytosolic Ca2+ concentration sensitizes the cell to an influx of Ca2+, ensuring a rapid response to environmental stimuli. C. Cytosolic Ca2+ concentration is kept low by the use of chelators such as EDTA. D. The primary mechanism by which Ca2+ acts as a signaling molecule is by increasing the net charge in the cytosol.

A. True. B. True. C False. Ca2+ concentrations in the cytosol are kept low by the action of ATP-driven calcium pumps in the endoplasmic reticulum membrane and the plasma membrane. D. False. Ca2+ binds tightly to many proteins in the cell, which in turn changes their activity. This interaction is the primary mechanism by which Ca2+ signaling occurs.

The oxidative reactions of cellular respiration were the focus of intense study in the 1930s. These reactions are represented in a linear pathway, as they were thought to occur. Each product is designated as a lettered compound (A through H) in Figure Q13-57. Figure Q13-57 A. What was the first observation that Krebs made when he added malonic acid to the minced muscle samples, and what was his conclusion about how and where it was acting in the reactions he was studying? B. What happens when the malonate block is introduced and subsequently compound A is added in excess? What is the result if compound G is added after the block, instead of A? How did Krebs attempt to reconcile these two results? C. What additional observation led Krebs to hypothesize that what was previously thought to be a linear sequence of reactions is actually a cyclic sequence of reactions? How did this idea further explain the earliest observations that the addition of any single compound in the pathway greatly increases oxygen uptake by the muscle tissue?

A. When malonic acid was added to the minced muscle samples, compound E accumulated and the concentrations of compounds F, G, and H were severely diminished. Krebs interpreted this to mean that malonic acid blocked one of the reactions by inhibiting the enzyme involved. The inhibitory effect of malonic acid on the reaction cycle became known as the malonate block. B. After the malonate block is in place, if compound A is added in excess, compound E will accumulate, inhibiting the pathway at this point and preventing the conversion of E to F. If the experiment is repeated and compound G is added in excess instead of compound A, the result is the same. The first explanation for this surprising result was that the F G H pathway somehow also led to an intermediate that could be converted into compound E. C. When Krebs combined pyruvate and oxaloacetate (compound H) with the muscle suspensions, he noticed an immediate production of citrate (compound A). This result made it logical to conclude that H is recycled and used in the first reaction. The model of a reaction cycle explained all the other previous results, including the observation that any one compound can be added to increase O2 uptake. A large amount of any intermediate will drive the production of the others, as long as pyruvate is available. The cofactors produced are used in the electron-transport chain, which terminates with the splitting of oxygen and its reduction to form water.

You have prepared lipid vesicles (spherical lipid bilayers) that contain Na+-K+ pumps as the sole membrane protein. All of the Na+-K+ pumps are oriented in such a way that the portion of the molecule that normally faces the cytosol is on the inside of the vesicle and the portion of the molecule that normally faces the extracellular space is on the outside of the vesicle. Assume that each pump transports one Na+ ion in one direction and one K+ ion in the other direction during each pumping cycle (see Figure Q12-24 for how the Na+-K+ pump normally functions in the plasma membrane). Figure Q12-24 Predict what would happen in each of the following conditions: A. The solutions inside and outside the vesicles contain both Na+ and K+ ions but no ATP. B. The solution outside the vesicles contains both Na+ and K+ ions; the solution inside contains both Na+ and K+ ions and ATP. C. The solution outside contains Na+; the solution inside contains Na+ and ATP.

A. Without any ATP to provide energy for the Na+-K+ pumps, no ions will be pumped. B. The pumps will use the energy from ATP hydrolysis to transport Na+ out of the vesicles and K+ into the vesicles. (The pumps will stop working either when the amount of ATP inside the vesicle is depleted or when the K+ outside the vesicles is depleted.) C. The pump will bind a molecule of Na+, causing the ATPase activity to hydrolyze ATP and transfer the phosphate group onto the pump. A conformational change will occur, leading to the release of Na+ from the vesicle. However, because there is no K+ outside the vesicle, the pump will get stuck at that step and subsequent steps of the cycle will not occur.

You are interested in studying kinesin movements. You therefore prepare silica beads and coat them with kinesin molecules so that each bead, on average, has only one kinesin molecule attached to it. You add these kinesin-coated beads to a preparation of microtubules you have polymerized. Using video microscopy, you watch the kinesin [labeled with green fluorescent protein (GFP)] move down the microtubules. A. Kinesin-GFP has been measured to move along microtubules at a rate of 0.3 μm/sec, and single-molecule studies have revealed that kinesin moves along microtubules progressively, with each step being 8 nm. How many steps can the kinesin molecule take in 4 seconds, assuming that the kinesin stays attached to the microtubule for the entire 4 seconds? B. Because each kinesin molecule is thought to take approximately 100 steps before falling off the microtubule, will you see your silica beads detach from the microtubule during your 4 seconds of observation? C. What would you predict would happen to the kinesin-coated silica beads if you were to add AMP-PNP (a nonhydrolyzable ATP analog)?

A. You would expect the kinesin molecule to travel 150 steps. The calculation is as follows: 0.3 μm = 300 nm. Therefore, in 4 seconds, the kinesin molecule could travel 1200 nm if it were to move at a rate of 0.3 μm/sec. Because the step size is 8 nm, 1200 nm/(8 nm per step) = 150 steps. B. Yes, you should see silica beads detach some time during the 4 seconds of observation, because each kinesin will take more than 100 steps in that 4-second time frame (see part A above). C. If you were to add AMP-PNP, you would no longer see the silica beads moving down the microtubule. It is thought that one molecule of ATP is hydrolyzed per step that kinesin takes; without ATP hydrolysis, translocation of the beads will be inhibited. However, you may still see the beads associated with the microtubules, because AMP-PNP does not inhibit the association of kinesin with the microtubule.

Indicate which of the three major classes of cytoskeletal elements each statement below refers to. A. monomer that binds ATP B. includes keratin and neurofilaments C. important for formation of the contractile ring during cytokinesis D. supports and strengthens the nuclear envelope E. their stability involves a GTP cap F. used in the eukaryotic flagellum G. a component of the mitotic spindle H. can be connected through desmosomes I. directly involved in muscle contraction J. abundant in filopodia

A. actin B. intermediate filaments C. actin D. intermediate filaments E. microtubules F. microtubules G. microtubules H. intermediate filaments I. actin J. actin

Circle the phrase in each pair that is likely to occur more rapidly in response to an extracellular signal. A. changes in cell secretion / increased cell division B. changes in protein phosphorylation / changes in proteins being synthesized C. changes in mRNA levels / changes in membrane potential

A. changes in cell secretion B. changes in protein phosphorylation C. changes in membrane potential

Indicate whether each of the following statements refers to a ciliary microtubule, a microtubule of the mitotic spindle, both types of microtubule, or neither type of microtubule. A. The basal body is the organizing center. B. The monomer is sequestered by profilin. C. It is arranged in a "9 + 2" array. D. It is nucleated at the centrosome. E. It uses dynein motors. F. It is involved in sperm motility. G. It is involved in moving fluid over the surface of cells.

A. ciliary microtubules B. neither C. ciliary microtubules D. microtubules of the mitotic spindle E. both F. neither (this involves flagellar microtubules) G. ciliary microtubules

When adrenaline binds to adrenergic receptors on the surface of a muscle cell, it activates a G protein, initiating an intracellular signaling pathway in which the activated α subunit activates adenylyl cyclase, thereby increasing cAMP levels in the cell. The cAMP molecules then activate a cAMP-dependent kinase (PKA) that, in turn, activates enzymes that result in the breakdown of muscle glycogen, thus lowering glycogen levels. You obtain muscle cells that are defective in various components of the signaling pathway. Referring to Figure Q16-36, indicate how glycogen levels would be affected in the presence of adrenaline in the following cells. Would they be higher or lower than in normal cells treated with adrenaline? A. cells that lack adenylyl cyclase B. cells that lack the GPCR C. cells that lack cAMP phosphodiesterase D. cells that have an α subunit that cannot hydrolyze GTP but can interact properly with the β and γ subunits

A. higher B. higher C. lower D. lower

If ATP production is blocked in an animal cell, the cell will swell up. Explain this observation.

ATP is required to power the Na+-K+ pump, which is necessary for maintaining osmotic balance. The pump requires ATP hydrolysis to drive its pumping cycle. So, in the absence of ATP production, the Na+ concentration inside the cell will increase. This is followed by passive diffusion of water across the membrane, causing the cell to swell.

A process that requires ATP energy to move a substance independently of its concentration gradient is called

Active transport

_____processes move substances across a membrane regardless of concentration gradients.

Active transport

What does the reaction center complex do?

After receiving electrons in their excited state from the light harvesting complex, there are 2 special chlorophyll molecules which donate these electrons to the primary electron accepter so that they may undergo the ETC.

What is similar between desmosomes and adherens junctions?

All the above

Which of the following statements about allostery is true?

Allosteric regulation is always used for negative regulation of enzyme activity.

64. Genomic sequences are now available for an increasing number of species, and scientists studying evolutionary processes can take advantage of this enormous amount of data to bring evolution into the arena of molecular studies. By aligning the sequences of homologous genes and looking for regions of similarity and where changes have occurred, it is possible to infer the sequence of the ancestral gene. b. It is possible to express an ancestral gene sequence in modern organisms and subsequently compare the function of its product with that of the modern protein. Why might this approach give misleading conclusions?

Although an inferred ancestral sequence can be reconstructed and the protein expressed, you would be placing an inferred, ancient protein in the context of a modern cell. If there are important interacting partners for the modern protein, there is a chance they may not recognize the ancestral protein, and therefore any information about its function may be inaccurate.

When the neurotransmitter acetylcholine is applied to skeletal muscle cells, it binds the acetylcholine receptor and causes the muscle cells to contract. Succinylcholine, which is a chemical analog of acetylcholine, binds to the acetylcholine receptor on skeletal muscle cells but causes the muscle cells to relax; it is therefore often used by surgeons as a muscle relaxant. Propose a model for why succinylcholine causes muscle relaxation. What might be the mechanism to explain the different activities of acetylcholine and succinylcholine on the acetylcholine receptor?

Although succinylcholine can bind to the acetylcholine receptor, it does not activate the receptor and therefore does not cause the muscle cell to contract. Instead, succinylcholine blocks the ability of acetylcholine to bind to the receptor and thereby prevents acetylcholine from stimulating muscle contraction.

It is thought that the glucose transporter switches between two conformational states in a completely random fashion. How is it possible for such a system to move glucose across the membrane efficiently in a single direction?

Although the opening of the glucose transporter on one side of the membrane or the other is random, the binding of glucose into the binding site of the transporter is not a random event. The affinity between the glucose molecule and the transporter governs the binding event: transporter + glucose ↔ transporter-glucose At high glucose concentrations, the complex formation is favored; at low glucose concentrations, dissociation of glucose from the transporter is favored. So, as long as there is a large concentration gradient, efficient transport can occur by the simple rules of binding equilibria.

3.30.c.The branched diagram in Figure Q3-30 represents several possible reaction pathways that substance X may follow. Each branch point represents two possible reactions, which would give rise to different products. Next to each of these points are values for the uncatalyzed activation energies and catalyzed activation energies (in kcal/mole), respectively. Compare your answers for parts A and B. How do you think the uncatalyzed reactions could influence the rates of catalyzed reactions inside the cell?

Although we can make these predictions based on activation energies, there are many other factors that determine how fast a certain product will accumulate: reactant concentration, catalytic efficiency of the enzyme in question, and the relative reactivity of the product, to name a few. If a reaction has a very low uncatalyzed activation energy, it could occur spontaneously at a rapid rate, which would decrease the concentration of substrate available for the enzyme-catalyzed reaction. If this is the case, the relative abundances predicted in part B will change.

The term N-terminus refers to the presence of a(n) ________ at the 5′ end of a polypeptide.

Amino Group

n proteins, secondary, tertiary, and quaternary levels of structure depend on primary structure. Which of the following most accurately lists elements of any protein's primary, secondary, tertiary, and quaternary structure, in that order? Amino acid sequence, hydrogen bonding between backbone groups, overall shape of a single polypeptide, and combinations of tertiary structures Amino acid sequence, hydrogen bonding between R-groups, overall shape of a single polypeptide, and multiple polypeptide subunits One polypeptide chain, two polypeptide chains, three polypeptide chains, and more than three polypeptide chains Covalent bonding, van der Waals interactions, hydrogen bonding, and hydrophobic interactions,

Amino acid sequence, hydrogen bonding between backbone groups, overall shape of a single polypeptide, and combinations of tertiary structures

An extracellular signal molecule can act to change a cell's behavior by acting through cell-surface __________________ that control intracellular signaling proteins. These intracellular signaling proteins ultimately change the activity of __________________ proteins that bring about cell responses. Intracellular signaling proteins can __________________ the signal received to evoke a strong response from just a few extracellular signal molecules. A cell that receives more than one extracellular signal at the same time can __________________ this information using intracellular signaling proteins. __________________ proteins can act as molecular switches, letting a cell know that a signal has been received. Enzymes that phosphorylate proteins, termed ___________, can also serve as molecular switches; the actions of these enzymes are countered by the activity of __________________. acetylase decouple GTP-binding AMP-binding decrease neurotransmitter amplify effector protein kinases autocrine esterases protein phosphatases cleavage integrate receptors convolute GMP-binding sterols

An extracellular signal molecule can act to change a cell's behavior by acting through cell-surface receptors that control intracellular signaling proteins. These intracellular signaling proteins ultimately change the activity of effector proteins that bring about cell responses. Intracellular signaling proteins can amplify the signal received to evoke a strong response from just a few extracellular signal molecules. A cell that receives more than one extracellular signal at the same time can integrate this information using intracellular signaling proteins. GTP-binding proteins can act as molecular switches, letting a cell know that a signal has been received. Enzymes that phosphorylate proteins, termed protein kinases, can also serve as molecular switches; the actions of these enzymes are countered by the activity of protein phosphatases.

What is the electron accepter in fermentation?

An organic molecule, such as pyruvate or acetaldehyde

Hemidesmosomes bind intermediate filaments to _________ to integrins to ECM fibrous proteins

Anchor Proteins

Anemia, a condition that results in individuals with a low red blood cell count, can be caused by a number of factors. Why do individuals with defects in the spectrin protein often have this condition?

Anemia, a condition that results in individuals with a low red blood cell count, can be caused by a number of factors. Why do individuals with defects in the spectrin protein often have this condition?

Cell membranes are fluid, and thus proteins can diffuse laterally within the lipid bilayer. However, sometimes the cell needs to localize proteins to a particular membrane domain. Name three mechanisms that a cell can use to restrict a protein to a particular place in the cell membrane.

Any combination of the following four answers is acceptable. 1. The protein can be attached to the cell cortex inside the cell. 2. The protein can be attached to the extracellular matrix outside the cell. 3. The protein can be attached to other proteins on the surface of a different cell. 4. The protein can be restricted by a diffusion barrier, such as that set up by specialized junctional proteins at a tight junction.

Any substance that will bind to a protein is known as its __________________. Enzymes bind their __________________ at the __________________. The enzyme hexokinase is so specific that it reacts with only one of the two __________________ of glucose. Enzymes catalyze a chemical reaction by lowering the __________________, because they provide conditions favorable for the formation of a __________________ intermediate called the __________________. Once the reaction is completed, the enzyme releases the __________________ of the reaction. activation energy inhibitors products active site isomers substrates free energy ligand transition state high-energy low-energy

Any substance that will bind to a protein is known as its *ligand*. Enzymes bind their *substrates* (or inhibitors) at the *active site*. The enzyme hexokinase is so specific that it reacts with only one of the two *isomers* of glucose. Enzymes catalyze a chemical reaction by lowering the *activation energy*, because they provide conditions favorable for the formation of a *high-energy* intermediate called the transition state. Once the reaction is completed, the enzyme releases the *products* of the reaction.

2.11. you explain to a friend what you have learned about avogadro's number. your friend thinks the number is so large that he doubts there is even a mole of living cells on the earth.You have recently heard that there are about 50 trillion (5 × 1013) human cells in each adult human body and that each human contains more bacterial cells (in the digestive system) than human cells, so you bet your friend $5 that there is more than a mole of cells on Earth. The human population is approximately 7 billion (7 × 109). What calculation can you show your friend to convince him you are right?

Avogadro's number, or 6 × 1023, is the number of atoms (or units) in a mole. If you multiply the number of people on Earth by the number of cells in the human body, then double it to account for the bacteria, you will calculate: (7 × 109) × (1 × 1014) = 7 × 1023. Thus, even when only considering the human population and the associated microbial populations, you can estimate more than a mole of living cells on Earth. You win $5.

15-55 A plasma membrane protein carries an oligosaccharide containing mannose (Man), galactose (Gal), sialic acid (SA), and N-acetylglucosamine (GlcNAc). These sugars are added to the protein as it proceeds through the secretory pathway. First, a core oligosaccharide containing Man and GlcNAc is added, followed by Gal, Man, SA, and GlcNAc in a particular order. Each addition is catalyzed by a different transferase acting at a different stage as the protein proceeds through the secretory pathway. You have isolated mutants defective for each of the transferases, purified the membrane protein from each of the mutants, and identified which sugars are present in each mutant protein. Table Q15-55 summarizes the results. Table Q15-55From these results, match each of the transferases (A, B, C, D) to its subcellular location selected from the list below. (Assume that each location contains only one enzyme.) 1.central Golgi cisternae 2.cisGolgi network 3.ER 4.trans Golgi network

A—3 (oligosaccharide protein transferase = ER) B—1 (galactose transferase = central Golgi cisternae) C—4 (SA transferase = trans Golgi network) D—2 (GlcNAc transferase = cis Golgi network Proteins are modified in a stepwise fashion in the Golgi apparatus, with early steps taking place in the cis Golgi network, intermediate steps taking place in the central Golgi cisternae, and late steps occurring in the trans Golgi network. If each enzyme produces the substrate for the next step, then a mutant lacking the enzyme that catalyzes the addition of the first sugar will be missing all of the sugars, a mutant lacking the enzyme that catalyzes the addition of the second sugar will contain the first sugar but will lack the other three, and so on. By this logic, mannose and GlcNAc must be the first sugars added, additional GlcNAc is the second added, galactose the third, and SA the last. Hence, the oligosaccharide protein transferase must be in the ER, the GlcNAc transferase in the cis Golgi network, the galactose transferase in the central Golgi cisternae, and the SA transferase in the trans Golgi network.

15-53 Match the set of labels below with the numbered label lines on FigureQ15-53. A.cisterna B.Golgi stack C.secretory vesicle D.trans Golgi network E.cis Golgi network

A—3; B—1; C—5; D—4; E—2

Match the following labels to the numbered lines on Figure Q17-36. A. minus end of microtubule B. tail of motor protein C. cargo of motor protein D. head of motor protein Which of the two motors in Figure Q17-36 is most probably a kinesin? Explain your answer.

A—4; B—2; C—1; D—3 The top motor is more likely to be kinesin, because kinesins usually move toward the plus end of the microtubules.

For each of the following indicate whether the individual folded polypeptide chain forms a globular (G) or fibrous (F) protein molecule. A. keratin B. lysozyme C. elastin D. collagen E. hemoglobin F. actin

A—F; B—G; C—F; D—F; E—G; F—G

A patient is rushed to the emergency room and immediately needs a blood transfusion. The patient is found to have only anti-A antibodies in circulation. The blood type of this individual must be A) A. B) B. C) AB. D) O.

B

Carbon dioxide (CO2) is released during which of the following stages of cellular respiration? A) glycolysis and the oxidation of pyruvate to acetyl CoA B) oxidation of pyruvate to acetyl CoA and the citric acid cycle C) the citric acid cycle and oxidative phosphorylation D) oxidative phosphorylation and fermentation E) fermentation and glycolysis

B

In glycolysis, for each molecule of glucose oxidized to pyruvate A) 2 molecules of ATP are used and 2 molecules of ATP are produced. B) 2 molecules of ATP are used and 4 molecules of ATP are produced. C) 4 molecules of ATP are used and 2 molecules of ATP are produced. D) 2 molecules of ATP are used and 6 molecules of ATP are produced. E) 6 molecules of ATP are used and 6 molecules of ATP are produced.

B

Most CO2 from catabolism is released during A) glycolysis. B) the citric acid cycle. C) lactate fermentation. D) electron transport. E) oxidative phosphorylation.

B

Which process in eukaryotic cells will proceed normally whether oxygen (O2) is present or absent? A) electron transport B) glycolysis C) the citric acid cycle D) oxidative phosphorylation E) chemiosmosis

B

What is a major difference between the extracellular matrix (ECM) of a plant cell and the ECM of an animal cell?

B) Plant ECM is primarily carbohydrate in nature, whereas animal ECM is mainly proteins.

receptor—signal complex brings about a series of events that result in contraction of skeletal muscle. Venom from black widow spiders causes an explosive release of acetylcholine. What would that do to its victims?

B) The victim's muscles would be unable to relax.

Which of the following is the best example of gene expression? A) a frog adapts to variation in its environmental temperature. B) mouse fur color results from pigment formed by gene-encoded enzymes C) DNA is replicated during the S phase of the cell cycle D) the percent of A versus a alleles in a population is altered by natural selection E) mutation alters the sequence of a region of DNA

B) mouse fur color results from pigment formed by gene-encoded enzymes

Osteocytes are bone cells. Collagen fibers and calcium salts are found in abundance between and among the osteocytes. The collagen and calcium salts are ________.

B) part of the extracellular matrix

2.72.b.Eukaryotic cells have their DNA molecules inside their nuclei. However, to package all the DNA into such a small volume requires the cell to use specialized proteins called histones. Histones have amino acid sequences enriched for lysines and arginines.B. Lysine side chains are substrates for enzymes called acetylases. A diagram of an acetylated lysine side chain is shown in Figure Q2-72. How do you think the acetylation of lysines in histone proteins will affect the ability of a histone to perform its role (refer to your answer in part A)?

B. A histone with acetylated lysine residues will not be as good at packaging the DNA. The addition of the acetyl group to the terminal amino on the lysine side chain lowers the histone's net positive charge, which makes it less effective at buffering the negative charges on the DNA backbone.

3.43.b. Consider the reaction XY in a cell at 37°C. At equilibrium, the concentrations of X and Y are 50 μM and 5 μM, respectively. Use this information and the equations below to answer questions A-E. ΔG° = -0.616 ln Keq ΔG = ΔG° + 0.616 ln [Y]/[X] Recall that the natural log of a number z will have a negative value when z < 1, positive when z > 1, and 0 when z = 1. Is the standard free-energy change of this reaction positive or negative? Is the reaction XY an energetically favorable or unfavorable reaction under standard conditions?

B. The standard free-energy change, ΔG°, is positive because Keq is less than 1. Under standard conditions (equal concentrations of X and Y), the reaction XY is unfavorable.

3.67. Consider an analogy between reaction-coupling and money. In a simple economy, barter provides a means of direct exchange of material goods. For example, the owner of a cow may have excess milk and need eggs, whereas a chicken owner has excess eggs and needs milk. Provided that these two people are in close proximity and can communicate, they may exchange or barter eggs for milk. But in a more complex economy, money serves as a mediator for the exchanges of goods or services. For instance, the cow owner with excess milk may not need other goods until three months from now, or may want goods from someone who does not need milk. In this case, the "energy" from providing milk to the economy can be temporarily "stored" as money, which is a form of "energy" used for many transactions in the economy. Using barter and money as analogies, describe two mechanisms that can serve to drive an unfavorable chemical reaction in the cell.

Barter is analogous to the direct coupling of a favorable to an unfavorable reaction by a single enzyme. Money is analogous to the storage of energy from a favorable reaction in the form of high-energy bonds in an activated carrier molecule. Such activated carrier molecules can later be used to drive a huge variety of other unfavorable reactions in the cell, either by being hydrolyzed to provide the needed energy for a reaction or by transferring an activated chemical group to another molecule.

14-71 Which of the phylogenetic trees in Figure Q14-71is the most accurate? (The mitochondria and chloroplasts are from maize, but they are treated as independent "organisms" for the purposes of this question.)

C

Which of the following statements comparing prokaryotic and eukaryotic transcription is not true? A) part of the RNA polymerase specifically recognizes and binds to the promoter in bacteria; in eukaryotes, transcription factors mediate the binding of polymerase B) the RNA transcript is immediately useable as mRNA; in eukaryotes, the RNA transcript is processed C) RNA polymerase requires a primer in bacteria; in eukaryotes it does not D) eukaryotic transcription has a polyadenylation signal sequence; in prokaryotes, a transcription terminator causes the polymerase to detach and release the transcript

C) RNA polymerase requires a primer in bacteria; in eukaryotes it does not

Which of the following is incorrectly identified in this illustration of a tRNA? A) 5' and 3' ends B) hydrogen bond C) amino acid attachment site D) anticodon loop

C) amino acid attachment site

Which of the following components does not form part of the transcription initiation complex at a eukaryotic promoter? A) TATA box B) start point C) transfer RNA D) transcription factors E) RNA polymerase

C) transfer RNA

Place the following in order of size, from the smallest to the largest. A. protofilament B. microtubule C. α-tubulin D. tubulin dimer E. mitotic spindle

C, D, A, B, E

Explain how alcohol fermentation starts with glucose and yields ethanol.

CO2 from pyruvate becomes acetaldehyde, which is then reduced by NADH to ethanol, this regenerates NADH --> NAD+

Explain the important events that occur in the carbon fixation stage of the Calvin cycle

CO2 is attached to a 5 carbon sugar. Rubisco catylizes this step. The product is a very unstable 6 carbon sugar that splits into 2 G3P.

Explain the Calvin cycle.

CO2 is incorporated into organic molecules already present in the chloroplast - this is called carbon fixation. This happens many times to form a carbohydrate.

Ca2+ can trigger biological effects in cells because an unstimulated cell has an extremely __________________ concentration of free Ca2+ in the cytosol, compared with its concentration in the __________________ space and in the __________________, creating a steep electrochemical gradient. When Ca2+ enters the cytosol, it interacts with Ca2+-responsive proteins such as __________________, which also binds diacylglycerol, and __________________, which activates CaM-kinases. adenylyl cyclase endoplasmic reticulum nuclear average extracellular peroxisome Ca2+ high phospholipase C calmodulin intracellular protein kinase A colorful low protein kinase C

Ca2+ can trigger biological effects in cells because an unstimulated cell has an extremely low concentration of free Ca2+ in the cytosol, compared with its concentration in the extracellular space and in the endoplasmic reticulum, creating a steep electrochemical gradient. When Ca2+ enters the cytosol, it interacts with Ca2+-responsive proteins such as protein kinase C, which also binds diacylglycerol, and calmodulin, which activates CaM-kinases.

Cells can signal to each other in various ways. A signal that must be relayed to the entire body is most efficiently sent by __________________ cells, which produce hormones that are carried throughout the body through the bloodstream. On the other hand, __________________ methods of cell signaling do not require the release of a secreted molecule and are used for very localized signaling events. During __________________ signaling, the signal remains in the neighborhood of the secreting cell and thus acts as a local mediator on nearby cells. Finally, _________________ signaling involves the conversion of electrical impulses into a chemical signal. Cells receive signals through a __________________, which can be an integral membrane protein or can reside inside the cell. amplification G protein phosphorylation contact-dependent K+ channel receptor endocrine neuronal target epithelial paracrine

Cells can signal to each other in various ways. A signal that must be relayed to the entire body is most efficiently sent by endocrine cells, which produce hormones that are carried throughout the body through the bloodstream. On the other hand, contact-dependent methods of cell signaling do not require the release of a secreted molecule and are used for very localized signaling events. During paracrine signaling, the signal remains in the neighborhood of the secreting cell and thus acts as a local mediator on nearby cells. Finally, neuronal signaling involves the conversion of electrical impulses into a chemical signal. Cells receive signals through a receptor, which can be an integral membrane protein or can reside inside the cell.

Phosphorylation of nuclear lamins regulates their assembly and disassembly during mitosis. You add a drug to cells undergoing mitosis that inhibits the activity of an enzyme that dephosphorylates nuclear lamins. What do you predict will happen to these cells? Why?

Cells should become arrested in mitosis. Normally, the lamins are phosphorylated during mitosis, causing disassembly of the nuclear envelope. At the end of mitosis, the nuclear lamins are dephosphorylated, causing the lamins to reassemble. Inhibition of this last step should therefore prevent the nuclear lamins from reassembling after mitosis.

Cells signal to one another in various ways. Some use extracellular signal molecules that are dissolved gases, such as __________________, which can diffuse easily into cells. Others use cytokines, which bind to cytokine receptors. Cytokine receptors have no intrinsic enzyme activity but are associated with cytoplasmic tyrosine kinases called __________________s, which become activated on the binding of cytokine to its receptor and go on to phosphorylate and activate cytoplasmic transcriptional regulators called __________________s. Some intracellular signaling pathways involve chains of protein kinases that phosphorylate each other, as seen in the __________________ signaling module. Lipids can also relay signals in the cell, as we observe when phospholipase C cleaves the sugar-phosphate head off a lipid molecule to generate the two small messenger molecules __________________ (which remains embedded in the plasma membrane) and __________________ (which diffuses into the cytosol). cyclic GMP MAP kinase STAT diacylglycerol NO TGFβ IP3 Ras JAK SMAD

Cells signal to one another in various ways. Some use extracellular signal molecules that are dissolved gases, such as NO, which can diffuse easily into cells. Others use cytokines, which bind to cytokine receptors. Cytokine receptors have no intrinsic enzyme activity but are associated with cytoplasmic tyrosine kinases called JAKs, which become activated on the binding of cytokine to its receptor and go on to phosphorylate and activate cytoplasmic transcriptional regulators called STATs. Some intracellular signaling pathways involve chains of protein kinases that phosphorylate each other, as seen in the MAP kinase signaling module. Lipids can also relay signals in the cell, as we observe when phospholipase C cleaves the sugar-phosphate head off a lipid molecule to generate the two small messenger molecules diacylglycerol (which remains embedded in the plasma membrane) and IP3 (which diffuses into the cytosol).

64. Genomic sequences are now available for an increasing number of species, and scientists studying evolutionary processes can take advantage of this enormous amount of data to bring evolution into the arena of molecular studies. By aligning the sequences of homologous genes and looking for regions of similarity and where changes have occurred, it is possible to infer the sequence of the ancestral gene. a. What term is used to describe the changes in gene sequences that have occurred? How can we use what we know about this process to construct a time line showing when various sequence changes occurred and when they led to the modern sequences that we know today?

Changes in gene sequence occur through mutation. Mutations accumulate over time, occuring independently and at different sites in each gene lineage. homologous gens that diverted recently will differ only slightly; genes that diverged long ago will differ more. Knowing the average mutation rate, you can estimate the time that has elapsed since the different versions of the gene diverged. By seeing how closely the various members of the family of the homologous genes resemble one another, you can draw up a family tree, showing the sequence of lineage splits that lead from the ancestral gene to its many modern descendants. Suppose this family tree shows that family members A and B diverged from one another long ago, but that C diverged from B more recently; and suppose that at a certain site in the gene, A and B have the same sequence but C is different. Then, it is likely that the sequence of A and B is ancestral, while that of C reflects a recent mutation that has occurred in the lineage of C alone.

What purpose does the phosphorylation of glucose to glucose 6-phosphate by the enzyme hexokinase serve as the first step in glycolysis? (a) It helps drive the uptake of glucose from outside the cell. (b) It generates a high-energy phosphate bond. (c) It converts ATP to a more useful form. (d) It enables the glucose 6-phosphate to be recognized by phosphofructokinase, the next enzyme in the glycolytic pathway.

Choice (a) is correct. Choice (b) is incorrect because the phosphate transferred to the glucose is not held by a high-energy covalent bond. Choice (c) is incorrect because the reaction converts ATP to ADP, which is not useful as an energy source for most cellular reactions, even though it still has one high-energy bond. Choice (d) is incorrect because the next enzyme in the pathway is phosphoglucose isomerase, not phosphofructokinase

14-43Which ratio of NADH to NAD+in solution will generate the largest positive redox potential? (a)1:10 (b)10:1 (c)1:1 (d)5:1

Choice (a) is correct. NAD+ is the electron acceptor; NADH is the electron donor. If there is an excess of NAD+ in solution, there is less capacity to donate electrons and more capacity to accept electrons. This is reflected by a redox potential that is more positive than the alternative conditions.

Acetylcholine binds to a GPCR on heart muscle, making the heart beat more slowly. The activated receptor stimulates a G protein, which opens a K+ channel in the plasma membrane, as shown in Figure Q16-32. Which of the following would enhance this effect of the acetylcholine? Figure Q16-32 (a) addition of a high concentration of a nonhydrolyzable analog of GTP (b) addition of a drug that prevents the α subunit from exchanging GDP for GTP (c) mutations in the acetylcholine receptor that weaken the interaction between the receptor and acetylcholine (d) mutations in the acetylcholine receptor that weaken the interaction between the receptor and the G protein

Choice (a) is correct. The heart is induced to beat more slowly by the binding of acetylcholine to a GPCR, activating a G protein whose βγ complex binds to and opens K+ channels. The addition of high concentrations of a nonhydrolyzable analog of GTP will increase the length of time that the G protein βγ complex remains free of the α subunit and able to activate the K+ channel; this will therefore enhance the effect of acetylcholine [choice (a)]. All the other choices will make it more difficult for the signal to proceed from the GPCR to the K+ channel.

15-14What is the role of the nuclear localization sequence in a nuclear protein? (a)It is bound by cytoplasmic proteins that direct the nuclear protein to the nuclear pore. (b)It is a hydrophobic sequence that enables the protein to enter the nuclear membranes. (c)It aids in protein unfolding so that the protein can thread through nuclear pores. (d)It prevents the protein from diffusing out of the nucleus through nuclear pores.

Choice (a) is correct. The nuclear localization signal typically contains positively charged amino acids, not hydrophobic ones [choice (b)]. Proteins are not unfolded as they enter the nucleus [choice (c)]. Proteins are actively transported in and out of the nucleus and do not diffuse through the nuclear pores [choice (d)].

15-56 Which of the following statements about secretion is true? (a)The membrane of a secretory vesicle will fuse with the plasma membrane when it discharges its contents to the cell's exterior. (b)Vesicles for regulated exocytosis will not bud off the trans Golgi network until the appropriate signal has been received by the cell. (c)The signal sequences of proteins destined for constitutive exocytosis ensure their packaging into the correct vesicles. (d)Proteins destined for constitutive exocytosis aggregateas a result of the acidic pH of the transGolgi network.

Choice (a) is correct. Vesicles for regulated exocytosis bud from the trans Golgi network and accumulate at the plasma membrane until the appropriate signal has been received [choice (b)]. There are no signal sequences for proteins destined for exocytosis [choice(c)]. Those proteins that are to be secreted by regulated exocytosis aggregate in the trans Golgi network as a result of the acidic pH and high Ca2+ concentrations [choice (d)];those proteins that do not aggregate are packed into transport vesicles for constitutive exocytosis.

Which of the following statements about molecular switches is false? (a) Phosphatases remove the phosphate from GTP on GTP-binding proteins, turning them off. (b) Protein kinases transfer the terminal phosphate from ATP onto a protein. (c) Serine/threonine kinases are the most common types of protein kinase. (d) A GTP-binding protein exchanges its bound GDP for GTP to become activated.

Choice (a) is false. GTP-binding proteins themselves hydrolyze their bound GTP to GDP, using their own intrinsic GTPase activity.

The ethylene response in plants involves a dimeric transmembrane receptor. When the receptor is not bound to ethylene, the receptor binds to and activates a protein kinase, which activates an intracellular signaling pathway that leads to the degradation of a transcriptional regulator important for transcribing the ethylene- responsive genes (see Figure Q16-60). You discover a phosphatase that is important for ethylene signaling, and you name it PtpE. Plants lacking PtpE never turn on ethylene-responsive genes, even in the presence of ethylene. You find that PtpE dephosphorylates serine 121 on the transcriptional regulator. Furthermore, plants lacking PtpE degrade the transcriptional regulator in the presence of ethylene. Figure Q16-60 Which of the following statements is inconsistent with your data? (a) When the transcriptional regulator is phosphorylated, it activates transcription of the ethylene-responsive genes. (b) When the transcriptional regulator is not phosphorylated, it binds to DNA. (c) Activation of the protein kinase that binds to the ethylene receptor leads to inactivation of PtpE. (d) Binding of ethylene to its receptor leads to the activation of PtpE.

Choice (a) is inconsistent. Cells lacking PtpE do not transcribe the ethylene-reponsive genes, suggesting that dephosphorylation of the transcriptional regulator is required before it can activate transcription. From your data, PtpE is either constitutively active or inactivated by the active protein kinase downstream of the ethylene receptor. The latter scenario would permit maximum repression in the absence of ethylene. All the other choices are consistent with your data.

Which of the following statements about microtubules is true? (a) Motor proteins move in a directional fashion along microtubules by using the inherent structural polarity of a protofilament. (b) The centromere nucleates the microtubules of the mitotic spindle. (c) Because microtubules are subject to dynamic instability, they are used only for transient structures in a cell. (d) ATP hydrolysis by a tubulin heterodimer is important for controlling the growth of a microtubule.

Choice (a) is true. Microtubules are nucleated by the centrosome [not the centromere, choice (b)]. Although microtubules are subject to dynamic instability, their interaction with microtubule-binding proteins can stabilize them so that they can be used to form stable structures such as cilia and flagella [choice (c)]. GTP (not ATP) hydrolysis is important for controlling the growth of a microtubule [choice (d)].

15-42An individual transport vesicle________. (a)contains only one type of protein in its lumen. (b)will fuse with only one type of membrane. (c)is endocytic if it is traveling toward the plasma membrane. (d)is enclosed by a membrane with the same lipid and protein composition as the membrane of the donor organelle.

Choice (b) is correct. An individual vesicle may contain more than one type of protein in its lumen [choice (a)], all of which will contain the same sorting signal (or will lack specific sorting signals). Endocytic vesicles [choice (c)] generally move away from the plasma membrane. The vesicle membrane will not necessarily contain the same lipid and protein composition as the donor organelle, because the vesicle is formed from a selected subsection of the organelle membrane from which it budded [choice (d)].

Which of the following statements is true? (a) Because endocrine signals are broadcast throughout the body, all cells will respond to the hormonal signal. (b) The regulation of inflammatory responses at the site of an infection is an example of paracrine signaling. (c) Paracrine signaling involves the secretion of signals into the bloodstream for distribution throughout the organism. (d) The axons of neurons typically signal target cells using membrane-bound signaling molecules that act on receptors in the target cells.

Choice (b) is correct. Choice (a) is not true because only the cells with a receptor for the hormone will respond to the signal. Choice (c) is not true because paracrine signaling involves signaling in a more local fashion, unlike endocrine signaling where signals are sent through the bloodstream. Choice (d) is untrue because axons typically signal using diffusible neurotransmitters that are released at the synapse.

14-47Which of the following reactions has a sufficiently large free-energy change to enable it to be used, in principle, to provide the energy needed to synthesize one molecule of ATP from ADP and Piunder standard conditions? See Table Q14-47. Recall that ΔG°= -n(0.023) ΔE0′ ,and ΔE0′= E0′(acceptor) -E0′(donor). (a)the reduction of a molecule of pyruvate by NADH (b)the reduction of a molecule of cytochrome by NADH (c)the reduction of a molecule of cytochrome by reduced ubiquinone (d)the oxidation of a molecule of reduced ubiquinone by cytochrome c

Choice (b) is correct. For a reaction to drive ATP synthesis under standard conditions, the ΔG° of the reaction must be less than -7.3 kcal/mole. Because ΔG° = -n (0.023) ΔE0′, the value of ΔE0′ must be greater than 317 mV/n, where n is the number of electrons transferred. ΔE0′ is 130 mV for the reduction of a molecule of pyruvate by NADH, 390 mV for the reduction of a molecule of cytochrome b by NADH, 40 mV for the reduction of a molecule of cytochrome b by ubiquinone, 200 mV for the oxidation of a molecule of ubiquinone by cytochrome c, and 590 mV for the oxidation of cytochrome c by oxygen. The numbers of electrons transferred in each of the above reactions are two, one, one, one, and one, respectively. Thus, only the second and fifth of these reactions are sufficient to drive ATP synthesis.

Intermediate filaments help protect animal cells from mechanical stress because ___________. (a) filaments directly extend from the interior of the cell to the extracellular space and into the next cell, linking one cell to the next, helping to distribute locally applied forces. (b) filaments in each cell are indirectly connected to the filaments of a neighboring cell through the desmosome, creating a continuous mechanical link between cells. (c) filaments remain independent of other cytoskeletal elements and keep the mechanical stress away from other cellular components. (d) filaments make up the desmosome junctions that connect cells; these junctions are more important than the internal network of filaments for protecting cells against mechanical stress.

Choice (b) is correct. Intermediate filaments do not directly extend from cell to cell [choice (a)]. The linking of intermediate filaments to other cytoskeletal elements (like actin) is thought to help protect cells from mechanical stress [choice (c)]. Desmosome junctions are made up of many different kinds of proteins including cadherins in the extracellular space (which mediate cell-cell adhesion) as well as proteins within the cytoplasm that mediate the attachment of desomosomes to intermediate filaments. These junctions alone are not sufficient for protection against mechanical stress and need the interaction with the intermediate filament network in the cell [choice (d)].

15-8 Which of the following statements is true? (a)Lysosomes are believed to have originated from the engulfment of bacteria specialized for digestion. (b)The nuclear membrane is thought to have arisen from the plasma membrane invaginating around the DNA. (c)Because bacteria do not have mitochondria, they cannot produce ATP in a membrane-dependent fashion. (d)Chloroplasts and mitochondria share their DNA.

Choice (b) is correct. Lysosomes are part of the endomembrane system and are not thought to have come from the engulfment of an ancient prokaryotic cell [choice (a)]. Bacteria use their plasma membrane for ATP production [choice (c)]. Chloroplasts and mitochondria have their own DNA and do not share [choice (d)].

Which of the following statements is true? (a) MAP kinase is important for phosphorylating MAP kinase kinase. (b) PI 3-kinase phosphorylates a lipid in the plasma membrane. (c) Ras becomes activated when an RTK phosphorylates its bound GDP to create GTP. (d) STAT proteins phosphorylate JAK proteins, which then enter the nucleus and activate gene transcription.

Choice (b) is correct. MAP kinases are phosphorylated by MAP kinase kinases [choice (a)]. Ras exchanges its GDP for GTP when activated [choice (c)]. JAK proteins are receptor-associated kinases that phosphorylate the transcriptional regulators called STAT proteins [choice (d)].

During the mating process, yeast cells respond to pheromones secreted by other yeast cells. These pheromones bind GPCRs on the surface of the responding cell and lead to the activation of G proteins inside the cell. When a wild-type yeast cell senses the pheromone, its physiology changes in preparation for mating: the cell stops growing until it finds a mating partner. If yeast cells do not undergo the appropriate response after sensing a pheromone, they are considered sterile. Yeast cells that are defective in one or more components of the G protein have characteristic phenotypes in the absence and presence of the pheromone, which are listed in Table 16-34. Table Q16-34 Which of the following models is consistent with the data from the analysis of these mutants? Explain your answer. (a) α activates the mating response but is inhibited when bound to βγ (b) βγ activates the mating response but is inhibited when bound to α (c) the G protein is inactive; either free α or free βγ complex is capable of activating the mating response (d) the G protein is active; both free α and free βγ complex are required to inhibit the mating response

Choice (b) is correct. Single mutations in the β and γ subunits of the G protein permit growth in the absence of pheromone and display a sterile phenotype in the presence of pheromone, whereas loss of the α subunit causes growth arrest in either the presence or the absence of pheromone. Because arrested growth is a normal response that occurs when the cells sense a pheromone, it must be the βγ complex that normally activates the cellular mating response when released from the α subunit. In cells lacking α, the βγ complex causes growth arrest inappropriately because α normally inhibits the action of the complex until it is activated by the binding of pheromone to the yeast cell. This interpretation is consistent with the analysis of the double mutants in which α is deleted together with either β or γ. In these double mutants, normal growth is seen in the absence of pheromone because β and γ act together as a complex to activate target proteins; a lack of either β or γ leads to no response to pheromone, even when α is not present to inhibit the response.

The amino acid sequences below represent the sequences of transmembrane helices. The characteristics of α helices that form a channel are different from those that form a single transmembrane domain. Select the helix that forms a single transmembrane domain. (a) VGHSLSIFTLVISLGIFVFF (b) IMIVLVMLLNIGLAILFVHF (c) ILHFFHQYMMACNYFWMLCE (d) VTLHKNMFLTYILNSMIIII

Choice (b) is correct. The peptide sequences for options (a), (c), and (d) are amphipathic—that is, they contain a hydrophilic amino acid every 3-4 amino acids in the sequence. This indicates they may be part of a protein that makes a channel in the membrane, and they require hydrophobic amino acids on the surface that interacts with the lipid bilayer and a hydrophilic surface on the inside of the channel. Option (b) is the only sequence that is completely hydrophobic, indicating that it forms a helix in which all the side chains interact with the lipid bilayer.

Cell lines A and B both survive in tissue culture containing serum but do not proliferate. Factor F is known to stimulate proliferation in cell line A. Cell line A produces a receptor protein (R) that cell line B does not produce. To test the role of receptor R, you introduce this receptor protein into cell line B, using recombinant DNA techniques. You then test all of your various cell lines in the presence of serum for their response to factor F, with the results summarized in Table Q16-1. Table Q16-1 Which of the following cannot be concluded from your results above? (a) Binding of factor F to its receptor is required for proliferation of cell line A. (b) Receptor R binds to factor F to induce cell proliferation in cell line A. (c) Cell line A expresses a receptor for factor F. (d) Factor F is not required for proliferation in cell line B.

Choice (b) is false. Expressing receptor R in cell line B causes proliferation in the absence of factor F, suggesting that something else in the serum can bind to receptor R in cell line B. We do not know the effect of eliminating receptor R from cell line A, and thus we cannot say whether binding of receptor R to its ligand is required for cell line A to proliferate. The data support all the other answers. Because cell line A proliferates only in the presence of factor F, binding of factor F must be required for cell line A to proliferate [choice (a)]. Because cell line A proliferates in response to factor F, it must express a receptor for factor F since it responds to factor F [choice (c)]. Because cell line B can proliferate in the absence of factor F once R is expressed, factor F is not required for cell line B to proliferate [choice (d)].

Which of the following statements about skeletal muscle contraction is false? (a) When a muscle cell receives a signal from the nervous system, voltage-gated channels open in the T-tubule membrane. (b) The changes in voltage across the plasma membrane that occur when a muscle cell receives a signal from the nervous system cause an influx of Ca2+ into the sarcoplasmic reticulum, triggering a muscle contraction. (c) A change in the conformation of troponin leads to changes in tropomyosin such that it no longer blocks the binding of myosin heads to the actin filament. (d) During muscle contraction, the Z discs move closer together as the myosin heads walk toward the plus ends of the actin filaments.

Choice (b) is false. Muscle contraction is triggered by an efflux of Ca2+ from the sarcoplasmic reticulum into the cytosol.

15-1 Which of the following statements about the endoplasmic reticulum (ER) is false? (a)The ER is the major site for new membrane synthesis in the cell. (b)Proteins to be delivered to the ER lumen are synthesized on smooth ER. (c)Steroid hormones are synthesized on the smooth ER. (d)The ER membrane is contiguous with the outer nuclear membrane.

Choice (b) is false. Proteins to be delivered to the ER lumen are synthesized on rough ER; these areas appear "rough" because ribosomes are attached to the cytosolic surface of these ER regions.

Acetylcholine is a signaling molecule that elicits responses from heart muscle cells, salivary gland cells, and skeletal muscle cells. Which of the following statements is false? (a) Heart muscle cells decrease their rate and force of contraction when they receive acetylcholine, whereas skeletal muscle cells contract. (b) Heart muscle cells, salivary gland cells, and skeletal muscle cells all express an acetylcholine receptor that belongs to the transmitter-gated ion channel family. (c) Active acetylcholine receptors on salivary gland cells and heart muscle cells activate different intracellular signaling pathways. (d) Heart muscle cells, salivary gland cells, and skeletal muscle cells all respond to acetylcholine within minutes of receiving the signal.

Choice (b) is not true. Only skeletal muscle cells express an acetylcholine receptor that belongs to the transmitter-gated ion channel family; salivary gland cells and heart muscle cells express a different receptor. The other choices are all true.

In humans, glycogen is a more useful food-storage molecule than fat because _____________________. (a) a gram of glycogen produces more energy than a gram of fat. (b) it can be utilized to produce ATP under anaerobic conditions, whereas fat cannot. (c) it binds water and is therefore useful in keeping the body hydrated. (d) for the same amount of energy storage, glycogen occupies less space in a cell than does fat.

Choice (b) is the answer. The breakdown of glycogen to glucose does not require oxygen; the glucose can then enter glycolysis and generate ATP by a fermentation process that produces lactic acid. In contrast, fats are broken down to acetyl CoA; this must enter the citric acid cycle, which requires oxygen to keep turning. Choice (a) is incorrect, because 1 g of glycogen (wet or dry) produces less energy than 1 g of fat. Choice (c) is incorrect, because the water bound by glycogen is not useful in keeping the body hydrated and merely contributes to making the glycogen weigh a lot. Choice (d) is incorrect, because the actual mass of glycogen required to store the same amount of energy is sixfold greater than the amount of fat.

14-26Which of the following types of ion movement might be expected to require co-transport of protons from the mitochondrial intermembrane space to the matrix, inasmuch as it could not be driven by the membrane potential across the inner membrane? (Assume that each ion being moved is moving against its concentration gradient.) (a)import of Ca2+into the matrix from the intermembrane space (b)import of acetate ions into the matrix from the intermembrane space (c)exchange of Fe2+in the matrix for Fe3+in the intermembrane space (d)exchange of ATP from the matrix for ADP in the intermembrane space

Choice (b) is the correct answer. Because the inside of the membrane (the mitochondrial matrix) is more negative than the outside, in principle any traffic resulting in an increase in the positive charge in the matrix can be driven by the membrane potential. Hence, exchange of Fe2+ (or ATP) in the matrix for Fe3+ (or ADP) in the intermembrane space can be driven by the membrane potential and need not require the co-transport of protons down the pH gradient. The same applies to the import of Ca2+. But import of acetate ions into the matrix and exchange of Ca2+ in the matrix for Na+ in the intermembrane space result in an increase in the amount of negative charge in the matrix and they therefore cannot be driven by the charge difference between the two mitochondrial compartments.

The first energy-generating steps in glycolysis begin when glyceraldehyde 3-phosphate undergoes an energetically favorable reaction in which it is simultaneously oxidized and phosphorylated by the enzyme glyceraldehyde 3-phosphate dehydrogenase to form 1,3-bisphosphoglycerate, with the accompanying conversion of NAD+ to NADH. In a second energetically favorable reaction catalyzed by a second enzyme, the 1,3-bisphosphoglycerate is then converted to 3-phosphoglycerate, with the accompanying conversion of ADP to ATP. Which of the following statements is true about this reaction? (a) The reaction glyceraldehyde 3-phosphate 1,3-bisphosphoglycerate should be inhibited when levels of NADH fall. (b) The ΔG° for the oxidation of the aldehyde group on glyceraldehyde 3-phosphate to form a carboxylic acid is more negative than the ΔG° for ATP hydrolysis. (c) The energy stored in the phosphate bond of glyceraldehyde 3-phosphate contributes to driving the reaction forward. (d) The cysteine side chain on the enzyme is oxidized by NAD+.

Choice (b) is the correct answer. This is another way of stating that the energetically favorable oxidation of glyceraldehyde 3-phosphate provides sufficient energy to ultimately drive the energy-requiring step of ATP synthesis from ADP. Choice (a) is untrue: NADH is an end product of the reaction glyceraldehyde 3-phosphate to 1,3-bisphosphoglycerate and therefore high (not low) levels of it would inhibit the reaction. Choice (c) is untrue: the reactions do not involve the 3-phosphate group on glyceraldehyde 3-phosphate at all. Choice (d) is untrue, because the cysteine on the enzyme is important in making a covalent intermediate with the substrate and is not oxidized by NAD+.

You are studying nuclear lamins in yeast. Using recombinant DNA technology,you alter the coding sequence of a nuclear lamin gene such that the gene now codes for a nuclear lamin protein that can no longer be phosphorylated when the nuclear envelope is broken down during mitosis. What do you predict would happen if the yeast cell only had the altered nuclear lamin gene (and not the unaltered version)? (a) Mitosis should proceed as usual because the dephosphorylation of the lamin is what is important for nuclear lamina assembly during mitosis, so phosphorylation will not be necessary. (b) Disassembly of the nuclear lamins will occur prematurely because the lamins cannot be phosphorylated. (c) Nuclear lamins will no longer disassemble properly during mitosis. (d) Nuclear lamins will be unable to produce dimers, as coiled-coil formation will be disrupted.

Choice (c) is correct. Although it is true that dephosphorylation of the lamin is necessary for the reassembly of the nuclear lamins at the end of mitosis, the cycle of phosphorylation and dephosphorylation is important for mitosis [choice (a)]. Disassembly of the lamins occurs when they are phosphorylated, as this weakens the bonds between the lamin tetramers. Lamins should be more stable if they cannot be phosphorylated [choice (b)]. Dimer formation depends on the α-helical rods; the binding of the lamin tetramers is what is affected by phosphorylation [choice (d)].

14-34NADH and FADH2carry high-energy electrons that are used to power the production of ATP in the mitochondria. These cofactors are generated during glycolysis, the citric acid cycle, and the fatty acid oxidation cycle. Which molecule below can produce the most ATP? Explain your answer. (a)NADH from glycolysis (b)FADH2from the fatty acid cycle (c)NADH from the citric acid cycle (d)FADH2from the citric acid cycle

Choice (c) is correct. NADH produced in glycolysis does not contribute directly to ATP production in the mitochondria because it cannot be imported into the matrix. If the energy is transferred to a different carrier, some of the stored energy is lost. FADH2, from either the fatty acid cycle or the citric acid cycle, contributes less energy than NADH from the citric acid cycle because the electrons are donated further down the chain. Fewer electron transfers means that fewer protons are pumped across the membrane.

3.69. The anhydride formed between a carboxylic acid and a phosphate (Figure Q3-69A) is a high-energy intermediate for some reactions in which ATP is the energy source. Arsenate can also be incorporated into a similar high-energy intermediate in place of the phosphate (Figure Q3-69B). Figure Q3-69C shows the reaction profiles for the hydrolysis of these two high-energy intermediates. What is the effect of substituting arsenate for phosphate in this reaction? Figure Q3-69 (a) It forms a high-energy intermediate of lower energy. (b) It forms a high-energy intermediate of the same energy. (c) It decreases the stability of the high-energy intermediate. (d) It increases the stability of the high-energy intermediate.

Choice (c) is correct. The activation energy of the arsenate compound is extremely low, as can be seen from the reaction profile, meaning that its high-energy intermediate is very unstable and will be spontaneously hydrolyzed more rapidly than the phosphate compound. In fact, this hydrolysis occurs rapidly without enzyme catalysis, even in cellular conditions. Arsenate is therefore quite deleterious for living organisms.

15-19 Which of the following statements is true? (a)The signal sequences on mitochondrial proteins are usually at theC-terminus. (b)Most mitochondrial proteins are not imported from the cytosol but are synthesized inside the mitochondria. (c)Chaperone proteins in the mitochondria facilitate the movement of proteins across the outer and inner mitochondrial membranes. (d)Mitochondrial proteins cross the membrane in their native, folded state.

Choice (c) is correct. The signal sequences on a protein destined for the mitochondria are on its N-terminus [choice (a)]. Although some mitochondrial proteins are synthesized inside the mitochondria from the mitochondrial genome, most mitochondrial proteins are encoded by genes in the nucleus and imported into the mitochondria after synthesis in the cytosol [choice (b)]. Mitochondrial proteins are unfolded as they enter the mitochondria through protein translocators[choice (d)].

You are interested in how cyclic-AMP-dependent protein kinase A (PKA) functions to affect learning and memory, and you decide to study its function in the brain. It is known that, in the cells you are studying, PKA works via a signal transduction pathway like the one depicted in Figure Q16-35. Furthermore, it is also known that activated PKA phosphorylates the transcriptional regulator called Nerd that then activates transcription of the gene Brainy. Which situation described below will lead to an increase in Brainy transcription? Figure Q16-35 (a) a mutation in the Nerd gene that produces a protein that cannot be phosphorylated by PKA (b) a mutation in the nuclear import sequence of PKA from PPKKKRKV to PPAAAAAV (c) a mutation in the gene that encodes cAMP phosphodiesterase that makes the enzyme inactive (d) a mutation in the gene that encodes adenylyl cyclase that renders the enzyme unable to interact with the α subunit of the G protein

Choice (c) is correct. cAMP phosphodiesterase is important for converting cAMP into AMP and thus down-regulating the activity of PKA. Without cAMP phosphodiesterase, transcription of Brainy will be increased. All the other choices will lead to inactivation of the signaling pathway and a decrease in Brainy transcription. A mutant form of Nerd that cannot be phosphorylated will not be active [choice (a)]. If PKA cannot be imported into the nucleus, it will be unable to phosphorylate Nerd [choice (b)]. Adenylyl cyclase interaction with the α subunit of the G protein is important for the G protein's activation [choice (d)].

15-15Which of the following statements about nuclear transport is true? (a)mRNAs and proteins transit the nucleus through different types of nuclear pores. (b)Nuclear import receptors bind to proteins in the cytosol and bring the proteins to the nuclear pores, where the proteins are released from the receptors into the pores for transit into the nucleus. (c)Nuclear pores have water-filled passages that small, water-soluble molecules can pass through in a nonselective fashion. (d)Nuclear pores are made up of many copies of a single protein.

Choice (c) is correct. mRNAs and proteins can move through the same nuclear pore [choice (a)]. Nuclear import receptors bind to proteins in the cytosol and transit with them across the nuclear pore into the nucleus [choice (b)]. Nuclear pores are made up of many copies of multiple proteins [choice (d)]

Figure Q16-61 shows that intracellular signaling pathways can be highly interconnected. Figure Q16-61 From the information in Figure Q16-61, which of the following statements is incorrect? (a) The GPCR and the RTK both activate phospholipase C. (b) Activation of either the GPCR or the RTK will lead to activation of transcriptional regulators. (c) CaM-kinase is only activated when the GPCR is active and not when the RTK is active. (d) Ras is activated only when the RTK is active and not when the GPCR is active.

Choice (c) is incorrect. CaM-kinase is activated by calmodulin, which is ultimately activated by phospholipase C. Either the GPCR or the RTK activates phospholipase C. All the other statements are correct.

The lab you work in has discovered a previously unidentified extracellular signal molecule called QGF, a 75,000-dalton protein. You add purified QGF to different types of cells to determine its effect on these cells. When you add QGF to heart muscle cells, you observe an increase in cell contraction. When you add it to fibroblasts, they undergo cell division. When you add it to nerve cells, they die. When you add it to glial cells, you do not see any effect on cell division or survival. Given these observations, which of the following statements is most likely to be true? (a) Because it acts on so many diverse cell types, QGF probably diffuses across the plasma membrane into the cytoplasm of these cells. (b) Glial cells do not have a receptor for QGF. (c) QGF activates different intracellular signaling pathways in heart muscle cells, fibroblasts, and nerve cells to produce the different responses observed. (d) Heart muscle cells, fibroblasts, and nerve cells must all have the same receptor for QGF.

Choice (c) is most likely to be true. Because heart muscle cells, fibroblasts, and nerve cells all respond to QGF with different outcomes, it is likely that QGF activates different effector proteins in these different cell types, leading to the diversity of outcomes observed in the experiment. QGF is unlikely to diffuse across the cell membrane, given that it is a large protein [choice (a)]. Although glial cells do not die or divide in response to QGF, they could have a receptor for QGF, as receptor activation could lead to some other response [choice (b)]. A signal molecule can often bind to different types of receptor on different cell types, so choice (d) may or may not be correct.

14-41Which of the following statements is true? (a)Only compounds with negative redox potentials can donate electrons to other compounds under standard conditions. (b)Compounds that donate one electron have higher redox potentials than those compounds that donate two electrons. (c)The ΔE0′of a redox pair does not depend on the concentration of each member of the pair. (d)The free-energy change, ΔG, for an electron-transfer reaction does not depend on the concentration of each member of a redox pair.

Choice (c) is the correct answer. By definition, E0′ refers to the standard state of equal concentrations of each member of the redox pair. Therefore, ΔE0′ does not vary with the actual concentrations. Compounds with positive redox potentials can donate electrons to other compounds under standard conditions, so long as the electron acceptor has a higher (more positive) redox potential; thus, option (a) is incorrect. Compounds that are able to donate only one electron do not necessarily have higher redox potentials than compounds that are able to donate two electrons; thus. option (b) is incorrect. (Water, for example, has a very high redox potential.) Although the ΔE0′ of a reaction is directly proportional to the ΔG° of a reaction and both are independent of the concentrations of substrates and products, the ΔG depends on these concentrations; thus, option (d) is incorrect.

All members of the steroid hormone receptor family __________________. (a) are cell-surface receptors. (b) do not undergo conformational changes. (c) are found only in the cytoplasm. (d) interact with signal molecules that diffuse through the plasma membrane.

Choice (d) is correct. All members of the steroid hormone receptor family are intracellular proteins [thus choice (a) is not correct] that interact with signal molecules that can diffuse through the plasma membrane. Once activated, steroid hormone receptors regulate gene transcription in the nucleus [choice (c)]. The binding of the signal molecule induces a large conformational change in the receptor protein [choice (b)]. This conformational change activates the steroid hormone receptors, allowing them to promote or inhibit the transcription of the appropriate genes.

Consider the in vitro motility assay using purified kinesin and purified polymerized microtubules shown in Figure Q17-63. The three panels are images taken at 1-second intervals. In this figure, three microtubules have been numbered to make it easy to identify them. Which of the following statements about this assay is false? Figure Q17-63 (a) Kinesin molecules are attached by their tails to a glass slide. (b) The microtubules used in this assay must be polymerized using conditions that stabilize tubule formation or else they would undergo dynamic instability. (c) ATP must be added for this assay to work. (d) Addition of the nonhydrolyzable ATP analog (AMP-PNP) would cause the microtubules to move faster.

Choice (d) is false. Addition of AMP-PNP would block movement, because ATP hydrolysis is required for the kinesin to step along a microtubule. Addition of AMP-PNP would cause the microtubules to attach to the kinesin heads without being released. Kinesin molecules are attached to the slide by their tails (the cargo-binding domain) so that the heads are available to move the microtubules along the slides. If they were in solution with the microtubules, there would be no force and thus no movement [choice (a)]. The microtubules used in this assay are stabilized with a nonhydrolyzable form of GTP, because otherwise they might shrink during the course of the assay [choice (b)]. ATP is required for kinesin movement, and thus must be added for this assay to work [choice (c)].

Akt promotes the survival of many cells by affecting the activity of Bad and Bcl2, as diagrammed in Figure Q16-58. Figure Q16-58 Which of the following statements is false? (a) In the presence of a survival signal, Akt is phosphorylated. (b) In the absence of a survival signal, Bad inhibits the cell-death inhibitor protein Bcl2. (c) In the presence of a survival signal, the cell-death inhibitory protein Bcl2 is active. (d) In the absence of a survival signal, Bad is phosphorylated.

Choice (d) is false. Bad is phosphorylated in the presence of a survival signal. When the survival signal is not present, Bad binds to the cell-death inhibitor protein Bcl2, promoting cell death. All the other statements are correct.

On a diet consisting of nothing but protein, which of the following is the most likely outcome? (a) loss of weight because amino acids cannot be used for the synthesis of fat (b) muscle gain because the amino acids will go directly into building muscle (c) tiredness because amino acids cannot be used to generate energy Page 4 of 27 (d) excretion of more nitrogenous (ammonia-derived) wastes than with a more balanced diet

Choice (d) is the correct answer. Because ammonia is given off when amino acids are metabolized to yield energy but not when sugars and fats are metabolized, you would expect more nitrogenous waste to be excreted. Choice (c) is incorrect because amino acids can be converted into pyruvate and acetyl CoA and used to generate energy. If more amino acids are consumed than are used, the body will not store them as protein in muscle tissue but will instead store them as fat, so choices (a) and (b) are incorrect.

14-51 Which of the following statements is true?(a)Ubiquinone is a small, hydrophobic protein containing a metal group that acts as an electron carrier. (b)A 2Fe2S iron-sulfur center carries one electron, whereas a 4Fe4S center carries two. (c)Iron-sulfur centers generally have a higher redox potential than do cytochromes. (d)Mitochondrial electron carriers with the highest redox potential generally contain copper ions and/or heme groups.

Choice (d) is the correct answer. Cytochrome c oxidase, which is the last carrier in the mitochondrial electron-transport chain and therefore has the highest redox potential, contains copper ions and a heme group. Ubiquinone is not a protein and does not contain a metal group [choice (a)]. Both 2Fe2S and 4Fe4S centers carry one electron [choice (b)]. Iron-sulfur centers generally have a lower redox potential than do cytochromes [choice (c)]. The heme group in cytochrome c contains a charged iron ion. The interiors of proteins are often hydrophobic, favoring a relatively high redox potential, because reduction of the iron ion decreases its charge, and charges are energetically unfavorable in a hydrophobic environment.

Which of the following statements about the cytoskeleton is true? (a) All eukaryotic cells have actin, microtubules, and intermediate filaments in their cytoplasm. (b) The cytoskeleton provides a rigid and unchangeable structure important for the shape of the cell. (c) The three cytoskeletal filaments perform distinct tasks in the cell and act completely independently of one another. (d) Actin filaments and microtubules have an inherent polarity, with a plus end that grows more quickly than the minus end.

Choice (d) is true. Not all eukaryotic cells have cytoplasmic intermediate filaments [choice (a)]. The cytoskeleton is not rigid and unchangeable; in fact, it can be quite dynamic [choice (b)]. Each of the three cytoskeletal systems is not completely independent. For example, proteins such as plectin are known to link intermediate filaments to the actin and microtubule cytoskeleton [choice (c)].

Microtubules are important for transporting cargo in nerve cell axons, as diagrammed in Figure Q17-33. Notice that the two types of cargo are traveling in opposite directions. Which of the following statements is likely to be false? (a) The gray cargo is attached to dynein. (b) The black cargo and the gray cargo require ATP hydrolysis for their motion. (c) The black cargo moving toward the axon terminal contains a domain that specifically interacts with the tail domain of a particular kind of motor. (d) The black cargo and the gray cargo are moving along microtubules of opposite polarity.

Choice (d) is unlikely to be the case. Microtubules in nerve cell axons are generally organized such that their plus ends are facing the axon terminal while the minus ends reside in the cell body. Thus, the gray cargo is likely to be attached to a dynein motor because it is moving toward the cell body [choice (a)]. Because cargo attaches to the tail domains of both dynein and kinesin motors, the attachment of a cargo to either tail is unlikely to affect directionality [choice (c)]. Both dynein and kinesin require ATP hydrolysis for their movement [choice (b)].

You wish to produce a human enzyme, protein A, by introducing its gene into bacteria. The genetically engineered bacteria make large amounts of protein A, but it is in the form of an insoluble aggregate with no enzymatic activity. Which of the following procedures might help you to obtain soluble, enzymatically active protein? Explain your reasoning. A. Make the bacteria synthesize protein A in smaller amounts. B. Dissolve the protein aggregate in urea, then dilute the solution and gradually remove the urea. C. Treat the insoluble aggregate with a protease. D. Make the bacteria overproduce chaperone proteins in addition to protein A. E. Heat the protein aggregate to denature all proteins, then cool the mixture.

Choices A, B, and D are all worth trying. Some proteins require molecular chaperones if they are to fold properly within the environment of the cell. In the absence of chaperones, a partly folded polypeptide chain has exposed amino acids that can form noncovalent bonds with other regions of the protein itself and with other proteins, thus causing nonspecific aggregation of proteins. A.) Because the protein you are expressing in bacteria is being made in large quantities, it is possible that there are not enough chaperone molecules in the bacterium to fold the protein. Expressing the protein at lower levels might increase the amount of properly folded protein. B.) Urea should solubilize the protein and completely unfold it. Removing the urea slowly and gradually often allows the protein to refold. Presumably, under less crowded conditions, the protein should be able to refold into its proper conformation. C.) Treating the aggregate with a protease, which cleaves peptide bonds, will probably solubilize the protein by trimming it into pieces that do not interact as strongly with one another; however, chopping up the protein will also destroy its enzymatic activity. D.) Overexpressing chaperone proteins might increase the amount of properly folded protein. E.) Heating can lead to the partial denaturation and aggregation of proteins to form a solid gelatinous mass, as when cooking an egg white, and rarely helps solubilize proteins.

Bacteria undergo chemotaxis toward amino acids, which usually indicates the presence of a food source. Chemotaxis receptors bind a particular amino acid and cause changes in the bacterial cell that induce the cell to move toward the source of the amino acid. Four types of chemotaxis receptor that mediate responses to different amino acids have been identified in a bacterium. The receptors are called ChrA, ChrB, ChrC, and ChrD. Each receptor specifically senses serine, aspartate, glutamate, or glycine, although you do not know which receptor senses which amino acid. You have been given a wild-type bacterial strain that contains all four receptors, as well as various mutant bacterial strains that are lacking one or more of the receptors. To figure out which receptor senses which amino acid, you conduct experiments in which you fill a capillary tube with an amino acid to attract the bacteria, dip the capillary tube into a solution containing bacteria, remove the capillary tube after 5 minutes, and count the number of bacteria in the capillary tube. Your results are shown in Table Q16-65. Table Q16-65 From these results, indicate which receptor is used for which amino acid.

ChrA senses glycine, ChrB senses aspartate, ChrC senses glutamate, and ChrD senses serine. To figure this out, you must match the pattern of intact receptors with the pattern of responses to the various amino acids. For example, ChrD is missing in strain 2, which does not sense serine. Therefore, ChrD is the receptor used to sense serine. Since we know that one of the receptors senses glutamate, but all strains respond to glutamate, ChrC must be the sensor for glutamate because it is present in all strains.

While many prokaryotic cells have a single membrane bilayer, all eukaryotic cells have a complex system of internal membrane-bound compartments. How might it be advantageous for the cell to have these additional compartments?

Compartmentalization using intracellular membranes allows eukaryotic cells to separate a variety of cell processes. Although this requires a higher degree of coordination, the cell also gains a more stringent degree of control over these processes (examples include: the separation of transcription and translation; the separation of enzymes involved in protein modifications for secreted versus cytosolic substrates; the separation of proteolytic events in the lysosomes versus the cytosol; the separation of anaerobic metabolism in the cytosol and aerobic metabolism in the mitochondria).

3.10. Assume that the average human adult requires 2000 kilocalories per day to sustain all normal processes and maintain a constant weight. If manufactured solar panels could somehow provide power directly to the human body, what size solar panel would be required (in cm2)? Assume there are 10 hours of sunlight per day, and that the usable energy output for a typical solar panel is 850 kJ/ft2 per hour. Note: 1 kcal = 4.184 kJ 1 ft2 = 929.03 cm2

Conversion factors: 1 kcal = 4.184 kJ 1 ft2 = 929.03 cm2 If there are 10 hours of sunlight each day hitting the solar panel, there are 8500 kJ/ft2 produced per day. The average adult human requires 8368 kJ per day; thus, with a conventional solar panel, we would require a surface area of about a square foot, or more precisely in cm2: 8368 kJ/X = 8500 kJ/929.03 cm2 929.03 μltiply 8368/8500 = X X = 914.57 cm2

In the transcription process shown here, in which process (1,2, or 3) does the RNA polymerase bind to the promoter, in which is the RNA transcript released, and in which is the RNA transcript extended? A) 1,2,3 B)2,3,1 C) 3,2,1 D) 1,3,2

D) 1,3,2

The extracellular matrix is thought to participate in the regulation of animal cell behavior by communicating information from the outside to the inside of the cell via which of the following?

D) integrins

One of the major categories of receptors in the plasma membrane reacts by forming dimers, adding phosphate groups, and then activating relay proteins. Which type does this?

D) receptor tyrosine kinases

3.43.d. Consider the reaction XY in a cell at 37°C. At equilibrium, the concentrations of X and Y are 50 μM and 5 μM, respectively. Use this information and the equations below to answer questions A-E. ΔG° = -0.616 ln Keq ΔG = ΔG° + 0.616 ln [Y]/[X] Recall that the natural log of a number z will have a negative value when z < 1, positive when z > 1, and 0 when z = 1. Imagine circumstances in which the concentration of X is 1000 μM and that of Y is 1 μM. Is conversion of X to Y favorable? Will it happen quickly?

D. Yes, the conversion is favorable because the value of [Y]/[X] is less than the equilibrium value. However, the speed of the reaction cannot be determined from the free-energy difference. For example, combustion of this piece of paper is a highly favorable reaction, yet it will not happen in our lifetime without a catalyst.

2.72.a.Eukaryotic cells have their DNA molecules inside their nuclei. However, to package all the DNA into such a small volume requires the cell to use specialized proteins called histones. Histones have amino acid sequences enriched for lysines and arginines. A. What problem might a cell face in trying to package DNA into a small volume without histones, and how do these special packaging proteins alleviate the problem?

DNA is a nucleic acid polymer in which each monomer has a negatively charged phosphate group. The negative charges will naturally repel each other, so in order to wrap the high density of negative charges into a small space, positively charged molecules must be present. Histones accomplish this because they are rich in lysines and arginines, which are positively charged in solution at pH 7.

calvin cycle

Dark reaction or light independent reaction. Carbon dioxide is fixed (incorporated) into organic materials (sugar).

proteins

Denotes function.

Do you agree or disagree with the following statement? Explain your answer. When skeletal muscle receives a signal from the nervous system to contract, the signal from the motor neuron triggers the opening of a voltage-sensitive Ca2+ channel in the muscle cells' plasma membrane, allowing Ca2+ to flow into the cell.

Disagree. The increase in intracellular Ca2+ during muscle contraction comes from an intracellular source. The Ca2+ is released from the lumen of the sarcoplasmic reticulum, which is a specialized region of endoplasmic reticulum inside a muscle cell. The signal from the nerve terminal triggers an action potential in the muscle cell plasma membrane, which causes a voltage-sensitive transmembrane protein in the membranous transverse tubules to open a Ca2+-release channel in the membrane of the sarcoplasmic reticulum.

Do you agree or disagree with this statement? Explain your answer. Minus-end directed microtubule motors (like dyneins) deliver their cargo to the periphery of the cell, whereas plus-end directed microtubule motors (like kinesins) deliver their cargo to the interior of the cell.

Disagree. The plus ends of microtubules usually point toward the cell periphery, whereas the minus ends point toward the cell center. This is because the γ-tubulin in the centrosome serves to nucleate microtubule growth. Because the centrosomes are near the center of the cell, the minus ends of microtubules are located there. Therefore, a minus- end directed microtubule motor would direct its cargo toward the center of the cell, and a plus-end directed microtubule motor would direct its cargo toward the cell periphery.

The biosynthetic pathway for the two amino acids E and H is shown schematically in the figure below. You are able to show that E inhibits enzyme V, and H inhibits enzyme X. Enzyme T is most likely to be subject to feedback inhibition by __________________ alone.

E

Which of the following is not a normal feature of prokaryotic mRNA? A) 5' end B) start codon C) stop codon D) 3' end E) poly-A tail

E) poly-A tail

What is the function of an aminoacyl-tRNA synthetase?

Each of these 20 enzymes catalyzes the attachment of a specific amino acid to a specific tRNA molecule.

Describe how Englemann was able to form an action spectrum long before the invention of a spectrophotometer.

Englemann illuminated a filament of algae through different wavelengths of light. Then aerobic bacteria were used to see where they would clump the most. Where they clumped the most, showed the most O2 released.

15-60 For each of the following sentences, fill in the blanks with the best word or phrase selected from the list below. Not all words or phrases will be used; each word or phrase should be used only once. Eukaryotic cells are continually taking up materials from the extracellular space by the process of endocytosis. One type of endocytosis is __________________, which uses __________________ proteins to form small vesicles containing fluids and molecules. After these vesicles have pinched off from the plasma membrane, they will fuse with the __________________, where materials that are taken into the vesicle are sorted. A second type of endocytosis is __________________, which is used to take up large vesicles that can contain microorganisms and cellular debris. Macrophages are especially suited for this process, as they extend __________________ (sheetlike projections of their plasma membrane) to surround the invading microorganisms. chaperone Golgi apparatus pseudopods cholesterol mycobacterium rough ER clathrin phagocytosis SNARE endosome pinocytosis transcytosis

Eukaryotic cells are continually taking up materials from the extracellular space by the process of endocytosis. One type of endocytosis is pinocytosis, which uses clathrin proteins to form small vesicles containing fluids and molecules. After these vesicles have pinched off from the plasma membrane, they will fuse with the endosome, where materials that are taken into the vesicle are sorted. A second type of endocytosis is phagocytosis, which is used to take up large vesicles that can contain microorganisms and cellular debris. Macrophages are especially suited for this process, as they extend pseudopods (sheetlike projections of their plasma membrane) to surround the invading microorganisms.

Which of the following is false with regard to the extracellular matrix? A) These structures are external to the cell membrane, but are still an integral component of cell function. B) Extracellular matrices are only found in association with eukaryotic cells. C) Extracellular matrices play a role in determining the shape of a cell. D) Bone and cartilage are examples of cells that have specialized extracellular matrices associated with their function. E) Extracellular matrices play a role in determining the mechanical properties of tissues and organs.

Extracellular matrices are only found in association with eukaryotic cells.

3.56.a. true or false:Enzymes alter the equilibrium point of a reaction.

False. An enzyme catalyzes its reaction in both directions, lowering the energy of activation for both the forward and reverse reactions. Enzymes do not affect the free energy of the reactants and products are the same, and thus they do not affect the reaction equilibrium.

3.32.d. true or false: enzymes increase the average energy of reactant molecules

False. By lowering the energy of activation, enzymes increase the number of molecules in a population that can overcome the activation barrier.

3.49.a. true or false:When two macromolecules form a complex, the free energy of the system increases because there is a net increase in the amount of order in the cell.

False. Even nonspecific interactions between macromolecules can be favorable if there is a large number of water molecules and ions displaced at the interaction interface. This would lead to an overall increase in disorder, even though the two larger molecules become associated and more ordered.

3.56.b. true or false: Vmax can be determined by measuring the amount of product accumulated late in the reaction.

False. Initial reaction velocities are measured to determine Vmax.

3.19.a. true or false:Photosynthetic organisms release only O2 into the atmosphere, while nonphotosynthetic organisms release only CO2.

False. Plants, as well as photosynthetic algae and bacteria, perform both photosynthesis and respiration. This means that photosynthetic organisms release both O2 and CO2 into the atmosphere.

3.39.d.On the basis of the two reactions below, decide which of the following statements are true and which are false. If a statement is false, explain why it is false. 1: ATP + Y Y-P + ADP ΔG = -100 kcal/mole 2: Y-P + A B ΔG = 50 kcal/mole Reaction 2 can be used to drive reaction 1 in the reverse direction.

False. This is false for two reasons: (1) reaction 2 is unfavorable, as indicated by the positive free-energy change associated with the reaction; (2) the reverse reaction, although possibly more favorable, will yield the product for reaction 1, not reactants to help drive it forward.

3.30.a.The branched diagram in Figure Q3-30 represents several possible reaction pathways that substance X may follow. Each branch point represents two possible reactions, which would give rise to different products. Next to each of these points are values for the uncatalyzed activation energies and catalyzed activation energies (in kcal/mole), respectively. Order the final products in the relative amounts you expect them to be produced in the absence of any enzymes (from greatest to least).

Following this principle, the relative amounts of products in the absence of the necessary enzymes (from greatest to least) are: C, D, A, B, H, G, E, F.

For an uncharged molecule, the direction of passive transport across a membrane is determined solely by its __________________ gradient. On the other hand, for a charged molecule, the __________________ must also be considered. The net driving force for a charged molecule across a membrane therefore has two components and is referred to as the __________________ gradient. Active transport allows the movement of solutes against this gradient. The transporter proteins called __________________ transporters use the movement of one solute down its gradient to provide the energy to drive the uphill transport of a second solute. When this transporter moves both ions in the same direction across the membrane, it is considered a(n) __________________; if the ions move in opposite directions, the transporter is considered a(n) __________________. antiport coupled membrane potential ATP hydrolysis electrochemical symport concentration light-driven uniport

For an uncharged molecule, the direction of passive transport across a membrane is determined solely by its concentration gradient. On the other hand, for a charged molecule, the membrane potential must also be considered. The net driving force for a charged molecule across a membrane therefore has two components and is referred to as the electrochemical gradient. Active transport allows the movement of solutes against this gradient. The transporter proteins called coupled transporters use the movement of one solute down its gradient to provide the energy to drive the uphill transport of a second solute. When this transporter moves both ions in the same direction across the membrane, it is considered a symport; if the ions move in opposite directions, the transporter is considered an antiport.

66. You have been following the recent presidential elections and have heard some candidates disparaging excessive and "unnecessary" federal expenditures.One particular candidate asks: "Why are we spending millions of dollars studying fruit flies? How can that possibly help us find a cure for cancer?" Use your knowledge of model organisms to explain why studies in D. melanogaster (the fruit fly) areactually an excellent use of research funding.

Fundingresearch on D. melanogaster is a worthwhile investment for several reasons: (1) working with insect animal models is relatively inexpensive; (2) fruit flies have historically proven useful in helping understand eukaryotic chromosome behavior; and (3) many of the genes in Drosophila are highly similar in sequence to the homologous human genes, and thus can be used to study human diseases.

G-protein-coupled receptors (GPCRs) all have a similar structure with __________________ transmembrane domains. When a GPCR binds an extracellular signal, an intracellular G protein, composed of __________________ subunits, becomes activated. __________________ of the G-protein subunits are tethered to the plasma membrane by short lipid tails. When unstimulated, the α subunit is bound to __________________, which is exchanged for __________________ on stimulation. The intrinsic __________________ activity of the α subunit is important for inactivating the G protein. __________________ inhibits this activity of the α subunit, thereby keeping the subunit in an active state. adenylyl cyclase cholera toxin GTPase AMP diacylglycerol phosphodiesterase ATP five seven ATPase four three Ca2+ GDP twelve cAMP GTP two

G-protein-coupled receptors (GPCRs) all have a similar structure with seven transmembrane domains. When a GPCR binds an extracellular signal, an intracellular G protein, composed of three subunits, becomes activated. Two of the G-protein subunits are tethered to the plasma membrane by short lipid tails. When unstimulated, the α subunit is bound to GDP, which is exchanged for GTP on stimulation. The intrinsic GTPase activity of the α subunit is important for inactivating the G protein. Cholera toxin inhibits this activity of the α subunit, thereby keeping the subunit in an active state.

Ions can travel directly from the cytoplasm of one animal cell to the cytoplasm of an adjacent cell through ________.

Gap Junctions

_____ are the polysaccharide portion of some protein glycans, which are characterized by repeating disaccharide units.

Glycoaminoglycans (GAGs)

Which observation suggested the involvement of a second messenger in epinephrine's effect on liver cells?

Glycogen breakdown was observed only when epinephrine was administered to intact cells.

Name the 3 stages of cellular respiration.

Glycolysis, Krebs Cycle, Electron Transport Chain.

What consists of a network of flattened membranes and vesicles?

Golgi apparatus

A mutation that disrupts the ability of an animal cell to add polysaccharide modifications to proteins would most likely cause defects in its ________.

Golgi apparatus and extracellular matrix

3.57.a. you are measuring the effect of temperature on the rate of an enzyme-catalyzed reaction. If you plot reaction rate against temperature, which of the graphs in Figure Q3-57 would you expect your plot to resemble?

Graph 1 is correct.

Describe the process by which gut epithelial cells use transporters to take up ingested glucose (against the concentration gradient) and to distribute glucose to other tissues by moving it back out of the cell (down the concentration gradient).

Gut epithelial cells use two different transporters to take glucose up from the gut and distribute it into the bloodstream and to other tissues. These transporters are located at opposite sides of the cell: the apical side of the cell (which faces the gut) contains a Na+-glucose symporter. This symporter couples the entry of Na+ down its electrochemical gradient to the active import of glucose against its concentration gradient. The Na+-glucose symporter is restricted to the apical side of the cell by tight-junction complexes in the plasma membrane, which link neighboring epithelial cells together. On the basolateral side of the cell, there is another transporter that facilitates movement of glucose down its concentration gradient, out of the cell. This transporter is a uniporter that only transports glucose in one direction: from the cytosol to the extracellular matrix. The location of this uniporter is also restricted by the presence of the tight junctions, so that the epithelial cell will not transport glucose back into the lumen of the gut.

Based on the periodic table shown here, which elements will most likely form a covalent bond?

H and H

List three places in the light reactions where proton-motive force is generated.

H+ from H2O, H+ pumped across the membrane by cytochrome complex, removal of H+ from stroma when NADP+ goes to NADPH.

ATP hydrolysis provides the energy to transport ions against their concentration gradients.

How does the sodium-potassium pump work to transport sodium and potassium against their concentration gradients?

Capping

IS when mature mRNA of eukaryotes have a modified form of guanine covalently attached at the 5' end. this event occurs while a pre-mRNA is being made by RNA polymerase.After mRNA is in the cytosol the cap structure helps to prevent mRNA degration and is recognized by other cap-binding proteins that enable the mRNA to bind to a ribosome for translation.

How would removing a terminator from a gene affect transcription?where would transcription end?

If a terminator was removed, transcription would occur beyond the normal stopping point.eventually,RNA polymerase would encounter a terminator from an adjacent gene,and transcription would end.

What is the relationship between glycolysis and oxygen?

If oxygen is present, we can get a bigger pay off from the pyruvate and take use of the NADH in making a lot of ATP by undergoing the citric acid cycle, and then oxidative phosphorilation via chemiosmosis and the ETC.

Write a prediction for the cuts you made concerning the regeneration of the planarians if the diffusion gradient is NOT correct (if/then).

If the diffusion gradient model is not correct, then the planaria will regenerate random body parts or die. The result will be an abnormal planarian that will not have all its body parts

Write a prediction (if/then) concerning the relationship between light intensity and oxygen production

If the light intensity(in) increases, then the production of oxygen(de) will increase.

One way in which an enzyme can lower the activation energy required for a reaction is to bind the substrate(s) and distort its structure so that the substrate more closely resembles the transition state of the reaction. This mechanism will be facilitated if the shape and chemical properties of the enzyme's active site are more complementary to the transition state than to the undistorted substrate; in other words, if the enzyme were to have a higher affinity for the transition state than for the substrate. Knowing this, your friend looked in an organic chemistry textbook to identify a stable chemical that closely resembles the transition state of a reaction that converts X into Y. She generated an antibody against this transition state analog and mixed the antibody with chemical X. What do you think might happen?

If your friend was lucky, she made a "catalytic antibody" that catalyzed the conversion of X into Y. Such catalytic antibodies have been isolated and shown to catalyze a variety of reactions, but with lower efficiency than genuine enzymes.

14-37Explain how scientists used artificial vesicles to prove that the generation of ATP by the ATP synthase was not powered by a single high-energy intermediate but rather by a proton gradient. Be sure to describe the two experiments that were negative controls (no ATP generated), the positive control (ATP generated as expected), and a fourth experiment proving that the gradient is the required energy source.

In all the experiments, artificial liposomes were generated and exposed to light, and the surrounding solution was checked for an increase in ATP. In the first experiment, the liposomal membranes contained only bacteriorhodopsin, a bacterial protein that pumps protons and is activated by light. In this negative control, ATP was not expected to be produced, and it was not. In the second experiment, also a negative control, the liposomes contained only ATP synthase. Again, if the chemiosmotic hypothesis was correct, ATP should not have been generated, which was what was observed. In the third experiment, both bacteriorhodopsin and ATP synthase were present in the liposomal membrane. When exposed to light, protons were pumped into the vesicle and ATP was generated. In the fourth experiment, to show that the ATP production was solely a result of the proton gradient, an uncoupling agent was added to the solution containing liposomes with bacteriorhodopsin and ATP synthase. In this case, even though the protons were being pumped into the liposomes, a gradient did not build up; this was because of the presence of the uncoupling agent, which made the membrane permeable to protons. No ATP was generated, proving that it was the proton gradient that was the energy source for ATP synthesis.

Some of the enzymes that oxidize sugars to yield usable cellular energy (for example, ATP) are regulated by phosphorylation. For these enzymes, would you expect the inactive form to be the phosphorylated form or the dephosphorylated form? Explain your answer.

In general the inactive form is the phosphorylated form. The main purpose of glycolysis and the citric acid cycle is to generate ATP; thus, the enzymes are inactive when the concentration of ATP is high and active when it is low. It makes sense that cells would not want to have to phosphorylate their enzymes to turn them on when ATP levels are already low, because phosphorylation requires ATP.

14-68 For each of the following sentences, fill in the blanks with the best word or phrase selected from the list below. Not all words or phrases will be used; each word or phrase should be used only once.In the carbon-fixation process in chloroplasts, carbon dioxide is initially added to the sugar __________________. The final product of carbon fixation in chloroplasts is the three-carbon compound __________________. This is converted into __________________ (which can be used directly by the mitochondria),into __________________ (which is exported to other cells), and into __________________ (which is stored in the stroma). The carbon-fixation cycle requires energy in the form of __________________ and reducing power in the form of __________________. 3-phosphoglycerate pyruvateATP ribose 1,5-bisphosphate glyceraldehyde 3-phosphate ribulose 1,5-bisphosphate NADH starch NADPH sucrose

In the carbon-fixation process in chloroplasts, carbon dioxide is initially added to the sugar ribulose 1,5-bisphosphate. The final product of carbon fixation in chloroplasts is the three-carbon compound glyceraldehyde 3-phosphate. This is converted into pyruvate (which can be used directly by the mitochondria), into sucrose (which is exported to other cells), and into starch (which is stored in the stroma). The carbon-fixation cycle requires energy in the form of ATP and reducing power in the form of NADPH.

Provide a brief description of the process by which the body derives energy from food. Include in your description the three stages of catalysis, and delineate the input and output that connect each stage.

In the first stage of catabolism, polymers are broken down into smaller subunits in the digestive system, which stretches from the mouth to the gut. Proteins are converted to amino acids, fats to fatty acids and glycerol, and carbohydrates simple sugars, including the monosaccharide glucose. In the second stage of catabolism, these simple subunits are further broken down to generate the activated carrier acetyl CoA. Acetyl CoA is the molecular input for the third stage of metabolism. In the last stage of catabolism, the acetyl CoA is oxidized to CO2, coupled to the production of large amounts of ATP, which is used as chemical energy for the cell.

Make a table of your results and use them to support or falsify your prediction

Include a table of results. The prediction was supported. As the light intensity increased the gas collected increased.

14-30 Bongkrekic acid is an antibiotic that inhibits the ATP/ADP transport protein in the inner mitochondrial membrane. Which of the following will allow electron transport to occur in mitochondria treated with bongkrekic acid? (a)placingthe mitochondria in anaerobic conditions (b)adding FADH2 (c)making the inner membrane permeable to protons (d)inhibiting the ATP synthase

Inhibition of the ATP/ADP translocase prevents the export of ATP generated by oxidative phosphorylation in exchange for an import of ADP into the matrix. The ensuing buildup of ATP at the expense of ADP inhibits the ATP synthase. Because protons are no longer being used to power the ATP synthase, the proton gradient is not dissipated; the increasingly steep proton gradient makes it increasingly difficult for the electron-transport proteins to pump protons out of the matrix, and electron transport quickly stops. Hence, the inner membrane becomes permeable to protons, allowing electron transport to resume (although no ATP will be synthesized).

Intermediate filaments are elongated fibrous proteins with an N-terminal globular _________________ region and a C-terminal globular _________________ region; these regions flank the elongated rod domain. The α-helical region of the rod interacts with the α-helical region of another monomer in a ____________________ configuration to form a dimer. ______________ dimers will line up to form a staggered tetramer. ______________ strands of tetramers come together and twist together to form the _________________ nm filament. The ___________________ domains are exposed on the surface of the intermediate filament, allowing for interaction with cytoplasmic components. antiparallel four tail β barrel globular ten coiled-coil head trimeric covalent rod twenty-five eight seven two

Intermediate filaments are elongated fibrous proteins with an N-terminal globular head region and a C-terminal globular tail region; these regions flank the elongated rod domain. The α-helical region of the rod interacts with the α-helical region of another monomer in a coiled-coil configuration to form a dimer. Two dimers will line up to form a staggered tetramer. Eight strands of tetramers come together and twist together to form the ten nm filament. The globular domains are exposed on the surface of the intermediate filament, allowing for interaction with cytoplasmic components.

Intermediate filaments are found mainly in cells that are subject to mechanical stress. Gene mutations that disrupt intermediate filaments cause some rare human genetic diseases. For example, the skin of people with epidermolysis bullosa simplex is very susceptible to mechanical injury; people with this disorder have mutations in their __________________ genes, which code for the intermediate filament found in epithelial cells. These filaments are usually connected from cell to cell through junctions called __________________s. The main filaments found in muscle cells belong to the __________________ family; people with disruptions in these intermediate filaments can have muscular dystrophy. In the nervous system, __________________s help strengthen the extremely long extensions often present in nerve cell axons; disruptions in these intermediate filaments can lead to neurodegeneration. People who carry mutations in the gene for __________________, an important protein for cross-linking intermediate filaments, have a disease that combines symptoms of epidermolysis bullosa simplex, muscular dystrophy, and neurodegeneration. Humans with progeria, a disease that causes premature aging, carry mutations in a nuclear ____________. desmosome lamin synapse keratin neurofilament vimentin kinase plectin

Intermediate filaments are found mainly in cells that are subject to mechanical stress. Gene mutations that disrupt intermediate filaments cause some rare human genetic diseases. For example, the skin of people with epidermolysis bullosa simplex is very susceptible to mechanical injury; people with this disorder have mutations in their keratin genes, the intermediate filament found in epithelial cells. These filaments are usually connected from cell to cell through junctions called desmosomes. The main filaments found in muscle cells belong to the vimentin family; people with disruptions in these intermediate filaments can have muscular dystrophy. In the nervous system, neurofilaments help strengthen the extremely long extensions often present in nerve cell axons; disruptions in these intermediate filaments can lead to neurodegeneration. People who carry mutations in the gene for plectin, an important protein for cross-linking intermediate filaments, have a disease that combines symptoms of epidermolysis bullosa simplex, muscular dystrophy, and neurodegeneration. Humans with progeria, a disease that causes premature aging, carry mutations in a nuclear lamin.

Which of the following best describes the behavior of a gated channel?

It opens more frequently in response to a given stimulus.

What is the function of the nuclear pore complex found in eukaryotes?

It regulates the movement of proteins and RNAs into and out of the nucleus

Which of the following is true of heterochromatin but not of euchromatin?

It remains tightly coiled at the G1 phase.

What changes did you notice in the red blood cells between the different environments?

It was isotonic because it was just a normal cell at first. After adding the NaCl solution, the blood cells shriveled and became hypertonic. Finally, after the water was added, the cells became hypotonic and bursted

D

LOOK AT FIGURE 53) The researcher measures the concentration of the polypeptides from different regions in the early embryo and finds the following pattern (darker shading = greater concentration): Which of the following would be his most logical assumption? A) The substance has moved quickly from region 5 to region 1. B) Some other material in the embryo is causing accumulation in region 1 due to differential binding. C) The cytosol is in constant movement, dispersing the polypeptide. D) The substance is produced in region 1 and diffuses toward region 5. E) The substance must have entered the embryo from the environment near region 1.

Three turns of the Calvin cycle nets one G3P because the other five must be recycled to RUBP. Explain how the regeneration of RUBP is accomplished.

Last step of Calvin cycle rearranges five carbon skeletons of G3P to 3 RUBP molecules. 3 molecules of ATP are needed for this.

Table of results. N = no germination 1 = predominantly 1-D growth; 2 = predominantly 2-D growth

Medium Day 21 normal fern medium 2-D fern medium + 2-thiouruacil (5mg/L) 1-D fern medium + 2-thiouruacil (10mg/L) 1-D (or dead) fern medium + 2-thiouruacil + excess uracil 2-D

Once researchers identified DNA as the unit of inheritance, they asked how information was transferred from the DNA in the nucleus to the site of protein synthesis in the cytoplasm. What is the mechanism of information transfer in eukaryotes? Messenger RNA is transcribed from a single gene and transfers information from the DNA in the nucleus to the cytoplasm, where protein synthesis takes place. Transfer RNA takes information from DNA directly to a ribosome, where protein synthesis takes place. Proteins transfer information from the nucleus to the ribosome, where protein synthesis takes place. DNA from a single gene is replicated and transferred to the cytoplasm, where it serves as a template for protein synthesis.

Messenger RNA is transcribed from a single gene and transfers information from the DNA in the nucleus to the cytoplasm, where protein synthesis takes place

The following mRNA transcript would result in what polypeptide sequence? 5′ ACU-UUC-ACU-AUG-UUU-UUA-UCC-UCC-ACU-CCU-UGA 3′ Use the following codons and the amino acids they encode. AUG = Start or Met; UUU, UUC = Phe; UUA, UUG = Leu; UCU, UCC = Ser; CCU, CCC = Pro; ACU, ACC = Thr; UGA = Stop. Thr-Phe-Thr Phe-Leu-Ser-Ser-Thr-Pro Met-Phe-Leu-Ser-Ser-Thr-Pro Thr-Phe-Thr-Phe-Leu-Ser-Ser-Thr-Pro Met-Phe-Leu-Ser-Ser

Met-Phe-Leu-Ser-Ser-Thr-Pro

59. you wish to explore how mutations in specific genes affecting sugar metabolism might alter tooth development. which organism is likely to provide the best model system for your studies, and why?

Mice, because they have teeth and have been used as a model organism for a long time. They can reproduce relatively rapidly and the extensive scientific community that works with mice has developed techniques to facilitate genetic manipulations. E. coli and arabidopsis do not have teeth. horse like sugar and have big teeth.

Microtubules are formed from the tubulin heterodimer, which is composed of the nucleotide-binding __________________ protein and the __________________ protein. Tubulin dimers are stacked together into protofilaments; __________________ parallel protofilaments form the tubelike structure of a microtubule. __________________ rings are important for microtubule nucleation and are found in the __________________ , which is usually found near the cell's nucleus in cells that are not undergoing mitosis. A microtubule that is quickly growing will have a __________________ cap that helps prevent the loss of subunits from its growing end. Stable microtubules are used in cilia and flagella; these microtubules are nucleated from a __________________ and involve a "__________________ plus two" array of microtubules. The motor protein __________________ generates the bending motion in cilia; the lack of this protein can cause Kartagener's syndrome in humans. α-tubulin dynein nine ATP four thirteen basal body γ-tubulin twenty-one β-tubulin GTP UTP centrosome kinesintwo vimentin δ-tubulin myosin

Microtubules are formed from the tubulin heterodimer, which is composed of the nucleotide-binding β-tubulin protein and the α-tubulin protein. Tubulin dimers are stacked together into protofilaments; thirteen parallel protofilaments form the tubelike structure of a microtubule. γ-Tubulin rings are important for microtubule nucleation and are found in the centrosome, which is usually found near the cell's nucleus in cells that are not undergoing mitosis. A microtubule that is quickly growing will have a GTP cap that helps prevent the loss of subunits from its growing end. Stable microtubules are used in cilia and flagella; these microtubules are nucleated from a basal body and involve a "nine plus two" array of microtubules. The motor protein dynein generates the bending motion in cilia; the lack of this protein can cause Kartagener's syndrome in humans.

The stimulation of a motor neuron ultimately results in the release of a neurotransmitter at the synapse between the neuron and a muscle cell. How is the chemical signal converted into an electrical signal in the postsynaptic muscle cell?

Most neurotransmitter receptors function as ligand-gated ion channels. These ion channels are similar to voltage-gated channels, except that they do not open in response to a change in voltage across the membrane, but to the binding of a neurotransmitter. In the neuromuscular junction, the neurotransmitter acetylcholine binds to the acetylcholine receptor, which allows Na+ to enter the muscle cell, altering its membrane potential. In this way, a chemical signal (acetylcholine) is converted back into an electrical signal (change in membrane potential).

NAD+

NAD+ is a Coenzyme that forms NADH when it is reduced. (Picks up electrons)

The hydrogens are held in the cell temporarily by what electron carrier?

NAD+, a coenzyme.

14-22For each of the following sentences, fill in the blanks with the best word or phrase selected from the list below. Not all words or phrases will be used; each word or phrase should be used only once.NADH donates electrons to the __________________ of the three respiratory enzyme complexes in the mitochondrial electron-transport chain. __________________ is a small protein that acts as a mobile electron carrier in the respiratory chain. __________________ transfers electrons to oxygen. Electron transfer in the chain occurs in a series of __________________ reactions. The first mobile electron carrier in the respiratory chain is __________________. cytochrome c plastoquinone cytochrome c oxidase reduction first second NADH dehydrogenase the cytochrome b-c1complex oxidation third oxidation-reduction ubiquinone phosphorylation

NADH donates electrons to the first of the three respiratory enzyme complexes in the mitochondrial electron-transport chain. Cytochrome c is a small protein that acts as a mobile electron carrier in the respiratory chain. Cytochrome c oxidase transfers electrons to oxygen. Electron transfer in the chain occurs in a series of oxidation-reduction reactions. The first mobile electron carrier in the respiratory chain is ubiquinone.

14-24Which of the following statements is true? (a)The NADH dehydrogenase complex can pump more protons than can the cytochrome b-c1complex. (b)The pH in the mitochondrial matrix is higher than the pH in the intermembrane space. (c)The proton concentration gradient and the membrane potential across the inner mitochondrial membrane tend to work against each other in driving protons from the intermembrane space into the matrix. (d)The difference in proton concentration across the inner mitochondrial membrane has a much larger effect than the membrane potential on the total proton-motive force.

NADH donates electrons to the first of the three respiratory enzyme complexes in the mitochondrial electron-transport chain. Cytochrome c is a small protein that acts as a mobile electron carrier in the respiratory chain. Cytochrome c oxidase transfers electrons to oxygen. Electron transfer in the chain occurs in a series of oxidation-reduction reactions. The first mobile electron carrier in the respiratory chain is ubiquinone.

3.70.In general, there is a positive change in free energy associated with reduction reactions, and most of them are coupled with oxidation reactions. The last step in the biosynthesis of cholesterol involves the reduction of a carbon-carbon double bond. What activated carrier molecule is used in this reaction (and generally for the reduction of lipids) and how would this reaction be influenced by the levels of available ATP?

NADPH is the activated carrier used in the final reduction reaction to produce cholesterol. The rate of this reaction will depend upon the concentration of NADPH, the regeneration of which depends upon ATP hydrolysis. If ATP levels are low, then we can expect that the levels of NADPH will be correspondingly low. The rate of cholesterol synthesis will be lower than if the cell is in a high-energy state with abundant levels of ATP, and consequently high levels of NADPH.

Under which of the following conditions would there be no net movement of potassium (K+) through K+ leak channels in a cell's plasma membrane (remember, [K+] is high on the inside of the cell and low on the outside)?

NOT There is a higher Na+ concentration gradient than that of K+.

Heme and 11-cis-retinaldehyde are two examples of ______________.

NOT ENERGY CARRIERS

Hydrolysis reactions are commonly used inside the cell to split high-energy covalent bonds. Use the ΔG° for reaction below to determine the equilibrium constant (K) for acetyl-P and actetate + P. ΔG° (kcal/mole) acetyl-P LaTeX: \leftrightarrow ↔ actetate + P -10.3

NOT NONE OF THESE

Energy required by the cell is generated in the form of ATP. ATP is hydrolyzed to power many of the cellular processes, increasing the pool of ADP. As the relative amount of ADP molecules increases, they can bind to glycolytic enzymes, which will lead to the production of more ATP. The best way to describe this mechanism of regulation is ___________.

NOT substrate-level phosphorylation.

15-58 For each of the following sentences, choose one of the two options enclosed in square brackets to make a correct statement. New plasma membrane reaches the plasma membrane by the [regulated/constitutive] exocytosis pathway. New plasma membrane proteins reach the plasma membrane by the [regulated/constitutive] exocytosis pathway. Insulin is secreted from pancreatic cells by the [regulated/constitutive] exocytosis pathway. The interior of the trans Golgi network is [acidic/alkaline]. Proteins that are constitutively secreted [aggregate/do not aggregate] in the trans Golgi network.

New plasma membrane reaches the plasma membrane by the constitutive exocytosis pathway. New plasma membrane proteins reach the plasma membrane by the constitutive exocytosis pathway. Insulin is secreted from pancreatic cells by the regulated exocytosis pathway. The interior of the trans Golgi network is acidic. Proteins that are constitutively secreted do not aggregate in the trans Golgi network.

Recall that one question you were studying was "Is RNA synthesis required for 1-D growth?" Using your data to support your answer, is it YES or NO?

No, RNA synthesis is not required for outgrowth. The experimental treatments (2-thiouracil 5mg/L and 2-thiouracil 10mg/L) did show 1D growth.

Your friend discovers a protein that she names EBP. EBP binds to microtubule plus ends, and she hypothesizes a role for EBP in increasing dynamic instability. To determine the function of EBP, she examines its effect on microtubules. She polymerizes microtubules from purified centrosomes in a Petri plate and determines the number of shrinking microtubules over a three-minute time interval for different concentrations of EBP. The data she obtained are shown in Figure Q17-25. Is this result consistent with her hypothesis? Explain.

No, the data graphed in Figure Q17-25 are not consistent with her hypothesis. If EBP were to increase the dynamic instability of microtubules, you would expect an increase in both the number of shrinking microtubules and the number of growing microtubules. Since the graph shows a reduction in the number of shrinking microtubules, dynamic instability has evidently decreased. Further experiments would be needed to determine the net effect of these changes on microtubules and whether they become longer or shorter on average following EBP treatment.

Thermal motion promotes lateral position exchanges between lipid molecules within a monolayer. In an artificial bilayer, this movement has been estimated to be ~2 μm/second. This represents the entire length of a bacterial cell. Do you expect the lateral movement of a lipid molecule within a biological membrane to be equally fast? Explain your answer.

No. Although the rate of movement may be similar, it will most likely be slower in a biological membrane. An artificial bilayer is primarily phospholipids. Biological membranes contain a large number of protein components and specialized membrane domains that could limit the rate of lateral diffusion.

Which of the following contains the only 4 basic components of vertebrate ECM?

None of the above

Polar covalent bonds are formed when the electrons in the bond(s) are not shared equally between the nuclei. Which one of these molecules is polar covalent?

None of these is correct.

You find a bacterium growing in pure oil, containing the usual hydrophobic lipids. This organism must be very good at breaking down ____.

Nonpolar covalent bonds

What are the experimental control(s) in this experiment?

Normal fern medium - This is a positive control. Normal gametophyte development is expected and all treatments are compared to this control. 2-thiouracil + uracil medium - This is a control to determine whether the effects of the base analog can be reversed by adding normal uracil.

Eukaryotic cells have membranous organelles which share many similarities with the cell plasma membrane. Which of the following is not a characteristic shared by the plasma membrane and organellar membranes?

Not flexible, two-dimensional field

The nucleus is surrounded by a membrane called the ________.

Nuclear envelope

What are some types of essential macromolecules found in living organisms that require nitrogen?

Nucleic acids, proteins, chlorophyll

67. Cellular processes are often regulated by unknown mechanisms. In many cases, biologists work backward in an attempt to understand a process in which they are interested.This was the case when Nurse and Hartwell were trying to understand how cell division is controlled in yeast. Describe the process by which they "broke" the system and then supplied the "missing parts" to get the cell cycle running again. What further evidence did they collect to show that human cells and yeast cells regulate the cell cycle using a similar mechanism?

Nurse and Hartwell first treated yeast cells with a chemical mutagen. The mutated population of cells was then grown and observed. Cells that demonstrated defects in cell-cycle regulation (characterized by cell-cycle arrest, larger-than-normal cells, and smaller-than-normal cells) were then isolated. The use of a library of plasmids that each express a normal gene from yeast cells allowed the scientists to identify exactly which gene could be used to "rescue" the mutant, because when the normal gene is expressed again, the cells return to a normal cell cycle. After this big result, the scientists went on to show that the homologous gene from other organisms could also rescue the mutant phenotype. The most exciting result was obtained with the human version of the cdc2 gene, which demonstrated that there are common principles underlying cell-cycle regulation across a large range of eukaryotic organisms.

Which of the following statements is true? (a) Extracellular signal molecules that are hydrophilic must bind to a cell- surface receptor so as to signal a target cell to change its behavior. (b) To function, all extracellular signal molecules must be transported by their receptor across the plasma membrane into the cytosol. (c) A cell-surface receptor capable of binding only one type of signal molecule can mediate only one kind of cell response. (d) Any foreign substance that binds to a receptor for a normal signal molecule will always induce the same response that is produced by that signal molecule on the same cell type.

Only choice (a) is true. A hydrophilic molecule cannot diffuse across the membrane and it can therefore only affect a cell if it binds to a cell-surface receptor [choice (a)]. Most signal molecules remain bound to the extracellular domain of the receptor, whereas the intracellular domain mediates signal transduction; although many signal molecules are endocytosed with their receptor, they remain inside membrane-bounded compartments and are therefore not transported into the cytosol [choice (b)]. A cell-surface receptor capable of binding only one type of signal molecule can stimulate more than one kind of cell response, depending on the types of intracellular signaling pathway it activates [choice (c)]. Foreign substances that bind to a receptor for a normal signal molecule can sometimes induce the same response as the natural signal molecule, but in other cases they can block the binding of the natural signal molecule without activating the receptor [choice (d)].

heterotroph

Organism that cannot synthesize organic molecules from inorganic nutrients, animals.

For each of the following sentences, fill in the blanks with the best word or phrase selected from the list below. Not all words or phrases will be used; each word or phrase may be used more than once. Oxidative phosphorylation is a process that occurs in the __________________ of mitochondria. It requires an electron-transport chain that operates on the high-energy electrons taken from the activated carrier molecules __________________ and __________________ that are produced by glycolysis and the citric acid cycle. These electrons are transferred through a series of molecules, and the energy released during these transfers is used to generate a gradient of __________________, or __________________. Because their concentration is much __________________ outside than inside the mitochondria, the flow of __________________, or __________________, down the concentration gradient is energetically very __________________ and can thus be coupled to the production of ATP from ADP. Thus, oxidative phosphorylation refers to the oxidation of __________________ and __________________ molecules and the phosphorylation of __________________. Without this process, the yield of ATP from each glucose molecule would be __________________ decreased. ADP GTP NAD+ ATP H+ NADH cytosol higher Pi electrons inner membrane protons FADH2 lower severely favorable matrix slightly glucose moderately unfavorable

Oxidative phosphorylation is a process that occurs in the inner membrane of mitochondria. It requires an electron-transport chain that operates on the high-energy electrons taken from the activated carrier molecules NADH and FADH2 that are produced by glycolysis and the citric acid cycle. These electrons are transferred through a series of molecules, and the energy released during these transfers is used to generate a gradient of protons, or H+. Because their concentration is much higher outside than inside the mitochondria, the flow of protons, or H+, down the concentration gradient is energetically very favorable and can thus be coupled to the production of ATP from ADP. Thus, oxidative phosphorylation refers to the oxidation of NADH and FADH2 molecules and the phosphorylation of ADP. Without this process, the yield of ATP from each glucose molecule would be severely decreased.

Which of the following is a product of photosynthesis that is a reactant in a campfire?

Oxygen

Which of the following is a trace element in the human body?

P

Titanium has an atomic number of 22. How many protons, neutrons, and electrons are in an isotope of titanium with mass number of 48?

P = 22 n = 26 e = 22

What term is used to define the structure that separates the contents of a human cell from it's surrounding medium

Plasma membrane

15-9For each of the following sentences, fill in the blanks with the best word or phrase selected from the list below. Not all words or phrases will be used; use each word or phrase only once. Plasma membrane proteins are inserted into the membrane in the __________________. The address information for protein sorting in a eukaryotic cell is contained inthe __________________ of the proteins. Proteins enter the nucleus in their __________________ form. Proteins that remain in the cytosol do not contain a __________________. Proteins are transported into the Golgi apparatus via __________________. The proteins transported into the endoplasmic reticulum by __________________ are in their __________________ form. amino acid sequence Golgi apparatus sorting signal endoplasmic reticulum plasma membrane transport vesicles folded protein translocators unfolded

Plasma membrane proteins are inserted into the membrane in the endoplasmic reticulum. The address information for protein sorting in a eukaryotic cell is contained in the amino acid sequence of the proteins. Proteins enter the nucleus in their folded form. Proteins that remain in the cytosol do not contain a sorting signal. Proteins are transported into the Golgi apparatus via transport vesicles. The proteins transported into the endoplasmic reticulum by protein translocators are in their unfolded form.

________ are cytoplasmic channels through openings in plant cell walls.

Plasmodesmata

What would an electron holding extra energy be useful for?

Powering a biochemical process that requires energy Storing energy for a time

What are some types of essential macromolecules found in living organisms that require nitrogen? Give some examples of how these molecules are used by plants.

Proteins - enzymes for metabolism, structural, transport, and etc. DNA and RNA - genetic information ATP - cellular energy Chlorophyll - photosynthesis

The hydrophobic amino acids would come into contact with the hydrocarbon tails of the phospholipid bilayer, whereas the hydrophilic amino acids would interact with the intracellular and extracellular environment.

Proteins are amphipathic molecules that contain nonpolar (hydrophobic) amino acids and polar (hydrophilic) amino acids. Where would you expect to find the hydrophobic and hydrophilic amino acid residues of a transmembrane protein?

15-37For each of the following sentences, fill in the blanks with the best word or phrase selected from the list below. Not all words or phrases will be used; use each word or phrase only once. Proteins are transported out of a cell via the __________________ or __________________ pathway. Fluids and macromolecules are transported into the cell via the __________________ pathway. All proteins being transported out of the cell pass through the __________________ and the __________________. Transport vesicles link organelles of the __________________ system. The formation of __________________ in the endoplasmic reticulum stabilizes protein structure. carbohydrate Golgi apparatus disulfide bonds hydrogen bonds endocyticionic bonds endomembrane lysosome endoplasmic reticulum protein endosome secretory exocytic

Proteins are transported out of a cell via the secretory or exocytic pathway. Fluid and macromolecules are transported into the cell via the endocytic pathway. All proteins being transported out of the cell pass through the endoplasmic reticulum and the Golgi apparatus. Transport vesicles link organelles of the endomembrane system. The formation of disulfide bonds in the endoplasmic reticulum stabilizes protein structure.

15-29Which of the following statements is true? (a)Proteins destined for the ER are translated by a special pool of ribosomes whose subunits are always associated with the outer ER membrane. (b)Proteins destined for the ER translocate their associated mRNAs into the ER lumen where they are translated. (c)Proteins destined for the ER are translated by cytosolic ribosomes and are targeted to the ER when a signal sequence emerges during translation. (d) Proteins destined for the ER are translated by a pool of cytosolic ribosomes that contain ER-targeting sequences that interact with ER-associated protein translocators.

Proteins destined for the ER are translated by cytosolic ribosomes and are targeted to the ER when a signal sequence emerges during translation.

14-28Explain how the F0 complex of ATP synthase harnesses the proton-motive force to help synthesize ATP. What would happen if the proton gradient were reversed?

Protons flow through a channel that exists between the subunits of the transmembrane H+ carrier, which forms a ring (the rotor). The flow of protons through this carrier makes the rotor and its attached stalk rotate. As the stalk rotates, it rubs against proteins in the stationary F1 portion of the ATP synthase. The resulting mechanical deformation produces a conformational change in the subunits of the F1 ATPase that causes them to produce ATP. When the proton gradient is reversed, the F1 portion of the ATP synthase catalyzes the hydrolysis of ATP to ADP and Pi, rather than the revers reaction of ATP synthesis; this causes protons to be pumped out of the matrix against their electrochemical gradient, as the rotor and its stalk rotate in the direction opposite to that involved in ATP synthesis.

14-16 For each of the following sentences, fill in the blanks with the best word or phrase selected from the list below. Not all words or phrases will be used; each word or phrase should be used only once.Mitochondria can use both __________________ and __________________ directly as fuel. __________________ produced in the citric acid cycle donates electrons to the electron-transport chain. The citric acid cycle oxidizes __________________ and produces __________________ as a waste product. __________________ acts as the final electron acceptor in the electron-transport chain. The synthesis of ATP in mitochondria is also known as __________________. acetyl groups NADH carbon dioxide NADP+ chemiosmosis NADPH fatty acids oxidative phosphorylation glucose oxygen NAD+ pyruvate

Pyruvate Fatty Acids NADH acetyl groups carbon dioxide oxygen oxidative phosphorylation

Which of the following proteins is involved in synthesizing messenger RNA in eukaryotes?

RNA polymerase II

How does temperature affect the reaction rate?

Rate of the reaction increases when particles hit each other faster and collide more frequently and the temperature makes the particles collide more

A photosystem is composed of a protein complex called a ________ complex surrounded by several ______ complexes.

Reaction center, light harvesting

Activation of which of the following is associated with the formation of focal adhesions and stress fibers?

Rho

Protein production is a function of the

Ribosomes

What do elements with atomic numbers 6, 14, and 22 have in common?

Same number of valence electrons and will form the same number of covalent bonds

Three different membrane components are shown in Figure Q11-10. Using the list below, identify the three components, and label the chemical groups indicated. A. glycerol B. sugar C. phospholipid D. glycolipid E. sterol F. unsaturated hydrocarbon G. saturated hydrocarbon H. sterol polar head group

See Figure A11-10.

A small membrane vesicle containing a transmembrane protein is shown in Figure Q11-31. Assume that this membrane vesicle is in the cytoplasm of a cell. Figure Q11-31 A. Label the cytosolic and noncytosolic faces of the membrane vesicle. This membrane vesicle will undergo fusion with the plasma membrane. B. Sketch the plasma membrane after vesicle fusion, indicating the new locations of the vesicle membrane and the transmembrane protein carried by the membrane vesicle. C. On your drawing for B, label the original cytosolic and noncytosolic faces of the vesicle membrane as it resides in the plasma membrane. Also label the extracellular space and the cytosol. Indicate the N- and C-terminus of the inserted transmembrane protein.

See Figure A11-31A.

Band 3 protein is important for erythrocyte shape and cortical flexibility. It participates in multiprotein complexes that include ankyrin and spectrin. There are three major populations of Band 3 in the plasma membrane: (1) unassociated with the cortex, with lateral diffusion limited only by spectrin fibers; (2) associated with spectrin fibers; and (3) associated with the actin junctional complex. Figure Q11-61 A. Redraw a portion of the erythrocyte cortex shown in Figure Q11-61, and indicate where you expect to find Band 3 protein, based on the description of the three major populations (label them 1, 2, and 3). B. Draw the single-particle tracking (SPT) profile you may expect to observe for each of these three populations. Explain your reasoning. Defects in any of the membrane proteins that participate in the multiprotein junctional complexes can disrupt the overall cell morphology. These erythrocyte pathologies are characterized by loss of the normal biconcave disc morphology and early clearance from the bloodstream. C. You have a patient who carries a mutation in the spectrin gene. The defect interferes with interactions between spectrin and Band 3. Consider your answer from part B and redraw SPT profiles for Band 3 populations as you expect to see them in the erythrocytes of your patient. Explain your reasoning.

See Figure A11-61 Figure A11-61 B. The lateral movement of populations 2 (dark gray) and 3 (black) would be fairly restricted by their association with spectrin fibers and the junctional complex, respectively. The lateral movement of population 1 (light gray) should be rapid, but contained within the particular region defined by spectrin fibers. Based on these parameters, potential SPT profiles are drawn below. C. Without the interaction with spectrin, it is likely that the Band 3 proteins that made up populations 2 and 3 will no longer be found associated with the junctional complex or the spectrin fibers. This implies that the majority of Band 3 molecules would be able to move rapidly through the membrane, restricted only by the spectrin fibers (essentially becoming part of population 1).

15-4 Label the structures of the cell indicated by the lines in Figure Q15-4.Figure Q15-4 A.nucleus B.peroxisome C.rough endoplasmic reticulum D.Golgi apparatus E.cytosol F.endosome G.plasma membrane H.lysosome I.mitochondrion J. smooth endoplasmic reticulum

See Figure A15-4.

In the three cell outlines in Figure Q17-18, indicate the arrangement of the microtubules, showing clearly their free and attached ends. On each figure, indicate the plus end and the minus end for one of the microtubules.

See Figure A17-18.

You isolate some muscle fibers to examine what regulates muscle contraction. When you bathe the muscle fibers in a solution containing ATP and Ca2+, you see muscle contraction (experiment 3 in Table Q17-61). Ca2+ is necessary, as solutions containing ATP alone or nothing do not stimulate contraction and thus the muscle remains in a relaxed state (experiments 1 and 2 in Table Q17-61). From what you know about the mechanism of muscle contraction, fill in your predictions of whether the muscle will be contracted or relaxed for experiments 4, 5, and 6. Explain your answers.

See Table A17-61. Table A17-61 In experiment 4, the muscle will be relaxed because troponin will not be able to bind Ca2+. By preventing troponin from binding to Ca2+, troponin will not be able to undergo the conformational change that causes tropomyosin to alter its association with actin. This altered association is normally required for myosin to bind actin. In the absence of troponin regulation by Ca2+, myosin cannot bind actin and the muscle cannot contract. In experiment 5, the muscle will contract because tropomyosin cannot bind to actin. If tropomyosin cannot bind actin, myosin can. The presence of ATP means that ATP will be available for myosin to hydrolyze, causing muscle contraction. In experiment 6, the muscle will remain relaxed. The presence of Ca2+ will induce a conformational change in troponin that causes tropomyosin to shift, exposing actin for myosin to bind. However, when myosin binds, the myosin molecule will attach and then release because the myosin will bind the nonhydrolyzable analog of ATP. Because no ATP hydrolysis can occur, the muscle will remain in the relaxed state.

Cell membranes allow certain molecules to pass, while blocking others.

Selectively permeable

Autotroph

Self-feeders; they sustain themselves without eating anything derived from other living beings

It can be useful to analyze the steps of glycolysis with respect to the four basic types of enzymes required by this central catabolic pathway and to consider whether each enzyme produces or harvests the energy of an activated carrier. For each step of glycolysis (see Figure 13-5 or Panel 13-1), indicate which type of enzyme (of the four listed below and in Table 13-1 is required—or if none apply). Also, indicate whether an activated energy carrier is involved, and, if so, how. Step 1 ___________ Step 2 ___________ Step 3 ___________ Step 4 ___________ Step 5 ___________ Step 6 ___________ Step 7 ___________ Step 8 ___________ Page 9 of 27 Step 9 ___________ Step 10 ___________ Enzyme types: kinase, isomerase, mutase, dehydrogenase

Step 1 kinase, energy in the form of ATP consumed Step 2 isomerase Step 3 kinase, energy in the form of ATP consumed Step 4 none of the above Step 5 isomerase Step 6 dehydrogenase, energy in the form of NADH produced Step 7 kinase (catalyzing its reverse reaction)*; energy in the form of ATP produced Step 8 mutase Step 9 none of the above Step 10 kinase (catalyzing its reverse reaction)*; energy in the form of ATP produced *During glycolysis, both phosphoglycerate kinase and pyruvate kinase remove rather than add phosphates. However, if the appropriate conditions are established in vitro, these enzymes can also catalyze the backward reaction that adds a phosphate—thus functioning like a typical kinase.

How We Know: Tracking Protein and Vesicle Transport 15-66 You have created a green fluorescent protein (GFP)fusion to a protein that is normally secreted from yeast cells. Because you have learned about the use of temperature-sensitive mutations in yeast to study protein and vesicle transport, you obtain three mutant yeast strains, each defective in some aspect of the protein secretory process. Being a good scientist, you of course also obtain a wild-type control strain. You decide to examine the fate of your GFP fusion protein in these various yeast strains and engineer the mutant strains to express your GFP fusion protein. However, in your excitement to do the experiment, you realize that you did not label any of the mutant yeast strains and no longer know which strain is defective in what process. You end up numbering your strains with the numbers 1 to 4, and then you carry out the experiment anyway, obtaining the results shown in Figure Q15-66 (the black dots represent your GFP fusion protein). Figure Q15-66Name the process that is defective in each of these strains. Remember that one of these strains is your wild-type control.

Strain A has protein accumulating in the ER, which means that this cell has a mutation that blocks transport from the ER to the Golgi apparatus. Strain B has secreted protein, and therefore is the wild-type control. Strain C has protein accumulating in the Golgi apparatus, and thus has a mutation that blocks exit of proteins from the Golgi apparatus. Strain D has protein accumulating in the cis Golgi network, and thus has a mutation that blocks the travel of proteins through the Golgi cisternae.

63. employ the principles of evolution discussed in this chapter to explain how the specific features of predatory behaviors of some primitive eukaryotes may have given them a selective advantage over others 1.5 billion years ago.

The Earth's atmosphere became oxygen-rich roughly 1.5 billion years ago. If some primitive predatory eukaryotic cells were similar to modern-day protozoans, they may have been mobile and able to engulf other cells. These characteristics would have been advantageous in the face of a changing atmosphere, and the establishment of a symbiotic relationship with an engulfed aerobe would have been selected for in the eukaryotic cell populations.

Antibodies are Y-shaped molecules that have two identical binding sites. Suppose that you have obtained an antibody that is specific for the extracellular domain of an RTK. When the antibody binds to the RTK, it brings together two RTK molecules. If cells containing the RTK were exposed to the antibody, would you expect the kinase to be activated, inactivated, or unaffected? Explain your reasoning.

The RTK will probably become activated on binding of the antibody molecule. This is because signal-induced dimerization usually activates RTKs. When RTK molecules are brought together, their cytoplasmic kinase domains become activated and each receptor phosphorylates the other.

The action potential is a wave of __________________ that spreads rapidly along the neuronal plasma membrane. This wave is triggered by a local change in the membrane potential to a value that is __________________ negative than the resting membrane potential. The action potential is propagated by the opening of __________________-gated channels. During an action potential, the membrane potential changes from __________________ to __________________. The action potential travels along the neuron's __________________ to the nerve terminals. Neurons chiefly receive signals at their highly branched __________________. anions hyperpolarization neutral axon less positive cytoskeleton ligand pressure dendrites more synaptic vesicle depolarization negative voltage

The action potential is a wave of depolarization that spreads rapidly along the neuronal plasma membrane. This wave is triggered by a local change in the membrane potential to a value that is less negative than the resting membrane potential. The action potential is propagated by the opening of voltage-gated channels. During an action potential, the membrane potential changes from negative to positive. The action potential travels along the neuron's axon to the nerve terminals. Neurons chiefly receive signals at their highly branched dendrites.

Which of the following statements about the carbohydrate coating of the cell surface is false?

The arrangement of the oligosaccharide side chains is highly ordered, much like the peptide bonds of a polypeptide chain.

15-5 For each of the following sentences, fill in the blanks with the best word or phrase selected from the list below. Not all words or phrases will be used; use each word or phrase only once.The __________________ makes up about half of the total cell volume of a typical eukaryotic cell. Ingested materials within the cell will pass through a series of compartments called __________________ on their way to the __________________, which containsdigestive enzymes and will ultimately degrade the particles and macromolecules taken into the cell and will also degrade worn-out organelles. The __________________ has a cis and transface and receivesproteins and lipids from the __________________, a system of interconnected sacs and tubes of membranes that typically extends throughout the cell. cytosol Golgi apparatus nucleus endoplasmic reticulum lysosome peroxisomes endosomes mitochondria plasma membrane

The cytosol makes up about half of the total cell volume of a typical eukaryotic cell. Ingested materials within the cell will pass through a series of compartments called endosomes on their way to the lysosome, which contains digestive enzymes and will ultimately degrade the particles and macromolecules taken into the cell and will also degrade worn-out organelles. The Golgi apparatus has a cis and trans face and receives proteins and lipids from the endoplasmic reticulum, a system of interconnected sacs and tubes of membranes that typically extends throughout the cell.

15-17A gene regulatory protein, A, contains a typical nuclear localization signal but surprisingly is usually found in the cytosol. When the cell is exposed to hormones, protein A moves from the cytosol into the nucleus, where it turns ongenes involved in cell division. When you purify protein A from cells that have not been treated with hormones, you find that protein B is always complexed with it. To determine the function of protein B, you engineer cells lacking the gene for protein B.You compare normal and defective cells by using differential centrifugation to separate the nuclear fraction from the cytoplasmic fraction,and then separatingthe proteins in these fractions by gel electrophoresis. You identify the presence of protein A and protein B by looking for their characteristic bands on the gel. The gel you run is shown in Figure Q15-17. Figure Q15-17On the basis of these results, what is the function of protein B? Explain your conclusion and propose a mechanism for how protein B works.

The data on the gel show that protein A is always found in the nucleus in the absence of protein B. Therefore, any mechanism that is proposed must explain this result. One possible answer is that protein B binds protein A and masks the nuclear localization signal. In the presence of hormone, protein B interacts with the hormone, which changes its conformation so that it can no longer bind protein A. When protein B no longer binds to protein A, the nuclear localization signal on protein A is now exposed and protein A can enter the nucleus. Therefore, in the absence of protein B, the nuclear localization signal on protein A is always exposed and protein A resides in the nucleus. Another possible answer is that protein B binds protein A and sequesters it by keeping protein A in some subcellular compartment, away from the nucleus. In the presence of hormone, protein B interacts with the hormone, changing its conformation so that it can no longer bind to protein A. When protein B is not present, protein A can enter the nucleus in the presence or absence of hormone.

Describe the two forces that drive an ion across the plasma membrane and explain how the Nernst equation takes into account both of these forces. Use the components of the equation to support your explanation and be sure to specify the assumptions being made when using the Nernst equation to calculate membrane potential.

The forces that drive the movement of an ion across the plasma membrane include a concentration gradient (that is, there is a negative change in free energy associated with an increase in entropy for ions in solution) and an electrical component (the force resulting from the attraction between molecules of opposite charges). The Nernst equation expresses the change in voltage across the membrane as it relates to a change in the ratio of ions on either side of the plasma membrane. As written below, the voltage changes by 62 millivolts with every tenfold change in the ion concentration ratio across the membrane. V = 62 log10(Co/Ci) As written in this simplified form, the equation assumes that the flow of ions has reached an equilibrium at 37°C and that the ions that are moving are positive ions. Because the ions that are moving across the membrane are typically K+ ions (resting membrane potential) or Na+ ions (action potentials) these assumptions hold true for most of the biological systems being examined.

3.74.b. Coenzyme A can be converted to acetyl CoA, which is an important activated carrier molecule that has a central role in metabolism and can be used to add two carbons in each successive cycle of fatty acid synthesis. How does the bond energy help promote the synthesis of fatty acids?

The generation of long-chain fatty acids requires the generation of new carbon-carbon bonds and the reduction of carbon-oxygen bonds. The energy stored in the acetyl CoA thioester bond promotes the addition of new carbons to an elongating fatty acid chain, two carbons at a time. However, the high-energy electrons required to reduce the carbonyl bond are derived from a second type of molecule, NADPH.

All three domains (Bacteria, Archaea, and Eukarya) follow the same genetic code. Therefore, which of the following statements would most likely be correct? The genetic code evolved three times. There were no mutations following the evolution of the genetic code. The genetic code evolved before the different domains diverged. The genetic code evolved before DNA replaced RNA as the unit of genetic information.

The genetic code evolved before the different domains diverged

The human immune system produces __________________ of different immunoglobulins, also called __________________, which enable the immune system to recognize and fight germs by specifically binding one or a few related __________________. The hypervariable structural element that forms the ligand-binding site is comprised of several __________________. Purified antibodies are useful for a variety of experimental purposes, including protein purification using __________________ chromatography. affinity billions ligands antibodies coiled coils loops antigens hundreds size-exclusion Beta strands ion-exchange

The human immune system produces *billions* of different immunoglobulins, also called *antibodies*, which enable the immune system to recognize and fight germs by specifically binding one or a few related *antigens*. The hypervariable structural element that forms the ligand-binding site is comprised of several *loops*. Purified antibodies are useful for a variety of experimental purposes, including protein purification using *affinity* chromatography.

3.75. The addition of a new deoxynucleotide to a growing DNA chain requires more energy than can be obtained by the hydrolysis of ATP to ADP + Pi. What alternative series of reactions is used, and how does this help overcome the energy barrier for DNA synthesis?

The hydrolysis of ATP to ADP is favorable, with a ΔG between -11 and -13 kcal/mole. However, this is not sufficient to drive the addition of a nucleotide to the end of a growing DNA strand. Instead, two reactions are used. The first reaction converts ATP to a DNA-linked AMP residue when a phosphodiester bond is formed during DNA synthesis; simultaneously, a pyrophosphate molecule (PPi) is released. In the second reaction, the PPi is hydrolyzed to form two molecules of Pi. This second reaction is also favorable, providing roughly another -13 kcal/mole. Adding up the ΔG for the entire process, there will be about -26 kcal/mole to drive the addition of the nucleotide to the growing DNA chain, which is sufficient to drive the reaction strongly in one direction.

Some lower vertebrates such as fish and amphibians can control their color by regulating specialized pigment cells called melanophores. These cells contain small, pigmented organelles, termed melanosomes, that can be dispersed throughout the cell, making the cell darker, or aggregated in the center of the cell to make the cell lighter. You purify the melanosomes from melanophores that have either aggregated or dispersed melanosomes and find that: 1. aggregated melanosomes co-purify with dynein; 2. dispersed melanosomes co-purify with kinesin. Given this set of data, propose a mechanism for how the aggregation and dispersal of melanosomes occur.

The melanosomes are transported in the cell on microtubules. When it is advantageous for the animal to become lighter, a signal is sent to the pigment cell that causes the melanosomes to associate with dynein. Because dynein is a minus-end directed motor, it will transport the melanosomes toward the center of the cell, causing the melanosomes to aggregate in the center and the cell to take on a lighter appearance. When the animal wants to become darker, a signal is sent to the pigment cell that causes the melanosomes to associate with kinesin. Kinesin is usually a plus-end directed motor and will move the melanosomes away from the center of the cell so that they are more dispersed, making the cell look darker.

H. V. Wilson worked with sponges to gain some insight into exactly what was responsible for holding adjacent cells together. He exposed two species of differently pigmented sponges to a chemical that disrupted the cell—cell interaction (cell junctions), and the cells of the sponges dissociated. Wilson then mixed the cells of the two species and removed the chemical that caused the cells to dissociate. Wilson found that the sponges reassembled into two separate species. The cells from one species did not interact or form associations with the cells of the other species. How do you explain the results of Wilson's experiments?

The molecules responsible for cell—cell adhesion (cell junctions) differed between the two species of sponge.

The movement of glucose into the cell, against its concentration gradient, can be powered by the co-transport of Na+ into the cell. Explain this movement with respect to the net entropy of the system (that is, thermodynamics).

The movement of Na+ ions from an area that has a high Na+ concentration to a new area of low Na+ concentration is energetically favorable because the net entropy in the system is increasing. As long as the difference in Na+ ion concentration across the membrane is large, the entropic factor will be sufficient to drive the import of glucose into the cell, which represents a decrease in entropy with respect to the population of glucose molecules inside the cell.

Activated GPCRs activate G proteins by reducing the strength of binding of GDP to the α subunit of the G protein, allowing GDP to dissociate and GTP (which is present at much higher concentrations in the cell than GDP) to bind in its place. How would the activity of a G protein be affected by a mutation that reduces the affinity of the α subunit for GDP without significantly changing its affinity for GTP?

The mutant G protein would be constantly active. Each time the α subunit hydrolyzed GTP to GDP, the GDP would dissociate spontaneously, allowing GTP to bind and reactivate the α subunit, especially because the intracellular concentration of GTP is higher than that of GDP. Normally, GDP is tightly bound by the α subunit, which keeps the G protein in its inactive state until interaction with an appropriate activated GPCR stimulates the release of GDP.

Explain why the signal molecules used in neuronal signaling work at a longer range than those used in contact-dependent signaling.

The neurotransmitter released from a neuron in neuronal signaling must diffuse across the synaptic cleft to reach receptors on the target cell. In contrast, in contact-dependent signaling, the signal molecule is attached to the plasma membrane of the signaling cell and interacts with receptors located on the plasma membrane of the receiving cell; thus, the cells must be in direct contact for this type of signaling to occur.

Why is the total count about 36 or 38 ATP rather than a specific number?

The number varies depending on what shuttle transports electrons from NADH in the cytosol.

15-22You are trying to identify the peroxisome-targeting sequence in the thiolase enzyme in yeast. The thiolase enzyme normally resides in the peroxisome and therefore must contain amino acid sequences that are used to target the enzyme for import into the peroxisome. To identify the targeting sequences, you create a set of hybrid genes that encode fusion proteins containing part of the thiolase protein fused to another protein, histidinol dehydrogenase (HDH). HDH is a cytosolic enzyme required for the synthesis of the amino acid histidine and cannot function if it is localized in the peroxisome. You genetically engineer a series of yeast cells to express these fusion proteins instead of their own versions of these enzymes. If the fusion proteins are imported into the peroxisome, the HDH portion of the protein cannot function and the yeast cells cannot grow on a medium lacking histidine. You obtain the results shown in Figure Q15-22. Figure Q15-22What region of the thiolase protein contains the peroxisomal targeting sequence? Explain your answer.

The peroxisomal targeting sequence lies between amino acids number 100 and number 125. Any fusion protein containing this sequence can be targeted for import into the peroxisome (because the yeast cannot grow on a medium lacking histidine), whereas the fusion proteins lacking this region do not target the fusion protein for import into the peroxisome (because the yeast do grow on medium lacking histidine). The most important pieces of data are from the fusion protein containing amino acids 100-200 of the thiolase protein fused to HDH and the fusion protein containing amino acids 1-125 of the thiolase protein fused to HDH. Neither of these fusion proteins allow growth on medium lacking histidine and can be used to define the minimal region necessary for targeting thiolase for import into the peroxisome. (Note that although these experiments show that amino acids 100-125 are necessary, these experiments do not show that this region is sufficient for peroxisomal targeting. It is possible that the region consisting of amino acids 100-125 is sufficient, or it could be that this region collaborates with redundant signals between amino acids 1 and 100 or between amino acids 125 and 200.)

3.59.Figure Q3-59 illustrates the amount of energy per molecule for a population in a contained, controlled environment. Most molecules will have the average energy of the population, shown in region 1. The number of molecules in the population with enough energy to be converted to product is shown in region 2. The number of molecules with enough energy to react in the presence of enzyme is shown in region 3. Use this information to explain how enzymes catalyze reactions.

The presence of enzyme in the mixture of reactant molecules does not change the energy distribution of the population of molecules. Their average energy will remain the same, and there still will be only a very small proportion of the molecules with high energy. Enzyme catalysis increases the total number of molecules that have sufficient energy to participate in the reaction because the total energy required per molecule that reacts is lowered

15-50 If you remove the ER retention signal from a protein that normally resides in the ER lumen, where do you predict the protein will ultimately end up? Explain your reasoning.

The protein would end up in the extracellular space. Normally, the protein would go from the ER to the Golgi apparatus, get captured because of its ER retention signal, and return to the ER. However, without the ER retention signal, the protein would evade capture, ultimately leave the Golgi via the default pathway, and become secreted into the extracellular space. The protein would not be retained anywhere else along the secretory pathway: it presumably has no signals to promote such localization because it normally resides in the ER lumen.

Explain how ETC is used in context with oxidative phosphorylation.

The protons naturally want to diffuse along their concentration gradient, the ETC helps the protons get to a lower energy level so that they can go to the final electron accepter (oxygen) and go through ATP synthase. So the ETC is important in 'breaking the falls' of the electrons so that oxidative phosphorylation could be successful.

Which of the following is true for all exergonic reactions?

The reaction proceeds with a net release of free energy

3.74.c. Coenzyme A can be converted to acetyl CoA, which is an important activated carrier molecule that has a central role in metabolism and can be used to add two carbons in each successive cycle of fatty acid synthesis.What function does the rest of the coenzyme A molecule serve in these reaction pathways?

The rest of the acetyl CoA molecule provides a specific surface recognized by the enzymes that catalyze reactions in which acetyl CoA is required.

3.9.Two college roommates do not agree on the best way to handle the clutter piled up in your dorm room. Roommate 1 explains that chaos is inevitable, so why fight it? Roommate 2 counters that maintaining an organized environment makes life easier in many ways, and that chaos is not inevitable. What law of thermodynamics drives the thinking of roommate 1? What thermodynamic argument can be used to support roommate 2?

The second law of thermodynamics supports Roommate 1's view. It is favorable for a system to become less ordered. However, if the energy used to create an ordered environment in the room is accompanied by enough release of heat, the Universe will become more disordered as the room becomes more organized. Thus, while increasing chaos is inevitable, the room—like a cell—can be kept highly organized, by interconverting types of energy.

Promoter

The site where transcription begins .signals the beginning of transcription

The specialized functions of different membranes are largely determined by the __________________ they contain. Membrane lipids are __________________ molecules, composed of a hydrophilic portion and a hydrophobic portion. All cell membranes have the same __________________ structure, with the __________________ of the phospholipids facing into the interior of the membrane and the __________________ on the outside. The most common lipids in most cell membranes are the __________________. The head group of a glycolipid is composed of __________________. amphipathic hydrophobic phosphatidylserine cholesterol lipid bilayer phospholipids fatty acid tails lipid monolayer proteins glycolipids lipids sterols hydrophilic head groups phosphatidylcholine sugars

The specialized functions of different membranes are largely determined by the proteins they contain. Membrane lipids are amphipathic molecules, composed of a hydrophilic portion and a hydrophobic portion. All cell membranes have the same lipid bilayer structure, with the fatty acid tails of the phospholipids facing into the interior of the membrane and the hydrophilic head groups on the outside. The most common lipids in most cell membranes are the phospholipids. The head group of a glycolipid is composed of sugars.

Intracellular steroid hormone receptors have binding sites for a signaling molecule and a DNA sequence. How is it that the same steroid hormone receptor, which binds to a specific DNA sequence, can regulate different genes in different cell types?

The specific genes regulated in response to an activated steroid hormone receptor depends not only on the genes having the appropriate DNA sequence for binding the receptor but also on a variety of other nuclear proteins that influence gene expression, some of which vary between different cell types.

On the diagram below, draw and label the pigment bands that you observed from the chromatography.

The top pigment is carotene (yellow). The next pigment is xanthophylls (light yellow). The next pigment is chlorophyll a (green). The bottom pigment is chlorophyll b (yellowish-green)

Circle the molecule in each pair that is more likely to diffuse through the lipid bilayer. A. amino acids or benzene B. Cl- or ethanol C. glycerol or RNA D. H2O or O2 E. adenosine or ATP

The two basic properties governing the likelihood of whether a molecule will diffuse through a lipid bilayer are the size of the molecule and the charge of the molecule. A smaller molecule will be more likely to diffuse through the lipid bilayer than a larger molecule. A nonpolar (hydrophobic) molecule will be more likely to diffuse through the lipid bilayer than a polar molecule, which is more likely to diffuse through the lipid bilayer than a charged molecule. A. benzene (small nonpolar versus larger uncharged) B. ethanol (polar versus charged) C. glycerol (small polar versus very large, highly charged) D. O2 (nonpolar versus polar) E. adenosine (polar versus highly charged)

14-49Consider a redox reaction between molecules A and B. Molecule A has a redox potential of -100 mV and molecule B has a redox potential of +100 mV. For the transfer of electrons from A to B, is the ΔG°positive or negative or zero? Under what conditions will the reverse reaction, transfer of electrons from B to A, occur?

The ΔG° is negative. The sign of ΔG° is the opposite of that of ΔE0′ = E0′ (acceptor) - E0′ (donor). The acceptance of electrons by B from A has a ΔE0′ = 100 - (-100) = 200. The reverse reaction, the donation of electrons from B to A, has a positive ΔG° and is therefore unfavorable under standard conditions. Remember that, by definition, the concentrations of A and its redox pair A′ are equal under standard conditions; similarly, the concentration of B is equal to the concentration of its redox pair B′. B will be able to donate electrons to A only when [B] > [B′] and/or [A] < [A′] to such an extent that the ΔG for electron transfer becomes negative.

There are several ways that membrane proteins can associate with the cell membrane. Membrane proteins that extend through the lipid bilayer are called __________________ proteins and have __________________ regions that are exposed to the interior of the bilayer. On the other hand, membrane-associated proteins do not span the bilayer and instead associate with the membrane through an α helix that is __________________. Other proteins are __________________ attached to lipid molecules that are inserted in the membrane. __________________ membrane proteins are linked to the membrane through noncovalent interactions with other membrane-bound proteins. amphipathic hydrophilic noncovalently cortical hydrophobic peripheral covalently integral transmembrane detergent micelle unfolded

There are several ways that membrane proteins can associate with the cell membrane. Membrane proteins that extend through the lipid bilayer are called transmembrane proteins and have hydrophobic regions that are exposed to the interior of the bilayer. On the other hand, membrane-associated proteins do not span the bilayer and instead associate with the membrane through an α helix that is amphipathic. Other proteins are covalently attached to lipid molecules that are inserted in the membrane. Peripheral membrane proteins are linked to the membrane through noncovalent interactions with other membrane-bound proteins.

Your friend is studying mouse fur color and has isolated the GPCR responsible for determining its color, as well as the extracellular signal that activates the receptor. She finds that, on addition of the signal to pigment cells (cells that produce the pigment determining fur color), cAMP levels rise in the cell. She starts a biotech company, and the company isolates more components of the signaling pathway responsible for fur color. Using transgenic mouse technology, the company genetically engineers mice that are defective in various proteins involved in determining fur color. The company obtains the following results. Normal mice have beige (very light brown) fur color. Mice lacking the extracellular signal have white fur. Mice lacking the GPCR have white fur. Mice lacking cAMP phosphodiesterase have dark brown fur. Your friend has also made mice that are defective in the α subunit of the G protein in this signaling pathway. The defective α subunit works normally except that, once it binds GTP, it cannot hydrolyze GTP to GDP. What color do you predict that the fur of these mice will be? Why?

These mice will have dark brown fur. The inability to hydrolyze GTP to GDP will lead to inappropriate activation of the signaling pathway that makes pigment. Too much pigment will be produced, as seen in the mice lacking cAMP phosphodiesterase (which lack the ability to damp the signal), and the mice will end up with dark brown fur.

The cell walls of bacteria, fungi, and plant cells and the extracellular matrix of animal cells are all external to the plasma membrane. Which of the following is a characteristic common to all of these extracellular structures?

They are constructed of polymers that are synthesized in the cytoplasm and then transported out of the cell.

Why do amoeba cells have an internal structure

They are eukaryotic organisms. The cells digest particles and are mobile, so they need to have elaborate structures in order to perform these functions

Which of the following is true regarding GAGs?

They are repeated disaccharide chain that is usually around 80 monosaccharides in length.

What is a function of the membrane proteins called transport proteins?

They regulate the passage of ions

What was the purpose of the treatment with 2-thiouracil plus uracil?

This is a control to determine whether the effects of the base analog can be reversed by adding normal uracil.

14-67In 1925, David Keilin used a simple spectroscope to observe the characteristic absorption bands of the cytochromes that participate in the electron-transport chain in mitochondria. A spectroscope passes a very bright light through the sample of interest and then through a prism to display the spectrum from red to blue. If molecules in the sample absorb light of particular wavelengths, dark bands will interrupt the colors of the rainbow. His key discovery was that the absorption bands disappeared when oxygen was introduced and then reappeared when the samples became anoxic. Subsequent findings demonstrated that different cytochromes absorb light of different frequencies. When light of a characteristic wavelength shines on a mitochondrial sample, the amount of light absorbed is proportional to the amount of a particular cytochrome present in its reduced form. Thus, spectrophotometric methods can be used to measure how the amounts of reduced cytochromes change over time in response to various treatments. If isolated mitochondria are incubated with a source of electrons such as succinate, but without oxygen, electrons enter the respiratory chain, reducing each of the electron carriers almost completely. When oxygen is then introduced, the carriers oxidize at different rates, as can be seen from the decline in the amount of reduced cytochrome (see Figure Q14-67). Note that cytochromes a1and a3cannot be distinguished and thus are listed as cytochrome (a1+ a3). How does this result allow you to order the electron carriers in the respiratory chain? What is their order?

This result allows you to order the electron carriers in the respiratory chain because when oxygen is added, the last carrier in the chain will be oxidized first. This is because oxygen is the final sink for the electrons that flow through the chain, and it participates directly in a redox reaction with the last electron carrier. The wave of oxidation will then proceed backward through the chain toward the first electron carrier in the chain; this is because the oxidation of each carrier will convert it to a form that can accept electrons from the "upstream" carrier in the chain, thereby oxidizing each upstream carrier sequentially. The order of cytochromes in the respiratory chain is the reverse of the order in which they are oxidized (that is, the order in which the reduced form is lost). Listed from first to last, the cytochromes in the chain are b, c1, c, and (a1 + a3).

What was the purpose of the treatment with 2-thiouracil plus uracil?

To prove that the 2-thiouracil is not toxic to the plant

Describe how synaptic signaling is influenced by the action of tranquilizers (such as Valium®) compared to the effects of antidepressants (such as Prozac®).

Tranquilizers promote inhibitory signaling at the synapse. They do this by making GABA-gated channels open more easily in response to the inhibitory neurotransmitter GABA. This channel, when open, allows Cl- to flow into the neuron, making the cell more difficult to depolarize. Antidepressants are used to enhance the neuronal signaling in neurons that have serotonin receptors. Prozac® specifically blocks the reuptake of serotonin in the synaptic cleft. This results in a net increase in serotonin available for excitatory signaling to the postsynaptic neuron.

Desmosomes are made of intermediate filaments and are intended for long-term use. They are meant to resist shear forces and are found in squamous and simple epithelium cells.

True

Due to the intermediate filaments, desmosomes adheres a cell long term to the membrane of its target.

True

Focal adhesions bind microfilaments to anchor protein to integrins to ECM fibrous proteins, while desmosomes bind intermediate filaments to anchor proteins to their own cadherins which bind to adjacent cell's cadherins.

True

Hemidesmosomes bind intermediate filaments to their anchor proteins then to their cadherins which then bind to the neighbor cell's cadherins to form a tight junction.

True

Intermediate filaments are constructed of 8 filaments twisted together. They are formed into strong rope-like filaments to provide cytosol and structural integrity through cell-to-cell junctions, which do not come apart easily.

True

RNA polymerase II of eukaryotes always requires more than one general transcription factor to initiate transcription.

True

The desmosomes bind intermediate filaments to anchor proteins to their own cadherins, which bind to the adjacent cell's cadherins.

True

How many times does the citric acid cycle occur for each molecule of glucose?

Twice

The nontemplate strand of a portion of a gene reads 5′-TTCACTGGTTCA. What is the sequence of the resulting transcript for this portion?

UUCACUGGUUCA

3.63.Enzymes A and B catalyze different reactions, but use the same reactant molecule as a substrate. The graph in Figure Q3-63 presents the reaction rates observed when enzyme A and enzyme B are mixed together in a single test tube containing molecule X. What are the Vmax and the apparent Km values for each enzyme under these conditions? How might these values change for enzyme B if it were analyzed in the absence of enzyme A? Explain your answer.

Under the mixed conditions, enzyme A has a Vmax of 6 and an apparent Km of 1 μM X; enzyme B has a Vmax of 10 and an apparent Km of 3 μM. Because enzyme A has a higher affinity for substrate, it more quickly binds to reactant X and converts it into the product Y, lowering the effective concentration of X reactant available for enzyme B. If enzyme B were tested separately, the Vmax should stay the same, but the Km might be smaller and be a more accurate reflection of the binding affinity of enzyme B for the reactant molecule X.

The DNA contains the nucleotide thymine, but RNA contains _______ instead.

Uracil

How does pH affect the reaction rate and enzymes?

Very low or very high pH values dramatically decrease enzyme activity because it disrupts the hydrogen bonds

The study of enzymes also includes an examination of how the activity is regulated. Molecules that can act as competitive inhibitors for a specific reaction are often similar in shape and size to the enzyme's substrate. Which variable(s) used to describe enzyme activity will remain the same in the presence and absence of a competitive inhibitor?

Vmax

Two solutions are separated by a semipermeable membrane with the same properties as a cell membrane. Solution A is 5% glucose and solution B is 10% glucose. Under these circumstances,

Water will move from solution A to solution B

built a model in which the backbones were antiparallel

Watson

2.34. the relative strengths of a covalent bonds and van der Waals interactions remain the same when tested in a vacuum or in water.However, this is not true of hydrogen bonds or ionic bonds, whose bond strengths are lowered considerably in the presence of water. Explain these observations.

We estimate bond strengths by measuring the amount of energy needed to break them. As explained in Panel 2-7 (p.78-79), in an aqueous solution, water can form hydrogen bonds with any polar molecules that are capably of forming hydrogen bonds with each other. This formation of bonds with water takes away from the net energy that would be gained from the molecules forming hydrogen bonds with each other, as they would in a vacuum. Similarly, water forms favorable electrostatic interactions with ions, thereby greatly weakening the ionic bonds that form between positive and negative ions in a vacuum (see Panel 2-7). Thus, for example, solid table salt (NaCl) readily dissociates in water, producing separate Na+ and Cl- ions as it dissolves. In contrast, covalent bonds and van der Waals attractions have an intrinsic bond strength that is independent of the aqueous environment, because changes in water molecule associations are not involved in the formation of these two types of bonds.

phopolipids have a hydorphilic head that wants to interact with water and a hydrophobic tail that wants to get away from water

What allows phopholibids to easily form membranes?

steroids fats phopholipids

What are the most common types of lipids?

carbon dioxide (nonpolar, small)

What would likely move through the lipid bilayer of the plasma membrane most rapidly?

How do light intensity, carbon dioxide levels, and availability of water affect the stomata?

When CO2 is high and water is low the guard cells lose water closing the stomata.

3.58.a. Consider a description of an enzymatic reaction pathway that begins with the binding of substrate S to enzyme E, and ends with the release of product P from the enzyme. E + S ES EP E + P In many circumstances, Km = [E][S]/[ES] What proportion of enzyme molecules is bound to substrate when [S] = Km?

When [S] is substituted for Km in the equation, it becomes clear that [E] = [ES]. Thus, half of the enzyme molecules are free and half are bound to the substrate.

3.7. in the cytoplasm, materials are organized, separated, and sorted by membranes. cells exploit the selective permeability of these membranes to partition populations of molecules and generate chemical energy for the cell. use the principles of the first and second laws of thermodynamics to explain how membranes can be used to produce chemical energy.

When one type of molecule accumulates at a higher concentration on one side of the membrane, the molecules become "organized" by having their movement limited to the space they occupy. The second law of thermodynamics indicates that, if allowed to do so, the molecules would move across the membrane until there is an equal distribution of molecules on either side. The accumulation of molecules on one side of the membrane represents a store of potential energy. The first law of thermodynamics tells us that this energy will not be lost, but rather converted into a different type of energy. First, when the molecules are then allowed to move across the membrane, this potential energy is converted into kinetic energy. The kinetic energy of molecules moving through a protein channel to cross the membrane is often linked to conformational changes in the protein, promoting specific chemical reactions.

15-65 Fibroblast cells from patients W, X, Y, and Z, each of whom has a different inherited defect, all contain "inclusion bodies," which are lysosomes filled with undigested material. You wish to identify the cellular basis of these defects. The possibilities are: 1.a defect in one of the lysosomal hydrolases 2.a defect in the phosphotransferase that is required for mannose-6-phosphate tagging of the lysosomal hydrolases 3. a defect in the mannose-6-phosphate receptor, which binds mannose-6-phosphate-tagged lysosomal proteins in the trans Golgi network and delivers them to lysosomes When you incubate some of these mutant fibroblasts in a medium in which normal cells have been grown, you find that the inclusion bodies disappear. Because of these results, you suspect that the constitutive exocytic pathway in normal cells is secreting lysosomal hydrolases that are being taken up by the mutant cells. (It is known that some mannose-6-phosphate receptor molecules are found in the plasma membrane and can take up and deliver lysosomal proteins via the endocytic pathway.) You incubate cells from each patient with medium from normal cells and medium from each of the other mutant cell cultures, and get the results summarized in Table Q15-65.Table Q15-65 Indicate which defect (1, 2, 3) each patient (W, X, Y, Z) is most likely to have.

W—3 (defect in mannose-6-phosphate receptor) X—2 (defect in phosphotransferase) Y—1; Z—1 (defect in lysosomal hydrolases); these will be defects in two different lysosomal acid hydrolases A cell that has no mannose-6-phosphate receptor will be able to make all the lysosomal hydrolases properly but will not be able to send them to the lysosome and will also not be able to scavenge hydrolases from the external media. Hence, this cell line cannot be rescued by a culture medium that has had lysosomal hydrolases secreted into it and thus will not be rescued by any of the media tested here. A cell line that has no phosphotransferase will be able to scavenge hydrolases from the external medium, but because all of the cell's own hydrolases will lack the mannose-6-phosphate tag, it will be rescued only by medium from a cell line that is able to make all of the hydrolases. Cell lines lacking one hydrolase will be rescued by medium from any cell line that is able to secrete that hydrolase in a mannose-6-phosphate-tagged form; in addition, media from cultures of cells lacking a hydrolase will rescue any cell line with another type of defect.

3.58.b.Consider a description of an enzymatic reaction pathway that begins with the binding of substrate S to enzyme E, and ends with the release of product P from the enzyme. E + S ES EP E + P In many circumstances, Km = [E][S]/[ES] Recall that when [S] = Km, the reaction rate is ½Vmax. Does your answer to part A make sense in the light of this rate information?

Yes. If half of the enzyme molecules are bound to the substrate, it makes intuitive sense that the reaction rate is half of the maximum possible rate, or half of the rate observed when all of the enzyme molecules are bound to the substrate.

In step 7 of the citric acid cycle, fumarase catalyzes the addition of a water molecule to a carbon-carbon double bond (see Panel 13-2). Can this be considered an oxidation reaction? Explain your answer.

Yes. In the citric acid cycle, the overall process oxidizes carbon molecules to produce carbon dioxide. Although, fumarase does not directly remove electrons from its substrate (as do the enzymes that catalyze steps 3, 4, 6, and 8), the addition of water across the double bond in fumarate leaves one of its carbon atoms in a more oxidized state. This is because one of the carbons becomes bonded to an oxygen atom. In this arrangement, the carbon atom shares its electrons unequally across the new bond (see Figure 3-11).

68. Your friend has just returned from a deep-sea mission and claims to have found a single-celled life-form. He believes this new life-form may not have descended from the common ancestor that all types of life on Earth share. You are convinced that he must be wrong, and you mange to extract DNA from the cells he has discovered. He says that the mere presence of DNA is not enough to prove the point: his cells might have adopted DNA as a useful molecule quite independently of all other known life-forms. What could you do to provide additional evidence to support our argument?

You could use modern technology to discover the sequence of the DNA. if you are right, you would expect to find parts of this sequence that are unmistakably similar to corresponding sequences in other, familiar, living organisms; it would be highly improbably that such similar sequences would have evolved independently. You could, of course, also analyze other feature of the chemistry of his cells; for examples, do they contain proteins made of the same set of 20 amino acids? This could all be supporting evidence that this newly discovered species arose from the same common ancestral cells as all other life on earth.

Actin-binding proteins bind to actin and can modify its properties. You purify a protein, Cap1, that seems to bind and cap one end of an actin filament, although you do not know whether it binds the plus end or the minus end. To determine which end of the actin filament your protein binds to, you decide to examine the effect of Cap1 on actin polymerization by measuring the kinetics of actin filament formation in the presence and the absence of Cap1 protein. You obtain the following results (see Figure Q17-48). Do you think Cap1 binds the plus end or the minus end of actin? Explain your reasoning.

You would predict that Cap1 binds the plus end of actin, because it seems to inhibit actin polymerization. Actin filaments grow through the addition of monomers to the plus end of the actin filament. A capping protein that binds the plus end of actin can block monomer addition to the actin filament. Thus, less actin polymerization will be seen in the presence of the Cap1 protein.

Rank the following types of cell signaling from 1 to 4, with 1 representing the type of signaling in which the signal molecule travels the least distance and 4 the type of signaling in which the signal molecule travels the largest distance. ______ paracrine signaling ______ contact-dependent signaling ______ neuronal signaling ______ endocrine signaling

____3__ paracrine signaling ____1__ contact-dependent signaling ____2__ neuronal signaling ____4__ endocrine signaling

2.62. each nucleotide in DNA and RNA has an aromatic base. what is the principal force that keeps the bases in a polymer from interacting with water? a. hydrophobic interactions b. hydrogen bonds c. covalent bonds d. van der waal interactions

a

2.64. there are 20^100 different possible sequence combinations for a protein chain with 100 amino acids. in addition to the amino acid sequence of a protein, what other factors increase the potential for diversity in these macromolecules? a. free rotation around single bonds during synthesis b. noncovalent interactions sampled as protein folds c. the directionality of amino acids being added d. the planar nature of the peptide bond

a

3.34For the reaction YX at standard conditions with [Y] = 1 M and [X] = 1 M, ΔG is initially a large negative number. As the reaction proceeds, [Y] decreases and [X] increases until the system reaches equilibrium. How do the values of ΔG and ΔG° change as the reaction equilibrates? (a) ΔG becomes less negative and ΔG° stays the same. (b) ΔG becomes positive and ΔG° becomes positive. (c) ΔG stays the same and ΔG° becomes less negative. (d) ΔG reaches zero and ΔG° becomes more negative.

a

3.38. Isomerization of glucose 1-phosphate to glucose 6-phosphate is energetically favorable. At 37°C, ΔG° = -1.42 log10K. What is the equilibrium constant for this reaction if ΔG° = -1.74 kcal/mole at 37°C? (a) 16.98 (b) 0.09 (c) -0.09 (d) 0.39

a

3.62. What information regarding an enzyme-catalyzed reaction is obtained in a plot of the inverse of the initial velocities against the inverse of the corresponding substrate concentrations? (a) 1/Vmax and 1/Km (b) 1/V and 1/[S] (c) Vmax and Km (d) V and [S]

a

3.71. The synthesis of glutamine from glutamic acid requires the production of an activated intermediate followed by a condensation step that completes the process. Both amino acids are shown in Figure Q3-71. Figure Q3-71 Which molecule is added to glutamic acid in the activation step? (a) phosphate (b) NH3 (c) ATP (d) ADP

a

In step 4 of glycolysis, a six-carbon sugar (fructose 1,6-bisphosphate) is cleaved to produce two three-carbon molecules (dihydroxyacetone phosphate and glyceraldehyde 3-phosphate). Which enzyme catalyzes this reaction? (a) aldolase (b) phosphoglucose isomerase (c) enolase (d) triose phosphate isomerase

a

In step 4 of the citric acid cycle, the reduction of NAD+ to NADH is coupled to the generation of CO2 and the formation of a high-energy thioester bond. The energy of the thioester bond is harnessed in step 5. What is the energy used for? Page 13 of 27 (a) to generate a molecule of GTP (b) to generate a molecule of ATP (c) to generate a proton gradient (d) to generate a molecule of NADH

a

Select the best option to fill in the blanks of the following statement: Fermentation is a/an _____________________ process that converts _____________ into carbon dioxide and _____________________. (a) anaerobic, pyruvate, ethanol (b) anaerobic, lactate, ethanol (c) eukaryotic, glyceraldehyde 3-phosphate, ethanol (d) prokaryotic, lactate, propanol

a

Several different classes of enzymes are needed for the catabolism of carbohydrates. Which of the following descriptions best matches the function of an isomerase? (a) An enzyme that catalyzes the rearrangement of bonds within a single molecule. (b) An enzyme that catalyzes a change in the position of a specific chemical group within a single molecule. (c) An enzyme that catalyzes the oxidation of a molecule by removing a hydride ion. (d) An enzyme that catalyzes the addition of phosphate groups to other molecules.

a

The citric acid cycle is a critical sequence of reactions for energy production, which take place in the matrix of the mitochondria. The reaction cycle requires materials from the cytosol to be converted into acetyl CoA, which represents the starting point of a new cycle. Which of the following statements about acetyl CoA is true? (a) Amino acids can be converted into acetyl CoA. (b) Pyruvate is converted into acetyl CoA in the cytosol. (c) Triacylglycerol molecules are transported into the mitochondrial matrix and cleaved by lipases to produce acetyl CoA. (d) Oxaloacetate is converted directly into acetyl CoA to feed the citric acid cycle.

a

The citric acid cycle is a series of oxidation reactions that removes carbon atoms from substrates in the form of CO2. Where do the oxygen atoms in the carbon dioxide molecules come from? (a) water (b) phosphates (c) molecular oxygen (d) acetyl CoA

a

The conversion of glyceraldehyde 3-phosphate to 1,3 bisphosphoglycerate in step 6 of glycolysis generates a "high energy" phosphoanhydride bond. Which of the following best describes what happens to that bond in step 7? (a) It is hydrolyzed to drive the formation of ATP. (b) It is hydrolyzed to drive the formation of NADH. (c) It is hydrolyzed to generate pyruvate. (d) It is oxidized to CO2.

a

The oxygen-dependent reactions required for cellular respiration were originally thought to occur in a linear pathway. By using a competitive inhibitor for one enzyme in the pathway, investigators discovered that these reactions occur in a cycle. What compound served as the inhibitor? (a) malonate (b) malate Page 14 of 27 (c) fumarate (d) succinate

a

When glucose is being used up and not replaced from food intake, the blood sugar level can be maintained by synthesizing glucose from smaller molecules such as pyruvate or lactate. This process is called gluconeogenesis. Which organ is principally responsible for supplying glucose to the rest of the body when glucose reserves are low? (a) liver (b) pancreas (c) spleen (d) gall bladder

a

10. select the option that correctly finishes the following statement: "a cell's genome contains all of ...."

a cell's dna

Initiation

a complex is formed between an mRNA molecule,the first tRNA,and ribosomal subunits.

When nucleotides polymerize to form nucleic acid ______

a covalent bond forms between the sugar of one nucleotide and the phosphate of a second

The partial negative charge at one end of the water molecule is attracted to the partial positive charge of another water molecule. What is this attraction called?

a hydrogen bond

Synthesis of which new strand(s) would be affected by mutations in the enzyme DNA polymerase I?

a lagging strand

Cellulose is

a major structural component of plant cell walls

Inborn error of metabolism

a mutation in a gene that is inherited from one or both parents

A glycosidic linkage is analogous to which of the following proteins?

a peptide bond

Which of the following is NOT a component of each monomer used to make proteins?

a phosphorous atom, P

Which one of the following is NOT a component of each monomer used to make proteins? a carboxyl group, COOH a phosphorus atom, P a side chain, R an amino functional group, NH2

a phosphorus atom, P

Sigma factor

a protein that binds to RNA polymerase.Also it recognizes the base sequence of promoter ans binds there.

A promoter is ______. one or more eukaryotic proteins that bind to DNA near the start of a gene a sequence in RNA that promotes the release of RNA polymerase from DNA a protein that associates with bacterial RNA polymerase to help it bind to DNA a sequence in DNA that brings RNA polymerase near the site for transcription

a sequence in DNA that brings RNA polymerase near the site for transcription

If a mutation eliminated the start codon from a gene,how would the mutation affect transcription,and how would it affect translation?

a structural gene would still be transcribed into RNA if the start codon was missing.however it would not be translated properly.

microtubule-associated proteins (MAPs)

account for 10-15% of microtubules

type of adhesive junction containing actin microfilaments

adherens junction

a type of adhesive junction connecting an epithelial cell to the basal lamina

adhesion belt

collection of adherens junctions particularly associated with epithelial cells

adhesion belt

Cadherins are associated with ________ junctions

adhesive

Focal contact is associated with which of the following type of cellular junction? A) adhesive junction B) gap junction C) plasmodesmata D) tight junctions E) all of the above

adhesive junction

anchors cytoskeleton of one cell with that of a neighboring cell

adhesive junction

aerobic and anaerobic

aer- O2 - glucose oxididized into 6 CO2 and water + 36 ATP ana- no O2 - glucose = lactate, no CO2 or 2 ATP

The process of generating monoclonal antibodies is labor-intensive and expensive. An alternative is to use polyclonal antibodies. A subpopulation of purified polyclonal antibodies that recognize a particular antigen can be isolated by chromatography. Which type of chromatography is used for this purpose?

affinity

61. genes that have homologs in a variety of species have been discovered through the analysis of genome sequences. in fact, it is not uncommon to find a family of homologous genes encoding proteins that are unmistakably similar in amino acid sequence in organisms as diverse as budding yeast, archaea, plants, and humans. even more remarkably, many of these proteins can substitute functionally for their homologs in other organisms. explain what it is about the origins of cells that makes it possible for proteins expressed by homologous genes to be functionally interchangeable in different organisms.

all living beings on Earth (and thus, all cells) are thought to be derived from a common ancestor. solutions to many of the essential challenges that face a cell (such as the synthesis of proteins, lipids, and DNA) seem to have been achieved in this ancient common ancestor. the ancestral cell therefore possessed sets of proteins to carry out these essential functions. many of the essential challenges facing modern-day cells are the same as those facing the ancestral cell, and the ancient solutions are often still effective. thus, it is not uncommon for organisms to use proteins usually show some species-specific diversification they still retain the basic biochemical characteristics of the ancestral protein. for example, homologous proteins often retain their ability to interact with a specific protein target, even in cells of diverse species. because the basic biochemical characteristics are retained, homologous proteins are often capable of functionally substituting for one another.

There is only one start codon, AUG. This means that all newly-made polypeptides have a methionine at their amino end. all newly-made polypeptides have a methionine at their carboxyl end. the first tRNA will have the anticodon loop 3'-AUG-5'. the 5' end of an mRNA must start with an A. the stop codons each have their own special tRNAs.

all newly-made polypeptides have a methionine at their amino end.

Alternative splicing

allows a signle gene to encode two or more polypetides with differences in their amino acid sequences/ allows complex eukaryotic species to use the same gene to make different proteins at different stages of development or in different cell types.

Selectively permeable membrane

allows some particles to pass through it and restricts the movement of others

a specific gene is known to code for three different but related proteins. this could be due to what?

alternative RNA splicing

What is the enzyme that catalyzes that attachment of amino acids to tRNA?

aminoacyl-tRNA synthetases

Nitrogen is more electronegative than hydrogen. Which of the following is a correct statement about the atoms in ammonia (NH3)

ammonia has an overall positive charge

2.55. cholesterol is an essential component of biological membranes. although it is much smaller than the typical phospholipids and glycolipids in the membrane, it is a BLANK molecule, having both hydrophilic and hydrophobic regions

amphipathic

Choose the answer that best fits the following statement: Cholesterol is an essential component of biological membranes. Although it is much smaller than the typical phospholipids and glycolipids in the membrane, it is a(n) _________________ molecule, having both hydrophilic and hydrophobic regions.

amphipathic

43) Phosphorylation cascades involving a series of protein kinases are useful for cellular signal transduction because they ________.

amplify the original signal many times

Gene

an organized unit of DNA sequences that enables a segment of DNA to be transcribed into RNA and ultimately results in the formation of a functional product.

3.2. when there is an excess of nutrients available in the human body, insulin is released to stimulate the synthesis of glycogen from glucose. this is a specific example of BLANK process, a general process in which larger molecules are made from smaller molecules

anabolic

When there is an excess of nutrients available in the human body, insulin is released to stimulate the synthesis of glycogen from glucose. This is a specific example of a(n) __________ process, a general process in which larger molecules are made from smaller molecules.

anabolic

Select the best option to fill in the blanks of the following statement: Fermentation is a/an _____________________ process that converts _____________ into carbon dioxide and _____________________.

anaerobic, pyruvate, ethanol

Which of the following is not a function associated with the cytoskeleton?

anchoring site for mRNA during transcription

36.b. what cell type has a nucleus?

animal, plant

36.g. what cell type has mitochondrion?

animal, plant

2.68. you are trying to make a synthetic copy of a particular protein but accidentally join the amino acids together in exactly the reverse order. one of your classmattes says the two proteins must be identical, and bets you $20 that your synthetic protein will have exactly the same biological activity as the original. after having read this chapter, you have no hesitation in staking your $20 that it won't. what particular feature of a polypeptide chain makes you sure your $20 is safe and that project must be done again.

as a peptide bond has a distinct chemical polarity, a polypeptide chain also has a distinct polarity. the reversed protein chain cannot make the same noncovalent inerations during folding and thus will not adopt the same three-dimensional structure as the original protein. the activities of these two proteins will definitely be different, because the activity of a protein depends on its 3d structure. it is unlikely that the reverse chain will fold into any well-defined, and hence functionally useful, structure at all, because it has not passed the stringent selective pressures imposed during evolution.

2.40. the amino acid histidine is often found in enzymes. depending on the pH of its environment, sometimes histidine is neutral and at other times it acquires a proton and becomes positively charged. consider an enzyme with a histidine side chain that is known to have an important role in the function of the enzyme. it is not clear whether this histidine is required in its protonated or its unprotonated state. which form of histidine is necessary for the active enzyme?

assuming that the change in enzyme activity is due to the change in the protonantion state of histidine, the enzyme must require histidine in the protonated, charged state. the enzyme is active only at low, acidic pH, where the proton (or hydronium ion) concentration is high; thus, the loss of a proton from histidine will be disfavored so that histidine is likely to be protonated

2.63. because there are four different monomer building blocks that can be used to assemble RNA polymers, the number of possible sequence combinations that can be created for an RNA molecule made of 100 nucleotides is... a. 100^4 b. 4^100 c. 4*100 d. 100/4

b

3.48.Which of the following statements would not be true of a favorable binding equilibrium? (a) The free-energy change is negative for the system. (b) The concentration of the complex remains lower than the concentration of the unbound components. (c) The complex dissociation rate is slower than the rate for component association. (d) The binding energy for the association is large and negative.

b

In the final step of the citric acid cycle, oxaloacetate is regenerated through the oxidation of malate and this is coupled with the production of which other molecule? (a) FADH (b) NADH (c) GTP (d) CO2

b

The template strand of a given gene includes the sequence 3′-GCCACGTATCA-5′ What is the sequence of the nontemplate strand? a)3′-CGGTGCATAGT-5′ b)5′-CGGTGCATAGT-3′ c)5′-CGGUGCAUAGU-3′ d)3′-CGGUGCAUAGU-5′

b

Fatty acids can easily be used to generate energy for the cell. Which of the following fatty acids will yield more energy? Explain your answer. (a) CH3-CH2-CH2-CH2-CH2-CH2-CH2-CH=CH-COOH (b) CH3-CH2-CH2-CH2-CH2-CH2-CH2-CH2-CH2-COOH (c) CH3-CH=CH-CH2-CH2-CH2-CH2-CH=CH-COOH (d) CH3-CH2-CH2-CH2-CH2-CH2-CH2-COOH

b) This fatty acid has 10 carbons [as opposed to 8 in (d)] and has a completely saturated hydrocarbon tail [options (a) and (c) are monounsaturated and diunsaturated, respectively). The total number of carbons determines how many acetyl CoA molecules can be derived from it; in this case it is 5. The fact that all hydrocarbon bonds are saturated means that an FADH2 molecule will be generated in the first step of the fatty acid cycle, converting a carbon-carbon single bond into a carbon-carbon double bond. If that bond is already reduced, the cycle begins at the second step. As a result, FADH2 is not generated in that cycle, lowering the total number of cofactors that can be used by the electron-transport chain to generate a proton gradient.

14-40Which of the following statements about "redox potential" is true? (a)Redox potential is a measure of a molecule's capacity to strip electrons from oxygen. (b)For molecules that have a strong tendency to pass along their electrons, the standard redox potential is negative. (c)The transfer of electrons from cytochrome c oxidase to oxygen has a negative redox potential. (d)A molecule's redox potential is a measure of the molecule's capacity to pass along electrons to oxygen.

b. For molecules that have a strong tendency to pass along their electrons, the standard redox potential is negative.

2.58.c. the nitrogen-containing aromatic ring is called

base

2.69.a. choose the class of amino acids most important for the interaction: forming ionic bonds with negatively charged DNA

basic

Which of the following can be found in parallel and anti-parallel conformations?

beta β sheets

The equilibrium constant for complex formation between molecules A and B will depend on their relative concentrations, as well as the rates at which the molecules associate and dissociate. The association rate will be larger than the dissociation rate when complex formation is favorable. The energy that drives this process is referred to as ___________.

binding energy.

In the extracellular matrix of animal cells, collagen and elastin fibers are embedded with this matrix. A) glycoaminoglycans B) proteoglycans C) pectins D) lignins E) both choices A and B

both choices A and B

Synthesis of which new strand(s) would be affected by mutations in the enzyme primase?

both leading and lagging strands

13. f.employs a light microscope and requires that samples be fixed and stained in order to reveal cellular details

bright-field

In anaerobic conditions, skeletal muscle produces _____________. (a) lactate and CO2. (b) ethanol and CO2. (c) lactate only. (d) ethanol only.

c

In step 1 of the citric acid cycle, citrate is generated by the enzyme citrate synthase. The enzyme combines the two-carbon acetyl group from acetyl CoA and the four-carbon oxaloacetate. What is the source of energy that drives this reaction forward? (a) a high-energy phosphodiester bond (b) a transfer of high-energy electrons (c) a high-energy thioester bond (d) the heat of molecular collision

c

In step 4 of the citric acid cycle, the reduction of NAD+ to NADH is coupled to the generation of CO2 and the formation of a high-energy thioester bond. Which molecule provides the sulfhydryl group necessary to form the thioester bond? (a) pyruvate (b) acetyl CoA (c) CoA (d) cysteine side chain in the catalytic pocket

c

Several different classes of enzymes are needed for the catabolism of carbohydrates. Which of the following descriptions best matches the function of a dehydrogenase? (a) An enzyme that catalyzes the rearrangement of bonds within a single molecule. (b) An enzyme that catalyzes a change in the position of a specific chemical group within a single molecule. (c) An enzyme that catalyzes the oxidation of a molecule by removing a hydride ion. (d) An enzyme that catalyzes the addition of phosphate groups to other molecules.

c

The reaction cycle that uses acetyl CoA to generate electron carrier molecules needed in the electron-transport chain is important for powering the cell. Which of the names below is not one of those commonly used to describe this reaction cycle? (a) tricarboxylic acid cycle (b) Krebs cycle (c) oxaloacetic acid cycle (d) citric acid cycle

c

Which of the following components does not form part of the transcription initiation complex at a eukaryotic promoter? a)TATA box b)start point c)transfer RNA d)transcription factors e)RNA polymerase

c

Which of the following statements comparing prokaryotic and eukaryotic transcription is not true? a)Part of the RNA polymerase specifically recognizes and binds to the promoter in bacteria; in eukaryotes, transcription factors mediate the binding of polymerase. b)The RNA transcript is immediately usable as mRNA; in eukaryotes, the RNA transcript is processed. c)RNA polymerase requires a primer in bacteria; in eukaryotes, it does not. d)Eukaryotic transcription has a polyadenylation signal sequence; in prokaryotes, a transcription terminator causes the polymerase to detach and release the transcript.

c

3.68.You are studying a biochemical pathway that requires ATP as an energy source. To your dismay, the reactions soon stop, partly because the ATP is rapidly used up and partly because an excess of ADP builds up and inhibits the enzymes involved. You are about to give up when the following table from a biochemistry textbook catches your eye. Figure Q3-68 Which of the following reagents are most likely to revitalize your reaction? (a) a vast excess of ATP (b) glucose 6-phosphate and enzyme D (c) creatine phosphate and enzyme A (d) pyrophosphate

c) An excess of ATP will initially restore the reactions, but as ATP is hydrolyzed, ADP will build up and inhibit the enzymes again. Pyrophosphate does not look like ATP and is therefore unlikely to be used by the enzymes as an alternative energy source. Pyrophosphate + enzyme D will just heat things up. What you need is a high-energy source of phosphate that can convert ADP back to ATP. Because the ΔG° of the reaction ATP + creatine ADP + creatine phosphate catalyzed by enzyme A is greater than zero, the addition of creatine phosphate and enzyme A can be used to form ATP from ADP, regenerating the ATP while also forming creatine as a waste product.

The advantage to the cell of the gradual oxidation of glucose during cellular respiration compared with its combustion to CO2 and H2O in a single step is that ________________. (a) more free energy is released for a given amount of glucose oxidized. (b) no energy is lost as heat. (c) energy can be extracted in usable amounts. (d) more CO2 is produced for a given amount of glucose oxidized.

c) The amount of free energy released by glucose oxidation is the same as combustion, the amount of CO2 released is the same, and some of the energy released is still lost as heat, whatever the route.

3.54.Protein E can bind to two different proteins, S and I. The binding reactions are described by the following equations and values: E + S ES Keq for ES = 10 E + I EI Keq for EI = 2 Given the equilibrium constant values, which one of the following statements is true? (a) E binds I more tightly than S. (b) When S is present in excess, no I molecules will bind to E. (c) The binding energy of the ES interaction is greater than that of the EI interaction. (d) Changing an amino acid on the binding surface of I from a basic amino acid to an acidic one will probably make the free energy of association with E more negative.

c.Choice (c) is true. The binding energy is the standard free energy of the binding reaction, and thus is proportional to ln Keq. As the binding energy increases, the equilibrium constant for the association reaction becomes larger. Choices (a) and (b) are false, because although E binds S more tightly than it does I, some E molecules will still be bound to I molecules. Choice (d) is false; although not enough information is given to be certain, it is more likely that binding would be weakened by this change, making the free energy of association more positive.

why does the incidence of cancer increase drastically with age?

cancer develops after multiple mutations have occurred which takes years to accumulate

3.1. chemical reactions carried out by living systems depend on the ability of some organisms to capture and use atoms from nonliving sources in the environment. the specific subset of these reactions that break down nutrients in food can be described as...

catabolic

Which proteins is associated with the precise control of where microtubules form and depolymerize?

catastrophins

1. living systems are incredibly diverse in size, shape, environment, and behavior. It is estimated that there are between 10 million and 100 million different species. Despite this wide variety of organisms, it remains difficult to define what is alive. Which of the following can be described as the smallest living unit?

cell

Despite the differences between eukaryotic and prokaryotic cells, prokaryotes have proteins that are distantly related to eukaryotic actin filaments and microtubules. What is likely to be the most ancient function of the cytoskeleton?

cell division

specialized modifications of the cell membrane in which two cells come together in multicellular organisms

cell junctions

3rd is a green line

chlorophyll a

24.c. In contrast, the BLANK is a type of organelle found only in the cells of plants and algae, and performs photosynthesis.

chloroplasts

Thylakoids, DNA, and ribosomes are all components found in ____

chloroplasts

You are investigating a clinical disorder in one of your patients. You discover that the patient has an autoimmune disease that targets and destroys fibroblasts. This disorder would most likely affect A) fiber production by the cells, as occurs with blood clotting. B) annulus formation. C) hyluronate production in the joints. D) collagen production. E) all of the above

collagen production

Elodea

common aquatic plant

13. a. a BLANK scans the specimen with a focused laser beam to obtain a series of two-dimensional optical sections, which can be used to reconstruct an image of the specimen in three dimensions the laser excites a fluorescent dye molecule, and the emitted light from each illuminated point is captured through a pinhole and recorded by a detector

confocal

LDL (low density)

considered bad cholesterol, builds up on arteries

HDL (high density)

considered good cholesterol, helps remove the bad cholesterol from arteries

The difference between the sugar in DNA and the sugar in RNA is that the sugar in DNA _____

contains one less oxygen atom

2.66. macromolecules in the cell can often interact transiently as a result of noncovalent interactions. these weak interactions also produce stable, highly specific interactsion between molecules. which of the factors below is the most significant in determining whether the interaction will be transient or stable? a. the size of each molecule b. the concentration of each molecule c. the rate of synthesis d. surface complementarity between molecules

d

3.45.If proteins A and B have complementary surfaces, they may interact to form the dimeric complex AB. Which of the following is the correct way to calculate the equilibrium constant for the association between A and B? (a) kon/koff = K (b) K = [A][B]/[AB] (c) K = [AB]/[A][B] (d) (a) and (c)

d

3.61. The maximum velocity (Vmax) of an enzymatic reaction is an important piece of information regarding how the enzyme works. What series of measurements can be taken in order to infer the maximum velocity of an enzyme-catalyzed reaction? (a) the rate of substrate consumption after the system reaches equilibrium, for several reactant concentrations (b) the rate of product consumption shortly after mixing the enzyme and substrate (c) the rate of substrate consumption at high levels of enzyme concentration (d) the rate of substrate consumption shortly after mixing the enzyme and substrate, for several substrate concentrations

d

3.65. Activated carriers are small molecules that can diffuse rapidly and be used to drive biosynthetic reactions in the cell. Their energy is stored in a readily transferable form such as high-energy electrons or chemical groups. Which of the molecules below is the most widely used activated carrier? (a) FADH2 (b) NADH (c) NADPH (d) ATP

d

3.72. The synthesis of glutamine from glutamic acid requires the production of an activated intermediate followed by a condensation step that completes the process. Both amino acids are shown in Figure Q3-72. Figure Q3-72 In the condensation step, _______________ is displaced by ________________. (a) OH, NH3. (b) ADP, NH2. (c) ATP, NH3. (d) phosphate, NH3.

d

3.73. NADH and NADPH are activated carrier molecules that function in completely different metabolic reactions. Both carry two additional ________ and one additional _____________. This combination can also be referred to as a hydride ion. (a) protons, electron. (b) electrons, phosphate. (c) hydrogens, electron. (d) electrons, proton.

d

In step 3 of the citric acid cycle, the oxidation of isocitrate and the production of CO2 are coupled to the reduction of NAD+, generating NADH and an α-ketoglutarate molecule. In the isocitrate molecule shown in Figure Q13-47, which carbon is lost as CO2 and which is converted to a carbonyl carbon? (a) 4 and 6 (b) 6 and 5 (c) 5 and 4 (d) 6 and 4

d

In the transcription process shown here, in which process order (1, 2, or 3) does the RNA polymerase bind to the promoter, in which is the RNA transcript released, and in which is the RNA transcript extended? a)1, 2, 3 b)2, 3, 1 c)3, 2, 1 d)1, 3, 2

d

The conversion of fructose 1,6-bisphosphate to fructose 6-phosphate is catalyzed by a fructose 1,6-bisphosphatase and is one of the final steps in gluconeogenesis. Which of the following molecules is an allosteric activator of this enzyme? (a) Pi (b) AMP (c) ADP (d) ATP

d

The oxygen-dependent reactions required for cellular respiration were originally thought to occur in a linear pathway. By using a competitive inhibitor for one enzyme in the pathway, investigators discovered that these reactions occur in a cycle. Which enzyme was inhibited? (a) aconitase (b) isocitrate dehydrogenase (c) malate dehydrogenase (d) succinate dehydrogenase

d

The functional product of a structural gene is a.tRNA b.mRNA c.rRNA d.a polypeptide e.a,b,and c.

d.a polypeptide

The movement of polypeptide from the tRNA in the p Site to the tRNA in the A site is referred to as a.peptide bonding b.aminoacyl binding c.translation d.the peptidyl transfer reaction e.elongation

d.the peptidyl transfer reaction

Gametes

develop in the multicellular haploid gametophyte

There are 20 different amino acids. What makes one amino acid different from another? different asymmetric carbons different structural and optical isomers different side chains (R-groups) attached to a carboxyl carbon different side chains (R-groups) attached to an α carbon different side chains (R-groups) attached to the amino groups

different side chains (R-groups) attached to an α carbon

According to the current model, which of the following is the correct sequence of microtubule assembly?

dimers, oligomers, protofilaments, sheets of protofilaments, closing of microtubule, elongating microtubule

52. many of the mechanisms that cells use for maintenance and reprodution were first studied at the molecular level in bacteria. which bacterial species had a central role in advancing the field of molecular biology?

e. coli

NADH and NADPH are activated carrier molecules that function in completely different metabolic reactions. Both carry two additional ________ and one additional _____________. This combination can also be referred to as a hydride ion.

electrons, proton.

2.31. although covalent bonds are 10-100 times stronger than noncovalent interactions, many biological processes depend upon the number and type of noncovalent interactions between molecules. what noncovalent bond interaction will contribute most to the strong and specific binding of two molecules, such as a pair of proteins?

electrostatic attractions

39. the cell constantly exchanges materials by bringing nutrients in from the external environment and shuttling unwanted by-products back out. which term describes the process by which external materials are captured inside vesicles and brought into the cell?

endocytosis

30. mitochondria contain their own genome, are able to duplicate, and actually divide on a different time line from the rest of the cell. nevertheless, mitochondia cannot function for long when isolated from the cell because they are...

endosymbionts

3.3.The second law of thermodynamics states that the disorder in any system is always increasing. In simple terms, you can think about dropping NaCl crystals into a glass of water. The solvation and diffusion of ions is favored because there is an increase in _____________.

entropy

isotonic

environment same as the cell; no net movement, cell is stable

Below is a list of molecules with different chemical characteristics. Knowing that all molecules will eventually diffuse across a phospholipid bilayer, select the option below that most accurately predicts the relative rates of diffusion of these molecules (fastest to slowest). alanine estrogen propanol sodium

estrogen > propanol > alanine > sodium

The RNA segments joined to one another by spliceosomes are _____. caps exons snRNPs tails introns

exons

muscle cells differ from nerve cells mainly because they

express different genes

pigmented cups

eyespots, respective photoreceptors to receive light from one side only

Which of the following terms does not directly pertain to translation? a)anticodon b)tRNA c)aminoacyl-tRNA synthetase d)polypeptide e)rRNA f)DNA

f

2.19.c. true or falseL atoms with unfilled outer electron shells are especially stable and are therefore less reactive

false, atoms that have their outer electron shells filled are the most stable and least reactive. atoms with unfilled outer shells are more reactive because they tend to share or transfer electrons.

2.35.a. true or false: any covalently bonded H atom can participate in a hydrogen bond if it comes in close proximity with an oxygen atom that forms part of a water molecule

false, hydrogen atoms that are covalently bonded to carbon atoms do not participate in hydrogran bonds because these hydrogens have almost no net postive charge

3.8.b. true or false: The ultimate source of energy for living systems is chlorophyll.

false, the ultimate source of energy for living organisms is sunlight

2.52.a. true or false: a disaccharide consists of a sugar covalently linked to another molecule such as an amino acid or nucleotide.

false. a disaccharide consists of two sugar molecules that undergo a condensation reaction to form a covalent bond

46. a.true or false: plants do not require a cytoskeleton because they have a cell wall that lend structure and support to the cell

false. although plant cells do have a cell wall that lends structure and support, they still need a cytoskeleton, which also helps with connections between cells and the transport of vesicles inside the cell

2.2.a. true or false:The chemistry of life is carried out and coordinated primarily by the action of small molecules.

false. although small molecules are important in many processes, the chemical reactions in living systems are regulated by the coordinated action of large polymeric molecules

2.24.b. true or false: copper, zinc, and manganese are among 11 nonessential trace elements that contribute less than 0.1% of all the atoms in the human body

false. cu, zn, and mn are essential trace elements in the body.

2.15.b. true or false: H,C,O,N are the most common elements in biological molecules because they are most stable

false. h,c,n,o are the most common elements in biological molecules because their outer shells are unfilled, making them highly reactive

32.a. true or false: the number of mitochondria inside a cell remains constant over the life of the cell.

false. in plants, only mitochondria perform cellular respiration (using oxygen to break down organic molecules to produce carbon dioxide) just as in animal cells. Chloroplasts perform photosynthesis in which water molecules are split to generate oxygen and fix carbon dioxide molecules.

2.a. True or False: the paramecium is a multicellular microorganism covered with hairlike cilia

false. it is a single-celled organism

35.c. true or false: lysosomes are small organelles where fatty acid synthesis occurs.

false. lysosomes house enzymes that break down nutrients for use by the cell and help recycle materials that cannot be used, which will later be excreted from the cell

2.24.c. true or false: approximately .9% of the atoms in the human body come from seven essential elements- Na, Mg, K, Ca, P, S, and Cl- all of which form stable ions in aqueous solution.

false. na, mg, k, ca, and cl form ions in aqeuous solution, but P and S form covalent bonds in order to fill their outer shells

21.c. true or false: photosynthetic bacteria contain chloroplasts

false. photosynthetic bacteria have enzyme systems similar to those found in chloroplasts, which allow them to harvest light energy to fix carbon dioxide

3.49.d. true or false:The diffusion rates for smaller molecules in the cytosol are much lower than what is observed for the same molecules in water.

false. small molecules diffuse through the cytosol nearly as rapidly as they diffuse in water

48.a. true or false: primitive plant, animal, and fungal cells probably acquired mitochondria after they diverged from a common ancestor.

false. the mitochondria in modern plant, animal and fungal cells are very similar, implying that these lines diverted after the mitochondrion was acquired by the ancestral eukaryote

3.8a. true or false:The second law of thermodynamics states that the total amount of energy in the Universe does not change.

false. the second law of thermodynamics states that the components of any system move toward greater disorder. it is the first law of thermodynamics that states that energy is neither created nor destroyed

You discover a dog suffering from a strange disorder that is due to an autoimmune response against its own cells. Upon further investigation, you find that the cells of various tissues are loosely arranged and do not have characteristic shapes and that the animal is prone to hemorrhage as the result of poor blood clotting. Based on these results, the target of the autoimmune response could be A) fibronectins. B) laminins. C) extensins. D) pectins. E) desmotubules.

fibronectins.

3.27.b. Chemical reactions occur only when there is a loss of BLANK energy.

free

The processes of transcription and translation are collectively known as RNA processing. gene duplication. protein synthesis. DNA synthesis. gene expression.

gene expression.

What is the overall equation for cellular respiration?

glucose + Oxygen → Carbon dioxide + Water + Energy

3.25.ΔG° indicates the change in the standard free energy as a reactant is converted to product. Given what you know about these values, which reaction below is the most favorable? (a) ADP + Pi ATP ΔG° = +7.3 kcal/mole (b) glucose 1-phosphate glucose 6-phosphate ΔG° = -1.7 kcal/mole (c) glucose + fructose sucrose ΔG° = +5.5 kcal/mole (d) glucose CO2 + H2O ΔG° = -686 kcal/mole

glucose CO2 + H2O ΔG° = -686 kcal/mole

ΔG° indicates the change in the standard free energy as a reactant is converted to product. Given what you know about these values, which reaction below is the most favorable?

glucose LaTeX: \leftrightarrow ↔ CO2 + H2O ΔG° = -686 kcal/mole

15-32 Briefly describe the mechanism by which an internal stop-transfer sequence in a protein causes the protein to become embedded in the lipid bilayer as a transmembrane protein with a single membrane-spanning region. Assume that the protein has an N-terminal signal sequence and just one internal hydrophobic stop-transfer sequence.

he N-terminal signal sequence initiates translocation and the protein chain starts to thread through the translocation channel. When the stop-transfer sequence enters the translocation channel, the channel discharges both the signal sequence and the stop- transfer sequence sideways into the lipid bilayer. The signal sequence is then cleaved, so that the protein remains held in the membrane by the hydrophobic stop-transfer sequence.

40. eukaryotic cells are able to trigger the release of material from secretory vesicles to the extracellular space using a process called exocytosis. an example of materials commonly released this way is...

hormones

Free ________ molecules have lubricating properties and are therefore abundant in the joints, for example.

hyaluronate

Some observations have suggested to researchers that substances associated with the extracellular matrix may be involved in cell migration. This has been observed with the cell marker CD44 on the surface of cancer cells and their ability to be separate from one another and to easily metastasize to other regions of the body. This is most likely due to the interaction of CD44 and A) chondrotin sulfate. B) ICAMs. C) hyaluronate. D) fibronectins. E) elastin.

hyaluronate

The molecule of the ECM of animal cells that is associated with joint lubrication and friction reduction is A) glycosaminoglycan. B) proteoglycan. C) hyaluronate. D) pectin. E) fibronectin.

hyaluronate

The plant cell wall network is comprised of all of the following except A) hyaluronate. B) cellulose. C) extensin. D) hemicellulose. E) pectin.

hyaluronate

2.61. both DNA and RNA are synthesized by covalently linking a nucleoside triphosphate to the previous nucleotide, constantly adding to a growing chain. in the case of DNA, the new strand becomes part of a stable helix. the two strands are complementary in sequence and antiparallel in directionality. what is the principal force that holds these two strands together?

hydrogen bonds

Cyclic AMP (cAMP) is a small molecule that associates with its binding site with a high degree of specificity. Which types of noncovalent interactions are the most important for providing the "hand in a glove" binding of cAMP?

hydrogen bonds

Which type of interaction stabilizes the α-helix and the β-pleated sheet structures of proteins? hydrophobic interactions ionic bonds peptide bonds hydrogen bonds disulfide bonds

hydrogen bonds

2.33. the pH of an aqueous solution is an indication of the concentration of available protons. however, you should not expect to find lone protons in solution; rather, the proton is added to a water molecule to form a BLANK ion.

hydronium

The pH of an aqueous solution is an indication of the concentration of available protons. However, you should not expect to find lone protons in solution; rather, the proton is added to a water molecule to form a(n) ______________ ion.

hydronium

Two or three α helices can sometimes wrap around each other to form coiled-coils. The stable wrapping of one helix around another is typically driven by ________________ interactions.

hydrophobic

2.43.c. inorganic or organice: sodium chloride

i

can be stimulated when a specific molecule binds to a regulatory site

inducible operon

In eukaryotes, when mature mRNA is hybridized to complementary DNA, ______. exons loop out as single-stranded regions of RNA introns loop out as single-stranded regions of DNA exons loop out as single-stranded regions of DNA introns loop out as single-stranded regions of RNA

introns loop out as single-stranded regions of DNA

Intervening sequences that are transcribed, but not translated into protein are called exons. introns. spliceosomes. transposons. transcription factors.

introns.

morphallaxis

involves the regeneration of new parts from differentiated cells that are "programmed" to perform a new function

Name the three main classes of cell-surface receptor.

ion-channel-coupled receptors; G-protein-coupled receptors; enzyme-coupled receptors

2.30. when atoms are held together by BLANK, they are typically referred to as BLANK. a. hydrogen bonds, molecules b. ionic interactions, salts. c. ionic interactions, molecules

ionic interactions, salts

2.56.c. in solutions of neutral pH, amino acids are BLANK, carrying both a positive and negative charge.

ionized

Anticodon

is a three-based sequence in tRNA molecule that is complementary to a codon in mRNA bind to each other.

ATP synthase

is an important enzyme that provides energy for the cell to use through the synthesis of adenosine triphosphate (ATP).

spliceosome

is comprised of several different subunits knowns as snRNPS(pronounced "snurps").they bind to specific sequences at three locations

Messenger RNA (mRNA)

is created from structural genes ,it's function is to carry infomation from the DNA to cellular components called ribosomes

The statement "DNA → RNA → Proteins" _____. describes a series of catalytic reactions is known as central dogma is the same in all organisms including viruses and prions depicts the regulation of gene expression

is known as central dogma

Chromatography

is the process in which different substances can be separated on a piece of paper because each will travel up the paper in a solvent at a different rate. the rate depends on the solubility of each substance

In isoelectric focusing, proteins migrate in an electrical field and "focus" at a particular location in the polyacrylamide gel and stop when they reach their _____________.

isoelectric point

how can miRNA and siRNA affect gene expression?

it degrades mRNA or blocks its translation

the working of operons is important because

it represents a principle means by which genes are regulated in prokaryotes

Match the type of intermediate filament with its appropriate location. lamins _________ A. nerve cells neurofilaments _________ B. epithelia vimentins _________ C. nucleus keratins _________ D. connective tissue

lamins ____C____ neurofilaments____A____ vimentins ____D____ keratins ____B____

Why is there a limit to the size of a cell?

large cells still have to transport materials but have a larger volume to supply to

Scaffolding proteins are ________.

large molecules to which several relay proteins attach to facilitate cascade effects

Prokaryotic cells are able to evolve very fast, which helps them to rapidly adapt to new food sources and develop resistance to antibiotics. Which of the options below lists the three main characteristics that support the rapid evolution of prokaryotic populations?

large population, rapid growth, can exchange DNA

You are investigating cell localization by targeting the carbohydrates associated with various cell types. Based on this molecule's ability to bind various carbohydrates, which of the following would help determine which carbohydrates are found on the cell surface? A) cadherins B) proteoglycans C) pectins D) connexins E) lectins

lectins

2.18.b. these covalent bonds have characteristic bond BLANK and become stronger and more rigid when two electrons are shared in a BLANK.

length; double bond

DNA methylation and histone acetylation can regulate eukaryotic gene expression by

making specific genes available/unavailable for transcription

Actin bundles give shape and support to the () found in cells of the small intestine.

microvilli

24.b. Another organelle found in essentially all eukaryotic cells is the BLANK, which generates the chemical energy for the cell.

mitochondrion

27. what organelle has both an outer and inner membrane?

mitochondrion

All of the following are roles of microfilaments except

mitotic spindle formation.

9. select the option that best finishes the following statement: "evolution is a process that can be best understood on the based principles of...."

mutation and selection

Cell biologists often use cell culture to study a variety of species' cellular functions, including those of humans. Cells from a given (usually mammalian) organism are often fused with cancer cells in order to immortalize them and grow continuously. This technique is called ___________________.

mutation.

Mutations in () VII result in an autosomal recessive disorder called Usher's, which is characterized by congenital profound hearing loss, problems in the vestibular system, and retinitis pigmentosum.

myosin

The following proteins are important for cell movement. Match the following proteins with their function. myosin _____ A. nucleation of new actin filaments at the side of an existing filament ARP proteins _____ B. regulation of the availability of actin monomers profilin _____ C. important for the growth of straight, unbranched actin filaments integrins _____ D. contracting the rear of the cell formins _____ E. involvement in focal contacts

myosin ___D_____ ARP proteins ___A_____ profilin ___B_____ integrins ___E_____ formins ___C_____

Calc of percent change % increase in weight

new weight- old weight -------------------------- old weight x 100

saturated fat

no double bonds between carbon atoms; contains as many hydrogens as possible; good fats

3.6.If you weigh yourself on a scale one morning then eat four pounds of food during the day, will you weigh four pounds more the next morning? Why or why not? Hint: What happens to the atoms contained in the food as useful energy is derived from metabolizing the food molecules?

no, you will not weigh four pounds more the next morning because only a small portion of the mass of the food will form components of the body. much of the mass of food is either released as co2 and h2o that are breathed out into the atmosphere or converted into materials excreted as waste products.

Collagens are associated with which of the following types of cellular junctions? A) gap junctions B) tight junctions C) plasmodesmata D) adhesive junctions E) none of the above

none of the above

2.69.e. choose the class of amino acids most important for the interaction: tightly packing the hydrophobic interior core of a globular protein

nonpolar

A phospholipid is a _____

nonpolar lipid molecule that is made amphipathic by the addition of a phosphate

fats

not actually polymers, most common are triglycerides

2.71.b.Your lab director requests that you add new growth medium to the mammalian cell cultures before heading home from the lab on a Friday night. Unfortunately, you need to make fresh medium because all the premixed bottles of medium have been used. One of the ingredients you know you need to add is a mix of the essential amino acids (those that cannot be made by the cells, but are needed in proteins). On the shelf of dry chemicals you find the amino acids you need, and you mix them into your medium, along with all the other necessary nutrients, and replace the old medium with your new medium. On Sunday, you come in to the lab just to check on your cells and find that the cells have not grown. You are sure you made the medium correctly, but on checking you see that somebody wrote a note on the dry mixture of amino acids you used: "Note: this mixture contains only ᴅ-amino acids." Are there any organisms that could grow using this mixture? Justify your answer.

not unless l-amino acids were also mixed in. certain types of bacteria use d-amino acids to product their cell walls, but they would still require l-amino acids for the rest of the proteins they make.

2.70. it is now a routine task to determine the exact order in which individual subunits have been linked together in polynucleotides (DNA) and polypeptides (proteins). however, it remains difficult to determine the arrangement of monomers in a polysaccharide. explain why this is the case.

nucleotides and amino acids have an intrinsic directionality, and the mechanism by which monomers are added into a growing polymer is always the same. this yields a linear polymer with the same directionality as the monomers. polysaccharides are produced by linking monosaccharides together. the monosaccharides can be either added directly or modified to produce various derivatives before addition. beyond this, there are multiples sites on each monosaccharide where addition can occur, producing highly complex, branched polymers.

24. a. Eukaryotic cells are bigger and more elaborate than prokaryotic cells. By definition, all eukaryotic cells have a BLANK, usually the most prominent organelle.

nucleus

2.43.b. inorganic or organice: ethanol

o

2.43.h. inorganic or organice: glycine

o

2.43.k. inorganic or organice: phospholipid

o

nucleation

oligomer formation

Which stage of microtubule assembly is the stage in which nucleation occurs?

oligomers

2.67. As a protein is made, the polypeptide is in an extended conformation, with every amino acid exposed to the aqueous environment. Although both polar and charged side chains can mix readily with water, this is not the case for nonpolar side chains. Explain how hydrophobic interactions may play a role in the early stages of protein folding, and have an influence on the final protein conformation.

one reason that nonpolar groups are excluded from an aqueous environment is that hydrophobic surface would organize water into a highly structured network of hydrogen bonds, which is energetically unfavorable.So, you would expect that nonpolar amino acids would group together early, forming "hydrophobic pockets," while the polar and charged side chains remain at the interface of the surrounding solution. In the final, folded protein, most of the nonpolar amino acids will remain buried inside the protein. This fold is more stable because nonpolar atoms are prevented from contact with water and remain in contact with each other.

3.14.Your body extracts energy from the food you ingest by catalyzing reactions that essentially "burn" the food molecules in a stepwise fashion. What is another way to describe this process? (a) reduction (b) oxidation (c) dehydration (d) solvation

oxidation

3.22.Seed oils are often dehydrogenated and added back into processed foods as partly unsaturated fatty acids. In comparison with the original oil, the new fatty acids have additional double carbon-carbon bonds, replacing what were once single bonds. This process could also be described as _____________. (a) isomerization. (b) oxidation. (c) reduction. (d) protonation.

oxidation

2.38.a. what is the pH of pure water?

pH 7

13. d.uses a light microscope with an optical component to take advantage of the different refractive indices of light passing through different regions of the cell

phase contrast

Which type of lipids are the most abundant in the plasma membrane?

phospholipids

2.18.d. these BLANK covalent bonds should not be confused with the weaker BLANK bonds that are critical for the 3d structure of biological molecules and for the interactions between these molecules

polar noncovalent

The transcription process in a eukaryotic gene directly produces _________. rRNA pre-mRNA mRNA DNA Protein

pre-mRNA

cellular respiration

process occurring in the mitochondria that produces ATP from carbon-based molecules in the presence of oxygen.

Structural genes

produce RNA molecule that contains the information to specify a polypeptide with a particular amino acid sequence.

Transcription

produces an RNA copy of a gene, transcription literally means the act of making a copy.

57. caenorhabditis elegans is a nematode. during its development, it produces more than 1000 cells. however, the adult worm has only 959 somatic cells. the process by which 131 cells are specifically arget for destruction is called BLANK

programmed cell death

62.d.match the following biological process with the model organism that is best suited or most specifically useful for its study: c. elegans

programmed cell death

Transcription begins near a site in the DNA called the ______, while the terminator specifies the end of transcription.

promoter

Transcription begins near a site in the DNA called the ______. promoter enhancer response element transcription unit regulatory sequence

promoter

Which element(s) from the following list is/are part(s) of a bacterial operon?

promoter

epidermis

protect the leaf

Large numbers of ribosomes are present in cells that specialize in producing which of the following molecules?

proteins

Biological macromolecules

proteins, lipids, nucleic acids, carbohydrates

2.4. which subatomic particles contribute to atomic mass for any given element

protons and neutrons

The photograph shows female cats Rainbow and CC (CC is Rainbow's clone). Why is CC's coat pattern different from Rainbow's given that CC and Rainbow are genetically identical?

random X chromosome inactivation heterozygous at coat color gene locus environmental effects on gene expression

Reading frame

refers to the order in which codons are read during translation.

epimorphosis

regeneration involving the replacement of lost parts by indifferent cells

48) The cancer-causing forms of the Ras protein are involved in which of the following processes?

relaying a signal from a growth-factor receptor

The bonding of two amino acid molecules to form a larger molecule requires the ______

release of a water molecule

The three-dimensional coordinates of atoms within a folded protein are determined experimentally. After researchers obtain a protein's structural details, they can use different techniques to highlight particular aspects of the structure. What visual model best displays a protein's secondary structures (α helices and β sheets)?

ribbon

2.47. cells require one particular monosaccharide as a starting material to synthesize nucleotide building blocks. which of these fills this important role: glucose, fructose, ribulose, ribose

ribose

2.58.i. five-carbon sugar found in rna is called

ribose

44. which of the following characteristics would not support the idea that the ancestral eukaryote was a predator cell that captured and consumed other cells? a. dynamic cytoskeleton b. large cell size c. ability to move d. rigid membrane

rigid membrane

makes polypeptides

rough endoplasmic reticulum

The enzyme responsible for carbon fixation in the Calvin cycle, and possible the most abundant protein on Earth is ____

rubisco

2.18. whereas ionic bonds form a BLANK, covalent bonds between atoms form a BLANK.

salt molecule

The function of integrin-linked kinase during integrin-based signaling is A) integrin activation. B) anchorage-dependent growth. C) scaffolding protein. D) cell movement. E) anchorage-independent growth.

scaffolding protein

13. e. requires coating the sample with a thin layer of a heavy metal to produce three dimensional images of the surface of the sample

scanning electron

2.48. oligosaccharides are short sugar polymers that can become covalently linked to proteins and lipids though condensation reactions. these modified proteins and lipids are called glycoproteins and glycolipids, respectively. within a protein, which of the amino acids is the most probable target for this type of modification?

serine

14-3Based upon what you know about metabolism, explain how electrons are stripped from food molecules and used to drive the electron-transport chain.

several protein complexes in the electron transport chain function as proton pumps, using the energy of the electron to pump protons into the intermembrane space of the mitochondria

Which of the following is not true of the main structural forms of actin?

severed to form filament bundles.

2.41. silicon is n element, like carbon, has four vacancies in its outer electron shell and therefore has the same bonding chemistry as carbon. silicon is not found to any significant degree in the molecules found in living systems, however. does this difference arise because elemental carbon is more abundant than silicon? what other explanations are there for the preferential selection of carbon over silicon as the basis for the molecules of life?

silicon is actually more abundant in the earths crust than carbon, so this is not likely to be the reason that carbon was used preferentially. carbon might have been the element of choice in living systems because it is lighter than silicon and forms shorter covalent bonds with other elements. shorter bonds are typically stronger and more stable

A newly defined autoimmune disorder has resulted in an immune response against Type V collagen fibers. As a result, the most likely organ to be targeted in this response would be the A) skin. B) heart. C) lungs. D) stomach. E) tongue.

skin

3.50.The net distance a molecule travels through the cytosol via diffusion is relatively short in comparison with the total distance it may need to travel. This is because movement governed by diffusion alone is a ________________ process that is most effective for the dispersion of small molecules over short distances. (a) slow (b) random (c) regulated (d) complicated

slow

What type of organelle or structure is primarily involved in the synthesis of oils, phospholipids, and steroids?

smooth endoplasmic reticulum

assembles lipids for membrane

smooth endoplasmic reticulum

Spliceosomes are composed of _____. snRNPs and other proteins polymerases and ligases introns and exons the RNA transcript and protein snRNPs and snurps

snRNPs and other proteins

2.21.on the basis of the information in the chart and what you know about atomic surfaces, which elements will form ions with a net charge of +1 in solution?

sodium, potassium

Tonicity

solute concentration in one solution as compared to another solution

Isotonic

solute concentration is equal in the cell and the surrounding solution

Hypertonic

solute concentration outside a cell is greater than the solute concentration inside a cell

Hypotonic

solute concentration outside the cell is less than that inside the cell

Transporters, in contrast to channels, work by ________________.

specific binding to solutes.

Red blood cells have been very useful in the study of membranes and the protein components that provide structural support. Which of the following proteins is the principal fibrous protein in the cortex of the red blood cell?

spectrin

3.23.Chemical reactions that lead to a release of free energy are referred to as "energetically favorable." Another way to describe these reactions is: _____________. (a) uphill. (b) uncatalyzed. (c) spontaneous. (d) activated.

spontaneous

3.24.Even though cellular macromolecules contain a large number of carbon and hydrogen atoms, they are not all spontaneously converted into CO2 and H2O. This absence of spontaneous combustion is due to the fact that biological molecules are relatively __________ and an input of energy is required to reach lower energy states. (a) large (b) polar (c) stable (d) unstable

stable

2.42. selenium (se) is an element required in the human body in trace amounts. selenium is obtained through the diet and levels of selenium found in food depend greatly on the soil where it is grown. once ingested and absorbed as selenate, it can become incorporated into a small number of polypeptides. these selenoproteins are formed when selenium replaces an element that is found in two of the twenty "standard" amino acids. using your knowledge of atomic structure, the periodic table, and structure of amino acids, deduce which 2 amino acids may be converted to "seleno" amino acids and used to make selenoproteins.

sulfur is the only element found exclusively in two of the twenty amino acids. this element is located directly above selenium in the periodic table, indicating that these elements have the same number of electrons in their outer shell and both prefer to form bonds with other atoms to fill their outer orbital. if selenium instead of sulfur is incorporated into cysteine or methionine, the alteres "seleno" amino acids will be produced. we can expect that this substitution will alter the nature of the proteins in which these amino acids are incorporated because selenium is a large atom than sulfur

Macromolecules in the cell can often interact transiently as a result of noncovalent interactions. These weak interactions also produce stable, highly specific interactions between molecules. Which of the factors below is the most significant in determining whether the interaction will be transient or stable?

surface complementarity between molecules

Membrane lipids are capable of many different types of movement. Which of these does not occur spontaneously in biological membranes?

switching between lipid layers

As each codon binds at the active site of a ribosome, it interacts with another molecule. This molecule is called

tRNA

Which of the following serves as the "translator" or intermediary between an mRNA codon and an amino acid?

tRNA

One drug blocks cell division because it stabilizes the microtubules. What is this drug, and for what is it used?

taxol; breast cancer

What enzyme compensates for replication-associated shortening of linear chromosomes?

telomerase

At which level of protein structure are interactions between the side chains (R-groups) most important? primary secondary tertiary quaternary primary, secondary, tertiary, and quaternary

tertiary

What is the purpose of negative and positive tests?

test to see if the substrate catechol is oxidized at the perceptible rate

What is the purpose of the extract control?

test tp see of the enzymatic modification of catechol causes the color change or if it's another factor

During transcription in eukaryotes, a type of RNA polymerase called RNA polymerase II moves along the template strand of the DNA in the 3'→5' direction. However, for any given gene, either strand of the double-stranded DNA may function as the template strand. For any given gene, what ultimately determines which DNA strand serves as the template strand? the location of specific proteins (transcription factors) that bind to the DNA which of the two strands of DNA carries the RNA primer the base sequence of the gene's promoter the location along the chromosome where the double-stranded DNA unwinds

the base sequence of the gene's promotor

50. scientists learned that cell death is a normal and even import part of life by studying the development of the nematode worm C. elegans. What was the most important feature of C. elgans for the study of programmed cell death?

the developmental pathway of each cell in the adult worm was known

Which of the following is an exception to the central dogma? the discovery of RNA viruses that synthesize DNA using reverse transcriptase the discovery of ribozymes the discovery that the Archaea and Bacteria are more distantly related than are Archaea and Eukarya the discovery of DNA as the unit of genetic inheritance

the discovery of RNA viruses that synthesize DNA and reverse transcriptase

The graph below illustrates the relationship between reaction rates and substrate concentration for an enzyme-catalyzed reaction. What does the Km value indicate with respect to enzyme-substrate interactions?

the enzyme-substrate binding affinity

2.71.a. Your lab director requests that you add new growth medium to the mammalian cell cultures before heading home from the lab on a Friday night. Unfortunately, you need to make fresh medium because all the premixed bottles of medium have been used. One of the ingredients you know you need to add is a mix of the essential amino acids (those that cannot be made by the cells, but are needed in proteins). On the shelf of dry chemicals you find the amino acids you need, and you mix them into your medium, along with all the other necessary nutrients, and replace the old medium with your new medium. On Sunday, you come in to the lab just to check on your cells and find that the cells have not grown. You are sure you made the medium correctly, but on checking you see that somebody wrote a note on the dry mixture of amino acids you used: "Note: this mixture contains only ᴅ-amino acids." What is the meaning of the note and how does it explain the lack of cell growth in your culture?

the note indicates that the mixture contains only one of the two possible stereoisomers (l or d). because mammalian cells use only the l stereoisomer, the d-amino acid mixture could not be used and therefore it is as though no amino acids were added at all.

What does a magnification of 40x mean?

the object that is being viewed under the microscope is being magnified 40x

within a cell, the amount of protein made using a given mRNA molecule depends partly on

the rate at which the mRNA is degraded

The study of enzyme kinetics is usually performed with purified components and requires the characterization of several aspects of the reaction, including the rate of association with the substrate, the rate of catalysis, and _____________.

the regulation of the enzyme activity.

what is an example of post-transcriptional control of gene expression?

the removal of introns and alternative splicing of exons

What is the normal fern medium used for

the spores are viable, healthy and can make the transition to 2D growth

2.60. the cell is able to harvest energy from various processes in order to generate ATP molecules. these ATPs represent a form of store energy that can be used later to drive other important processes. explain how the cell can convert the chemical energy store in ATP to generate mechanical energy, for example changing the shape of a protein

the terminal phosphate group is typically hydrolyzed and the energy released from this chemical bond is often "reinvested" to generate a new bond that links the phosphate group to a protein. this addition of a phosphate group can cause a change in the protein's conformation. this conformational change is usually associated with change in function of transient interactions with other macromolecules, generating a domino effect within the cell.

release factor

the three-dimmensional structure of a release factor protein mimics the structure of tRNAs which allows it to fit into the A site.

what is the fern meduim + 2-thiouracil used for

these are experimental treatments. It lets you ask RNA transcription is required for 2D growth

Heterotroph

they get energy from compounds produced by other organisms

In step 4 of the citric acid cycle, the reduction of NAD+ to NADH is coupled to the generation of CO2 and the formation of a high-energy thioester bond. The energy of the thioester bond is harnessed in step 5. What is the energy used for?

to generate a molecule of GTP

What is the purpose of the light reactions?

to make energy for the Calvin cycle

Which of the following regions of a structural gene contains the information that specifies an amino acid sequence?

transcribed region

What protein influences the ability of RNA polymerase to transcribe genes? DNA polymerases DNA helicases transcription factors snRNPs tRNA

transcription factors

The process that produces mRNA from DNA is called transcription. translation. replication. processing. post-translational modification.

transcription.

Which of the following are products of nonstructural genes, and are therefore never translated?

transfer RNA and ribosomal RNA

2.17. an ionic bond between two atoms is formed as a result of the BLANK

transfer of electrons from one atom to another

Beginning within the nucleus, the first step leading to the synthesis of a polypeptide is _____. translation of a DNA nucleotide sequence into a sequence of amino acids linking of nucleotides to form a polypeptide transferring of information from DNA to messenger RNA translation of an RNA nucleotide sequence into a sequence of amino acids removal of introns from RNA and the stitching together of exons

transferring of information from DNA to messenger RNA

Tubulin may assemble and disassemble simultaneously in a process known as

treadmilling

2.54. many types of cells have stores of lipids in their cytoplasm, usually seen as fat droplets. what is the lipid most commonly found in these droplets?

triacylglycerol

14.b. true or false: the presence of the plasma membrane can be inferred by the well-defined boundary of the cell

true

19.b. true or false: prokaryotes can adopt several different basic shapes, including spherical, rod-shaped, and sprial

true

2.15.a. true or false: electron shells fill discrete regions around the nucleus of the atom and limit the number of electrons that can occupy a specific orbit

true

2.15.c. true or false: some atoms are more stable when they lose one or two elections, even though this means they will have a net positive charge.

true

2.19.a. true or false: electrons are constantly moving around the nucleus of the atom, but they can move only in discrete regions

true

2.24.a. true or false: there are 4 elements that constitute 99% of all the atoms found in the human body.

true

2.35.b. true or false: protons are constantly moving between water molecules, which means there is an overall equilibrium between hydroxyl ions and hydronium ions in aqueous solutions

true

2.35.c. true or false: a strong base is defined as a molecule that can readily remove protons from water

true

2.52.b. true or false: the hydroxyl groups on monosaccharides are reaction hot spots and can be replaced by other functional groups to produce derivatives of the original sugar

true

2.65.c. true or false: all proteins and RNAs pass through many unstable conformations as they are folded, finally settling on one single, preferred conformation.

true

2.b. true or false: cells of different types can have different chemical requirements

true

2.c. true or false: the branchlike extensions that sprout from a single nerve cell in a mammalian brain can extend over several hundred micrometers

true

21.a. true or false: oxygen is toxic to certain prokaryotic organisms

true

3.19.c. true or false: The oxidation of one molecule is always coupled to the reduction of a second molecule.

true

3.19.d. true or false:During cellular respiration, carbon-containing molecules become successively more oxidized until they reach their most oxidized form, as CO2.

true

3.32.c. true or false: enzymes increase the probability that any given reactant molecule will be convert to product.

true

3.39.a.On the basis of the two reactions below, decide which of the following statements are true and which are false. If a statement is false, explain why it is false. 1: ATP + Y Y-P + ADP ΔG = -100 kcal/mole 2: Y-P + A B ΔG = 50 kcal/mole Reaction 1 is favorable because of the large negative ΔG associated with the hydrolysis of ATP.

true

3.39.b.On the basis of the two reactions below, decide which of the following statements are true and which are false. If a statement is false, explain why it is false. 1: ATP + Y Y-P + ADP ΔG = -100 kcal/mole 2: Y-P + A B ΔG = 50 kcal/mole Reaction 2 is an example of an unfavorable reaction.

true

3.39.c.On the basis of the two reactions below, decide which of the following statements are true and which are false. If a statement is false, explain why it is false. 1: ATP + Y Y-P + ADP ΔG = -100 kcal/mole 2: Y-P + A B ΔG = 50 kcal/mole Reactions 1 and 2 are coupled reactions, and when they take place together, reaction 2 will proceed in the forward direction.

true

46.b. true or false: the cytoskeleton is used as a transportation grid for the efficient, directional movement of cytosolic components

true

46.c. true or false: thermal energy promotes random movement of proteins, vesicles, and small molecules in the cytosol

true

48.b. true or false: protozoans are single-celled eukaryotes with cell morphologies and behaviors that can be as complex as those of some multicellular

true

60.a. true or false: the human genome is roughly 30 times larger than the arabidopsis genome, but contains approximately the same number of protein-coding genes

true

The tertiary structure of a protein is the _____. overall protein structure resulting from the aggregation of two or more polypeptide subunits order in which amino acids are joined in a polypeptide chain organization of a polypeptide chain into an α-helix or β-pleated sheet unique three-dimensional shape of the fully folded polypeptide bonding together of several polypeptide chains by weak bonds

unique three-dimensional shape of the fully folded polypeptide

15-44 v-SNAREs and t-SNAREs mediate the recognition of a vesicle at its target membrane so that a vesicle displaying a particular type of v-SNARE will only fuse with a target membrane containing a complementary type of t-SNARE. In some cases, v-SNAREs and t-SNAREs may also mediate the fusion of identical membranes. In yeast cells, right before the formation of a new cell, vesicles derived from the vacuole will come together and fuse to form a new vacuole destined for the new cell. Unlike the situation we have discussed in class, the vacuolar vesicles contain both v-SNAREs and t-SNAREs. Your friend is trying to understand the role of these SNAREs in the formation of the new vacuole and consults with you regarding the interpretation of his data. Your friend has designed an ingenious assay for the fusion of vacuolar vesicles by using alkaline phosphatase. The protein alkaline phosphatase is made in a "pro" form that must be cleaved for the protein to be active. Your friend has designed two different strains of yeast: strain A produces the "pro" form of alkaline phosphatase (pro-Pase), whereas strain B produces the protease that can cleave pro-Pase into the active form (Pase). Neither strain has the active form of the alkaline phosphatase, but when vacuolar vesicles from the strains A and B are mixed, fusion of vesicles generates active alkaline phosphatase, whose activity can be measured and quantified(Figure Q15-44A).Figure Q15-44Your friend has taken each of these yeast strains and further engineered them so that they express only the v-SNAREs, only the t-SNAREs, both SNAREs (the normal situation), orneither SNARE. He then isolates vacuolar vesicles from all strains and tests the ability of each variant form of strain A to fuse with each variant form of strain B, by using the alkaline phosphatase assay. The data are shown in the graph in Figure Q15-44B. On this graph, the SNARE present on the vesicle of the particular yeast strain is indicated as "v" (for the presence of the v-SNARE) and "t" (for the presence of the t-SNARE).What do his data say about the requirements for v-SNAREs and t-SNAREs in thevacuolar vesicles? Is it important to have a specific type of SNARE (that is, v-SNARE or t-SNARE) on each vesicle?

v-SNAREs and t-SNAREs mediate the recognition of a vesicle at its target membrane so that a vesicle displaying a particular type of v-SNARE will only fuse with a target membrane containing a complementary type of t-SNARE. In some cases, v-SNAREs and t-SNAREs may also mediate the fusion of identical membranes. In yeast cells, right before the formation of a new cell, vesicles derived from the vacuole will come together and fuse to form a new vacuole destined for the new cell. Unlike the situation we have discussed in class, the vacuolar vesicles contain both v-SNAREs and t-SNAREs. Your friend is trying to understand the role of these SNAREs in the formation of the new vacuole and consults with you regarding the interpretation of his data. Your friend has designed an ingenious assay for the fusion of vacuolar vesicles by using alkaline phosphatase. The protein alkaline phosphatase is made in a "pro" form that must be cleaved for the protein to be active. Your friend has designed two different strains of yeast: strain A produces the "pro" form of alkaline phosphatase (pro-Pase), whereas strain B produces the protease that can cleave pro-Pase into the active form (Pase). Neither strain has the active form of the alkaline phosphatase, but when vacuolar vesicles from the strains A and B are mixed, fusion of vesicles generates active alkaline phosphatase, whose activity can be measured and quantified (Figure Q15-44A) Your friend has taken each of these yeast strains and further engineered them so that they express only the v-SNAREs, only the t-SNAREs, both SNAREs (the normal situation), or neither SNARE. He then isolates vacuolar vesicles from all strains and tests the ability of each variant form of strain A to fuse with each variant form of strain B, by using the alkaline phosphatase assay. The data are shown in the graph in Figure Q15-44B. On this graph, the SNARE present on the vesicle of the particular yeast strain is indicated as "v" (for the presence of the v- SNARE) and "t" (for the presence of the t-SNARE). What do his data say about the requirements for v-SNAREs and t-SNAREs in the vacuolar vesicles? Is it important to have a specific type of SNARE (that is, v- SNARE or t-SNARE) on each vesicle? To get maximal levels of vacuolar vesicle fusion, vesicles from each strain must carry both v-SNAREs and t-SNAREs. Experiment 1, which represents the normal scenario, is the only experiment in which 100% alkaline phosphatase activity is measured. However, as long as complementary SNAREs are present on the vesicles, some vesicle fusion does occur (see experiments 3, 4, 6, 7, 8, and 9). If both vesicles are missing v-SNAREs (experiment 2) or t-SNAREs (experiment 5) or both SNAREs (experiments 10 and 11), the level of fusion is very low. It does not matter whether a t-SNARE or a v-SNARE is on the vesicle of a particular strain, as long as the vesicle from the other strain contains a complementary SNARE (compare experiments 3 and 4, 6 and 7, and 8 and 9).

2.27. polar covalent bonds are formed when the electrons in the bond are not shared equally between the two nuclei. which one of these molecules contain polar bonds: molecular oxygen, methane, propane, water

water

The citric acid cycle is a series of oxidation reactions that removes carbon atoms from substrates in the form of CO2. Where do the oxygen atoms in the carbon dioxide molecules come from?

water

3.40.The potential energy stored in high-energy bonds is commonly harnessed when the bonds are split by the addition of _______________ in a process called _____________. (a) ATP, phosphorylation. (b) water, hydrolysis. (c) hydroxide, hydration. (d) acetate, acetylation.

water, hydrolysis

15. cell biologists employ targeted fluorescent dyes or modified fluorescent proteins in both standard fluorescence microscopy and confocal microscopy to observe specific details in the cell. even though fluorescence permits better visualization, the resolving power is essentially the same as that of a standard light microscope because the resolving power of a microscope is limited by the BLANK of light.

wavelength

Cell biologists employ targeted fluorescent dyes or modified fluorescent proteins in both standard fluorescence microscopy and confocal microscopy to observe specific details in the cell. Even though fluorescence permits better visualization, the resolving power is essentially the same as that of a standard light microscope because the resolving power of a microscope is limited by the __________ of light.

wavelength

29. You fertilize egg cells from a healthy plant with pollen (which contains the male germ cells) that has been treated with DNA-damaging agents. You find that some of the offspring have defective chloroplasts, and that this characteristic can be passed on to future generations. This surprises you at first because you happen to know that the male germ cell in the pollen grain contributes no chloroplasts to the fertilized egg cell and thus to the offspring. What can you deduce from these results?

your results show that not all of the information required for making a chloroplast is encoded in the chloroplasts own DNA; some, at least, must be encoded in the DNA carried in the nucleus. The reasoning is as follows: genetic info is carried only in DNA, so the defect in the chloroplasts must be due to a mutation in DNA. But all of those chlorplasts in the offspring (and thus all of the chloroplast DNA) must derive from those in the female egg cell, since chloroplasts only arise from other chloroplasts. Hence, all of the chloroplasts contain undamaged DNA from the female parent's chloroplasts. In all the cells of the offspring, however, half of the nuclear DNA will have come from the male germ-cell nucleus, which combined with the female egg nucleus at fertilization. Since this DNA has been treated with DNA-damaging agents, it must be the source of the heritable chloroplast defect. Thus, some of the info required for make a chloroplast is encoded by the nuclear DNA.

Which of the following proteins belongs to the same family (GPCRs) as Rhodopsin?

β -adrenergic receptor

14-50 For each of the following sentences, choose one of the options enclosed in square brackets to make a correct statement. An electron bound to a molecule with low affinity for electrons is a [high/low]-energy electron. Transfer of an electron from a molecule with low affinity to one with higher affinity has a [positive/negative] ΔG°and is thus [favorable/unfavorable] under standard conditions. If the reduced form of a redox pair is a strong electron donor with a [high/low] affinity for electrons, it is easily oxidized; the oxidized member of such a redox pair is a [weak/strong] electron acceptor.

• 1) High • 2) Negative • 3) Favorable


Kaugnay na mga set ng pag-aaral

Chemistry - Industrial Chemistry - Important Industrial Processes

View Set

Virology 1+2+3+4+5+6+7+8+9+10+11

View Set